Sunteți pe pagina 1din 169

ACP Medical Knowledge Self-Assessment Program•

Rheumatology

22AMAPRA
Category 1 Credits™
available until
ACP'
American College of Physicians
leading Internal Medicine,lmpr011ing Uv&s
July 31,2021.
Table of Contents

Approach to the Patient with Febuxostat . . . . . . ........................ 13


Rheumatologic Disease Uricosuric Agents ...................... . .... 13
Inflammatory Versus Noninflammatory Pain ......... 1 Pegloticase .............. ............ ....... 13
The Musculoskeletal Examination ......... ....... .. 1 Medications and Pregnancy ...... . ... ..... ....... . 14
ArthritiS ........................................ 1 Vaccination and Screening In Immunosuppression ... 14
Monoarthrltls ........ •..... . ............ . .... 1 Nonpharmacologlc and Nontraditional Management... 14
Oligoarthrltis ........ . ....................... 2 Physical and Occupational Therapy ..... ....... 14
Polyarthritis ................................. 2 Complementary and Alternative Medicine .. . ... 14
Soft-Tissue Abnormalities ........................ . 2 Role of Surgery ............................. l6
Extra-Articular Manifestations of
Rheumatologic Disease ..................... ... ... 2 Rheumatoid Arthritis
Constitutional Symptoms ...................... 2 Pathophysiology and Risk Factors .................. 16
Skin Involvement ............................ 2 Genetic Factors ............................. 16
Eye Involvement ............................ . 2 Environmental Factors ....................... 16
Internal Organ Involvement .................... 2 Infectious Agents... . ...................... 16
Laboratory Studies . . . . . . . . . ..................... 2 Hormones . . . . . . . . . ........ ... ............ 16
Tests That Measure Inflammation ............... 2 Diagnosis ...................................... 16
Autoantibody Tests . . . . ................ ..... 5 Cllmcal Manifestations ....................... 16
Imaging Studies... . . . . ......................... 5 Laboratory Studies .......................... 17
Radiogmphy ...... . ... . ...................... 5 Imaging Studies ............................. 18
cr ............................. ............ 5 Complications and Extra-Articular Manifestations .... 18
MRl ........................................ 5 Joints............... .. .............. ..... .. l8
Ultrasonography ...... . ...................... 5 Skin. . ................................. .. 19
Joint Aspiration .......................... .... ... . 7 Eyes .......... ...... .. ....... . . ............ 19
Tissue Biopsy .................................... 7 Lungs ............ • ........................ 19
Heart. ................. .................... 19
Principles of Therapeutics Hematologic ........... . .................... 19
Overview ........................... . ........... 8 Blood Vessels ............................... 20
Anti-Inflammatory Agents .............. ... ...... .. 8 Management ................................... 20
GlucocortJcolds .............................. 8 General Considerations....................... 20
NSAJDs ............................. ... . .... 8 Disease-Modifying Antirheumatic Drugs ........ 20
Colchicine .................................. 9 NSAIDs ................................. 21
Analgesics and Pain Pathway Modulators ............ 9 Glucocorticolds ............................. 21
Acetamlnophen ........................... ... 9 Surgery .............................. . ... .. 21
Tramadol ................................... 9 Pregnancy . . . . . . . . . . . . . . . . . . . . . . . . . . . . . . . . . . . . . 21
Duloxetine .................................. 9
Gabapentinoids .............................. 9 Osteoarthritis
Disease-Modifying Antirhewnatic Drugs............. 9 Pathophysiology ................................ 21
Nonblologic Disease-ModifYing Epidemiology and Risk Factors .................... 22
Antirheumatic Drugs ......................... 9 Qassiflcation ................................... 22
Biologic Disease-Modifying Primary Osteoarthritis ....................... 22
Antirheumatic Drugs ........... · · . . . . . . . . . . . 11
Secondary Osteoarthritis ..................... 22
Urate-Lowering Therapy ......................... 12
Diffuse Idiopathic Skeletal Hyperostosis ........ 23
Allopurinol. ......... ....................... U

ix
Diagnosis .............................. . ....... 24 Diagnosis . ... .................................. 38
Clinical Manifestations ....................... 24 General Considerations.............. . ........ 38
Laboratory and Imaging Studies ............. . . 24 Laboratory Studies ................. . ........ 38
Differential DiagnosiS ............................ 25 Differential Diagnosis ........................ 39
Management ....................... . .......... . 26 Management .................................. . 40
Nonpharmacologic Therapy ................... 26 Pregnancy and Childbirth Issues................... 41
Pharmacologic Therapy ...................... 26 Prognosis ........ . .............. . .............. 4J
Smgical Therapy ................. . .......... 27
Sj ijgren Syndrome
Fibromyalgla Epidemiology and Pathophysiology ... .... ......... 42
Epidemiology and Pathophysiology ................ 27 Clinical Manifestations ........................... 42
Diagnosis . ................... ...... ......... ... 27 Diagnosis ................ . .. . ... . .............. 42
Management ................................... 27 Management ..................... . ............. 43
Prognosis ................ . ..................... 43
Spondyloarthritis
Overview ............................. , ........ 28 Inflammatory Myopathies
Pathophysiology ...... . ....... . ......... ........ 29 Overview .................... • ................. 43
Genetic Factors ... ... . . ................ ..... 29 Evaluation of the Patient with
Envirorunental Factors ..... ..... ............. 29 Muscle Pain or Weakness ......................... 43
ClasSification ................................... 29 Epidemiology and Pathophysiology of
Ankylosing Spondylitis ....................... 30 lnflammatory Myopathies ........................ 44
Psoriatic Arthritis . . . . . . . . . . . . . . . . . . . . . . . . . . . 31 Polymyositis ...................... • ............. 45
fnflammatory Bowel Clinical Manifestations ....................... 45
Disease-Associated Arthritis .................. 32 Dermatomyositis ................................ 45
Reactive Arthritis.... .... ... ................. 32 Clinical Manifestations ................. . ..... 45
Diagnosis ...................................... 32 Immune-Mediated Necrotizing Myopathy ........... 45
Laboratory Studies .......................... 32 Clinical Manifestations ......... . ....... . ..... 46
Imaging Studies ......... . ...... ..... ....... . 33 Inclusion Body Myositis .......................... 46
Management .................... •. ............. 34 Clinical Manifestations ....................... 46
General Considerations..... . ....... ..... ..... 34 Inflammatory Myopathies and Malignancy ... . . ..... 46
Ankylostng Spondylitis ...... ........ ......... 34 Diagnosis ...................................... 46
Psoriatic Arthritis ........................... 34 Muscle-Related Enzymes ..................... 46
lnflanunatory Bowel Autoantibodies .. ..... ...... ................ 47
Disease-Associated Arthritis .................. 34 hnaging Studies ... ..... ... .................. 47
Reactive Arthritis. ... . . ................ .. .... 34 Electromyography ........................... 47
Muscle Biopsy ....... . ..... . ........ . ....... 47
Systemic Lupus Erythematosus Skin Biopsy ................................ 47
Epidemiology and Pathophysiology ................ 35 Management ................................... 47
Clinical Manifestations ........................... 35 Prognosis ... ..................... ... ....... .... 48
Mucocutaneouslnvolvement .................. 35
Musculoskeletal Involvement.................. 36 Systemic Sclerosis
Kidney Involvement ......................... 36 Epidemiology and Pathophysiology ...... ... .... ... 48
Neuropsyctrlabdclnvolverneot ..... . . ... ...... . 37 Classification ................................... 48
Cardiovascular Involvement............. ..... . 37 Clinical Manifestations and Diagnosis .............. 49
Pulmonary Involvement ...... ................ 37 CUtaneous Involvement ... .... ....... ........ 49
Hematologic Involvement. .................... 37 Musculoskeletal Involvement. ................. 49
Gastrointestinal Involvement .................. 38 Vascular Involvement ........................ 51
Association with Malignancy ...................... 38 Gastrointestinal Involvement ................. . 51

X
Kidney Involvement . . . .. .. . .. .. . . . .. ... . . . .. 52 Mycobacterium tuberculosis . . ... .. ... .. . . . .. . 62
Lung Involvement ....... .. . .. . . .... .. ... . ... 52 Fungal infections . ..... . ..... . . , ......... . ... 62
carcUacJnvolvement ......... ....... . . -.- .. -. 52 Viral infections . ..... . ... . . . ... .. . . . . .. .. . .. 62
Management . . . ..... ... . . . ... . .. . . ... ..... .. . . . 53 Prosthetic Joint Infections . . .... .. . .. .... . . . .. 63
Pregnancy .. . .... . .......... . . . . . .......... . ... S4 Management .. . . .. .......... . . . ..... . . .. . . ..... 64

Mixed Connective Tissue Disease Systemic Vasculitis


Overview . . . . . . . . . . . . . . . . . . . . . . . . . . . . . . . . . . . . . 54 Overview . . . . . . . . . . . . . . . . . . . . . . . . . . . . . . . . . . . . . 65
Epidemiology and Pathophysiology .. . . .. . ...... . . . 55 Large-Vessel Vasculitis ............ .. ........ .. ... 66
Clinical Manifestations and Diagnosis . .... . . . .. . . . . 55 Giant Cell Arteritis .. ... . . .. . . . ... ... . ... .. .. 66
Management ........ .. .. . ...... . ............... 55 Polymyalgia Rheumatica .......... . ....... . .. 67
Prognosis ... . ...... .. ....... • . . ... . . .. .. .. ... . . 55 Takayasu Arteritis .. .. ..... . ...... . . . . ... . . .. 67
Medium-Vessel Vasculitis ..... . .... .. .. . ....... . .. 68
Crystal Arthropathies Polyarteritis Nodosa . . ... . ................ . .. 68
Gout .. . ........................ . . . ........ .. .. 55 Primary Angiitis of the Central Nervous System . .. 68
Epidemiology ............ . .. . ........ . .. .. . . 55 Kawasaki Disease .. .. ... . ... . .. . . . ... . .. .... 69
Pathophysiology . . . . . .. ..... . .... .. ......... 55 Small-Vessel Vasculitis .. . .... ... ... . . .. . . . . ... . . . 69
Clinical Manifestations ...... . ..... . . . ..... . .. 55 ANCA-Associated Vasculitis ......... . ...... . .. 69
Diagnosis ............. . ............ . ....... 56 Immune Complex-Mediated Vasculitis .. . ... . . . 71
Management ........ . ........... . .. . ....... 57
Calcium Pyrophosphate Deposition ..... ..... ..... . 59 Other Rheumatologic Diseases
Epidemiology and Pathophysiology . ...... .. ... 59 Beh9et Syndrome ... ... . .... . ... .. .............. 73
Clinical Manifestations and Diagnosis ... .. ..... 59 Relapsing Polychondritis .......... . .............. 73
Management .. .• .......... . .... , .... . ...... 59 Adult-Onset Still Disease , . ... . . .... . .. .. , ..• , , . •. 74
Basic Calclum Phosphate Deposition ... . .... .. ..... 60 Autoinflammatory Diseases ... ... ....... . .. . .... . . 74
Sarcoidosis ...... . . . ..... .. . . ..... ... .. . ........ 75
lnfertious Arthritis Genetic Diseases of Connective Tissue .. . ....... • ... 76
Diagnosis ...... . ......... . .... .. .... . ... • . .... . 60 IgG4-Related Disease .... . ........ .. ........ . .... 76
Clinical Manifestations . . ..... . . .. ... . . . .. .. .. 60
Laboratory and Imaging Studies ... . ...... . . .. . 60
Bibliography ..... . ........ . ................... 78
causes........... - . . .................. - .... ' ... 61
Infection with Gram-Positive Organisms . ....... 61 Self·AssessmentTest. ..... . ..... . ............... 83
Infection with Gram-Negative Organisms ....... 61
Lyme Arthritis ....................... . ...... 62
Index .... ...... .. ... . .. ..... . .. . .. ... .. ...... 155

xl
Rheumatology High Value
Care Recommendations

The American College of Physicians, in collaboration with • Plain radiography is usually the first-line imaging test
multiple other organizations, is engaged in a worldwide performed in the evaluation of rheumatologic diseases.
initiative to promote the practice of High Value Care • Ultrasonography is an inexpensive means to assess
(HVC). The goals of the HVC initiative are to improve soft-tissue abnormalities, assess disease activity, and
health care outcomes by providing care of proven benefit assist with tendon or joint injections.
and reducing costs by avoiding unnecessary and even • Acetaminophen is not beneficial for osteoarthritis and
harmful interventions. The initiative comprises several lower back pain, even at high doses.
programs that integrate the important concept of health • Topical NSA!Ds are beneficial for patients at high risk for
care value (balancing clinica I benefit with costs and toxicity from oral NSAlDs (see Hem 20).
harms) for a given intervention into a broad range of edu- • The most useful laboratory studies to aid in the diagnosis
cational materials to address the needs of trainees, prac- of rheumatoid arthritis are rheumatoid facwr and anti-
ticing physicians, and patients. cyclic citrullinated peptide antibodies.
• Weight loss can lower the risk for developing osteoanhrltls
HVC content has been integrated into MKSAP 18 in several
(see Item 78).
important ways. MKSAP 18 includes HVC-identified key
• Laboratory testing is usually not necessary to diagnose
points in the text, HVC-focused multiple choice questions,
osteoarthritis.
and, for subscribers to MKSAP Digital, an 1-rvc custom quiz.
• A muscle strengthening program is part of the manage-
From the text and questions, we have generated the following
ment plan for knee osteoarthritis (see ftem 73).
list of HVC recommendations that meet the definition below
• Arthroscop1c surgery iS generally not indicated for knee
of high value care and bring us closer to our goal of improving
osteoarthritis unless there is a mechanical disorder.
patient outcomes while conserving finite resources.
• Nonpharmacologic therapy (education, exercise. psycho-
High Value Care Recommendation: A recommendation to social support) remains a cornerstone of treatment for
choose diagnostic and management strategies for patients fibromyalgia.
in specific clinical situations that balance clinical benefit • Plain radiography of the sacroiliac joints is the first imag-
with cost and harms with the goal ofimproving patient ing test in patients with suspected ankylosing spondylitis
outcomes. (see Item 94) .
• HLA-B27 cannot independently confirm or exclude a
Below are the High Value Care Recommendations tor the
diagnosis of spondylitis.
Rheumatology section of MKSAP 18.
• Antibiotics are not usually effective in treating reactive
• An accurate history and a thorough musculoskeletal arthrttis.
examination can avoid unnecessary testing. • Lofgren syndrome (a triad of bilateral hilar lymphade-
• Do not obtain antinuclear antibody tesUng for patients nopathy, erythema nodosum, and migratory polyarthral-
with nonspecific symptoms and a normal clinical exam- gia) is highly specific for sarcoidosis, obviating the need
ination. for tissue biopsy.
• Do not obtain antinuclear antibody subserology testing • Plain radiography is the initial radiographic study of
without a strong clinical suspicion of an underlying con- choice for osteonecrosis, and MRI is indicated when plain
nective tissue disease. radiographs are normal (see Item 81).

xiii
Rheumatology

Approach to the Patient KEY POINTS


• Inflammatory symptoms include pain. erythema,
with Rheumatologic swelling. and warmth; Wtth the exception of pain.
Disease noninflammatory conditions usually lack these
features.
Inflammatory Versus • An accurate history and a thorough musculoskeletal HVC
Noninflammatory Pain physical examination are essential to diagnose and
The differentiation between inflammatory and noninflamma- differentiate inflammatory and noninflammatory
tory signs and symptoms is central to the evaluation of symptoms and can help to avoid unnecessary testing.
patients with musculoskeletal pain. Autoimmune conditions
typically present with inflammation, whereas mechanical or
degenerative disorders are characteristically noninflamma- Arthritis CJ
tory. The cardJnal signs of inflanunation are pain, ezythema, Monoarthritis
swelling, and warmth; with the exception of pain, noninflam- Monoarthritis involves a single joint and is classified as acute
matory conditions usually lack these features. Importantly, or chronic. Acute monoarthritis can be noninflammatory (for
patients may simultaneously experience more than one type example, trauma, hemarthrosis, or internal derangement) or
of pain. Table 1 compares the features of inflammatory and inflammatory (for example, crystal induced or infectious).
noninflammatory pain. Evaluation for Infectious arthritis should be guided by the
clinical presentation and examination, but suspicion should
always be high. Joint aspiration Is usually the most effective
The Musculoskeletal Examination means of diagnosing the underlying cause.
An accurate history and a thorough musculoskeletal physical Chronic inflammatory monoarthritis (>26 weeks) can be
examination are essential to diagnose and differentiate inflam- caused by chronic infection (for example, mycobacterial, fun-
matory and noninflammatory symptoms and can help to gal. or Borrelia burgdor{erO or by autoimmune rheumatologic
avoid unnecessary testing. Musculoskeletal pain may be artic- disease. Synovial fluid analysis Will confirm inflammation but
ular, periarticular, or referred. Pain with passive range of may be Inadequate for diagnosis; assessment for systemic dis-
motion suggests an articular condition, whereas pain With ease (serologies and other laboratory studies) and, In some
active range of motion suggests a periarticular condition. cases, synovial biopsy may be required.
See MKSAP 18 General Internal Medicine for more Chronic noninflammatory monoarthrltis is usually
information. caused by osteoarthritis.

TABLE 1. Features of Inflammatory Versus Noninflammatory Pain


Feature Inflammatory Pain Noninflammatory Pain
Physical examinatron frndings Erythema; warmth; soft-tissue swelhng; jornt Minimal or no warmth; no soft-tJssue
effusions; reduced ROM is frequent swelling; bony enlargement and joint
effusions may occur in osteoarthritis;
reduced ROM may occur
Morning stiffness >60 min; worsens with immobility <30min
Constitutronal symptoms Fever; fatigue; malarse Generally absent
Synovial fluid Leukocyte count >2000/J.tl (2.0 x 109fL), Leukocyte count between 200-2000/j.tL
predominantly neutrophils in acute inflammation (0.2-2.0 x 109/L), predominantly monocytes
and monocytes in chronic inflammation
Other laboratory findings Elevated inflammatory markers (ESR. CRP); Inflammatory markers usually normal or
anemia of chronic disease minrmally elevated
C~ P - C·re~ctive protain; ES8 • erythrocyte $edimentnion rato; ROM • rang~ of motion.

1
Approach to t he Patient with Rheumat ologic D isease

C] Oligoarthritis and spiking in some conditions (for example. adult-onset Still


CONT. Oligoarthritis involves two to four joints. typically in an asym disease and autointlammatory diseases). Morning stiffness last-
metric pattern. Acute intlammatory oUgoarthritis may be ing more than 60 minutes is most commonly described in rheu-
caused by gonorrhea or rheumatic fever. Chronic inflamma matoid arthritis but also occurs In other forms ofinflammatory
tory oligoarthritis can be caused by autoimmune ronditions arthritis. Significant and even disabling fatigue is a prominent
such as spondyloarthritis. teature of fibromyalgia and chronic fatigue syndrome.
Chronic noninflammatory oligoarthritls is usually caused
by osteoarthritis. Skin Involvement
Skin involvement is common in rheumatologic conditions and
Polyarthritis may go unnoticed by the patient (Table 2). Skin involvement
Polyarthritis involves five or more joints. In many cases, it may also occur as an adverse effect of medications used to
involves the small joints of the hands and/or feet. Acute polyar- treat rheumatologic conditions, including skin infections sec-
thritis (<6 weeks in duralion) can be caused by viral infections ondary to immunosuppressive therapy.
(for example, parvovirus Bl9, 1-lfY, hepatitis B virus, or rubella)
or may be an early manifestation of a chronic inflammatory Eye Involvement
polyarthritis (>6 weeks in duration) such as rheumatoid arthri- Eye involvement in different rheumatologic diseases usually
tis, systemic lupus erythematosus, or psoriatic arthritis. follows fairly distinct patterns, and the location and type of
involvement can help narrow the differential diagnosis
Soft-Tissue Abnormalities (Table 3) . If not quickly recognized and treated, certain
Common nonarticular sources of musculoskeletal symptoms forms of eye involvement can have devastating conse-
are the soft tissues (tendons, ligaments, and bursae) around or quences, including permanent loss of vision.
away from the joints. Isolated tendon and/or ligament involve-
ment Is usually suggestive of noninflammatory disorders such Internal Organ Involvement
as mechanical injury/irritation, overuse, or degeneration (for Rheumatologic diseases frequently affect internal organs, with
example, rotator cuff disorders or tennis elbow). Disorders of different diseases tending to follow characteristic patterns
widespread musculoskeletal pain (such as tibromyalgia} also (1'able 4}. Cl
have symptoms localizing to these structures. KEY POINT
The enthesls Is a complex structure at the site of the inser-
• Rheumatologic disease can cause constitutional symp-
tion of a tendon or ligament onto the bone. Inflammation of the
toms and extra-articular manifestations affecting the
enthesis (enthesitis) is highJy suggestive of spondyloarthritis.
skin. eyes, and internal organs.
When enthesitis is particularly severe. the inflammation may
extend along the associated tendon and local ligaments. result-
ing in dactylitis (wsausage digits"). Laboratory Studies
See MKSAP L8 General (nternal Medicine for more Laboratory studies are useful for diagnosing rheumatologic
information. CJ diseases, identifYing the extent/severity ofinvolvement, evalu-
KEY POINTS ating disease activity, and monitoring therapeutic responses.
HVC • Joint aspiration is usuaUy the most effective means of Because of limited speCificity, these tests should always be
diagnosing the underlying cause of acute monoarthritis. interpreted in the context of the clinic~ history and physical
examination and should be used with great caution, ifat all, in
• Chronic Inflammatory arthritis can be caused by auto-
the setting of low pretest probability.
immune diseases, whereas chronic noninflammatory
arthritis is usually caused by osteoarthritis.
Tests That Measure Inflammation
• Isolated tendon and/or ligament involvement is usually Erythrocyte Sedimentation Rate
suggestive of noninflammatory disorders such as The erythrocyte sedimentation rate (ESR) measures the fall of
mechanical injury/irritation, overuse, or degeneration. erythrocytes (mm /h) through anticoagulated plasma.
• Inflammation of the enthesis (enthesitis} is highly Erythrocytes tend to be negatively charged on their surfaces,
suggestive of spondyloarthritis. leading to repulsion and a prolonged ESR. Fibrinogen and
other acute phase reactants neutralize the erythrocytes' sur-
CJ Extra-Articular Manifestations face charges, promoting their ability to settle at a faster rate.
Elevated fibrinogen levels and high ESRs are seen in many
of Rheumatologic Disease rheumatolog!c diseases, as well as in nonrheumatologic
Constitutional Symptoms Inflammatory conditions such as chronic infections and
Fever. morning stiffness, and fatigue occur In numerous rheu- malignancies. The normal ESR increases with age and is usu-
matologic conditions. Fever is usually low grade but may be high ally higher in women. A well-accepted rule of thumb is to

2
Approach to the Patient w ith Rheumatologic Disease

Systemte lupus erythematosus Butterfly (malar) rash; photosensitive rash; discoid lupus erythematosus; subacute cutaneous lupus
erythematosus; oral ulcerations (on the tongue/hard palate; usually painless); alopecia; lupus
panniculitis (painful, indurated subcutaneous swelling with overlying erythema of the skin)
Dermatomyositis Gottron papules (erythematous p laques on extensor surfaces of MCP and PIP jornts);
photodistributed poikiloderma, including shawl sign (over the back and shoulders) and V sign
(over the posterior neck/back or neck/upper chest); heliotrope rash (violaceous rash on the upper
eyelids); mechanic's hands (hyp erkeratotic, fissured skin on the palmar and lateral aspects of
fingers); nail fold capillary abnormalities; holster sign (poikilodermic rash along lateral thigh); can
occur in the absence of myositis (amyopathic dermatomyositis)
Systemic sclerosis Skin thickening and hardening; nailfold capillary changes
Vasculitis Palpable purpura; cutaneous nodules; ulcers; necrosis
Beh<;et syndrome Painful oral and genital ulcers, erythema nodosum; acne/folliculitis; pathergy(skrn inflammation/
ulceration from minor trauma)
Sarcoidosis Erythema nodosum; infiltrated plaques; maculopapular and papular lesions; nodules; soft
infiltrates of the nose (lupus pernio); on blanching with a glass slide, sarcoid skin lesions reveal
"apple jelly'' discoloration
Psoriatic arthritis Plaque psoriasis typically on extensor surfaces, umbilicus, gluteal fold, scalp, and behmd ears;
pustular psorias1s on palms and soles; nail prtting; onychodystrophy
Reactive arthritis Keratoderma blennorrhagicum (psoriasiform rash on soles, toes, palms); circinate balanitis
(psoriasiform rash on penis)
Adult-onset Still disease Evanescent, salmon-colored rash on trunk and proximal extremities
Rheumatic fever (secondary to Erythema marginatum (annular pink to red non pruritic rash with central clearing)
streptococcal infection)
Lyme disease Erythema chronicum migrans (slowly expanding, often annual lesion with centra l clearing)

MCP- metoca<pophalangeal; • proximallnte•rph•alol'9""'·

TABLE 3. Ocular Manifestations of Systemic Inflammatory Disease


Systemic Inflammatory Disease Ocular Manifestations

Ankylosing spondylitis, reactive arthritis, and 1nflammatory bowel Uveitis (inflammation of the anterior and/or posterior chamber
disease (anterior chamber); sarcoidosis and Behc;et syndrome and/or retina)
(anterior and/or postenor chamber}; granulomatosis with
polyangiitis (posterior chamber)
Rheumatoid arthritis; spondyloarthritis; systemic vasculitis Episderitis
Rheumatoid arthnt1s; relapsing polychondritis; inflammatory Scleritis
bowel disease; systemic vasculitis
Systemic vasculitis; anti phospholipid syndrome Retinal ischem ia
Sjogren syndrome Dryness of the eyes (keratoconjunctivitis sicca)
Giant cell arteritis Anterior/posterior ischemic optic neuropathy; central retinal artery
occlusion; loss of vision
SarcoidosiS; granulomatosis with polyangiit1s Exophthalmos/retrobulbar inflammatory 1nfiltrate
Reactive arthritis Conjunctivitis

adjust the upper limit of normal as age in years divided by 2 for or heart failure and in conditions promoting rouleaux forma-
men and (age in years+ 10)/2 for women. tion (for example, polycythemia vera). Sickle cell disease and
In addition to inflammatory conditions, elevated ESRs microcytosis (including spherocytosis) may also lower ESR.
can be seen in pregnancy. diabetes mellitus, and end-stage A markedly elevated ESR (>100 mm/h) should alert phy-
kidney disease. Due to rheostatic properties, anemia and mac- sicians to conditions such as giant cell arteritis, multiple mye-
rocytosis are also associated with an incr eased ESR. An exces- loma, metastatic cancer, or other overwhelming inflanunatory
sively low ESR can occur in low fibrinogen states such as liver states (infection or autoimmune disease).

3
Approach to the Patient with Rheumatologie Disease

Kawasaki disease Coronary artery vasculitis


Systemic sclerosis Arrhythmia; myocardial fibrosis
SLE Pericarditis: valvular disease; myocarditis
RA Pericarditis; myocarditis
Rheumaticfever; anti phospholipid syndrome Valvular disease
GCA Aortic aneurysm/dissection; aortitis; large-vessel obstruction

RA Serositis; ILD, rheumatoid nodules


SLE; CTDs; Henoch·Schonlein purpura Serositis; pneumonitis; pulmonary hemorrhage from vasculitis
AAV Pulmonary hemorrhage; cavitary nodules
Diffuse cutaneous systemic sclerosis ILD; pulmonilry hypertension
Limited cutaneous systemic sclerosis Pulmonary hypertension
Antiphospholipid syndrome Pulmonary embolism
Sarcoidosis Hilar lymphadenopathy; ILD
Goodpasture syndrome Pulmonary hemorrhage

SLE; CTDs; AAV; systemic vasculitis (except PAN) Glomerulonephritis


PAN Renal artery vasculitis; pseudoaneurysms
Antiphospholipid syndrome Renal infarct; renal vein thrombosis
Sjogren syndrome Acute interstitial nephritis/renal tubular acidosis
Goodpasture syndrome Glomerulonephritis

PAN Mesenteric vasculitiS


Henoch-Schonlein purpura Intestinal vasculitis and ulcerations
Diffuse and limited cutaneous systemic sclerosis Esophageal and small bowel hypomotility
Behc;et syndrome M ucosal ulcerations
Familial Mediterranean fever Peritonitis

SLE; CTDs; AAV; systemic vasculitis Mononeuritis multiplex; peripheral neuropathy


PACNS CNS vasculitis

AAV • ANCA-ossociated voscul~ls; CNS - central nervous system; CTO • coMe<:tive tissue dis.,ue; GCA ~ giant cella~teritis; ILD • inl81'$tilial lung disease; I'ACNS • primary
angotis of the C4ntl'81 nei'IIOus system; PAN • polya~teritls nodosa; RA • rl\eum~toid a~thrids; SlE • systemic lupus erythematosus.
- -- - - - - -

C-ReacUve Protein inflammation in spondyloarthrltis. In contrast, In some


C-reactlve protein (CRP) Is produced by the Uver mainly in patients with systemic lupus erythematosus (SLE), the CRP
response to interleukin-6 generated by leukocytes during the may remain normal despite active disease. CRP can be elevated
inflammatory state. CRP levels and ESR usually follow a com- in obesity, and a low CRP can occur with the use of certain
mon pattern, but CRP is often more rapidly responsive to antibiotics and lnterleukin-6 blockers.
changes In Inflammation, In rheumatologic conditions, CRP is
typically elevated 2 to 10 times the normal level; a higher level Complement
(especially >10 mg/dL [100 mg/L)) should prompt considera- The complement system is an essential part of the immune
tion of an altemati.ve diagnosis such as Infection. CRP is response, promoting vasodilation, attracting leukocytes, and assist-
thought to be a better marker than ESR in measuring ing In the lysis ofopsonized bacteria during hwnoral immunity.

4
Approach to t he Patient with Rheumat ologic Disease

Complement components are acute phase reactants KEY POINTS


and rise in many inflammatory states. However, in
• l:.rythrocyte sedimentation rate and C reactiVe protein
response to immune complex formation d iseases (SLE and
(CRPl levels usually follow a conunon pattern, but CRP is
cryoglobulinemic and urticarial vasculitis) and certain
often more rapidly responsive to changes in inflammation.
other states, complement cascades are activated, and
complement levels fall due to excessive consumption. • An isolated positive antinuclear antibody with nonspe- HVC
Paradoxically, genetic deficiency of early complement ciflc symptoms and normal clinical examination does
components may increase the ris k for lupus-like autoim- not establish the diagnosis of a connective tissue disease.
mune diseases. • Antinuclear antibody (ANA) subserology testing should HVC
C3 and C4 are the commonly measured complement not be routinely performed, even in the setting of a pos-
components. The CHso assay should not be performed itive ANA, without strong clinical suspicion of an
routinely due to cost and limited utility. underlying connective tissue disease.

Autoantibody Tests
Rheumatologic diseases are commonly associated with Imaging Studies
autoantibodies, but their presence does not equate with the
diagnosis of :111 underlying condition because they Jack
Radiography C]
specificity and may be seen in other conditions and healthy Plain radiography is an essential modality In the evaluation of
persons. In commercial laboratories, autoantibody testing many rheumatologic diseases and can assess and differentiate
inflammatory arthritis. osteoarthritis, and crystal arthropa-
has been automated using enzyme-linked immunosorbent
assay in a sequential algorithm. which may simplify physi- thies (Table 6). Plain radiography has limitations because it
gives a !\No-dimensional picture of three-dimensional struc-
cian assessment but tends to have reduced sensitivity and
specificity. tures, is limited in its ability to visua.ll.re soft tissues, and may
Rheumatoid factor is an lgM antibody directed against the not detect early or small erosive changes. Despite these limita-
Fe portion of lgG immunoglobulin. Although cbamcteristi- tions, plain radiography is usually the first imaging test ordered
caUy associated with rheumatoid arthritis (RA), rheumatoid In the evaluation ofrheumatologlc diseases because it Is read-
factor Is present .in fewer than 70% of pat!e11ts wtth RA and Is Ily available. is inexpensive, exposes patients to only a low level
common in several other diseases. Anti- cyclic citruUinated of ionizing radiation. and is useful In monitoring arthritis
progression.
peptide antibodies are more specific for RA but less sensitiVe.
The presence of both autoantibodies together Increases the
likelihood ofRA. CT
Antinuclear antibodies (ANA) are directed against CT provides multiple vif\1\-'S and orientations from a single

nuclear antigens and are traditionally associated with SLE. study but Is more useful for bony abnormalities than for soft-
About one third of the healthy population has a low-titer tissue inflammation or fluid collections. CT is more sensitive
(1:40) ANA, and 3% to S% have a liter of 1:160 or more. ANA for detecting bone erosions than plain radiographs or MRI.
can also be seen in other autoimmune conditions, infection, However, CT is more expensive than plain mdiography and
and malignancy, and it may be drug induced. An isolated exposes the patient to more radiation.
positive ANA with nonspecific symptoms and normal clinical
examination does not establish the diagnosis of a connective MRI
tissue disease. MRl is the most sensitive routine radiologic technique for
A hlgher titer of ANA is more often associated with an detecting soft-tissue abnormalities, inflammation. and fluid
underlying rheumatologic disease, although not always SLE. collections but is less effective than CT in demonstrating bony
However, almost all patients with SLE (>95%) have a positive abnormallties or erosions. MRI is sensitive for detecting early
ANA. Al-IA titer does not correlate with disease activity and spine and sacroiliac joint inflammation and may be indicated
should not be used for activity assessment. for the evaluation ofsuspected spondyloarttuitis If plain radio-
ANA specificity or subserology testing (that is, testing for graphs are negative. MRI does not expose patients to radiation
an tibodies to specific nuclear components such as DNA or but is associated with high cost, limited availability, and pos-
centromeres) should be reserved for patients with a positive sible patient intolerance due to claustrophobia or body habi-
ANA and a clinical syndrome suggestive of an underlying con- tus. The American College of Rheumatology Choosing Wisely
nective tissue disease. ANA subserology testing should not be list recommends against routine MRI of the peripheral joints to
routtnely performed, even in the setting of a positive ANA. monitor RA due to inadequate data supporting its use. CJ
without a strong clinical suspicion of underlying connective
tissue disease. Ultrasonography
Table S provides details on these and other autoantibodies The use of ultrasonography to evaluate patients with rheuma-
and their associations with specific conditions. tologic diseases has expanded dramatically in the past 10 years.

5
Approach to the Patie nt with Rh e umatolog ic Disease

TABLE 5. Autoantibodies in Rheumatologic Disease


Autoantibody Rheumatologlc D lseas. Sensitivity/Specificity Comments
ANA SLE; also SSe, Sjogren. SLE: >95% sensitivity, poor speofic•ty; Does not correlate With disease activity
MCTD indirect IFA is the most appropriate
methodology
Anti-double-stranded SLE SLE: 50%-60% sensitivity, >95% Found in more severe disease,
DNA specificity; Crithidia IFA or Farr assays especially kidney disease; antibody
more specific than ELISA levels commonly follow disease activity
and are useful to monitor
Anti-Smith SLE SLE· 30% sensitivity. 99% specificity Most specific test for SLE; does not
correlate with d1sease activity
Anti·U1 -RN P MCTD; SLE High sensitivity for MCTD High titer seen in MCTD(>1 : 10,000);
does not correlate with disease activity
Anti-Ro/SSA; anti-La/ Sjogren; SLE; RA; SSe Sjcigren : 70% sensitiVity; SLE 20% Sicca symptoms; tn SLE, associated
SSB sensitiVity with photosensitive rash; offspnng of
mothers who are positive for anti-Ro/
SSA or anti-La/SSB are at tncreased nsk
for neonata II upus erythematosus {rash
and congenital heart block)
Antiribosomal P SLE 15% sensitivity Associated with CNS lup us and lupus
hepatitis
Anti-Scl-70 De SSe 10%-30% sensitivity Seen more often in patients wrth DcSSc
(antitopoisomerase-1) who have pulmonary fibrosis
Anticentromere LcSSc (CREST) 10%-30% sensitivity Patients with LcSSc with this antibody
are more hkely to develop pulmonary
arterial hypertension
c-ANCA GPA 90% sensitivity when disease is active; Correlation with disease activity is
(antiproteinase-3) h1gh specificity in classic presentations unclear
p-ANCA MPA;EGPA MPA: 80% sensitivity; EGPA: 60% Atypical p-ANCA (antimyeloperoxidase
(antimyeloperoxidase) sensitivity; Iess specific than c-ANCA negative) can be seen in inflammatory
bowel disease and with positive ANA
Antl-Jo-1 Polymyositis 20%-30% sensitivity Associated wtth a nt•synthetase
syndrome. including lung inflammation
1 Rheumatoid factor RA; Sjogren; RA: 70% sensitivity; limited specificity, RF is common in multiple other
cryoglobulinemia especially in patients without a classic diseases (e.g., hepatitis C. endocarditis,
disease presentation SLE); 30% with RA are RF negative but
may become positive later in RAcourse
Anti-cyclic RA RA: 70% sensitivity; 95% spec•ficity Can be posit1ve 1n RF-negative RA
citrullinated peptide patients; often present before RF
becomes positive; assoc1ated w1th
erosions; predicts disease progression
in undifferentiated arthritis
A ntihistone DILE 95% sensitivity; poor specificity Also seen in primary 5LE
Cryoglobulins Vasculitis; hepatitis C; Type II or Ill cryoglobulins seen in May be present in connective tissue
myeloma; SLE; RA cryoglobulinemic vasculitis diseases in the absence of vasculitis

ANA- antint.,~.<;leer antibodfes; CNS -centro I nervous sy;Stem; CREST - c;illcinosi$, Raynat;d pl'lenomenon, esophageal dy.smot.ility, sclerodactyty, and telangioete-t ia;
DcSSc- difru.e cutaneous systemic sclerosis; OILE • drug·lndueed lupus e rythematosus: EGPA- oo,;nopnillcgranulo"'at<»ls with polyangiitlo; EUSA- onzymot-flnked
irnrnynosorbentassay; GPA o;: granulomatosis with polyang11t1s; fFA • immunoAuorescence assay; lcSSc -limited cutaneous systemic sclerosis.; MCJO • mixed connect1ve tissue
disease; MPA • microscopoc polyangiilis; RA - rheumatood allhritis: RF • rheumatoid factor; RNP- robonucleoproteln; SlE • systemic lupus erythematosus: SSe • systRmlc
sclerosis..

Ultrasonography is relatively inexpensive, can scan across KEY POINTS


three-dimensional structures, and can provide real-time data • Plain radiography is usually the first imaging test HVC
in the clinic without exposure to ionizing radiation. It can ordered in the evaluation of rheumatologic diseases
assess soft- tissue abnonnalitles, including synovitis, tendoni- because it is readily available. is inexpensive, exposes
tis, bursitis, and effusions; assess disease activity using
patients to only a low level of ionizing radiation, and is
Doppler; and assist with tendon or joint injections. However, useful in monitoring arthritis progression.
it is operator dependent, and training/practice is needed to
(Continued)
achieve competence.

6
Approach to the Pat ie nt w ith Rh e um atologic Dise ase

Rheumatoid arthritis Bony erosions; periarticular osteopenia; subluxations; soft-tissue swelling; MCP, PIP. and wnst involvement
Osteoarthritis Asymmetric joint-space narrowing; osteophytes; subchondral sclerosis and cystic changes; degenerative
disk disease with collapse of disks; degenerative joint disease with facet joint osteophytes;
spondylolisthesis (anterior/posterior misalignment of the spine); kyphosis
D1ffuse idiopathic skeletal CalcificatiOn of the anterior long1tudinalligament; bridging horizontal syndesmophytes; usually seen in the
hyperostosis thoracic spine and more prominent on the right s1de of the spme
Ankylosing spondylitis Sacrolliitis; squaring of the vertebral bodies; bridging vertical syndesmophytes; shiny corners; ankylosis
does not skip vertebrae
Psonatic arthntas Destructive arthritiS with eros1ons and osteophytes; DIP involvement IS common; pencil-in-cup deformity
on hand radiograph; arthritiSmutilans, syndesmophytes
Gout Punched-out erosions with sclerotic borders and overhanging edges; periarticular soft-tissue swelling with
calcifications in tophaceous deposits
Calcium pyrophosphate Chondrocalcmos1s, most commonly of the knees. shoulders, wrists, pubic symphysis; osteoarthritis,
deposition including in locations atypical for primary osteoarthritis (MCPs, wrists, shoulders)

DIP • dinollmerpholongool; MCP • meticorpopholongeal: PIP • proximal interphalangeal.

KEY P 0 I NTS (continued} aspiration should be performed ro diagnose the underlying


cause. Aspirated synovial fluid should be sent for leukocyte
HVC • cr Is more sensitive for detecting bony abnormalities or count, Gram stain, and cultures, as weU as evaluation for crys-
erosions than plain radiographs or MRI, whereas MRI
tals under polarized light. See Table 7 for more information.
JS the most sensitive routine radiologic technique for
There is no absolute cutoff of synovial fluid leukocyte
detecting soft- tissue abnormalities, intlammation, and
counts for rullng out infectious arthritis: however. counts
fluid collections.
greater than 50,000/ J.!L (50 x 109 / L) wi th polymorphonuclear
HVC • Ultrasonography is an inexpensive means to assess soft- cell predominance have a high likelihood of infection. Counts
tissue abnormalities, assess disease activity. and assist less than 2000/J.lL (2.0 x 109 /L) are usually associated with
with tendon or joint injections, but it is operator noninflammatory etiologies. Notably, crystals can coexist with
dependent. infection, and their presence does not rule out infection if
suspicion is high. Cl

CJ Joint Aspiration
Joint aspiration and synovial fluid analysis are essential for Tissue Biopsy
discriminating between inflammatory and noninflammatory When appropriate, tissue biopsy of involved organs can be
effusions or for distinguishing between infectious arthritis helpful in diagnosing numerous rheumatologic conditions
and acute crystal arthropathies. In the evaluation of any such as vasculitis (lung, kidney, or temporaJ artery biopsy) and
monoarthritis or when infection is being considered, joint SLE (skin biopsy). Tissue biopsy may also help assess disease

TABLE 7. Synovial Fluid Analysis


Normal Noninflammatory Inflammatory Crystallndu~ed Infectious Hemonhagi~

Appearance Clear/yellow/ Clear/yellow/ Yellow/wh1te/ Yellow/white/ Yellow/white/ Red/opaque


transparent transparent translucent/ translucent! opaque
opaque opaque
Leukocyte <200/~tl 200·2000/).IL 2000·20,000/~tl 10,000-50,000/lll >50,000/jll
count (0.2 X 109/ L) (0.2-Z.Ox 10 9/L) (2.0·20x 109/l) (10-SOx 10 9/L) (50 X 109/l)
(may be higher) (may be higher) (may b e lower)
Other Negative Gram Negative Gram Negative Gram Negative Gram Positive Gram Negative Gram
studies stain; negative stain; negattve stain; negative stain; positive stain°; positive stain; negative
culture culture culture crystals• culture< culture

'Crystal description: Urate crystals ""' naedlo shap<td and b rtgl>t. Viewed •nd•r polantad lign~ they ore negotively birefringent; they app ear yellow when J>a!allel to the oxls of
the polarized nold and blue when pe rpendic•lor to the axis. C.lcium pyropho•phate crystab .,. rhomboid, pale, end weakly (not as vividly! positively bif<tfringonl; they appeor
bJue whf<l parallel tO the aXiS and yellow Wh&n perpendicular.

"Gram S1ain sensitivity fo r infection i$ approxtmately 30'l!o to 50%.


I •Nearly all culture.s are positive excep t for lnt".etion caused by Neisseria gonorrl>oeae. which may be positive in 50% or fewer ca.es.

1
Principles of Therapeutics

activity (for example, kidney biopsy in SLE). The benefits Patients anticipated to be taking ~.5-mg prednisone for
should be appropriately balanced with possible risks of the 2:3 months should be risk- assessed for glucocorticoid-induced
procedure. osteoporosis and appropriately managed. Those who are at
moderate or high risk for osteoporotic fractures and placed on
KEY POINTS
chronic glucocorticoid therapy should begin oral bisphospho-
• Synovial fluid leukocyte counts greater than 50,000/).I.L nates.
(SOx 109 / L) with polymorphonuclear cell predomi-
nance have a high likelihood of Infection; counts less NSAIDs
than 2000l f.!L (20 x 109 / L) are usually associated with NSAIDs prevent prostaglandin production by inhibiting the
noninflammatory etiologies.
two lsoforms of cyclooxygenase (COX), COX-1 and COX-2.
• Tissue biopsy of involved organs can be helpful in diag- COX-2 Is an Inducible enzyme typically expressed In Inflam-
nosing numerous rheumatologic conditions and ln matory milieus, whereas COX-1 is constitutively expressed and
assessing disease activity in some conditions. helps maintain organismal homeostasis. Nearly all available
COX inhibitors are nonselective (inhibit both lsoenzymes),
down-regulating prostaglandin production in inflammatory
states and interfering with housekeeping functions of pros-
Principles of Therapeutics tanoids (for example, renal blood flow and gut mucosal integ-
rity maintenance) (Table 8). Nonselective COX inhibitors also
Overview inhibit thromboxane A1 , inhibiting platelet function and pro-
This section reviews the Indications for use, mechanisms of moting bleeding.
action. major toxicities. and monitoring requirements of med- Although they alleviate symptoms, COX inhibitors are not
Ications used in rheumatotogic disease. Drug applications in disease modifying, with the apparent exception of ankylosing
specific disease states are elaborated upon In their respective spondylitis. Major concerns surrounding all COX inhibitors
sections. include Increased risk for gastrointestinal bleeding (particu-
larly in those already at risk) and adverse cardiovascular
events; therefore, they should be prescribed at the lowest dose
Anti-Inflammatory Agents for the shortest time possible. COX inhibitors should generally
Glucocorticoids be avoided in patients on concomitant anticoagulation.
Glucocorticoids are effective in many rheumatologic diseases, SelecUve COX-2 Inhibitors were developed to spare COX-1
including rheumatoid arthritis (RA), acute crystal arthropa- and reduce gastrointestinal riSk; although effective for this
thy, systemic vasculitis, polymyalgla rheumatica, systemic purpose, the most selective COX-2 inhibitors were found to
lupus erythematosus (SLE), inflammatory myopathies, and increase the risk of cardiovascular events and were removed
autointlammatory diSeases. Advantages include rapid onset of from the market.
action, ease of use, low cost, and universal availability; in NSAIDs vary with regard to kinetics, COX-1/2 selectivity,
many disease states, they are disease modlfytng and some- and other features, and carry somewhat different degrees of
times lifesaving. rtsk; having experience with several different NSAlDs iS poten-
Glucocorticoids have numerous adverse effects, which are tially beneficial in clinical practice.
more likely to occur with higher doses and longer treatment. Topical NSAIDs such as diclofenac are available by pre-
These include osteoporosis, immunosuppresSion, skin fragility, scription for arthritis and pose a lower risk for systemic side
glaucoma, cataracts, weight gain, diabetes mellitus, hyperten- effects than oral NSAIDs. They may be preferred for patients at
sion, psychomotor agitation, osteonecrosis, and hypothalamic- high risk for toxicity from oral NSAlDs and/or for those
pituitary-adrenal axis suppression. ~75 years of age. However, they are often expensive.

TABLE 8. Potential Toxicities of NSAID Use


Category Toxicity
Cardiovascular Myocardial infarction; exacerbation of heart failure
Hemostatic Platelet dysfunction
Gastrointestinal Dyspepsia; reflux; peptic ulcer disease; gastrointestinal bleeding
ObstetridGynecologic Bleeding; delayed labor; premature ductus arteriosus closure
Pulmonary Asthma exacerbation
Renal Hypertension; decreased glomerular filtration; increased salt and water retention; increased renin
production; uncommonly, allergic interstitial nephritis or acute tubular necrosis

8
Principles of Th er ape utics

Colchicine FDA approved for fibromyalgia. Common side effects (dizzi-


Colchicine inhibits microtubules and impairs neutrophil func- ness, disequilibrium, somnolence, weight gain, peripheral
tion. rt is most commonly employed tor gout and acute cal- edema, cognitive difficulties) may limit its utility, and discon-
cium pyrophosphate crystal arthritis (pseudogout). 1t is also tinuation is sometimes warranted. Gabapentfn also modestly
used to treat hypersensitivity vasculitis and familial improves fibromyalgia symptoms, with a similar side-effect
Mediterranean fever. profile.
Gastrointestinal side effects (particularly diarrhea) are KEY POINT
common. With overdose, severe (even fatal) myelosuppression
• The efficacy of acetaminophen for osteoarthritis and HVC
can occur. Dosing must be adjusted for kidney disease. When
lower back pain is increasingly questioned. wJrh recem
given chronically, colchicine can rarely cause neuromuscular
controlled trials and meta-analyses demonstrating no
toxicity, particularly if coadmlnlstered with statln drugs.
benellt from the drug. even at high doses.
Concomitant adminlstratlon of strong CYP3A4 inhibitors (for
example, clarithromycin) that reduce the hepatic catabolism
of colchicine should be avoided. Disease-Modifying
K£Y POINTS Antirheumatic Drugs
• Although effective inflammatory agents in many rheu- Nonbiologic Disease-Modifying
matologic diseases. glucocorticoids have numerous Antirheumatic Drugs
adverse side effects. Including osteoporosis, immuno- Table 9 summarizes the mechanisms of action, lndlcatlons,
suppression, skin fragility, glaucoma, cataracts, weight and common monitoring parameters of various nonblologic
gain. diabetes mellitus, hypenension. psychomotor disease-modifying antirheumatic drugs (DMARDs). See
agitation, osteonecrosis. and hypothalamic-pituitary- Medications and Pregnancy for information on these drugs in
adrenal axis suppression. pregnancy.
• Because of the Increased risk for gastrointestinal bleed-
ing and adverse cardiovascular events, cyclooxygenase ~ethotrexate
Methotrexate is a first-line medication for treating RA and
CJ
inhibitors should be prescribed ar rhe lowest dose for
the shortest time possible. other autoimmune diseases. Once-weekly dosing is generally
between 10 to 25 mg and can be given orally or subcutane-
ously. At doses above IS mg. parenteral administration is more
Analgesics and Pain Pathway reliable but much more expensive.
Modulators Potential side effects include headaches, fatigue, and nau-
sea (particularly around the time of weekly dosing) .
Acetaminophen Hepatotoxicity and cytopenias can occur (especially macro-
The efficacy of acetaminophen for osteoarthritis and lower cytic anemia), and dose adjustment is required With kidney
back pain is increasingly questioned, with recent controlled disease. Methotrexate should be avoided in patients with sig-
trials and meta-analyses demonstrating no benefit from the nificant hepatic or kidney disease. Folic acid supplements
drug, even at high doses (3000-4000 mg/d). minimize toxicity while preserving efficacy. Limiting alcohol
intake is recommended. I:J
Tramadol
Tramadol is a mixed opioid analgesic and weak serotonin- Hydroxychloroquine
norepinephrine reuptake inhibitor; potential for addiction is Hydroxychloroquine is an im.munomoduJator that is Widely
lower than for traditional opioids, which are generally avoided used in SLE, in which it decreases mortality and the likelihood
In rheumatolog!c treatment of developing nephritis. It is rarely sufficient as single-drug
therapy for RA but is useful as an adjunctive therapy.
Duloxetine
Duloxetine is a serotonin-norepinephrine reuptake inhibitor SulfasaJazine
that is FDA approved for the management of chronic muscu- Sulfasalazine is used to treat RA and nonaxial psoriatic arthri-
loskeletal pain and fibromyalgia. Duloxetine provides modest tis, but use has decreased because of the relative effectiveness
pain relief tor knee osteoarthritis, chronic lower back pain, of methotrexate and letlunomlde. It Is now most frequently
and fibromyalgla. Patients must be slowly weaned off the drug used as pan of combination DMARD therapy for RA and ln
when discontinuing to avoid withdrawal symptoms. women considering pregnancy. Serious side effects Include
blood dyscraslas, hepatitis, and hyperse.nsit!vity reactions.
Gabapentinoids Because of its benefit for Inflammatory bowel disease (lBD),
Gabapentlnolds (gabapentin and pregabaJin) inhibit volmge- it may constitute a useful strategy for patients with IBD-
gated calcium channels, reducing patn signaling. Pregabalin is associated arthritis.

9
Principles of Th erapeutics

TABLE 9. Nonbiologic Disease-Modtfying Antirheumatic Drugs


Agent Mechanism Ind ications Common Monitoring Parameters
Methotrexate Low dose: anti-inflammatoty agent via RA; psonasis; psoriatic Baseline· chest radiography, hepatitis
up·regulation of adenosine A~A signaling arthritis; IBD; SLE screening, CBC, LCTs, serum creatinine
(arthritis only); reactive
High dose: antimetabolite/folate Thereafter; CBC, LCTs, serum creatinine
arthritis; DM; PM;
antagonist used in neoplastic disease after the first month, then approximately
vasculitis
every 2-3 months•
Hydroxychloroquine Uncertain; appears to involve stabilization SLE;RA Baseline: CBC, LCTs, serum creatinine
of lysosomal vacuoles, leading to
inhibition of antigen processing and/or Retinal examinations at baseline and
annual examination after 5 years of
inhibition of Toll-like receptor activation
therapy to evaluate for retinopathy
Sulfasa laztne Unknown; the prod rug is broken down RA; SpA; lBO Baseline: CBC, LCTs, serum creati nine
into 5-amino salicylic acid (active
Thereafter: CBC, LCTs. serum creatinine
metabolite in the gastrointestinal tract)
and sulfapyridine (exerts systemic action) evety 3-6 months

Leflunomide lnhtbits the mitochondrial enzyme RA Baseline: hepatitis screening, CBC, LCTs,
dihydroorotate d ehydrogenase to block serum creatinine
pyrimidine synthesis (decreasing
Thereafter: CBC, LCTs, serum creatinine
lymphocyte production); anti proliferative
after 4 weeks, then every 3 months
Azathioprine A prod rug of 6-mercaptopurine; purine SLE; DM;PM; Baseline: CBC, LCTs, serum creatintne
analogue; inhibits DNA synthesis essential "asculitis; IBD
Thereafter. CBC, LCTs, serum creatinine
for proliferating T- and B·lymphocytes
evety 3 months•
Cyclophosphamide A!kylating agent; blocks DNA synthesis Severe and life- Close monitoring clinically and
I and causes cell death, especially of threatening disease in measuring CBC, chemistries, LCTs.
Tcells SLE, DM. PM, and urinalysis evety 4-8 weeks
vascu litis; may be used
when other agents fail
Mycophe nolate Active metabolite (mycophenofic acid) SLE (especially lupus Baseline: CBC. LCTs, serum creatinine
mofetil inhibits purine synthesis; preferentlally nephritis); vasculitis
Thereafter: CBC, LCTs. serum creat inine
inhibits T- and B-lymphocytes (maintenance
after 4 weeks and then evety 3 months•
therapy); DM; PM; SSe
Cyclosporine Inhibits calcineurin(a transcription SLE; psoriasis; RA Baseline: CBC, LCTs, serum creatinine
activating factor); preferentially targets
Thereafter: CBC, LCTs. serum creatinine
Tcelfs
evety 2-3 months•
Tofacitinib Janus kinase inhibitor RA Baseline: CBC, LCTs, creatinine,lipid panel
Thereafter CBC, LCTs, creatinine evety
8 weeks. lipids after 8 weeks and then
evety 6 months
Apremilast Inhibits phosphodiesterase 4 Psoriasis; psoriatic Baseline: weight
arthritis
Thereafter: weight. neuropsychiatric effects

CBC- complete blood count; OM • dermatomyositis; 180 • innammatory bowel disease; lCTs • liver chemlwy tes1s: PM • polymyositis; RA • rheumatoid arthritis:
SLE • sySiemic lupus erythematosus; SpA • spondyloarthritis; SSe • $)'$1emic scle<OSl$.
%>commended monitoring intetval is lor a stahl• dose but may be shorter alter initiation or in the case of abnormal results and must be individuali<ed to the patient"s risl: of
tO)ticity.

Lefiunomid.e AzathJoprtne
Leflunomide is FDA approved for rheumatoid and psoriatic Azathioprine is an immunosuppressant used in various inflamma-
arthritis. with comparable efficacy to methotrexate. Patients tory diseases. Concomitant use with xanthine oxidase inhibitors
must be monitored for hepatotoxicity and myelosuppression. (allopurinol, febuxostat) Is contraindicated. Azathioprine's pri-
Other common side effects include nausea, headaches, rash, mary toXicity is myelosuppression. Th!opurlne methyltransfernse
diarrhea, and transamln!tis. An uncommon side effect is enzyme testing allows for identification of patients with decreased
peripheral neuropathy, but it is usually self~ limited if the drug or absent enzyme actMty at hJgh myelosuppression risk.
is discontinued. The active metabolite ofletlunomide (teri.tlu-
Cyclophosphamide
nomide) has a half-life of nearly 3 weeks; therefore, when the
drug needs to be eliminated quickly, an 11-day cholestynunlne Cyclophosphamide is a powerful immunosuppressant with a Cl
washout is necessazy. rapid onset of action (days to weeks). It treats vasculitis,

10
Principles of Therapeutics

life threatening complications of SLE, and interstitial lung Biologic Disease-Modifying


Cl disease. Cyclophosphamide has largely been displaced by Antirheumatic Drugs
CONT newer and safer drugs for tirsl line treatment of ANCA- Biologic OMARDs are highly specific. parenterally adminis- CJ
associated vasculitis and lupus nephritis (riruxlmab and tered. protein-based agents wtth extracellular targets (spe-
mycophenolate mofetil. respectively) but Is still used in severe clflc prolntlammatory cytokines. cytoktne receptors. or
cases or when these agents fail. Serious potential side effects clusters of differentiation cell surface molecules on immune
include severe immunosuppression, leukopenia hemorrhagic cells; Figure 1). The end of the genertc name of a biologic
cystitis, and ov-arian failure. as well as long-terrn risk for blad- agent indicates what type or molecule it is: - mob indicates
der cancer, leukemia. and lymphoma. CJ monoclonal antibody: -kin indicates an interleukin-type
substance: -ra is for a receptor antagonist; and -cepe is for
Mycophenolate Mofetll receptor molecules.
Mycophenolate mofetil is currently the first-line agent for Table 10 and Table U summarize the structures. targets,
lupus nephritis and may be effective for systemic sclerosis and Indications, and common monltoting parameters of various
associated interstitial lung disease. Gastrointestinal side effects biologic DMARDs. See Medications and Pregnancy for infor-
are common, particularly diarrhea. Myelosuppression may mation on these drugs in pregnancy. Biologic DMARDs
occur. increase the risk for infection to variable degrees. Targeted
screening is therefore necessary before initiation (see
Calcineurin Inhibitors Vaccination and Screening in Immunosuppression).
Calcineurtn inhibitors include cyclosportne and tacrolimus. The cost of biologic agents is significant and may be a
Although cyclosporine iS now rarely used in rheumatology, barrier to access.
one of its most common side effects is hyperuricemia, and
cyclosporine-lnduced gout Is an lmportant consideration in Tumor Necrosis Factor alnhibitors
patients taking the drug who present With acute monoarthri- Tumor necrosis factor (TNF)-a inhibitors are large protein·
tls. There Is currently renewed Interest ln tacrollmus as a pos- based molecules that require parenteral administration (see
sible altemattve therapy for lupus nephritis. Table 10). They are widely used for treating RA. psoriasis,
psortauc arthritis. and ankyloslng spondylitis. and are also
Tofacitinlb approved for several nonrheumatolog!c diseases.
Tofacltlnib is an oral agent that Inhibits Ja nus kinase (JAK) TNF a inhibitors are generally well tolerated, with
signaling. Tofacitinib is FDA approved for RA and has efficacy Increased risk for infection the primary safety concem. They
equal to biologic DMARDs. Risks include hyperlipidemia, pose a particularly high risk for reactivation of tuberculosis,
hepatotoxicity, and leukopenia so all patients being considered for TN F-a inhibitor therapy
need to be screened and receive appropriate prophylaxis.
Apremilast TNF a inhibitors do not appear to increase the risk of new
Apremilast is modestly effective for psoriasis and psoriatic cancers, aside from nonmelanoma and possibly melanoma
arthritis. It does not cause immunosuppression or myelosup- skin cancer; the risk for malignant recurrence is unclear.
pression. However, apremilast is Jess efficacious than biologic These agents may also exacerbate heart failure and rarely
DMA RDs and has a slow onset of action, and Its effect on pro- provoke a demyelinating condition. Over time, TN F-a inhib
gression of erosive damage is unknown. Adverse events !tors may often lose efficacy owing to formation of anti-drug
Include gastrointestinal side effects (mainly ru1usea and diar- antibodies.
rhea) and weight loss. It should be used with caution in
patients with a history or depression. Other Biologic Disease- Modifying
Antirheumatic Drugs
KEY POINTS
Multiple biologic DMARDs with addirional extracellular
• Methotrexate is a first-line medication for treating and cell-surface targets have been approved by the FDA in
rheumatoid arthritis and other autoimmune diseases. the past decade. Most of these agents are started after one or
• Hydroxychloroquine is widely used ln systemic lupus two TNF-a inhibitors have failed. See Table 11 for more
erythematosus, in which it decreases mortality and the Information. Cl
likelihood of developing nephritis.
• Cyclophosphamide has been largely displaced by newer Btoslmilars
drugs for first-line treatment of ANCA-associated vas- BlosimUar agents are •copycat" versions of brand-name bio-
culitis and lupus nephrir1s but Is still used in severe logic medications. The drugs are not exact replicas (hence the
cases or when other agents fall. term "bloslmllar· rather than "generic"); therefore, they
must go through phase tn testing to garner regulatory
• The oral agent tofacltinib is FDA approved for rheuma-
approval In 2016, the FDA approved three biosimilar antt-
toid arthritis and has efficacy equal to biologic agents.
TNFagents.

11
Princip les of Therapeutics

Anlnra

Neutrophil
"11 LTB.. PGEu
/' o2·· IL-8,
l
I
ll l -
- . ~

:-. Proteases
-
~hondrocyte_ ..,-rNF-a~( ~
Cartrl~ , MMPs \
IL·l
;~ ~
) / ~ll-6
Tocllizumab
T 7 Tcell
~

_,...-- - - --.....~'·'Synovial Tofacitinib


~"'fibroblast
8one MMPs @ ) ~ TNF-a
Osteoclast lnflixiab
RF, anti-<:CP
Etanercept antibodies
( Adalimumab
Golimumab
\ Certilizumab

FIGURE 1 . The Inflammatory cascade in rheumatoid arthritis (RA). Oendriti<eells, macrophages, and Bcells present inciting
antigens toT cells. Macrophages secrete multiple cytokines, indudinglls and TN F-a, which are also secreted by activated synovial
fibroblasts. The activated fibroblasts seuete matrix metalloproteinases (MMPs) and other enzymes that contribute to the
degradation of articular cartilage and activate neutroph ils, which mediate joint damage through proteases and other enzymes.
Activated osteoclasis additionally secrete MMPs that contribute to marginal erosions of bone Its, TNF·cx, Tcells, and Bcells may all
be targeted for inhibition by the various disease-modifying antirheumatic drugs useful in RA. CCP = cyclic citrullinated peptide;
IL = interleukin; LTB,; leukotriene 84; MMP- matrix metalloproteinase; 02 ~oxygen; PG~ ~prostag landin~; RF • rheumatoid
factor; TNF; tumor necrosis factor.

TABLE 10. Tumor Necrosis Factor ex Inhibitors•


Agent Agent Structure Indications

Adalimumab Humanized monoclonal antibody RA; psoriatic arthritis; ankylosing spondylitis; IBD
Etanercept Fusion protein made of two p75 TN F-a receptors RA; psoriatic arthritis; ankylosing spondylitis
linked to lgG Fe segment
Certolizumab pegol Fab' segment of humani~ed monoclonal antibody RA; psoriatic arthritis; ankylosing spondylitis
attached to polyethylene glycol strands
Golimumab Humanized monoclonal antibody RA; psoriatic arthritis; ankylosing spondylitis
lnfliximab Chimeric (mouse-human) monoclonal antibody RA; psoriatic arthritis; ankylosing spondylit1s; IBD

lBO • inflammatory bowel disease: RA- rheumatoid artllritJs; TNF- wmor necrosis factor.
'Common monitoring parameters for TNF-Cl irlhibitors indude wberc.utosi$, fungal. and other infections as well as complete blood count• .serum creatinjn&, and liver chemistry
tasts at b<tsetif'l& and t~eteafter &v&ry 3 to 6 months.

K£V POINTS Urate-Lowering Therapy


• All biologic agents Increase the risk for infection; there- Allopurinol
fore, targeted screening is necessary before initiation. Allopurinol is the most commonly used urate-lowering agent.
• Tumor necrosis factor a inhibitors are widely used for lt competitively inhibits the enzyme xanthine oxidase, block-
treating rheumatoid arthritis, psoriasis, psoriatic arthri- ing the conversion of hypoxan thine (a breakdown product of
tis, and ankylosing spondylitis; they are also approved purines) to uric acid. AllopurlnoJ Is metabolized to oxypurinol,
for treating some nonrheumatologic diseases. which also inhibits xanthine oxidase. Allopurinol Is FDA
HVC • Non-tumor necrosis factor (TNF)-o: biologic agents are usu- approved for doses up to 800 .mgtd. According to the American
aUy started after one or two TNF-a inhibitors have failed. College of Rheumatology, allopurinol should be initiated at
100 mg/d and titrated in 100-mg increments as needed, and

12
Principles of Therapeutics

TABLE 11. Other Biologic Disease· Modifying Antirheumatic Drugs•


Agent Agent Structure Target Indications Comments
Abatacept Soluble CTlA4 receptor/lgG Fe CD80/CD86; RA Preferred for those with history of
segment chimera blocks T-cell severe Infection; relatively
costi mu lation contraindicated in COPD
Rituximab Chimeric (mouse-human) CD20•Bcells RA; ANCA-associated Given as an intravenous infusion over
monoclonal antibody vasculitis: occasionally several hours; has a higher risk of infusion
for SL€ (off-label); reactions than other biologic DMARDs;
hyper-lgG4 syndrome can cause hypogammaglobulinemia
Tocilizumab Humanized monoclonal IL-6 receptor RA; JIA, Castleman Can cause transaminitis, hyperlipidemia,
antibody disease; GCA leukopenia, thrombocytopenia; avoid in
those with h1story of diverticulitis because
of an attendant risk of bowel perforation
Belimumab Human monoclonal antibody BlyS/BAFF SLE Phase Ill trials showed a small but
statistically significant improvement
versus standard therapy alone, with a
glucocorticoid -sparing effect
Ustekinumab Human monoclonal antibody IL-12/IL-23 Psonasis, psoriatic Injectable; less robust effect than other
arthritis biologic DMARDs
Secukinumab Human monoclonal antibody IL-17a Psoriatic arthritis; Phase Ill trials suggest equal efficacy to
ankyloslng spondylitis TN F-a inhibitors; can cause flares of IBD
Anakinra Recombinant receptor ll-11} receptor RA; CAPSb; AOSD (off- Rarely used in RA because efficacy is
antagonist label); acute gouty inferior to other biologic OMARDs;
arthritis (off-label) reversible neutropenia can develop
Canakinumab Human monoclonal antibody ll-t~ CAPSb Newer, more expensive IL-t inhibitor
Rilonacept Dualll-113 receptors chimerically IL-t CAPSb; refractory Newer, more expensive IL-1 inhibitor
attached to lgG Fe segment gout
AOSO • adult-onset Still d isease; BAFF • B-cell-octiYoting factor; BlyS- a-lymphocyte stimulator; CAPS • cryopyrfr>oasooeiated periodic syndtomes; DMARD • dls.,ase·
modifying antlrheuma~c drug; GCA • giant cell ar1eritls; lBO • lnOammatory bowel disease; ll • lnterlaukin; JIA • juvenile Idiopathic atthritis; RA • rhe<~matold anhritis;
SlE • systemic lupus erythematosus; TNF -rumor ne<:rosis factor.

•Prior to initiating any biologic. tuberculosis screening must be performed Complete blood counts should be performed every 3 i'o 6 months for all biologics, and aspartate
aminottensferawalanine aminotransferase and a lipid panel should be checked every 2 to 3 months for tocilizumab.

bThe cryopyrin~anociated poriodic syndromes (CAPS) lndude famili3f cold autoinflimmatory 'yndrome. Mudd&-WaUs syndcomo, ai'Kf ne onatal.on.$et mlllt.lsyttem ;nflammatory
disease (chronic infantile neurologic. cutaneous, Jrticular syndrome}.

for those with stage 4 or s chronic kidney disease, aJlopurtnol Uricosuric Agents
should be Initiated at so mg/d and titrated in 50-mg Incre- Probenecid is an organic acid transport inhibitor that decreases
ments as needed. renal reuptake of uric acid. Probenecid is uncommonly used
The biggest rtsl< the drug poses is DRESS (drug reaction with because of limited efficacy, as well as inconvenience and limi-
eosinophilia and systemic symptoms) syndrome, a rare reaction tations on use.
that usually occurs in the presence of chronic kidney disease and Lesinurad is a recently approved, highly potent inhibitor
diuretic use and has a high mortality rate (see MKSAP 18 of renal uric acid reuptake targeting the urate transporter
Dermatology). A recently discovered DRESS risk factor is the URATI. lt Is approved only for combination use with febux-
HLA-B·ssm allele, which is more common in persons of Han ostat or allopuJinol. Leslnurad can rarely cause renal toxicity
Chinese. ThaJ, and Korean descent. Screening for HLA-s·sso1 in and should not be given to those With a n estlmated creatinine
high-riSk populations is recommended before initiating therapy. clearance less than 45 m L/min.
Xanthine oxidase Inhibitors cannot be coadministered with
purine analogues (such as azathioprine). CJ Pegloticase
Unlike most o ther mammals, humans lack a functioning
Febuxostat uricase to break down uric acid. Peglotlcase Is a recombinant,
Febuxostat is a noncompetitive xanthine oxidase inhibitor. As nonhuman, infusible pegylated uricase that is highly effective
with allopurinol, transaminltls rarely occurs, and liver at lower! ng serum urate. Pegloticase Is reserved for severe and/
enzymes should be monitored. Concomitant use with purine or refractory gout. Because of its extreme potency. mobiliza-
analogues is contraindicated. Incidence of DRESS syndrome tion flares of gout are common, and prophylaxis against acute
is rare. gouty attacks is req uired. Peglotlcase Is administered

13
Principles of Therapeutics

intravenously every 2 weeks; if the pre-infusion serum urate • Hepatitis B and C serologies (for biologic DMARDs and
increases to more than 6.0 mg/dL (0.35 mmol!L) on two occa- drugs that can cause hepatotoxicity)
sions, it is likely that antibodies have formed, and the drug • HIV screening
should be discontinued to prevent infusion reactions.
Patients with latent or active tuberculosis, active hepatitis
KEY POINTS B, or untreated HIV infection require initiation of appropriate
• Allopurinol is the most commonly used urate-lowering therapy before initiating immunosuppression. Repeat screen-
agent; the biggest risk the drug poses is DRESS (drug ing for tuberculosis should be performed annually If there are
reaction v-rith eosinophilia and systemic symptoms) risk factors for ongoing tuberculosis exposure.
syndrome, which has a high mortality rate. KEY POINTS
• Concomitant use of xanthine oxidase inhibitors (allopu- • Vaccinations should be updated before initiating bio-
rinol or tebuxostat) with purine analogues is contrain- logic disease-modifying antirheumatic dntg regimens.
dicated.
• Screening (and therapy tf needed) for tuberculosis, hep-
atitis Band C serologies. and HJV is appropriate before
initiating immunosuppressive therapy.
Medications and Pregnancy
Some rheumatologic medications can have adverse effects on
pregnancy. See Table 12 for a discussion of these agents and Nonpharmacologic and
their relative risks. Nontraditional Management
KEY POINTS Because rheumatologic diSeases frequently affect the muscu-
• Methotrexate is highly teratogenic and abortifacient; loskeletal system, nonpharmacologic measures are often
it must be discontinued at least 3 months before employed to address pain not eliminated by medications.
pregnancy. These measures include physical therapy, occupational ther-
apy. surgery, weight reduction, psychosocial support, and self-
• HydroxychJoroquine is relatively safe in pregnancy and
should not be discontinued if it is needed. management programs. Many patients tum to complementary
and alternative medicine as adjuncts to traditional medical
• Leflunomide is extremely teratogenic and must not be interventions.
used before or during pregnancy: upon discontinua-
tion. cholestyramine administration is required to Physical and Occupational Therapy
remove the drug !Tom the body in all women of child-
The physical therapist can aid the primary care proVider in
bearing potential and specifically in those Wishing to
assessing a patient's aerobic fitness and conditioning as well as
become pregnant.
the patient's ability to carry out activities of daily living. Pain
and functional limitation can be addressed through manual
Vaccination and Screening therapy, assistive devices, joint protection techniques, and
thermal treatments. A targeted exercise program can be initi-
in Immunosuppression ated, and adapting the program for home use is critical.
Patients should be updated with vaccinations before initiat- Tendinitis, bursitis, many fonns of arthritis, and chronic soft
ing biologic DMARD regimens. Vaccine response may be tissue pa.ln due to overuse, injury, and chronic pam syndromes
diminished once on treatment, and patients on some immu- (such as tlbromyalgta) are among the diagnoses appropriate
nosuppressants (including all biologic DMARDs and tofaci- for physical therapy referrnl.
t!nib) should not receive Jive attenuated vaccines (such as for Occupational therapists assess upper extremity function-
herpes zoster, Jive attenuated influenza, and yellow fever) Ing, lncludlng the ability to perfonn self-care and job-related
because of risk tor viral activity in an lmmunocompromlsed tasks. Braces and splints may be provided for pa.lnt'ul or unsta-
host. Despite diminished efficacy, patients already on a bio- ble joints. An ergonomic evaluation of the workstation may
logic DMARD should receive any non-live vaccines that are accompany instruction in improved body mechanics and
indicated as per standard of care. Patients receiving tradi- avoidance of repetitive trauma.
tional oral DMARDs (for example, hydroxychloroqulne,
methotrexate, and sulfasalazine) may receive any and all Complementary and Alternative Medicine
vaccines as needed. Nontraditional options for symptom management are
Before initiating immunosuppressive therapy. the follow- employed by about one third of patients overall and up to 90%
Ing screening Is recommended: of patients with chronic pain, Including arthritis and rheuma-
• Tuberculosis screening with tuberculin skin testing or tologic diseases. Commonly used over-the-counter supple-
interferon-y release assay, particularly for patients initiat- ments include fish oil. vitamins, glucosamine, and chondroi-
ing biologic DMARDs tin. Providers should ask about supplement use because

14
Principles of Therapeutics

TABLE 12. Rheumatologic Medications and Pregnancy•


Medication/Class Comments
Anti-Inflammatory Agents

NSAIDs May ampede implantation and may be assocaated with a small increased risk of miscarriage when used
before 20 weeks' gestat1on. Use of NSAIDs after 30 weeks' gestation can lead to premature closure of t he
ductus arteriosus
Glucocorticoids When taken in the first trimester, can increase the risk offetal deft palate. and can raise the risk of maternal
gestational diabetes throughout the pregnancy.
Useful in the management of active autoimmune disease in pregnancy. Non-fluorinated glucocorticoids
(e.g., prednisone, prednisolone, methylprednisolone) have limited ability to cross the placenta and are
preferred, except when treating the fetus (e.g., neonatal lupus erythematosus).
Colchicane Should be used only if the potentaal benefit justifies the potential risk to the fetus.

Analgesics

Acetaminophen Generally conSldered safe at standard dosing, but does cross the placenta.
Opiates Some opiates/opioids cross the placenta; may cause fetal opioid withdrawal at birth.
Tramadol Should be used only if the potential benefit JUstifies the potential risk to the fetus; postmarkettng reports
suggest the possibilfty o f neonatal seizures, withdrawal syndrome, and stillbirth.
Topical agents Topical use may limit serum levels; individual agents should be reviewed for pregnancy impact prior to use.
I
Nonbiologic DMARDs

Methotrexate Highly teratogenic and abortifacient; muSt be discontinued at least 3 months before pregnancy.
Hydroxychloroquine Relatively safe in pregnancy and should not be discontinued if needed.
Sulfasalazine Relatively safe dunng pregnancy.
Leflunomide Extremely teratogenic; must not be used before/during pregnancy; upon discontinuation, cholestyramine
administration is required to remove the drug from the body in all women of childbeari ng potential and
specifically in those wishing to become pregnant; should be followed up with measurement ofleflunomide
and its metabol ite levels to ensure removal of the drug.
Azath ioprine Routine use in pregnancy is not recommended; however, azathioprine may be safer than some other
DMARDs and may be used if an immunosuppressive agent ts imperative.
Cyclophosp hamide Not used in pregnancy unless absolutely necessary.
Mycophenolate mofetil Teratogenic; should not be used in pregnancy; discontinue for 3 months before attempting pregnancy.
Cyclosporine May be used in pregnancy only if benefits outweigh the risks.
Tofacitlnib May be teratogenic at high doses.

Biologic OMARDs

TN F·a inhibitors Accumulating retrospective data suggest low risk in pregnancy, but evidence is limited; can be continued if
absolutely needed; different agents may have different considerations regarding crossing the placenta
Ustekinumab; anakinra; Should be used only if the potential benefit justifies the undefined risk to the fetus.
secukinumab
Abatacept; belimumab; Should be used only if the potential benef1t JUStifies the potential risk to the fetus.
canakinumab; rilon acept;
rituximab; tocilizumab

Urate-Lowering Therapy (rarely needed In premenopausal women)

Allopurinol Should be used only if the potential benefit justifies the potential nsk to the fetus.
Febuxostat Should be used only if the potential benefit justifies the potential rrsk to the fetus.
Probenecid No current evidence for adverse impact on pregnancy
Pegloticase Should be used only if the potential benefit justifies the potential risk to the fetus.
DMARO • di>ease-mod;fying ontirhevmat•c dt\J9; TNF • tumor necrosis !actor.
-s•e MKSAP 18 General Internal Medicine for lnfotm.ation on 1he FDA pregnancy categorios,
-- ---------------- -- -

15
Rheumatoid Arthritis

patients rarely volunteer this Information. Significant drug immune responses to provide a milieu for the development of
interactions may occur; for example, some herbal preparations autoantibodies.
can interact with anticoagulants.
Mind-body interventions such as tal chi, meditation, and Environmental Factors
yoga can improve psychological well-being. strength, balance, EnVironmental factors provide 40% of the risk for RA One of
and pain level. Chiropractic and osteopathic manipulation as the most provocative environmental factors is smoking.
well as massage remain popular. Randomized controlled trials Smoking can lead to lung inflammation. which activates
support the use of tai chi for arthritis; smaller trials suggest enzymes such as PADI. and may promote citrullination.
benefit from meditation techniques, yoga, massage, and Patients who smoke and are at risk for RA because of family
manipulative medicine for various musculoskeletal problems. history must be counseled about smoking cessation.

Role of Surgery Infectious Agents


Surgical procedures such as carpal tunnel release or rotator A potential risk factor for the development of RA is periodontal
cuff tendon repair can address conditions that arise from disease. Porphyromonas gtngiual!s, a bacterium associated
repetitive trauma, injury, and degenerative changes in the with periodontitis, produces PAD! enzymes and provides a
soft tissue. Synovectomy of inflammatory pannus is occa- potential link to cttrullinated peptide formation. Other infec-
sionally employed when a single or limited number of joints tious agents Implicated include mycoplasma, Epstein-Barr
in patients with RA do not respond to medications. Total joint virus. and parvovtrus 819. However, a direct infectious cause
arthroplasty, particularly of the knee or h.ip, can reduce or of RA has not been Identified. There Is also interest in the role
eliminate pain and restore function in patients with an inad- of the intestinal microbiome i.n RA. Gut dysbiosis has been
equate response to medication and physical or occupational postulated to promote early RA, possibly by activating proin-
therapy. flammatory lymphocytes.
KEY POINT
Hormones
• Nonpharmacologic measures used in rheumatologic
Women are two to three times as likely as men to develop RA.
diseases include physical or occupational therapy, sur-
The role of estrogen and other gender-spt:cific factors ls
gery, weight reduction, psychosocial support, and self-
incompletely understood but appears to promote a proinflam-
managemenliJrograms.
matory and/or proautoimrnune milieu. Estrogen receptors are
present on synovial fibroblasts and may lead to the production
of cartilage-damaging metalloproteases. Stimulation of estro-
Rheumatoid Arthritis gen receptors on macrophages can increase tumor necrosis
factor a production. a key RA inflammator:y cytokine.
Pathophysiology and Risk Factors KEY POINT
Rheumatoid arthritis (RA) Is a systemic autoimmune disease • Potential risk factors for rheumatoid arthritis include
characterized by a chronic inflammatory polyarthritis affect- genetic and environmental factors. infectious agents,
ing large and small joints with a predilection for tl1e small and hormones; genes provide 60% of the risk
joints of the hands and feet. RA has a prevalence of 0.5% to 1%
in the general population. with some specific populations hav-
ing rates as high as 7%. Diagnosis
The 2010 American College of Rheumatology (ACR) and the
Genetie Factors European League Against Rheumatism (EULAR) classification
Genes provide 60'X, ofthe risk for RA. Among some 100 genetic criteria for RA are more sensitive but less specific tha n the
loci currently recognized as associated with R.A risk, the most prior 1987 criteria and emphasize early diagnosis and treat-
important is the class 11 HLA group, especially HLA-D alleles. m ent to prevent the permanent consequences of chronic
These rtsk alleles code for the shared epitope, a five amino acid inflammation (Table 13).
sequence that binds and presents citrullinated peptide anti-
gens important in the pathophysiology of RA Because citrul- Clinieal Manifestations
line is an amino acid that does not nonnatly occur in hwnans, RA is a chronic disorder, and onset of symptoms is usually
citrullinated proteins are immunogenic, especially in people gradual. Patients with RA typically report joint pain and
who also have the shared epitope. inflammatory symptoms. Including swelling and morning
Otrulllne is formed by the action of the enzyme peptidyl- stiffness often lasting several hours. Stiffness is generally worse
arginine deiminase (PADl). which is found at sites of inflam - with rest but alleViated by ongoing actiVity. Joint swelling
mation and serves to deiminate arginine to form citrullinated (softness or bogginess of the affected joint) is palpable on joint
peptides. Many of the genes associated with RA modify examlnatlon.

16
Rheumatoid Arthritis

TABLE 13. The 2010 American College of Rheumatology/


European League Against Rheumatism Classification Criteria
for Rheumatoid Arthritis
Criteria Score•
Joint lnvolvementb
1 large joint (shoulders, elbows, htps, knees, ankles) 0
2·10 large joints 1
I 1-3 small joints (MCPs, PIPs, wrists, 2-5 MTPs) 2
j 4-10 small joints 3
More than 10 small joints 5
: Serology
I Negative RF or anti-CCP antibodies 0
· Low-positive RF or anti-CCP antibodies (under 3 2
times the upper limit ot normal)
High-level RF or anti-CCP antibodies (above 3 times 3
the upper limit of normal)

i Acute Phase Reactants


Normal CRP or ESR 0 FIGURE 2. Early rheumatoid arthritis of the hands, with swelling in the third

l Abnormal CRP or ESR

Duration
1 and fourth proximal interphalangeal joints.

Less than 6 weeks 0


More than 6 weeks 1
CCP • cyclk citrullln~ted peptide: CRP • C·reoctlve proto in; ESR • trythrcxyte I
.s&dimentation rato: MCP • metac.arpophalangeal: MTP • met.otarsophalangeal:
PIP • proximal interphalangeat; RF - rheumatoid factot. I
I
•Six p oints- needed for dasslfication M rhevmatoJd &rtiH"itis.

bAt least one JOint with definfte din«:al syn<Witis thDt ts not better explained by
another disease.
from Aleta "a 0, Noogo T, Silman AJ, Funovils J. Fefson OT, Bingham CO Jrd, et al.
2010 Rhe<Jmatoid arthritis dassification criteria: an American College of
Rheumatolog-y/Europt~ar•leagve Against Rhwmatism cotla.bor'ative initiative
Arthritis Rh~um. 2010 Sop;62(9J:2S69·81. doi· 10 . I 002/ort.27584. (PMID:
20872595) Copyrigh< 2010, American College of Rhevm<>tology. Ad~pted with
petmi$$ioc\ from John Wiley & Sons, Inc.

Cl RA. The pattern of joint involvement is useful for diagnosing


RA c haracteristically affects the metacarpophalangeal
joints, metatarsophalangeal joints, and proximal interphalan- Fl GURE 3. Severe rheumatoid arthritis of the hands, with ulnar deviation and
geal joints of the hands and feet but spares the distal inter- subluxation at the metacarpophalangeal joints on both sides.
phalangeal joints of both the· upper and lower extremities
(Figure 2 and Figure 3). Wrists, elbows, shoulders, hips, knees,
and ankles also can be Involved. RA affects joints symmetri- 70% of patients with RA and may be present at the time of
cally (that is. joints on both sides of the body are generally disease onset. Because other diseases can be associated with
involved), but severity may be asymmetric. RA may occasion- rhewnatoid factor, its specificity for RA is somewhat limited.
ally present as persistent involvement in a single joint. RA Anti-CCP antibodies are also present in 70% of patients with
spares the thoracic and lumbar spine but affects the cervical RA but have a specificity of 95%. Anti-CCP antibodies are also
spine, especially the Cl-C2 (atlantoaxial) articulation. See more predictive than rheumatoid factor for erosive disease.
Table 14 for more information. Cl Approximately 10% to 20% of patients diagnosed with RA
are seronegative (that is, neither rheumatoid factor nor anti-
Laboratory St udies CCP antibodies Is positive). Although categoriZed as having
The most useful laboratory studies to aid In the diagnosis of RA RA, these patients have somewhat different genetics and risk
are rheumatoid factor and anti-cyclic citrullinated peptide factors and generally have a better prognosis than those with
(CCP) antibodies. Rheumatoid factor is found in approximately seropositive RA.

17
Rheumatoid Arthritis

TABLE 14. Consequences of Persistent Inflammation


on Joints and Supporting Structures
Joint Area lmpfications
C 1-C2 articulation and Laxity of the transverse ligament
transverse ligament results In increased posterior motion
of the dens on C2; with neck flexion,
the dens can impact the midbrain
and other vital neurologtc structures
Shoulder and rotator Restricted range of motion of the
cuff tendons glenohumeral joint and rotator cuff
tears
Elbow joint Elbow contractu res and difficulty w1th
hand pronation and supination
' Wrist carpa l joints and Restricted range of motion of wrist;
finger tendons carpal tunnel syndrome; rupture of
finger extensor tendons, especially
the fourth and fifth
MCPs and surrounding Ulnardevtation and subluxation of
structures MCPs
PIPs and surrounding Swan neck or boutonniere deformity
structures due to inflammatory disruption of
periarticular support structures
Hip joint Axial migration of the femoral head in
the acetabulum (protrusio acetabuli) I
Knee joint Tricompartmental jo1nt-space narrowing
Ankle and mid-foot Restricted range of motion of ankle;
joints and tendons progressive pronated flat foot deformity
MTPs and surrounding Rbular deviation of MTPs; cock-up F I GU R£ 4. Radiograph showing advanced rheumatoid arthritis In the hand.
structures deformities; skin ulceration There is ulnar deviation at the meta<arpophalangeal joints; margi nal erosions
underneath subluxed MTP heads most prominently at the second through fourth metacarpcpha!angeal joints and
MCf>• metacarpophalangeal; MTP- metatarsophalangeal; PIP • proXimal
the second and third proximal interphalangeal joints (arrows); and joint·space
interphalangeal. narrowing at the wrist, metac<~rpophalangeal, and proximal Interphalangeal joint.s,
which also represents erosive disease. Nole the loss of the ulnar styloid
(arrowhead), another common sign of bony erosion in rheumatoid arthritis.

Erythrocyte sedimentation rate and C-reactiVe protein are KEY POINTS


elevated In 75% of patients with RA and can be used to monitor
• Clinical manifestations of classic rheumatoid arthritis
treatment response. The systemic inflammation inherent to
typically include pain, swelling, and prolonged morning
RA is commonly reflected by an anemia of inflammation and
stiffness in symmetric small joints.
modest thrombocytosis.
• The most useful laboratory studies to aid in the diagno- HVC
Imaging Studies sis of rheumatoid arthritis (RA) are rheumatoid factor
Plain radiography of the hands and/or feet is a standard imaging and anti-cyclic citrullinated peptide (CCP) antibodies:
study for RA and can aid in diagnosis and assessing progression, anti-CCP antibodies have a specificity of 95% for RA.
although early radiographs may be normal. Radiographic • Plain radiography of the hands and/or feet is a standard HVC
changes include periarticular osteopenia, mazgi.nal erosions, imaging study for rheumatoid arthritis: radiographic
and joint-space narrowing (Figure 4 and FigureS). Radiography changes include periarticular osteopenia, marginal
of the cervical spine with flexion/extension views is appropri- erosions, and joint-space narrowing, although early
ate if Cl-C2 subluxation Is suspected. radiographs may be normal.
MRI and ultrasonography are more sensitive than plain
radiography and may have utility in following disease, assess-
lng risk of progression, and determlning response to therapy
Ultrasonography is becoming a standard tool for detecting
Complications and Extra-Articular
joint fluid, synovial tissue thickening, early erosions, and Manifestations
increased vascularity. MRI is used for measuring bone marrow Joints
edema, synovitis, and erosions; it is also specifically indicated RA joint damage is a consequence of the development of syno-
if atlantoaxial involvement ls suspected. v itis within the joint. The synovial lining, normally only a few

18
Rheumatoid Arthritis

Eyes
The most common eye manifestation ln RA is dry eye (kerato-
conju nctivitis sicca). It occuiS in lO'X, to IS% of patients and
can be severe. Most of these patients also have dry mouth and
are classified as having secondary SjOgren syndrome. Less
common (1 %) are episclerttis (l.nfiammatton of the superficial
sderaJ vessels) and sclelitis (inflammation of deep scleral ves-
sels). RA is one of the most common diseases associated with
scleritis, which can be vision-threatening and lead to thinning
of the sclera and perforation. Keratitis (corneal inflammation)
can occur, which is ulcerative and occurs at the periphery of
the cornea; severe keratitis is known as corneal melt. Both
scleritis and keratitis require immediate referral to an
ophthalmologist.

Lungs
Air trapping reflecting small airway disease occurs in up to
E:J
SO% of patients with RA. Pleural disease occurs in up to So/o;
pleural effusions are exudative and can be large. RA pleural
effusions are characterized by low glucose and pH (mimicking
bacterial or tubercular infection and malignancy) and low
complement levels, as well as elevated levels of total protein,
rheumatoid factor, and lactate dehydrogenase. Inflammatory
cells in the RA effusion are characteristically mononuclear; a
neutrophil- predominant effusion suggests Infection.
Interstitial lung disease contributes to excess mortality ln RA
and may develop in 50% of patients, but clinically significant
disease is seen in 10%. Bronchiectasis and bronchiolitis occur,
and the bronchiolitis can be obliterative/constrictive. Upper
airway involvement from cricoarytenoid arthritis occurs
FIGURE S. Ra.diograph of rheumatoid artllr~is in the foot showing marginal rarely; symptoms include hoarseness. sore throat. dysphagia.
erosions and joint~pace narrowing at the thi rd. fourth, and fifth metatarsophalangeal
joints (arrows). Erosion atthe fifth metatarsophalangeal joint is often the first and stridor. Cricoarytenoid arthritis can pose problems for
r.~dlographic sign of rheumatoid arthritis foot involvement. endotracheal intubation.

Heart
cells thick, becomes significantly expanded and produces Atherosclerotic heart disease remains the major cause of
cartilage-damaging metalloproteases. Receptor activator of excess death in patients with RA, although recent data suggest
nuclear factor kappa B ligand (RANKL), produced by inflam- that cardiovascular disease risk may be decreasing toward that
matory cells, activates osteoclasts to erode bone. The metallo- of the general population. Nonetheless. patients with RA
proteases and activated osteoclasts result in various irreversi- should be considered at high cardiovascular risk for purposes
ble changes to the joints (see Table 14). of perioperative evaluation, and cardiovascular disease risk
factors such as dyslipidemia and hypertension should be
Skin addressed. Clinically significant pericarditis is rare.
The most common RA skin changes are rheumatoid nodules, Granulomatous myocarditis, valvular disease (mainly mitral).
present in up to 30% of patients. Nodules typically occur in the conduction block, and aortitis are reported to occur in RA but
olecranon region and can be confused with gouty tophi, but are very rare.
can also occur over the hand and feet joints and even in the
lungs. Nodulosis can rarely be induced by certain drugs (for Hematologic
example, methotrexate and leflunomide) but may respond to Anemia of inflammation is the most common RA hematologic
others (for example, hydroxychloroqui.ne and colchicine). abnormality. Felty syndrome consists of neutropenIa and sple-
Patients with RA are also at an 1ncreased risk for neutrophilic nomegaly and occurs in patients with long-standlng, severe,
dermatoses such as pyoderma gangrenosum or Sweet syn- seropositive RA. These patients are at risk for serious bacterial
drome. Palpable purpura may result from small-vessel cutane~ infection, lower extremity ulceration, lymphoma. and vascu-
ous vasculitis. litis. With current treatment, Felty syndrome has become rare.

19
Rheumatoid Arthritis

Patients with RA can also have a large granular lymphocyte w ho remain in remJssion or a low disease activity state for
Cl syndrome that can progress to large granular lymphocyte leu- 6 months or longer may be able to reduce treatment intensity.
CONT. kemJa. Findings overlap with Felty syndrome and Include Treatment decisions for established RA are more complex. An
neutropenia, anemia, thrombocytopenia, splenomegaly, and initial approach to the treatment of both early and established
recurrent Infections. Patients with RA are at increased risk for RA is outlined in Figure 6.
lymphomas (particularly large B-celllymphomas). and risk is
correlated with disease activity. CJ Disease-Modifying Antirheumatic Drugs
Nonbiologic Disease-ModifYing Antirheumatic Drugs
Blood Vessels Methotrexate ls the anch.or drug in RA. used in both mono-
A small-vessel cutaneous vasculitis occurs In a small percent- therapy and combination therapy. lt can be titrated to doses as
age of patients with RA, leading to palpable purpura or peri- high as 25 mg per wee!.< In partial responders; the treating phy-
ungual infarcts. A very rare, larger-vessel vasculitis similar to sician should generally maximize methotrexate dosing before
polyarteritis nodosa can affect multiple organ systems; prior to adding other agents. At doses greater than 1.5 mg weekly, meth-
current therapy, It had a 5-year mortality of 30% to SO%. otrexate oral absorption approaches its effective limit; switching
to subcutaneous administration allows for higher serum drug
KEY POINT
levels. Folic acid supplements minimize toxicity without dimin-
• Extra-articular manifestations and complications of Ishing efficacy. Of patients with RA taking methotrexate alone,
rheumatoid arthrttis include rheumatoid nodules, 30% to 50% achieve remJssion or low disease activity.
rheumatoid vasculitis. dry eye, small airway disease, Sulfasalazine, leflunomide, and hydroxychloroquinealso
interstitial lung disease. pleural effusions, and anemia can be used as monotherapy agents in R.A. Leflunomide may
of inflammation.

Management
See Principles of Therapeutics for details on the uses, mecha-
nisms of action, major toxicities, and/or monitoring require-
OMARD
ments of the medications used In RA. monotherapy
(usually MTX)
General Considerations
The 2015 ACR RA treatment guidelines (https://www.rheurna
to logy. org/Pract ice-Quality I Clin ica 1-Su pport/Clinical-
Practice- Guidelines/Rheumatoid-Arthritis) advocate for early
diagnosis and aggressive early therapy of RA to prevent irre-
versible cartilage and bone damage. The 2010 ACR RA classifi-
cation criteria emphasize sensitivity to permit early diagnosis Combination DMARD or Continue current
TNFi +/· MTX or therapy; if in
and institution of disease-modifying therapy (see Table 13). A Non-TNF biologtc +/- MTX or remission, consider
key treatment goal is to treat to target, with the target being Tofadtinib +1- MTX tapering medications•
achievement of remission or low disease activity. Disease activ-
ity assessment involves making a measured determination
using combinations of numbers of tender and swollen joints
(typically utilizing 28 joints that exclude the feet), patient and
physician impressions of disease activity, and, in some activity
scoring systems, measurement of the erythrocyte sedimenta- Consider alternate
tion rate or C-reactive protein. These parameters are combined biologiclanti-TNF/small
molecule/MTX therapy
into a composite score, thus assigning a disease actiVity tangl.ng
from remission, to low, moderate, or high. The Clinical Disease
FIGURE 6. Asimplified algorithm presenting an initial approach to the
Activtty Index (COAl) and Disease Activity Score 28 (DAS28) are treatment of both early and established rheumatoid arthritis (RA). All patients with
two commonly used instruments to assess disease activity and RA should receive a disease·modilying antirheumali< drug initially and be
response to treatment Treating to target In RA results In less advanced to more aggressive and/or combination therapy as needed to control
radiographic damage, reduced cardiovascular risk. and disease. Disease activity should be assessed, wherever possible, using a formal,
validated, and consistent disease activity index. Refer to the American College of
Increased work productivity compared with conventional care.
Rheumatology RA treatment guidelines for more complex algorithms accounting
The 2015 ACR RA treatment guidelines can assist in mak- for differences between agents and patient-specific complexities. DMARD •
ing initial treatment decisions in early RA. Treatment ts typi- disease-modifying antirheumatic drug; MTX =methotrexate; RA .. rheumatoid
cally advanced at 12-week IntervalS with the goal of reaching arthritis; TNFi "' rumor necrosis factor inhibitor.
remission or low disease actiVity as rapidly as possible. Patients •oo not dlscon~nu• all RA treatments.

20
Osteoarthritis

be useful in those who cannot tolerate methotrexate. carpal tunnel release, repair of a ruptured tendon, total joint
Hydroxychloroquine is the least potent agent but can be used replacement (shoulder, metacarpophalangeal joints, hip, knee,
in very early disease when the disease activity score is low. It or ankle), or joint fusion for a painful damaged joint (wrist or
is also used as part of triple therapy (methotrexate, sulfasala- ankle). See MI<SAP 18 General Internal Medicine for a discus-
zine, and hydroxychloroquine). Data suggest that triple ther- sion of perioperative RA medication management.
apy is comparable to methotrexate combined with a tumor
KEY POINTS
necrosis factor a inhibitor, except in the area of radiographic
progression. • In rheumatoid arthritis, treating to target of remission
or low d1sease activity results in less radiographic dam-
Biologic Disease-Modifying Antirheumatic Drugs age, reduced cardiovascular risk, and Increased work
Biologic DMARDs can be used as monotherapy but are typi- productivity compared with conventional care.
cally added to methotrexate when moderate to high disease • Methotrexate Is the anchor drug in rheumatoid arthrl- HVC
activity persists. 1\lmor necrosis factor (TN F)- a inhibitors are tis; It is used as monotherapy and as a component of
the most frequently used biologic DMARDs; these agents have combination therapy.
a relatively rapid onset of action and demonstrate synergy with • Tumor necrosis factor a inhibitors are the most fre-
methotrexate. The combination of methotrexate and a TNF-a quently used biologics to treat rheumatoid arthritis;
inhibitor has also been shown to have a "disconnect effect, • they have a relatively rapid onset of action and demon-
meaning that even patients with continued clinical disease strate synergy with methotrexate.
activity may demonstrate little to no damage to cartilage and
• In rheumatoid arthritis, NSAIDs are not disease modify- HVC
bone. This effect also has been shown with rltuximab or toci-
ing and do not prevent joint damage; they are used pri-
Jizumab m combination wlth methotrexate. Other biologic
DMARDs used in RA include abatacept (a selective T-cell marily to control symptoms while waiting for the full
effect of disease-modifying antirheumatic drugs to be
costimulation modulator) and tofacitinib (a small- molecuJe
rea1!1..ed or in patients with postinflammatory osteoar-
Janus kinase inhibitor).
thritis.
Until better data are available to guide therapeutic deci-
sions, choice of a biologic DMARD remains empiric and based • low-dose prednisone can be used in rheumatoid
on patient characterfsttcs, Including what agents should be arthritis to rapidly improve symptoms until long-term
avoided due to patient comorbidity (for example, avoiding medications become effective, or they can be used short
abatacept in a patient with COPD). term for disease flares.

NSAIDs
NSAIDs were once the mainstay of therapy of RA. These agents Pregnancy
are not disease modifying and do not prevent joint damage. There is an increased risk for developing RA in the first year
They are used primarily to control symptoms while waiting for after a .first pregnancy. Breastfeeding may decrease this risk
the full effect of DMARDs to be realtzed or In patients with For women with established RA, two thirds will go Into remis-
postmtlanunatory osteoarthritis. sion or low disease activity during pregnancy, and one third
will not improve or will get worse. Medication management is
Glucocorticoids a major issue, and pregnancy plans should be discussed with
Unlike NSAlDs, glucocorticoids may have a disease-modifying any woman of childbearing age who will be placed on therapy.
effect. A recent study tested 10 mg/d of prednisone versus pla- A discussion of RA medications in pregnancy is located in
cebo for 2 years in combination with methotrexate; the pred- Principles of Therapeutics.
nisone group had no more side effects than the placebo group
KEY POINT
but gained better control of disease activity, used less metho-
trexate, needed fewer additional medications, and had less • There is an increased risk for developing rheumatoid
radiographic damage. Low-dose prednisone (5-10 mg/d) can be arthritis (RA) in the tirst year after a first pregnancy; for
used to rapidly improve RA symptoms while waiting for long- women with established RA, two thirds will go into remis-
term medications to become effective, or they can be used sion or achieve low disease activity during pregnancy,
short term for disease flares. Long-term therapy with glucocor- whereas one third will not improve or will get worse.
t1coids, however, may be associated with substantial adverse
effects including osteoporosis, diabetes mellitus, and Infection.
Osteoarthritis
Surgery
Surgical therapy bas become much less common in RA with Pathophysiology
current treatment strategies. However, some patients may Osteoarthritis (OA) ls a chronic progressive multifactorial dis-
require a synovectomy for a single persistently swollen joint, order of maladaptive cellular repair responses to joint stress.

21
Osteoarthritis

Previously deemed a ~wear and tear" disease and an inevitable Given its multifactorial etiology, an individual patient may
consequence of aging, OA is now recognized as a disorder develop OA as a consequence of one or more risk factors: how
drlven by a complex Interplay of genetics. joint illjury, cell the interaction of these risk factors culminates in OA is com-
stress, extracellular matrix degradation, and inflammation. It plex and not fully understood.
affects all the tissues of the joint and is characterized by carti-
KEY POINTS
lage and meniscal degradation, subchondral bone changes
(bone marrow lesions, subchondral sclerosis). and osteophyte • Osteoarthritis is a chronic progressive disorder charac-
formation. Other findings Include synovial inOammation with terized by cartilage and meniscal degradation, subchon-
hypertrophy and effusion as well as weakening of the periar- dral bone changes, and osteophyte formation.
ticular muscle. Morphologic changes in cartilage reflect colla- • Risk factors for osteoarthritis include epidemiological
gen and proteoglycan alterations driven by imbalanced ana- factors (age, race/ethnicity, female sex. genetics, obe
bolic and catabolic repair processes. In addition, intlanunatozy sity) and joint-specific factors (malallgnment, cartilage
cytokines such as interleukin-1~ and tumor necrosis factor a. defects, joint injury).
are produced in the synovium and cartilage; they drive joint
tissue destruction and stimulate synthesis of additional
inflammatory mediators.
Classification
Primary Osteoarthritis
The most common type of OA is primary OA, in which no
Ep idemiology and Risk Factors Identifiable proXimal cause is recognized. Although almost
OA is the most common form of artluitis worldwide and a any joint may be affected by OA, primary OA typically involves
leadil1g cause of pain and disability, affecting approximately spedftc joints, including the distal interphalangeal (DIP) and
30 million U.S. adu It~. OA is the leading cause oflower extrem- proximal interphalangeal (PIP) finger joints, the first carpo-
ity disability among older adults, With an estinlated lifetime metacarpal joint at the base of the thumb, the hip and knee
risk for knee OA approaching SO%. Prevalence is projected to joints, and the cervical and lumbar spine.
more than double by the year 2030, largely due to increaslng
obesity rates and aging of the population. Incidence and prev- Erosive Osteoarthritls
alence rates vary by joint and depend upon whether a clinical Erosive (lnflammatozy) OA 1s a subset of primary hand OA.
or radiographic definition is being applied. Recent efforts Clinically, erosive OA mostly affects the DIP and PIP joints, is
examining whether patients With OA are at increased risk for more inflammatozy and painful than typical hand OA (with
cardiovascular morbidity and all-cause mortality have been erythema and swelling), and is more common in women.
inconclusive, With positive associations appeanng to relate Radiographs reveal diagnosis-defining central erosions (in
more to functional decline and disabiUty than to OA-specific contrast to marginal erosions seen in rheumatoid arthritis)
pathologic processes. with a "seagull" or ~gull-wing" appearance in the finger joints
Epidemiological risk factors for OA include age greater (Figure 7); joint ankylosis (bony fusion) may also occur.
than 55, race/ethnlcity, female sex, obesity, genetics, and occu- Whether erosive OA comprises a separate disease entity or is
pations that include repetitive motions or physical labor. Age part of the continuum ofOA remains controversial.
and female sex are strong nonmodifiable rlsk factors for OA of
different sites. Obesity is the most important modifiable risk Secondary Osteoarthritis
factor, especially for knee OA. Increased body mass Is also a Secondary OA Is historically defined in the presence of a
risk factor for hand OA, perhaps underscoring the systemic predisposing disorder; however, an Increasing number of
nature of the disease and a role for metabolic and/or inflam- r isk factors are being identified for primary OA, thus blurring
matozy mechanisms in its pathogenesis. Single inherited con- the lines between the primary and secondary forms of the
ditions rarely predispose to OA; however, mutations in proteins disease. Pathologic changes, clinical presentations, symp-
involved in bone or articular cartilage structure or metabolism toms, and management are indistinguishable between pri-
are possible risk factors. mary and secondary OA. When OA is observed in a joint not
Risk factors involving the joint itself include abnormal typically affected by primazy OA, however, secondary causes
loading, injury, malalignment, and intrinsic cartilage or should be entertained. Commonly recognized causes of sec-
bone tissue defects. Injury may be acute (for example, ante- ondary OA include a history of frank damage such as trauma,
rior cruciate ligament tear) or may be more minor and joint infection. or surgical repair (such as anterior cruciate
repetitive (for example, physical labor or overuse); regard- ligament repair or meniscectomy); congenitally abnormal
less of cause, injuries have been linked w ith the future joints (for example. hip dysplasia); systemic metabolic, endo-
development of OA of various sites. Hip dislocation, con- crine, and neuropathic disorders, particularly those that
genital dysplasia, femoroacetabular impingement, knee affect cartilage; and underlying inflam ma tory arthritis With
rnalalignment. and shape of the knee joint are also associ- accompanying damage (for example. rheumatoid or psoriatic
ated with Incident OA. arthritis) (Table 15).

22
Osteoarthritis

F1GURE 7 . Plain radtographs showing erosiv~ hand osteoarthritis (OA). Note the dassic •gul~wing" appearance ofthe fourth and fifth distal interphalangeal (DIP) joints
on tbe left and the fifth DIP joint on the right (green arrows), reflecting centtal erosion of cartilage on the proximal surface of the joint. Also seen is bony ankylosis, a feature of
erosive OA. of the right fourth DIP joint and developing in the right third DIP joinI(red arrows). Additionally, the left second DIP joint exhibits the findings of dassic
(nonerosive) hand OA(joint·space nar10wing, subchondral sclerosis, and osteophytes without the 'gull·wing' appearance) (yeflow auow).

TABLE 15. Secondary Causes of Osteoarthritis most often involved. Although spinal ligamentous ossification
is also seen in an.kylosing spondylitis, the spinal calciflcatlons
Secondary Cause Joint(s) Typicalty Involved
in DISH are more "flowing," wider and less vertically oriented
Hemochromatosis Second/third MCP joints than those seen With ankylosing spondylitis. There is addi-
Calcium pyrophosphate MCP joints; wrists; knees; hips; tionally no involvement of the sacroiliac joints in DISH.
deposition shoulders; atlanto·axial joint Radiographic changes characteristic of DTSH Include con-
Alkaptonuria/ochronosis Spine; hips; knees fluent ossification of at least four contiguous vertebral levels.
Acromegaly Knees; shoulders; spine usually on the right side of the spine (Figure 8). Patients with
Hyperparathyroidism Wrists; MCP JOints DISH have less Involvement of the left side of the thoracic
spine, perhaps secondary to the aorta serving as a mechanical
Joint injury Knees
barrier to the production of bony hyperostosis.
MCP- metbc•'l'<>Phol•ngeal.
KEY POINTS
• The most common type of osteoarthritis (OA) Is primary
Diffuse Idiopathic Skeletal Hyperostosis OA, in which no identifiable proximal cause is recognized.
Diffuse idiopathic s keletal hyperostosis (DISH) is a noninflam- • Erosive osteoarth ritls (OA) mostly affects the distal Inter-
matory conclition characterized by calcification and ossifica- phalangeal and proximal interphalangeal joints, is more
tion of spinal ligaments (especially the anterior longltudln.al inflammatory and painful than typical hand OA (with
ligament) and entheses (tendon and ligament attachments to erythema and swelling), and is more common in women.
bone). Unlike OA, DISH is more common ln. men. DrSH usu- (Continued)
ally presents as back pain and stiffness, with the thoracic spine

23
Osteoarthritis

OA, clinical findings may not be accompartied by radiographic


changes; conversely, some patients with prominent radio-
graphic changes may have minimal or no symptoms. Patients
with OA are typically over so years of age; diagnosis at an
earlier age should prompt inquiry into a history of prior joint
damage, endocrine or metabolic disorders. or a genetic pro-
clivity for early disease.
Joints most commonly affected are the hands (DIPs and
PIPs) (Figure 9). feet (first metatarsophalangeal joint and mld
foot), knees, hips, and spine, but the distribution is variable.
Localized OA, In whJch a single joint Is affected, is more often
a consequence of injury or joint asymmetry and often occurs
in weight-bearing joints (hip or knee}. Generalized OA, which
affects multiple joint groups (for example, hands, spine, knees,
and hips), is more likely the result of a combination of genetic
and environmental factors and may be more symmetric in
distribution.
Patients with OA usually describe an insidious onset of
Intermittent symptoms, which become more persistent ~d
severe over time. The most common symptom is joint pain
that is exacerbated by activity and alleviated with rest. Patients
also describe morning stiffness usually lasting less than
30 minutes, in contrast to the prolonged morning stiffness of
inflammatory arthritis. A single joint may initially be involved
with eventual involvement of multiple joints.
On joint examination, crepitus, decreased range of
motion. bony enlargement, and sometimes effusion may be
present. Patients with long-standing hand OA may have
Heberden and Bouchard nodes (bony enlargement of the DIP
fIGURE 8. Plain lumbar radiograph of a patient with diffuse idiopathic skeletal and PIP joints, respectively) and squaring of the first carpo-
hyperostosis (DISH}. Note the calcification at sites of tendinous and ligamentous metacarpal joint. Hlp Involvement typically manifests as groin
insertion of the spine, taking the form offlowing ossification of multiple contiguous pain and decreased range of motion, especially internal rota-
vertebrae (arrows).Also seen here is the typical extensive involvement of the right
side of the spine and relative sparing of the left (calcification of the anterior
tion. Knee symptoms include pain on walking, especially on
longitudinal ligament is also common, but not readily observed In this view}. stairs, and difficulty transferring from a seated to standing
Courte$y of Sotorio• Gyftopoulo•, MO. MSc.
position. Spondylosis, or OA affecting the spine, can affect the
vertebral bodies, facet joints, and neural foramina, and may
KEY P 0 IN TS (continutd) lead to spinal stenosis.
Patients with OA generally do not have systemic features.
• Commonly recognized causes of secondary osteoarthri-
Pain and structural changes, however, ultimately result in
tis include a history of frank damage: congenitally
functional impairment, pain, disability, psychosocial isolation,
abnormal joints: systemic metabolic, endocrine, and
and reduced quality oflife.
neuropathic disorders; and underlying inflammatory
arthritis. Laboratory and Imaging Studies
• Diffuse idiopathic skeletal hyperostosis is a noninflam- Laboratory testing Is usually not necessary for diagnosis of
matory condition characterized by calcification and OA but is helpful if other causes of arthritis are being con-
ossification of spinal ligaments and entheses; radio- sidered, such as crystal arthropathy, rheumatoid arthritis,
graphs usually show confluent "flowing" ossification of psoriatic arthritis, or hemochromatosis (all of which can
at least four contiguous vertebral levels, usually on the coexist with OA). Acute phase reactants should be normal
right side of the spine. in OA. Routine laboratory testing (complete blood count,
kidney and hepatic function) are not necessary for diagnos-
ing OA but may be important when considering pharmaco-
Diagnosis logic therapy, especially In the elderly and those with
Clinical Manifestations comorbidlties.
OA diagnosis is based on history and physical examination; If effusion is present, evaluation for concurrent crystal
radiography is confirmatory but may not be necessary. In early arthritis, infections, or other inflammatory causes should be

24
Osteoanhritls

FIGURE 9. Progression of hand osteoarthritis (OA). This image illustrates the progression from normal hand joints (left), to distal interphalangeal (DIP) joint bony
hypertrophy (Heberden nodes) of hand OA (middle), to extensive hand OA findings of Heberden nodes and proximal interphalangeal (PIP) joint bony hypertrophy (Bouchard
nodes)(right). The hand on the right also displays a gouty nodule (tophus) overlying the Heberden node on the second DIP joint underlining the potential coexistence of
these two arthropathies. White arrowheads- Heberden nodes; black arrows - Bouchard nodes; black asterisk =tophus.

considered, ideally by synovial fluid analysis. OA synovial fluid anti-cyclic citrullinated peptide antibodies, and elevated
Is typically dear In appearance and noninflammatory, with a inflammatory markers favors the diagnosis of rheumatoid
leukocyte count S.2000/llL (2.0 x 109/ L). arthritis but does not eliminate the possibility of concurrent
Although imaging Js not necessary to make an OA diagno- OA. Psoriatic arthritis can involve the DIP joints, but also com-
sis, it can be helpful to confirm the diagnosis, establish base- monly includes prolonged morning stiffness, joint swelling,
line severity, and exclude other diagnoses. Radiographic dacl.ylitis, and a history of psoriasis. Synovial fluid analysis can
features of OA include asymmetric joint-space narrowing, also be helpful in distinguishing between OA and these
subchondral sclerosis, ostcophytes, and bone cysts; however, inflammatory disorders.
these changes may not be present in early disease. Even In Other conditions to consider when evaluating a patient for
established OA, symptoms may correlate poorly with imaging OA include nonarticular sources of pain such as bursitis and
fin<lings. Although MRI and ultrasonography can detect subtle tendinitis. Hip pain that is not In the anterior groin but instead
OA changes at an earlier stage and are Increasingly used in OA around the lateral hip and buttock may indicate trochanteric
research, they are not needed for routine OA diagnosis. MRl bursitis or lumbosacml rad.iculopathy. Similarly, knee pain may
may be Indicated in the setting of symptoms suggestive of a be secondary to pes anserine bursitis or iliotibial band syndrome
concomitant mechanical disorder Uolnt catclting, locking, rather than intra-articuJar pathology. The pes anserine bursa is
instability), but incidental abnormalities such as meniscal located along the proximal medial aspect of the tibia; thus, pes
tears are commonly seen on MRI in patients wtth OA and may anserine bursitis should be considered when there is spontane-
prompt unnecessary surgical intervention. ous pain along the inner (inferomedial) aspect of the knee joint.
Patients With iliotibial band syndrome typically present with pain
at the lateral aspect of the knee, usually exacerbated by physical
Differential Diagnosis activity. such as walking up or down stairs. numing, or cycling.
Diagnosing OA may be problematic when OA is present in
KEY POINTS
atypical joints, when an accurate history is difficult to obtain, or
when a concurrent Inflammatory arthropathy may be present. • The most common symptom of osteoarthritis is joint
Calcium pyrophosphate deposition may occur in joints also pain that is exacerbated by activity and alleviated wHh
typical for OA (bands or knees) but Is associated with Intermit- rest; morning stiffness usually lasts less than 30 minutes.
tent "flares, • and radiographs show cartUage calcification • Laboratory testing is usually not necessary to diagnose HVC
(chondrocalclnosis). G<lut and OA commonly co-associate, osteoarthritis but is helpful ifother causes of arthritis are
part!cularly In the DIP joints. Rheumatoid arthritis typically being considered, such as crystal arthropathy, rheuma-
affects the hands, but unlike OA, the DIP joints are rarely toid arthritis, psoriatic arthritis, and hemochromatosis.
affected, and signs and symptoms of chronic and persistent
(Continued)
inflammation are seen. The presence of rheumatoid factor,

25
Osteoarthritis

KEY POl NTS (continued} Oral and Topica.J Agents


• Radiographic features of osteoarthritis (OA) Include Oral agents include acetaminophen, NSAIDs, duloxetine,
asymmeLrlc joint-space narrowing, subchondral sclero- and tramadol. Recent systematic reviews and meta-analyses
sis, osteophytes, and bone cysts; however, these changes suggest that acetaminophen provides no benefit for hip or
may not be present in early disease, and symptoms may knee OA. NSAIDs are efficacious in OA. but their side-effect
correlate poorly with imaging findings, even in estab- profiles make sustained use problematic, especially In the
lished OA. elderly and those with comorbidities. Tf used, the proper
choice of an NSAID, considering drug pharmacokinetics as
well as gastrointestinal and cardiovascular effects, may
Management improve tolerance and minimize risk. Duloxetine, a seroto-
nin-norepinephrine reuptake Inhibitor with central nervous
To date, no agents have been FDA approved to prevent, delay, system activity, has shown efficacy for knee OA pain, impli-
or remit the structural progression of disease in OA. Instead, cating the role of central sensitization in OA pain modula-
current treatment is directed to the management of pain and tion. A 2018 randomized controlled trial demonstrated that
disability. Evidence-based guidelines for OA management are opioids were not superior to nonopioid medications for
available from the American College of Rheumatology (ACR), improving pain- related function for chronic back pair! or
the Osteoarthritis Research Society International (OARS!), the OA- related hlp or knee pain; pain intensity was significantly
European League Against Rheumatism (EULAR), and others; improved in the nonopiold group. Tramadol may be useful
however, there is poor consensus among these guidelines. in some patients who cannot take other analgesics, but has
Nevertheless, there is agreement that optimal OA management substantial adverse effects.
requires a combination of nonpharmacologic and pharmaco- Topical NSAIDs are an alternative to oral NSAIDs and may
logic modalities. Patients with OA should receive individual- be beneficial for those who should avoid or cannot tolerate oral
ized multidisdplinary treatment that takes Into account their NSA1Ds, including those who are ~75 years old, have a history
expectations, functional and activity levels, occupational and of peptic ulcer disease, or have c hronic kidney disease.
vocational needs, joints affected, severity of disease, and any However, topical NSAIDs are associated with more skin reac-
coexisting medical problems. tions and are more expensive than oral NSAIDs. Other topicals
such as capsaicin, lidocaine, and methyl salicylate prepara-
Nonpharmacologic Therapy tions may also be used as adjunctive measures, especially
The nonpharmacologic approach to OA starts with assessment of when NSA!Ds are either ineffective or not tolerated.
physical status, activities of daily living, health education, moti- The oral alternative therapies glucosamine and chondroitin
vation, beliefs, and other biopSJchosocial :fuctors. An individual- sulfate are frequently used by patients; although they appear to
ized management plan includes education on OA and joint pro- be safe, the qual !ty of evidence for their efficacy is poor.
tection, an exettise regimen, welght loss, proper footwear, and
assistive devices as appropriate. Physical activity includes gradu- Intra-Articular Injections
ated aerobic exercise and strength trnining, With attention paid Patients with knee or hlp OA who have Inefficacy, intolerance,
to strengthening periarticular structures and minimiZing lnjury. or contraindication to oral and topical therapies may benefit
Tal chi has also been shown to be as beneficlal as physical ther- from intra-articular glucocorticoid or hyaluronic acid injec-
apy for knee OA pain. A recent study reports that the combina- tions. There Is no formal evidence for the util!ty of Intra-
tion of d!etand exercise is more effeCtive at decreasing OA-related articular therapy in joints other than the knee and hip.
knee pain and dysfunction than either diet or exercise alone. Intra-articular glucocort!coids are efficacious in knee and
Furthermore, a 2015 Cochrane review, which included 54 knee hlp OA and are supported by multiple treatment guidelines.
OA studies, concluded that land-based therapeutic exercise pro- Long-term harm has not been demonstrated; however, benefit is
vides short-term benefit in terms of reduced knee pain and usually short term and usually wanes within 3 months. Injections
Improved physical fu nction that ls sustaJned for at least 21o 6 can be administered repe-atedly but are not usually given more
months after cessation of formal treatment. often than every 3 months. A recent 2-year study has called the
efficacy and long-term safety of intra-articular glucocorticoid
Pharmacologic Therapy injections into question, with possible negative effects on carti-
In the absence of disease-modifying OA drugs, pharmacologic lage thickness, an outcome with unclear clinical meaning.
treatment is considered when symptoms are present and both- Patients with inadequate response to intra-articular glu-
ersome to the patient. Pharmacologic therapy for OA includes cocorticoids may benefit from intra-articular hyaluronic acid
oral, topical, and intra- articular medications. Choice of injection. The quality of trials assessing intra-articular hyalu-
treatment(s) depends upon individualized assessment, with ronan efficacy ls low, and the degree of benefit is unclear.
particular attention to comorbidities, concomitant medications, Nonetheless, intra-articular hyaluronans are widely used,
and especially adverse effects of the treatments. See Principles espedally when other treatments have failed and surgery is
ofTherapeutics for details on the medications used ln OA. not a feasible option. When benefit is seen, it is usually more

26
Fibromyalgia

gradual and more persistent than that of Intra-articular Early paradigms of fibromyalgia as an inflammatory or
glucocorticoids. psychosomatic condition have yielded to an understanding of
flbromyalgia as a disorder of pain processing. It Is likely a form
Surgical Therapy of Mcentral sensitization,~ In which the pain centers of the
Surgery for OA is considered when nonpharmacologic and brain and spinal cord are hyperresponsive. Allodynia (a height-
pharmacologic approaches fail to control pain or functional ened sensitivity to stimuli that are not normally painfuJ) and
limitation. Multiple hlgh-quality studies have shown that hyperalgesia (an Increased response to painful stimuli) are
arthroscopic surgery for knee OA provides no better outcomes common. An additional characteristic feature is •wmd- up, • or
than conservative management unless there iS joint buckling, temporal summation. When repeatedly exposed to a mildly
instability, or locking or a concomitant and symptomatic uncomfortable stimulus, patients with fibromyalgia experi-
mechanical disorder. ence progressively additive pain, indicating that the stimuli are
In contrast, total joint replacement is a "curative" option both persistent and Inadequately damped.
for those who have failed conservative therapies, providing The underlying changes seen in the neurologic system
pa.l.n relief and funct1onal improvement. Overall, long-term are complex. Ascending fibers from the dorsal root ganglia
outcomes are excellent, although hardware loosening and late convey Inappropriately strong or persistent signals to hypo-
Infection may occur. thalamic pain centers. Patients withfibromyalgia have higher
Recent studies suggest that for morbidly obese patients cerebrospinal fluid levels of the pain-promoting neurotrans-
with knee OA, bariatric surgery may result In significant mitters substance P and glutamate, along with sensitization
Improvement in pain and function even without further OA of glutamate receptors. Concurrently, descending inhibitory
management pathways utilizing adrenergic neurotransmitters (serotonin,
norepinephrine) are impaired. The resultant circuit of
KEY POINTS
chronic pain is self-sustaining but may be amplified by both
HVC o Nonpharmacologic therapy for osteoarthritis includes psychic diStress and peripherally generated tissue pain (such
education, an exercise regimen, weight loss, proper as arthritic joint pain). Genetic Influences are a focus of
footwear, and assi~>tlve devices as appropriate. ongoing investigation.
HVC 0 Recent systematic reviews and meta-analyses suggest
that acetaminophen provides no benefit for hip or
knee osteoarthritis; NSAIDs and duloxet!ne are effica- Diagnosis
cious. The characteristic clinical features of fibromyalgia are wide-
o Pharmacologic therapy for osteoarthritis !ncJudes oral, spread chronic pain (including hypersensitivity to painful
topical, and intra-articular medications; choice of stimuli), fatigue, and sleep disorders (both disrupted and non-
treatment(s) depends upon individualized assessment, restorative sleep). These are frequently accompanied by
with particular attention to comorbidities and concom- Impaired cognitive funCtion. mood disorders. and symptoms
itant medications. such as headache, gastrointestinal symptoms. and paresthesia.
Although the diagnosis of fibromyalgla traditionally centered
HVC • Arthroscopic surgery is not indicated in patients with
on the presence of specific tender points elicited on physical
osteoarthritis unless there iS joint budding, instability,
examination, tender point examination Is subject to physician
or locking, or a concomitant and symptomatic mechan-
expertise (such as appropriate application of force). In addi-
Ical disorder.
tion, male patients report tender point pain less frequently
• Total joint replacement provides pain relief and func- than female patients, leading to possible underdiagnosis. The
tional Improvement in patients with osteoarthritis who 2010 American College of Rheumatology Preliminary
have failed conservative thernpies. Diagnostic Criteria forgo physical examination to emphasize a
careful characterization of symptoms (Table 16}. Many physi-
cians rely on these diagnostic criteria but perform tender point
assessment as part of the examination.
Fibromyalgia Treatable alternative diagnoses that may provoke or be
Epidemiology and mistaken for fibromyalgia Include hypothyroidism, hypoad-
renalism, and depression.
Pathophysiology
Fibrornyalgia iS characterized by widespread pain, fatigue,
disturbed sleep, and cognitive dysfunction. It is common Management
(prevalence, 2%-3%), particularly among lndlvfduals oflower Optimal management of tlbromyalgta requires a holistic
socioeconomic status and/or educational level. Prevalence approach, Including education, exercise, and psychosocial
Increases with age, peaking around the seventh decade. The support. Pharmacotherapy Is often warranted, although non-
female- to-male rntio is approximately 3:1. pharmacologic measures remain a cornerstone of treatment.

27
Spondyloarthritis

TABLE 1 6. Comparison of the 1990 ACR Classification Criteria and the 2010 ACR Diagnostic Criteria for Fibromy<>lgia
19 90 ACR Classfficatlon Criteria 2010 ACR Preliminary Diagnostic Criteria

Widespread pain by self-report Widespread pain by the Wtdespread Pain Index (WPI, 19-point scale) assessing the number of
regions 1n which the patient has pain'b
2:11 tender points on digital palpation Symptom severity by the Symptom Severity (SS) Scale ( 12-point scale) assessing 1) fatigue;
or using a dolorimeter 2) nonrestorative sleep; 3) cognitive symptoms•.t>
Duration of symptoms 2:3 months

J
No other disorder explaining the pain

ACR • American College of Rheumatology.


•Fibromyalgia is diagnosed in the setting of WPI >:7 plus SS score >5. or WPI ~3 plus SS score :.9
"The Widespread Po in Indo• and the Symptom Severity Scale con bo viow~>d at http<://www.meumatology.org/Port•ls/O/Files/2010%'0Fibromyalgia%200"'9'0<tic%20Crlteria_
Excerpt pdf.
----

Patients should be educated regarding the disease, with including the FDA-approved agents duloxetine and milnacip-
v:~Iidation that the symptoms are real and that the painful ran. They or a TCA may be particularly appropdate in the
areas are not injured and will not lose function. Aerobic exer- patient With concomitant depression.
ciSe can improve well-being and function as well as reduce Tramadol, which has a complex mechanism of action, has
pain. Because patients Initially experience postexercise pain shown some benefit and may be considered as a second-line
that may threaten their wtllingness to continue, exercise must approach. Finally, In some cases, combination pharmacologic
be introduced gradually and supported encouragingly. therapy may be helpful.
Strength training may also be helpful. Although the evidence Evidence does not support the benefit of NSAJDs for
base supporting their use is modest, alternative medicine fibrornyalgia, and pure opioids should not be used.
approaches, including yoga, tai chi, acupuncture, and mas- Fibromyalgia is a chronic disease, and the patient and
sage, may also help alleviate symptoms. physician should understand that the benefit of treatment will
Patients with fibromyalgia should be assessed for psy- likely be partial and palliative, rather than complete or cura-
chosocial stressors and psychiatric Illness, including a his- tive. Nonetheless. proper treatment can help most patients
tory of trauma. lf present, referral for psychological care Is manage and cope with their symptoms as well as maintain
mandatory, because psychic distress may both promote and function and autonomy.
result from fibromyalgia. Cognitive behavioral therapy has
KEY POINTS
shown modest benefit in reducing pain, negative mood, and
disability. • The characteristic features of fibromyalgia are wide-
Choice of pharmacologic therapy is based on symptom spread chronic pain, fatigue, and sleep disorders,
profile, patient comorbidities, and medication side effects which are frequently accompanied by impaired cog-
because few trials directly compare the efficacy of medica- nitive function, mood disorders, and symptoms such
tions. Effective pharmacologic therapies target the underlying as headache, gastrointestinal symptoms, and pares-
pathophysiology and inhibit the ascending pain pathways, thesia.
enhance the descending inhibitory pathways to the dorsal • The diagnosis of fibromyalgia is no longer based upon HVC
roots, or inhibit release of the pain-promoting neurotransmit- physician examination findings but a careful character-
ter glutamate. ization of symptoms using a v:~lidated scoring tool.
The antiepUeptic agents gabapentin and pregabalin (the • Non pharmacologic therapy (education. exercise, psy- HVC
latter FDA approved) inhibit a 2o calcium channels to inhibit chosocial support) remains a cornerstone of treatment
glutamate release. They have been shown to improve quaUty of for fibromyalgia, although pharmacotherapy is often
life and decrease pain. warranted.
Tricyclic antidepressants (TCAs) (such as amitriptyline)
raise norepinephrine levels and have documented benefit,
although efficacy may wane with time. TCAs induce drowsi-
ness, with potential benefit for di.sordered sleep. The muscle Spondyloarthritis
relaxant cyclobenzaprine, another tricyclic, may be useful in
patients with muscle spasms. Selective serotonin reuptake Overview
inhibitors have Jlttle benefit alone but may complement the Spondyloarthrit1s refers to a group of arthritic disorders that
activity ofTCAs. tends to Involve the spine and sacroiliac joints and share
Among the more effective fibromyalgia therapies are the genetic, pathophysiologic, and clinical features. HLA-B27 is
dual serotonin-norepinephrine reuptake lnhibttors (SNRTs). variably expressed but is more frequently present in this group

28
Spondylo arthritis

of patients than in the general population. Peripheral arthritis, KEY POINTS


enthesitis (inflammation of the insertion points of tendons
• Spondyloarthntls refers to a group of arthritic disorders
and ligaments onto bone), inflammatory eye disease, psortatic
that tends to involve the spine and sacroiliac joints and
rashes. and gastrointestinal and genitourinary inOanunation
share geneuc. pathophysiologic and clinical features
may occur in varying degrees.
• The most important genetic risk factor for spondyloar-
tnritis is the presence ofHLA-827. however, only S'X. to
Pathophysiology 6% of patients canying HLA 827 develop spondyloar-
Pathophysiologic processes involved in these disorders thrltis, and not all patients wtth spondyloarthritis are
Include T-cell activation and proliferation; elaboration of positive for HLA 827.
cytokines. particularly tumor necrosis factor (TNF)-a and
the lnterleukl ns (ll)-1 and -23117; and bony proliferation and
destruction. Although the exact trigger for these processes ls Classification
unknown, genetic and environmental factors are likely to Spondyloarthritls is divided into five categories based on the
play a role. predominant clinical features, although overlap of clinical
features among these categories is common {Table 17):
Genetic Factors
The most Important genetic risk factor for spondyloarthrttis is 1. Ankyloslng spondylitis: Spinal involvement is the defining
the presence of HLA-827. The prevalence of HLA-827 is high feature.
among Northern Europeans {about 6%) and low tn Africans; 2. Psoriatic arthritis: Psoriasis co-occurs with spinal and/or
accordingly, spondyloarthrltis ls seen with greater frequency peripheral arthritis.
in the former group. However, only 5% to 6% of those canying 3.lnflarnmatory bowel disease associated arthritis: Clinically
HLA-BZ7 develop spondyloarthritls, and not all patients with apparent intestinal inflammation is present along with spi-
spondyloarthritls a re positive for HLA-827. For these reasons, nal and/or peripheral arthritis.
HLA-827 should not be used as a screening test for these dis- 4. Reactive arthritis: Inflammatory synovitis (primarily pe-
orders, although Its presence in a patient with a high pretest
ripheral) occurs following specific gastrointestinal or geni-
probability can support the diagnosis.
tourinazy infections.
Several theories have been advanced to explain the rela-
tionship between HLA-827 and the spectrum of disease 5. Undifferentiated spondyloart.hrltis: Spondylitis occurs in
observed. The arthritogenlc peptide theozy suggests that the the absence of diagnostic features (Including radiographic
presentation of an unknown peptlde(s), perhaps of bacterial changes) that would permit diagnos1s of one of the other
origin, activates cytotoxic T lymphocytes in the context of categories of disease.
HLA-827. Other theories propose that HLA-827 molecules on Clinical features common to all forms ofspondyloarthritis
antigen-presenting cells can either directly activate natural are back pain, enthesltls, and dactylitis. Back pain is often the
killer and T cells even in the absence of antigen or can Induce presenting symptom for spondyloarthrltis, although spondy-
a proin flammatory environment. Similarly, HLA-827 expres- Joarthritls only accounts for about S')(, of chronic low back
sion on intestinal epithelium may interact with the gut micro- pain. Unli ke most forms of back pain, the pain in spondyloar-
biome to favor inflammation. Finally, tlssue-spectflc chronic thritis is inflammatory and typically occurs in patients under
inflammation triggered by bacterial or mechanical stress may the age of 40 years. It is characterized by an insidious onset.
underlie a more autoin.tlammatory rather than autoimmune duration of more than 3 months, presence at rest and at night
role for HLA-827. (waking the patient), morning stiffness lasting more than
Other genes that have been identified as risk factors for 30 minutes, and improvement with exercise. Enthesitis, or
spondyloarthrltls include those associated with the TNF Inflammation at the site where a ligament or tendon attaches
receptor signaling pathway as ~11 as IL-la and IL-23 receptor to bone, presents as pain and tenderness In areas such as the
polymorphisms. Achilles tendon (flgure 10). Dactylitis ("sausage digits") is a
diffuse fusiform swelUng of the fingers or toes, consistent with
Environmental Factors tendon and ligament involvement beyond the points of inser-
Considerable Interest has focused on the role of the microbi- tion (Flgure 11).
ome, particularly because gut inflammation is prevalent in Although the categories listed here are extremely helpful
many forms of spondyioarthritis. It is hypothesized that in identifying and diagnosing disease, recently developed clas-
microbe- associated intestinal inflammation could cause loss sification criteria emphasize the commonalities rather than
of Integrity of the bowel epithelium, permitting macrophage t he dJfferences among the various forms of spondyloarthritls
stimulation, Increases In IL-23, and Th17 cell activation. (for example, Assessment of SpondyloArthrttis International
However, speclfic causative pathogen exposure has not been Society [ASAS] criteria for peripheral and axial disease). Such
identified except In some cases of reactive arthritis. criteria may perm It diagnosis and treatment at an earlier stage

29
Spondyloarthritis

TABLE 17. Clinical Features of Spondyloarthritis


Ankylosfng Spondylitis Psoriatic Arthritis lBO·Associated Arthritis Reactive Arthritis

Musculoskeletal
Axial Axialmvolvement May occur at any level; May be asymptomatic but Less common than 1n
involvement predom<nates; initially may start in the can follow a course stmilar to other forms of
symmetrically involves the cervical spine; may ankylosing spondylitis; 51 spondyloarthritis
51 joints and lower sp1ne, skip reg<ons involvement often
progressing cranially; does asymmetric; axial arthritis
not skip regions does not parallel lSD activity
Peripheral Enthesitis; may have Five subtypes Acute polyarticular peripheral Enthesitis and
.nvolvement asymmetnc large-joint (symmetric arthritis: especially knee; asymmetric large-joint
oligoarthritis, including polyarthritis; early; can parallel lSD activity oligoarthritis; usually self-
hips and shoulders; hip asymmetric limited; nonerosive;
involvement can cause oligoarthritis; DIP- Chronic polyartiCUlar some patients experience
significant f unctiona l predominant; peripheral arthritis: of PIPs, recurrent or persistent
limitation; dactylitis MCPs, wrists, elbows, arthritis; may develop
spondyloarthritis;
uncommon arthritis mutilans); also, shoulders, knees, ankles, and features of otherforms of
enthesitis, dactylitis, MTPs; does not parallel lSD; spondyloarthritis
dactylitis; enthesitis
and tenosynovitis
Dermatologic Slc:in findings not Psoriasis typically Pyoderma gangrenosum; Keratoderma
characteristic, but precedes joint erythema nodosum blennorrhagicum
psoriatic-like lesions may involvement; (psoriasiform rash on
occasionally occur nail pitting; soles, toes, palms); circinate
onychodystrophy balanitis (psoriasiform rash
on penis)
' Ophthalmologic Anterior uveitis (unilateral. Conjunctivitis more Ante nor uveitis (can be ConJunctivitis more
recurrent) common than anterior b ilateral, insidious, or chronrc); common than anterior
uveitis (can be bilateral, conJunctivitiS, kerat1tJs, and uveitis
insidious, or chronic) ep1scleritis are rare
Gastrointestinal Asymptomatic intestinal Crohn disease; ulcerative PriorGI infection in some
ulcerations (rare) colitis patients
Genitourinary Urethritis (rare) Nephrolithiasis Prior GU infection in
some patients; sterile
urethritis; prostatitis;
cervtcttis; salpingitis
Cardiovascular Aortic valve disease; Association with Thromboembolism
aortitis; conduction uaditional CAD risk
abnormalities; CAD factors
Pulmonary Restrictive lung disease
from costovertebral rig1dity;
apical fibrosis (rare)
Bone quality Falsely elevated bone Increased risk of High risk for vitamin D Localized osteopenia
mineral density from fracture (multifactorial) deficiency, low bone density,
syndesmophytes; increased and fracture
risk of spine fracture

CAD • oorona<y artery d isease; DIP ~ di•tal interphalangeal; Gl • ga>rrointestnal; GU • genitourinary; lBO • inftamm•to<y bowel disease; MCP- metacarpophalangeal;
MTP • metat<orsophalangeal; PIP • proximal interphalangeal; Sl e sacroi~ac.

of disease before the unique teatures of the conditlon become described, women usually have mUder and underappreclated
we.ll defined. disease; prevalence may actually be simUar between genders.
Disease can begin as early as adolescence, but peak age of
Ankylosing Spondylitis diagnosis is between 20 and 30 years.
Ankylosing spondylitis is a chronic inflammatory disease Symptoms of inflammatory back pain are most frequently
affecting the axial skeleton, entheses, and peripheral joints. In the presenting complaint. Spinal involvement begins in the
contrast to other forms of spondyloarthritis, sacroiliac joint lumbar region and tends to ascend. Buttock pain may indicate
involvement is considered an essential feature of ankylosing sacroiliac joint involvement. which is characteristicaJ.Jy bilateral.
spondylitis. Ankyloslng spondylitis has a strong famtlfal predi- About one third of patients develop hJp joint Involvement and
lection and the strongest association with HLA-827 among the about one third develop peripheral arthritis elsewhere, often In
forms of spondyloarthritis, being present in up to 95'){, of the shoulders. Enthesltis most often manifests as Achilles tend-
patients. Although a male predominance of 2-3:1 has been initis. Dactylitis occurs in fewer than 10% of patients.

30
Spo nd yloarthritls

The course of ankylosing spondylitis is variable; progres-


sion may occur over many years. Markers of a poorer progno-
sis include a longer duratton of disease. the presence of hip
joint involvement. elevation ofinflammatory markers, and the
presence ot' radiographic changes in the spine. Mortality is
increased in men and correlates with disease activity.

Psoriatic Arthritis
Psoriatic arthritis is an inflammatory joint disease associated
with psoriasis. Estimates of the frequency of psoriatic arthri-
tis among patients with psoriasis vary from 7% to 42%. Men
and women are equally affected, and the usual age of onset is
between 30 to so years. Psoriasis precedes or co-occurs with
arthritis symptoms in about 90% of patients with psoriatic
arthritis; less commonly, the rash may follow the arthritis,
with a latency period as long as a decade. Skin involvement
may be subtle, involving only areas such as the umbilicus,
f 1GURE 1 0 . Marked thickening of the distal right Achilles tendon and Achilles
perineum, or gluteal cleft. There is no correlation between
insertion on the Cillcaneus as the result of chronic Achilles tendinitis In a patient
with psoriatic arthritis (normal contralateral Achilles insertion). the extent of skln involvement and the severity of joint symp-
toms. ln addition to skin lesions, nail pitting and onychodys-
trophy are seen; nail involvement is a risk factor for develop-
ing joint disease, particularly of the distal interphalangeal
joints (Figwc 12).
Five clinical subtypes of psoriatic arthritis, which may
overlap, are recognized as follows: symmetric polyarthritis;
asymmetric oligoarthritis; distal interphalangeal- predomi-
nant disease; spondyloarthrltis; and arthritis mutilans.
Arthritis mutilans represents the rare end stage of progressive.
destructive arthritis in the small joints of the hands. It results
in subluxation, ligamentous laxity, and telescope-like retrac-
tion of the fingers. Additional joints usually become involved
overtime.
Enthesitis, tenosynovitis, and dactylitis often occur.
Fl GURE 11 . Diffuse swelling oft he first and second toes oft he left foot due to Common locations for enthesltls include the Achilles tendon
dactylitis in a patient with psoriatic arthritis resulting in a •sausage digit"
appearance. (see Figure 10), the calcaneal insertion of the plantar fascia,
and ligamentous insertions into the pelvic bones. Dactylitis
typically involves one or two digits; the feet are more com-
Physical examination, particularly In more advanced monly affected than the hands (see Figure 11).
disease, can be highly characteristic. With active inflamma- Symptomatic and radiographic spine involvement is unu-
tion, tenderness over the sacroiliac joints is often the first sual at disease onset but increased over long-term follow-up,
sign of disease. Over time, marked reduction in spinal
mobility can occur in all planes, and chest expansion
becomes reduced. Flexion deformity at the neck and tho-
racic hyperkyphosts may lead to a hunched posture. A loss of
lumbar lordosis and restricted range of motion in the hips
may also be evident.
Extra-articular manifestations of ankylosing spondylitis
CJ are not uncommon (see Table 17). Cardiac and pulmonary
manifestations. Including aortic valve disease, cardiac conduc-
tion abnormalities. and restrictive lung disease, are more com-
mon in ankylosing spondylitis than other forms of spondy-
loarthritis. In addition, vertebral compression fractures may
occur with little to no trauma as the disease progresses. The fiGURE 1 Z. Dystrophic nail changes, including onycholysis and pitting, and
cervical spine Is the most common site for fractures and can be inflammatory arthritic changes in the distal interphalangeal joints in a patient with
associated with neurologic complications. Cl psoriasis and psoriatic arthritis.

31
Spondyloarthritis

particularly in the cervical spine. The presence ofsacroillitis is monoarticular or oligoarticular: joints commonly affected
associated with an increased likelihood for HLA-827 positivity. include the knee, ankle, and Wrist Enthesitis is especially com-
In contrast to the almost universally bilateral sacrollittis of mon; Achllles tendinitis and plantar fasciitiS occur in up to 90%
ankylosing spondylitis, sacroiliitis iS not always present in of cases. Dactylitis can also occur. The spine and sacroiliac
psoriatic arthritis and is more likely to be unilateral. Extra- joints are less commonly involved. In about half of patients.
articular manifestations are listed in Table 17. symptoms self-resolve within 6 months; most other cases self-
Poor prognostic factors in psoriatic arthritis Include the resolve within 1 year, with a small proportion of cases convert-
presence of polyarticular or erosive disease at the time of diag- ing to chronic disease. Extra-articular manifestations are
nosis. Patients additionally have an increased risk for cardio- described in Table 17. Asubset of patients may demonstrate the
vascular disease. "complete triad" of conjunctivitis, arthritis, and urethritis.
KEY POINTS
Inflammatory Bowel Disease-Associated Arthritis
• Ankylosing spondylitis is a chronic inflammatory dis-
Inflammatory bowel disease (IBD). including Crohn disease
and ulcerative colitis, may be associated with inflammatory ease affecting the axial skeleton (including sacroiliac
joints). entheses. and peripheral joints; HLA-827 is pre-
arthritis in 6% to 46% of patients. Onset of anhritls may occur
sent in up to 95% of patients.
at any time in the course of the bowel disease.
IBD-assoclated arU1ritis occurs in three patterns: • Psoriatic arthritis is an inflammatory joint disease asso-
ciated with psoriasis, multiple possible joint patterns,
1. SacroUUtls/spondyUtis: occurs in up to 26% of patients; SO% enthesitis, tenosynovitis, and dactylitis.
to 75% are positive for HLA-827; men are more frequently
affected than women; radiographic abnormalities of the • Inflammatory bowel disease-associated arthritis occurs
in three patterns: sacroillitis/spondylitis, acute polyar-
sacroiliac joints may be seen in the absence of symptoms.
ticular peripheral arthritis, and chronic polyarticular
2. Acute polyarticular peripheral arthritis: self-limited; af- peripheral arthritis; only the acute polyarticular form is
fects 5% of patients and occurs early in the course of IBD; related to flares of bowel disease.
the presence of arthritis is not as highly associated with
• Reactive arthritis can occur 2 to 3 weeks following spe-
HLA-827 as in axial disease; the knee is most frequently in-
cific gastrointestinal and genitourinary infections; an
volved; flares ofjotnt symptoms may parallel exacerbatlons
asymmetric monoartlcular or ollgoarticular arthritis.
of bowel disease, but 90% resolve within 6 months.
dactylitis. and enthesitis can occur.
3. Chronic polyarticular peripheral arthritis: affects less than
5% of patients; men and women are equally affected; the
presence of arthritis is not as highly associated with HLA-
827 as in axial disease; the metacarpophalangeal joints,
Diagnosis
knees. ankles, elboM, shoulders, wrists, proXimal inter- Laboratory Studies
phalangeal. and metatarsophalangeal joints may be affect- Spondyloarthrltis is considered seronegative because rheuma-
ed; flares of arthritis may recur repeatedly over years and toid factor and other autoantibodies are typically absent. HLA-
last for months. but joint symptoms are unrelated to bowel 827 is present In many patients, particularly those with anky-
disease activity. losing spondylitis and spinal involvement; however, it cannot
independently confirm or exclude a diagnosis. With active
See Table 17 for information on extra-articular
disease, evidence of systemic inflammation may be diagnosti-
manifestations.
cally helpful, including elevations of e.rythrocyte sedimenta-
tion rate and C-reactive protein as well as normochromic,
Reactive Arthritis normocytic anemia. Each of these findings, however, is non-
Reactive arthritis Is a rare cause ofinflammatory arthritis that specific and may be normal even in the presence of active
can occur following specific genitourinary and gastrointestinal disease. Several other laboratory findings are suggestive of the
infections, including Chlamydia trachoma tis and Ureaplasma underlying disorder but are of limited utility diagnostically.
urealyticum in the urethra and Campylobacter, Escherichia Elevated lgA levels have been reported in patients with anky-
coli, Salmonella, Shigella, and Yerslnla in the intestine. losing spondylitis and psoriatic arthritis, consistent with
Reactive arthritis occurs following 2'X, to 33% of cases ofbacte- increased mucosal immune activity. Serum alkaline phos-
rial dysentery, with a higher incidence reported after single phatase may be elevated In anl<ylosing spondylitis. Serum
source outbreaks. About 90% of Yersinio-related cases occur In urate levels may be high in psoriatic arthritis, and these
patients who are HLA-827 positive, whereas only 30'X, to SO% patients are at increased risk for co-occurrence of gout.
of patients with reactive arthritis associated with other infec- Synovial fluid may show an elevated leukocyte count with a
tious agents are HLA-827 positive. predominance of neutrophils.
Symptoms of inflammatory arthritis typically appear 2 to In cases of suspected reacttve arthritis, stool and urine
3 weeks postlnfection. Arthritis is asymmetric and may be cultures, genital swabs, nucleic actd ampllfl.catlon testing, and

32
Spondyloarthritis

rising serum antibody titers against the suspected causative


organism can be helpful in establishing the diagnosis.
Pathogens can generally not be cultured from joint fluid In
these patients, and antecedent infection is verified in less than
half of affected individuals.

Imaging Studies
Radiography of the sacroiliac joints is an essential part of the
initial evaluation of patients being assessed for spondyloar-
thriUs but may be normal early In the course of disease.
Radiographic evidence of sacroilliti<> Includes pseudo-widen-
ing of the joints, erosions, sclerosis, and ankylosis. In the
spine, bony proUferation between vertebral bodies can result
in formation of syndesmophytes (bony bridges) that can lead
to a "bamboo spine" appearance ln 10% to 15% of affected
patients with ankylosing spondylitis (Figure 13). Other changes
in the spine include vertebral squaring, disk calclfication, and
vertebral and facet joint ankylosis. ln psoriatic arthritis, the
syndesmophytes are often described as less delicate, more
~chunky, • patchy, and asymmetric compared With those asso-
ciated with ankyJosing spondylitis FIGURE 14. Radiograph showing pencif·ln·cup deformrty of the fihh
Plain radiography of peripheral joints may aid in the diag- met•r.rsal joint and ankylosis of the fourth metatarsal joint in a patient with
nosis of spondyloarthtitis. particularly when erosive and pro- psoriatic arthritiS.
Uferatlve changes are present concurrently. Bony proliferative
changes at entheseal sites may be seen In any spondyloarthri- recognized as a useful diagnostic and prognostic modality. MRI
tis. Features parrtcu.larly characteristic of psoriatic arthrltls is more sensitive for detecting early spine and sacroiliac joint
Include asymmetric distribution, distal interphalangeal joint Inflammation and may be lndicate<iln the evaluation of sus-
involvement, osteolysis leading to pencil-in-cup deformity pected spondyloarthritis if plain radiographs are negative. MRI
(Flgure 14), and proliferative new bone formation along the can also detect inflammatory changes even in the absence of
shaft of metacarpal or metatarsal bones. There are no charnc- bony lesions. For example, the presence of bone marrow
lerlstlc findings of reactive arthritis on plain radiographs. edema, although nonspecific, can suggest active inflammation
Although plain radiography remains the cornerstone of in the sacroiliac joints. MRI can also detect soft-tissue abnor-
radiographic diagnosis in spondyloarthrltis, MRI is increasingly malities (such as bursitis and enthesitis), erosions. sclerosis,
and ankylosis.
In older patients, the diagnostic specificity of radiographic
evaluation for spondyloarthrltis may decline. Other conditions
such as osteitis condensans nu.
osteoarthritis, degenerative
disk disease, and diffuse idiopathlc skeletal hyperostosis
(DISH) may cause sclerotic changes of the sacroiliac joints and
osteophytes that may be difficult to distinguish from the syn-
desmophytes of ankylosing spondylitis. In particu.lar, DISH
typically causes multilevel bridging osteophytes. In ankylosing
spondylitis, however, the bony bridges tend to be thinner and
more vertically oriented than those seen In DISH.
KEY POINTS
• In patients with spondyloarthritis, rheumatoid factor HVC
and other autoantibodies are typically absent; HLA-B27
is positive in many patients, but it cannot indepen-
fIG URl 13 . Spinal features of ankylosing spondylitis. left. Sacroiliac erosive dently confirm or exclude a diagnosis.
disease, charKterited by areas of joint·space natrowing in some locations, app<~rent
• Markers ofsystemic inflammation such as erythrocyte
relative widening in others, and secondary bony sderosis.AJthough the left saaoifiac
joint Is more severely involved, both sides are affected. Righ~ Oeli<ate (gracile) sedimentation rate and C-reactive protein may be nor-
syndesmophytes bridging lumbar vertebral bodies. feaclmg to~ "bamboo spine" mal even in the presence of active spondyloarthrltls.
appearance. Note that the syndesmophytes are "flowmg" (contiguous wrlhout
(Continued)
skipping any vertebBI joints~

33
Spondyloarthrit is

k £ Y P 0 I NT S (continued} Total hip arthroplasty can be highly effective in reducing


pain and restoring function in the management of progressive
• Radiography of the sacroiliac joints is essential in
hlp Involvement and may be Indicated at an earlier age than in
patients with suspected spondyloarthritis; radiographic
patients with osteoarthritis.
evidence of sacroiliitis mcludes pseudo-widening of the
joints. erosions. sclerosis, and ankylosis.
Psoriatic Arthritis
HVC • MRI is more sensitive for detecting early spine and sac· Treatment choices in psoriatic arthtitis take into account the
roiliac joint inflammation and may be indicated in the extent of both skin and joint Involvement. NSAlDs may be
evaluation of suspected spondyloarthritis if plain radio- effective for mild joint disease. Methotrexate is the most com-
graphs are negative. monly used DMARD for psoriatic arthritis. Although benefit
• Radiographic changes in psoriatic arthritis can Include can be demonstrated in control of skin disease and joint pain,
asymmetric distribution, distaJ interphalangeal joint methotrexate has not been shown to reduce progression of
involvement, osteolysis leading to a pencil-m-cup joint damage. Sulfasalazine and leflunomide can also improve
deformity, and proliferative new bone formation along joint symptoms. Apremilast is modestly effective for psoriasis
the shaft of metacarpal or metatarsal bones. and psoriatic arthritis.
In more severe psoriatic arthritis, TNF-a inhibitors have
been shown to have superior efficacy in the management of
joint symptoms and to slow the progression of radiographic
Management damage, Including joint-space narrowing and erosions.
General Considerations The biologic agents ustekinumab (anti-IL-U/23 antibody)
Management goals in spondyloarthritis are to control pain and and secukinumab (anti-IL-l7A antibody) have also been
inflammation, preserve function, and prevent progressive approved for treating both psoriasis and psoriatic arthritis,
structural damage, including spine and joint ankyloses and can lmprove daczytitls and enthesltls.
(fusion). Patient education regarding the course, treatment,
and prognosis of disease. as well as the importance of general Inflammatory Bowel Disease-Associated Arthritis
health, avoiding smoking, and participating In regular exercise Various phannacologic agents may be useful in the treatment
are critical components of the overall treabnent program. of both intestinal and peripheral arthritis related to Crohn
disease and ulcerative colitis, Including sulfasalaztne, azathio-
An kylosing Spondylitis prine, 6-mercaptopurine, methotrexate, glucocorticoids, and
NSAIDs are recommended as first-line treatment in ankylos- the TNF-a lnhJbltors infllxlmab and adalimumab. NSAIDs can
ing spondylitis and remain important in management provide symptomatic relief of arthritis symptoms but may
throughout the course of disease. Patients with ankylosing occasionally exacerbate IBD.
spondylitis are more likely to respond to NSAlDs and to do so
more rapidly and completely than patients with chronic low Reactive Arthritis
back pain from other causes. Several studies suggest that, in Treatment of the antecedent infection Is indicated in patie nts
contrast to most forms of arthritis. continuous use of NSAlDs with reactive arth ritis who have an Identifiable cause.
may help slow disease progression In ankylosing spondylitis. However, most patients present postJnfecUon, and antibiot-
Oral glucocortlcoids are not recommended. but intra-articu- ics are not usually effective in treating the arthritis. Most
lar glucocorticoid injections, including in the sacroiliac joints, patients have self-limited disease, and short- tenn use of
can help alleviate pain. Nonbiologlc disease- modifying anti- dally NSA!Ds often improves symptoms until the condition
rheurru~tlc drugs {DMARDs) such as methotrexate have no
resolves. If relief is incomplete, intra-articular glucocorti-
efficacy in axial disease and limited efficacy In peripheral coid Injections and oraJ glucocorticolds can be used. If
disease. symptoms persist beyond 3 to 6 months, the use ofDMARDs
ln patients with persistent symptoms on NSAID therapy, such as sulfasalazine, methotrexate, or TNF-a inhibitors may
TNF-a inhibitors have efficacy in reducing signs and symp - be necessary for symptom control and to prevent joint ero-
toms of ankylosing spondylitis, wllh about half ofthose treated sion. Therapy is discontinued 3 to 6 months following dis-
improving by SO%. Whether TN F-a Inhibitors can slow radio- ease re mission.
graphic progression in ankylosing spondylitis remains an area
of ongoing investigation. These agents are also effective in KEY POINrS
treating ankyloslng spondylitis-associated anterior uveitis. • In ankylosing spondylitiS, NSAlDs are recommended as HVC
Physical therapy is the most important nonphannaco- first- line treatment and may help slow disease progres-
logic intervention in ankyloslng spondylitis. The goals of ther- sion; physical therapy is the most Important nonphar-
apy include improving pain and stiffness, maintaining range macologic intervention.
of motion, and reducing disability; transition to an ongoing (Continued)
dally exercise program is optimal.

34
Sy st emic Lupus Ery thematosus

KEY POl NT S (continued} Clinical Manifestations


• In more severe psoriatic arthritis. tumor necrosis factor Mucocutaneous Involvement
<X Inhibitors have been shown to have superior efficacy Skin disease occurs in up to 90'X, patients with SLE and is clas-
in the management of joint symptoms and to slow the sified as acute, subacute, or chronic.
progression of radiographic damage. Acute cutaneous lupus erythematosus (ACLE) presents as
• Treatment options for both Intestinal and peripheral an erythematous, macular, patchy eruption, sometimes with
arthritis related to Crohn disease and ulcerative colitis desquamation. The facial eruption of ACLE (malar or buttertly
include sulfasalazine. azathioprine. 6 mercaptopurine. rash) is characterized by erythema/edema over the cheeks and
methotrexate, glucocorticoids, mfliximab. and adali- bridge of the nose, sparing the nasolabial folds; it occurs in
mumab about SO% of patients with SLE (Figu re 15). Less characteristi-
HVC • AntibiOtics are not usually effective in treating cally, ACLE can also involve the neck, upper chest, and dorsum
reactive arthritis, but shorr term NSAII)s can be used of the arms and hands; it affects the skin between the fingers
to improve symptoms in this typically self-limited but spares the knuckle pads. In some patients, a bullous erup-
disease. tion can occur. ACLE usually responds to therapy and heals
without scarring or atrophy.
Subacute cutaneous lupus erythematosus (SCLE) is a
photosensitive rash occurring especially on the arms, neck,
Systemic Lupus and face (Figure 16). It consists oferythematous annular/poly-
cyclic or patchy papulosquamous lesions, often with a fine
Erythematosus scale, that may leave postinflammatory hypo- or hyperpig-
mentation. SCLE is associated with anti-Ro/SSA autoantibod-
Epidemiology and Ies (prevalence >7S'J(,) and can occur in isolarion or as a
Pathophysiology manifestation of underlying SLE.
Systemic lupus erythematosus (SLE) is a multisystem autoim- Discoid lupus erythematosus (OLE) is the most common
mune disease characterized by a heterogeneous constellation chronic cutaneous manifestation of SLE, occurring in 20% of
of organ involvement and the presence of antinuclear antibod- patients (Figure 17). DtE presents as hypo- or hyperplgmented
ies (ANA) and other autoantibodies. patches or plaques. with erythema during active disease.
In SLE, a complex and varying lnteraction of genes, which may be variably atrophic or hyperkeratotic. Unlike
enVIronment, and random events leads to a breakdown of A.CLE and SCLE, OLE can cause scarring, atrophy, and perma-
self-tolerance and autoimmunity. Defects In cellular apopto- nent alopecia. OLE also occurs as an Isolated finding in the
sls result In inadequate clearance of intracellular proteins, absence of SLE. Isolated OLE is usually limited to the neck,
especially nuclear antigens, promoting the generation of face, and scalp, whereas diScoid lesions in SLE are more dif-
self-directed T and B cells and t he initiation/propagation of fusely distributed. Patients with isolated OLE tend to be ANA
autoimmunity. Cytokine generation supports the autoreac- negative and usually do not progress to SLE. It is important to
tlvlty, with type- t i nterferons playing a major role. differentiate isolated DLE from DLE as a manifestation of
Autoantibodies may di rectly Induce tissue damage or pro-
mote the formation of immune complexes that lead to com-
plement activation and tissue inflammation and damage.
Inheritance of SLE risk is polygenic, including major histo-
compatibillty complex.
The risk of SLE developing in genetically predisposed
individuals increases at puberty and peaks in the third decade.
Approximately 90'X, of adult patients are women. The disease
is more common, and perhaps more severe, In black, Asian,
and Hispanic ethnidties.
KE't' POINTS
• Systemic lupus erythematosus is a multisystem autoim-
mune disease characterized by a heterogeneous constel-
lation of organ involvement and the presence of antinu-
clear antibodles and other autoantibodies.
• Approximately 90'X. of adult patients with systemic fIG Ua E 1 5. The faci~l eruption of acute curaneous lupus erythematosus
(malar or butterfly rash~ This patient has fixed erythematous raised lesions over the
lupus erythematosus are women.
malar eminences, the bridge of Ihe nose wi1h sparing ol the nasolabiatfolds. and
the chin.

35
Systemic Lupus Erythematosus

B
fIG U RE 16. Subatute cutaneous lupus erythematosus ischaracterized by erythematous. macular, or patchy skin lesions that are scaly
and can evolve as (A) annularlpolycydic lesions or (8) papulosquamous plaques.

large joints are also affec ted. In contrast to rheumatoid arthri-


tis, SLE arthritis is nonerosive, but reducible subluxation of
the digits, swan neck deformities, and ulnar deviation (Jaccoud
arthropathy) can occur.
A serious complication of SLE is osteonecrosis, which
most commonly affects the hips but can also involve other
large joints, and should be suspected when there is otherwise
unexplained pain and/or reduced range of motion. Chronic
prednisone doses {>20 mg/d), severe/active SLE, and vasculitis
are all associated With increased risk. MRI is the modality of
choice for sensttive evaluation of early disease. with plain radi-
ography useful to diagnose and follow later stages. Small
lesions can improve and resolve spontaneously, but larger
lesions usually lead to bony collapse and structural sequelae.
FIGURE 17. Discoid lupus erythematosus. This patient has hyperpigmented, Myalgia and subjective weakness are common; frank
raised patches with keratotic scaling and follicular plugging involving the malar
and perioral areas as well as the bridge of the nose.Areas of atrophic scarring are
myosltls ts rare. Assessment is complicated by the potential
also present. effect on muscle of antimalarials (rarely) and glucocorticoids
(commonly), which must be dlfferentlated from active SLE
underlying SLE when making therapeutic and prognostic disease. Fibromyalgla Is a common comorbid1ty (30%); symp-
decisions. toms may be similar to active SLE disease.
Painless oral or nasopharyngeal ulcerations occur In S% of
patients with SLE, with involvement of hard palate suggestive Kidney Involvement C]
of the diagnosis. Rarely, DLE can be associated with painful KJdney disease occurs frequently among patients with SLE
ulcers. Nonscarring alopecia is a common feature of active (70%) and was the major cause of SLE mortality prior to the
SLE, With hair regrowth a sign of disease control. Raynaud advent of dialysis. Lupus nephritis can present with minimal
phenomenon occurs frequently, reflecting arterial vasospasm laboratory abnormalities (non-nephrotic proteinuria. hema-
of digital arteries. turia), frank nephritis (hypertension, lower extremity edema.
See MKSAP 18 Dermatology for more information. active urine sediment. and elevated serum creatinine), and/or
nephrosis (nephrotic-range proteinuria, dependent edema.
Musculoskeletal Involvement and thrombosis). Untreated active disease may progress to
Joints are affected in 90'%of patients with SLE. The most com- kidney failure.
mon Involvement is polyarthralgia, with frank arthritis occur- All patients with SLE should be regularly evaluated for
ring ln 40'X,. Typical dlstributlon is small peripheral joints, but kidney disease by assessing serum creatinine and urine for

36
Sy st emic: Lupus Eryt hematosus

protein and microscopic evaJuation. Patients with significant occurs in S% to lO'X, of patients with SLE and usually presents
CJ orabnonnalitles,lncluding
CONT.
proteinuria greater than soo mg/24 h
active urine sediment. should be urgently evaluated for
as Insidious heart failure but can be acute. CJ

active disease. Anti-double-stranded DNA antibody titers are Pulmonary Involvement


a marker for risk, and complement consumption is a common Pulmonary involvement is common in SLE, with most patients
phenomenon during active kidney disease. presenting with pleuritis (4S%-60'X.). Pleural effusions occur
Kidney biopsy defines both the histological subtype and in approximately half of these patients and are typically exu-
the activity/chronicity of disease and is usually essential to dative; fluid analysis may reveal a lymphocytic pleocytosis and
make therapeutic decisions. Indications for kidney biopsy mildly depressed gLucose levels.
include an otherwise unexpLained rise in serum creatinine, Parenchymal lung involvement occurs in less than 10% of C]
proteinuria greater than 1000 mg/24 h, proteinuria greater patients with SLE. A nonspecific interstitial pneumonia pat-
than 500 mg/24 h with hematuria, or an active urine sediment. tern is most common. and evaluation centers on assessing SLE
Patients who have SLE with hypercoaguLable states (for activity and excluding other causes of diffuse parenchymal
example. nephrosis or anti phospholipid syndrome) may be at lung disease. Two rare but potentially life-threatening compli-
risk for renal artery or vein thrombosis. cations of SLE lung disease are acute lupus pneumonitis (pre-
See MKSAP 18 Nephrology for Information on the classes senting as fever, cough, dyspnea, hypoxemia, pleuritic chest
and treatment of lupus nephritis. CJ pain, and infiltrates) and diffuse alveolar hemorrhage (pre-
senting with dyspnea, hypoxemia, diffuse alveolar infiltrates,
Neuropsychiatric Involvement a dropping hematocrit, and a high Dtco). Both carry a high
Neuropsychiatric systemic lupus erythematosus (NPSLE) may mortality rate (>50'%.); early recognition, rapid evaluation (Cf
involve the central and /or peripheral nervous systems and has and/or bronchoscopy with bronchoalveolar lavage or biopsy).
19 defined manifestations. NPSLE prevalence is high (75%), and aggressive respiratory support combined with high-dose
with the most common manifestations being headache, mild glucocorticoids and immunosuppression are required. With
cognitive dysfunction, and mood disorder. Peripheral neu- new pulmonary lnfiltrates. differentiation between these dis-
ropathy occurs in 10% to 14% of patients. Severe acute presen- orders and infection can be difficult. and antibiotics and
tations, including seizures and psychosis, happen infrequently immunosuppressive therapy are often administered simulta-
(<5%) but require aggressive symptomatic as well as disease- neously until the diagnosis Is clear. CJ
specific treatment. Shrinking lung syndrome is a rare but characteristic syn-
Patients wtth suspected serious centtal NPSLE such as drome consistlng of pleuritic chest pain and dyspnea, with
Cl meningitis, stroke, and psychosis should undergo central progressive decrease in lung volumes. The cause is uncertain,
nervous system imaging (cr. MRl, or PET) and cerebrospinal but pleuropulmonary disease and/or diaphragmatic dysfunc-
fluid analysis as appropriate. In some patients with severe tion may contribute. Immunosuppression may reverse the
disease, measurement of cerebrospinal fluid for NPSLE- process in some patients.
associated autoantibodies (anti neuronal. anU-N-methyl-o-as-
partate receptor, antiribosomal P, and others) may be useful. Hematologic Involvement
In patients with SLE. a normocytic. normochromic Inflamma-
c:J
For patients with suspected peripheral neuropathies. electro-
myography and nerve conduction studies should be per- tory anemia is common; autoimmune hemolytic anemia
fanned. Neuropsychologic testing may help distinguish occurs in approximately 10% and correlates with SLE activity.
organic versus functional cognitive changes. lymphopenia/leukopenia is also common but usually mild.
Thrombocytopenia occurs fn 30% to SO%, and approximately
Cardiovascular Involvement 10%of patients develop severe thrombocytopenia (<SO ,000/~tL
Asymptomatic pericarditis is the most frequent cardiac mani- [SO x 109/L]) in isolation or In conjunction with hemolytic
festation of SLE (40%). When symptomatic, features include anemia.
chest pain, exudative effusion, and ra rely tamponade or The cytopenias in SLE may be caused by Immune and
chronic constriction. Patients with SLE have a 2· to 10-fold nonimmune destructive mechanisms (including microangi-
increased prevaJence of coronary artery disease (CAD), the opathy), medications. and kidney and liver disease. Moderate
most common cause of death among older patients. High SLE :and severe or rapidly progressive cytopenias require prompt
disease activity and prednisone doses greater than 10 mg/d are evaluation with serologic studies and/or bone marrow biopsy.
independent risk factors for future CAD. An exact cause of cytopenias may be difficult to ascertain, and
ValvuJar abnormalities occurring in SLE include those a trial of medication adjustment In concert with evaluation for
associated with antiphospholipid syndrome (nonspecific other causes Is often necessary.
thickening of the mitral and aortic valve leaflets. vegetations, Anti phospholipid antibodies/lupus anticoagulant (APlA/
regurgitation, and stenosis). Ubman-Sacksendocarditis (non- LAC) are frequently present in patients with SLE (about 40%)
infectious verrucous vegetations) preferentially affects the and may be associated with a false-positive rapid plasma
mitral valve and can cause embolic complications. Myocarditis reagin test for syphilis. Most patients are asymptomatic.

37
Sys t emic Lupus Erythematosu s

. . , Thrombotic events occur in about 30% and are associated with activation and/or medications (azathioprine or cyclophospha-
1.1.1 moderate or high titer of the antibodies; these include venous mide). Other hematologic malignancies, including Hodgkin
CONt and arterial thrombosis, miscarriage, stiUbirth, livedo reticu- lymphoma and leukemia, are also increased. Lung cancer rates
laris, and cardiac valve thickening/vegetations. The highest are increased slightly compared with the general population,
risk of thrombosis occurs in the presence of triple positivity probably related to smoking. Cervical cancer risk is increased,
for LAC. anti-1}2 -glycoproteln I, and anticardiolipin antibodies. likely due to immunosuppression and increased prevalence of
Patients with SLE are at increased risk for thrombotic events human papillomavirus.
even In the absence of APLA.
KEY POINT
See MKSAP 18 Hematology and Oncology for more infor-
mation. CJ • The greatest malignancy nsk among pattems with sys-
temic lupus erythemarosus is non- Hodgkin lymphoma.
Gastro inte stinal Involve ment
Gastrointestinal disease is a common (40%) and frequently Diagnosis
underrecognized SLE manifestation. Serositis presents as
abdominal pain, is usually associated with active disease, and
General Considerations
improves with treatment. Mesenteric vasculitis, inflammation The diagnosis of SLE should be considered in any patient with
of the small and large bowel, pancreatitis, protein-losing unexplained symptoms affecting multiple organ systems or
enteropathy, and diffuse peritonitis are uncommon but may be with any Individual manifestation of SLE, especially In young
severe and associated with cutaneous vasculitis. Patients with women. At Initial presentation, skin and joint manJfestations
APLA can present with mesenteric thrombosis. are most common, along with constitutional symptoms (fever,
Noninfectious hepatitis can occur and is associated with the weight loss, or severe tatigue}. Patients with subjective com-
presence of antiribosomal P antibodies. Patients with SLE who plaints of fatigue, myalgia, and/or arthralgia, but lacking
have Raynaud phenomenon and anti-Ul-ribonucleoprotein objective findings, most likely have an alternative diagnosis
antibodies can develop esophageal disease and reflux. and should not be evaluated for SLE.
Medications used to treat SLE (NSAJDs, prednisone. Classification criteria for SLE were developed by the
mycophenolate, azathioprine) also frequently affect the gas- American College of Rheumatology (ACR) in 1982 and revised
trointestinal system and may cause esophagitiS, gastritis, pan- in 1997; although intended for accruing homogenous SLE
creatitis, and other manifestations. populations for research studies, they can also be used to sug-
gest a clinical diagnosis ofSLE (Table 18). fn 2012, the Systemic
KEY POINTS Lupus International Collaborating Clinics (SLTCC) proposed
• The facial eruption of acute cutaneous lupus erythema- and validated alternatiVe SLE classification criteria that are
tosus (malar or butterfly rash) is characterized by ery- similar to the ACR criteria but require the following: 1) fulfill-
thema/edema over the cheeks and bridge of the nose, ment of at Least four criteria, with at least one clinical criterion
spating the nasolabial folds. and one immunologic criterion; or 2) biopsy-proven lupus
• Polyarthralgfa of the small peripheral joints is the most nephritis as sufficient clinical criterion in the presence of ANA
common joint manifestation In systemic lupus erythe- or anti-double-stranded DNA antibodies. When compared
matosus; a nonerosJve arthritis occurs in 40% of patients. with the ACR classification criteria, the SLICC classification
criteria are associated with fewer mlsclassifications and have
• All patients with systemic lupus erythematosus should
greater sensitivity but less specificity.
be regularly evaluated for kidney disease by assessing
serum creatinine and urine for protein and microscopic
Laboratory Studies
evaluation.
Initial evaluation for SLE includes routine laboratory testing to
• The most common manifestations of neuropsychiatric establish organ-specific involvement, including complete
systemic lupus erythematosus are headache. mild cog- blood count, chemistry panel, and urinalysis with microscopy.
nitive dysfunction. and mood disorder. ANA should be obtained to screen for nuclear-directed
• Patients with systemic lupus erythematosus (SLE) have autoantibodies. The most appropriate methodology for testing
a 2- to 10-fold increased prevalence of coronary artery ANA is the indirect immunofluorescence assay, which 1s highly
disease (CAD): high SLE disease activity and prednisone sensitive (95~.) forSLE. ANA tests shou ld be interpreted in the
doses greater than 10 mgtd are independent risk factors context of the probability of disease because ANA may be pre-
for developing CAD. sent in other autoimmune diseases, and low-titer positivity
may be seen with aging and even in healthy individuals.
If ANA Is positive. SLE-speciftc autoantibodies (anti-
Association with Malignancy double-stranded DNA, anti-Smit h, anti-Ul-ribonucleopro-
The greatest malignancy risk among patients with SLE Is non- tein, anti-Ro/SSA, and anti-L..a/SSB), as well as tests for other
Hodgkin lymphoma. presumably due to chronic 8-cell autoimmune diseases under consideration, should be obtained

38
Systemic Lupus Erythematosus

Malar rash Fixed erythema, flat or raised, over the malar eminences
Discoid rash Erythematous, circular, raised patches with keratotic scaling and follicular plugging; atrophic scarring may occur
PhotosensitiVity Rash after exposure to ultraviolet light
Oral ulcers Oral and nasopharyngeal ulcers (observed by physician)
Arthntis Nonerosive arthritis of 2:2 peripheral joints, with tenderness, swelling. or effus1on
Serositis Sterile pleuritis, pericarditis, or peritonitis (documented by electrocardiogram, rub, or evidence of effusion or
ascites)
Kidney disorder Urinalys1s. 3+ protein or urine prote1n >500 mg/24 h; cellular casts
Neurologic disorder Seizures or psychosis (without other cause)
Hematologic disorder Hemolytic anemia or leukopenia (<4000/JJL [4.0 x 10~/LJ) or lymphopenia (<1500/J.I.L [1.5 >< 109/L]) or
thrombocytopenia ( <1 00,000/l!L I100 x 10 9/L]) in the absence of offending drugs
Immunologic disorder Anti-double-stranded DNA, anti-Smith, and/or antiphospholipid antibodies
ANA An abnormal titer of ANA by Immunofluorescence or an equivalent assay at any po•nt in the absence of drugs
known to 1nduceANA
ANA • antlnudear anttbodies.

•Any combination of 4 or more of the 11 criteria, well documented at any time dunng a patiet~rs history, makes'' l•k•ly that the parlent has systemic lupus erythematosus
(specilkity and oensitivity are 95% and 75%, respectively).

From Hochberg MC. Updating the Americatl College of Rheumatology revised criteria forth<> classification of sy>1emlc lupus erythematosus. Arthritis Rheum. 1997;40(9): 1725
(PMIO: 9324032) Copyright 1997,Arn&rkan College of Rheurn.rology. Adapted wrth permiuion from Jo~n Wiley & Sons,lne.
------------------------~

to further characterize the disease (Table 19). A small percent- Differential Diagnosis
age of patlenrs with SLE are negative for ANA but poslt1ve for The differential diagnosis of SLE includes multisystem dis-
antl-Ro/SSA antibodies. A negative ANA plus a negative anti- eases, acute and chronic infections, medication effect. malig-
Ro/SSA essentially rules out SLE. nancies (particularly hematologic), and neurologic diseases
Disease activity markers (complements C3 and C4) should
CJ be assessed initially and regularly thereafter. Complement lev-
(for example. multiple sclerosis). Multisystem autoimmune
diseases (ANCA-associated vasculitis, rheumatoid arthritis,
els are reduced during SLE activity, reflecting immune complex adult-onset Still disease, dermatomyositis. SjOgren syndrome.
formation and complement consumption. Anti- double- and mixed connective tissue disease) have overlapping fea-
stranded DNA antibody levels may rise with SLE kidney disease tures but maybe distinguished through a careful assessment
activity. Other SLE autoantibodies, Including the ANA test, do of their unique manifestations.
not reflect disease activity and need not be repeated. SLE should also be distinguished from undifferentiated
Erythrocyte sedimentation rate (ESR) and C-reactlve protein connectWe tissue disease, which presents with milder symp-
(CRP) are variably associated with disease activity; some toms and objective abnormalities that cannot be categorized or
patients with SLE do not generate CRP during SLE flares, which diagnosed as a specific connective tissue disease (see Mixed
may be helpful in distinguishing flares from infection. CJ Connective Tissue Disease}.

Antinuclear ;>95% Useful as an initial scceening test; assesses multiple antigens simultaneously
Anti-double-stranded DNA 50%-60% Found in more severe disease, especially kidney disease; antibody levels commonly
follow disease activity and are useful to monitor
Anti-Ro/SSA 30% Associated with photosensitive rashes, discoid lupus erythematosus, and neonatal
lupus erythematosus; also common when secondary Sjogren syndrome is present
Anti-U1 ·ribonucleoprotein 35% Associated with Raynaud phenomenon and esophageal dysmotility; also seen in MCTD
Ant1-Sm1th 30% Specific for SLE; often associated with more severe disease
Anti-La/SSB 20% Common in Sjogren syndrome; less common in SLE and neonatal lupus erythematosus
Anttribosomal P 15% Associated with CNS lupus and lupus hepatitis

CNS • ce1'111•1 neNous

39
Systemic lupus Erythematosus

Certain medications can cause drug-induced lupus erythe- II £Y P 0 I N TS (continued)


matosus (DILE), which mlmics SLE (Table 20). The syndrome is
• If antinuclear antibody testing is positive, autoantibod-
usually mild with symptoms of malaise, fever, arthritis. and ies specific to systemic lupus erythematosus (anti-
rash associated with a transient positive ANA and antihistone
double-stranded DNA, anti-Smith. anti-Ul -ribonucleo-
antibodies. Symptoms resolve after discontinuing the offending protein, anti- Ro/SSA, and anti-La/SSB) should be
agent. Kidney and central nervous system disease are uncom-
obtained to further characterize the disease.
mon. Patients with SLE are at no more risk of DILE than the
general population, and there is no contraindication to using
medications associated with DILE in patients with SLE.
Management
KEY POINTS SLE management requires close monitoring of disease activity
• Patients with systemic lupus erythematosus typically and frequent adjustment of therapy. Pharmacologic therapy is
initially present with skin and joint manifestations, almost always required and is usually directed toward specific
along with fever, weight loss. or severe fatigue. organ involvement (Table 21).
• Initial evaluation for systemic lupus erythematosus Hydroxychloroquine should be Initiated in every patient
includes antinuclear antibody testing as well as routine who can tolerate Jt, because evidence suggests it reduces dis-
laboratory testing to establish organ-specific involve- ease-associated damage, prevents disease flares, and improves
ment, including complete blood count, chemistry kidney and overall survival. Hydroxychloroquine can be used
panel, and urinalysis with microscopy. alone for mild disease (especially sl<in and joints) and In com-
bination with other agents in severe disease. Tn addition,
(Continued)
hydroxychloroquine may reduce the risk of thrombosis, liver

TABLE 20. Medications Commonly Associated woth Drug· Induced Lupus Erythematosus
Medication Antibodies Detected Comments
Procainamide ANA(75%); antihistone 20% develop DILE; fever; arthritis; serositis
Hydralazine ANA (20%); anti histone !:i%-8% develop DILE; fever; arthritis; rare vasculitis and kidney disease
Minocycline ANA; ANCA; anti-dsDNA rare Arthritis; vasculitis; autoommune hepatitis
Antithyroid drugs ANA; ANCA; antihistone Vasculitic rash; rare pulmonary and kidney disease
Statins ANA; anti histone; anti-dsDNA SLE, SCLE, dermatomyositis, and polymyositis all reported
Calcium channel blockers ANA; anti-Ro/SSA; antihistone rare SCLE
Thiazide diuretics ANA; anti-Ro/SSA; antihistone rare SCLE
ACE inhibitors ANA; anti-Ro/SSA; antihistone rare SCLE
TNF-a inhibitors ANA{23%-57%); chromatin and DILE most common with infliximab, uncommon for etanercept;
anti-dsDNA common; antihistone rare SLE, SCLE, OLE all reported
ANA • antinucle..- ontobO<fw.s; DILE • drug·•nduced lupus erythom<otosus: OLE~ discoid lupus erythemotosus; dsDNA • doubl~·strMded ONA; SCLE • subacute cutaneous
lupus ef)'thematosus; SLE- systemic: lupus eryt"emato5vs; TNF • tumor necrosis factor.

TABLE 21. Medications Commonly Used to Treat Systemic lupus Erythematosus


Medication Common Us" In SL£ Important Side Effects
NSAIDs Arthritis; pain; fever Hypertension; Gl bleed•ng; AKI
Prednisone Used for all manifestations in varying doses Hypertension; glucose intolerance; weight gain;
infection; osteonecrosis
Hydroxychloroquine Used in all patients; especially useful for skin Gl intolerance; rash; blurry vision; retinopathy;
involvement and to prevent disease flares vacuolar myopathy
Mycophenolate mofetil Moderate to severe d isease; as effective as Bone marrow suppression; elevation of liver
cyclophosphamide for remission induction for nephritis enzymes; infection
Azathioprine Moderate to severe disease Bone marrow suppression; elevation of liver
enzymes; hematologic malignancy
Cyclophosphamide Severe organ or life-threatening disease Bone marrow suppression; hemorrhagic cystitis;
infection; malignancy; infertility
Belimumab Add-on therapy for moderate to severe disease Infusion reactions; infections
AJ<I • acute kidney injury; Gl • gostrointestinal; SLE • oyotemic lvpu• erythematosus.

40
Syst emic l upus Erythematosus

disease, and myocardial infarction: improve lipid profites; and disease, nephritis. or antl-Ro/SSA and/or APLA antibodies.
improve outcomes in high-risk pregnancies. The best time to consider pregnancy is when SLE is quiescent,
Glucocortlcoids are a mainstay of SLE management, par- and conception should be considered only after at least 6
Cl ticularly in acute disease. The glucocorticoid dose should be months of adequate disease control.
determined by the level of disease activity and organ systems Proteinuria may increase during pregnancy in patients
threatened. For severe disease activity (Including profound with SLE, making distinction between Sl.E and preeclampsia/
cytopenias. class lll/IV nephritis. and NPSLE). high-dose glu- eclampsia a challenge. lncreases in anti-double-stranded DNA
cocorticoids are recommended. For life- or organ-threatening antibody levels, decreasing complement levels, or the develop-
disease (such as rapidly progressive glomerulonephritis or ment of active urine sediment suggests SLE as the cause. In
seizures), high-dose Intravenous glucocorticoids are given fol- contrast. serum urate levels are increased in preeclampsia but
lowed by high-dose daily prednisone. After disease stability is not durtng SLE tlares.
achieved, glucocorticolds are tapered to the lowest effective Fetuses of women who have anti-Ro/SSA or anti-La/SSB
dose, Ideally to eventual discontinuation. antibodies are at nsk for neonatal lupus erythematosus, which
Immunosuppressive therapy Is usually Initiated concur- is characterized by rash and congenital heart block (CHB}.
rently with glucocorticoids to achieve and maintain disease Although the risk of CHB in the offspring of an anti-Ro/SSA-
control and to allow tapering of glucocorticoids. Intravenous positive woman is only 2'X,, it is associated with significant
cyclophosphamide is used as induction therapy for severe or fetal and neonatal morbidity and mortality. After a woman
refractory disease (for example, severe active nephritis, acute bears a child with neonatal lupus erythematosus, the risk of
central nervous system lupus, diffuse alveolar hemorrhage, or CHB Is substantially Increased (20%) in subsequent pregnan-
myocarditis) followed by maintenance therapy with mycophe- cies. Hydro.xychloroquine may reduce the overall risk.
nolate mofetil or azathioprine. Mycophenolate mofetil is cur- Management of medications d uring SLE pregnancy is
rently the preferred oral agent for lupus nephritis and is as complicated. Hydroxychloroquine and low-dose gl ucocorti-
effective as cyclophosphamide for induction therapy. The bio- coids can be started during or continued throughout the
logic agent belimumab is FDA approved for patients with pregnancy. Higher-dose glucocorticotds can be used to treat
incomplete response to conventional treatments and has been flare-ups or end-organ involvement. The preferred immu-
shown to be useful in s.kin/joint involvement and moderate/ nosuppressive agent for SLE during pregnancy is azathio-
severe disease. Cl prine, which should be used only jf absolutely necessary.
NSAIDs can be used as adjunct therapy for arthritis and Belimumab, methotrexate, mycophenolate mofetil, and
pleuropericarditis, but are not disease modifying and may cyclophosphamide should be avoided. Cyclophospham ide is
adversely affect kidney function and blood pressure. associated with age- and dose-dependent infertility. See
See Principles of Therapeutics for information on SLE Principles of Therapeutics for informat ion on medications
medication toxicities, monitoring parameters, and more. See and pregnancy.
MKSAP 18 Nephrology for details on the treatment of lupus
KEY POINTS
nephritis.
• Systemic lupus erythematosus is associated with a five-
KEY POINTS to eightfold increase in miscarriage, stillbirth. prema·
HVC • Hydroxychloroquine should be initiated in every patient ture delivery, and intrauterine growth retardation.
with systemic lupus erythematosus who can tolerate it. • The best time to consider pregnancy is when systemic
because it can reduce disease-associated damage, prevent lupus erythematosus Is quiescent, and conception
disease flares, and improve kidney and overall survival. should be considered only after at least 6 months of
• Glucocortlcoids are a mainstay of systemic lupus ery- adequate disease control.
thematosus management, particularly in acute disease: • Fetuses of women who have anti-Ro/SSA or anti-La/SSB
after disease stability is achieved, glucocorticoids are antibodies are at risk for neonatal lupus erythematosus
tapered to the lowest effective dose, ideally to eventual (rash and congenital heart block).
disconti nuation .
• Hydroxychloroquine and low-dose glucocorticoids can be
• In systemic lupus erythematosus, immunosuppressive started during or continued throughout the pregnancy.
therapy is usually initiated concurrently with glucocor-
ticoids to achieve a nd mai ntain d isease control and to
Prognosis
allow tapering of glucocort icoids.
The prognosis in SLE has Improved signlficantJy, and there is
now a 90% 5-year survival rate. Early mortality is usually
Pregnancy and Childbirth Issues related to SLEdisease and infections, and late mortality related
SLE is associated with a five- to eightfold increase in miscar- to cardiovascular disease. Factors adversely affecting survival
riage, stlllbirth, premature delivery, and intrauterine growth include myocarditis, nephritis, low socioeconomjc status,
retardation. Outcomes are worse in patients With active male gender, and age over SO years at diagnosis. SLE tends to

41
Sjogren Syndrome

be a chronic waxing and waning disease with only 2'¥o of


patients achieving remission at 5 years.
KEY POINT
• Factors adversely <lffecting survival in systemic lupus General Fatigue{70"k), fever(6%)
erythematosus include myocarditis. nephritis. low soci- Ski n Dry skin (xerosis}, cutaneous vasculitis: 10%-16%
oeconomic status, male gender. and age over SO years at Jo1nt Arthralgia/arthritis: 36%
diagnosis.
Lung Interstitial pneumonitis: 5%-9%
Kidney Interstitial nephritis, distal (type 1) renal
tubular aetdos1s, glomerulonephritis: 5%-6%

Sjogren Syndrome Neurologic Central nervous system (CNS): demyelinating


disease, myelopathy, cranial nerve neuropathy
Epidemiology and Peripheral nervous system: small-fiber
neuropathy, mononeuritis multiplex,
Pathophysiology penpheral neuropathy
Sjogren syndrome is an autoimmune exocrinopathy affect- 8%-27% for CNS and peripheral
ing salivary and lacrimal glands. ft is more prevalent in Gastrointestinal Autoimmune hepatitis, primary biliary
women and white persons and commonly presents ln the cirrhosis: 3%·20%
fourth and flfth decades. Primary SjOgren syndrome occurs Hematologic lymphoma, cytopenia: 2%
in isolation; secondary SjOgren syndrome occurs in the set- Other Systemic vasculitis (7%), cryoglobulinemia
ting of other rheumatologic diseases, most commonly rheu- (4%-12%), Raynaud phenomenon ( 16%),
matoid arthlitis and systemic lupus erythematosus. Whereas thyroid disease (1 0%·15%}
the prevalence of primary Sjogren syndrome ranges from o.s
to S patients per thousand, secondary Sjogren syndrome is
common (10%-30'Yo) among populations with predisposing TABLE 23. American College of Rheumatology/European
league Against Rheumatism Classification Criteria for
rheumatologic conditions. SJOgren syndrome etiology
Pnmary Sjogr~n Syndrome·'
remains unclear, although genetic associations include sev-
The presence of a subjective finding of Sj(;gren syndrome
eral HLA subtypes.
(e.g.• sicca complaint) plus any combination of the follow-
ing resulting in a score of l!4:

Clinical Manifestations Item Sc:ore

The most common presentation of Sjogren syndrome is Salivary gland b1opsy with C>: 1 foci of lymphocytic 3
infiltrate/4 mmZ
sicca, consisting of dryness of the eyes (keratoconjunctivi-
tis sicca) and mouth (xerostomia). Patients report gritty Anti·Ro/SSA autoantibodies 3
eyes or a foreign body sensation. Oral dryness can cause Ocular staining score <::5 in at least one eye 1
caries and difficulty eating unmoistened food. Symmetric Schirmer test ~5 mm/5 min in at least one eye 1
parotid and lacrimal sweUing is common, as is skin and Unstimulated whole saliva flow rate !>0. 1 mUmin l
vaginal dryness.
•&elusion criteria Include prior head/necl< radiation and histO<y of active hepatitis
Extraglandular manifestations may also occur (Table 22). C virus infection. AIDS, sarcoidosis. amyloidosis. graft-versus-host disease, and
Arthralgia and arthritis are common: the joint distribution lgG4-telat&d dinaso.

and presentation resemble rheumatoid arthritis but without From Shilx>ski CH, Shiboslci SC, Serer R, CriswG!ItA l•l>etoulle M.lietman TM,
eta'; lntemationa1SjOgren's Syndrome Criteria Working Group. 2016American
bone erosions (Jaccoud arthropathy). Patients with Sj~gren College of Rheumatology/European league Against Rhev~tism OaJSif<calion
Criteria for Primary Sjog"ln's Syndrome: A Consensus and Oata·Oriven
syndrome are at increased risl< for non-Hodgkin lymphoma, Methodology Involving Three lntemalional Patient Cohorts. Anhritis Rt..umatol.
presumably related to chronic lymphocyte activation. 2017 Jan;69(11:35-4S. doi: 10.1002/art.39SS9. (PMID: 27785888). Copyright2016,
American College of Rheumatology. Adapted with permission from John Wiley &
Sons.l<'c.

Diagnosis
A diagnosis of Sjogren syndrome is typically triggered by a by an ophthalmologist using tluorescein and lissamine
complaint of sicca and requires objective confirmation of green dyes. Dry mouth is assessed by direct examination,
exocrinopathy along with demonstration of au toimmunity with dental caries and a lack of saliva suggesting the diag-
(Table 23). Dry eyes can be assessed using the Schirmer nosis. Measurement of salivary flow (sialography) is easily
test, in which a strip of filter paper Is placed under the done but is less commonly performed. Salivary gland imag-
lower eyelid and wetting is measured: less than 5 mm in ing wHh MRI or ultrasonography may assist in Identifying
5 minutes indicates dryness. More formally, dryness and characteristic abnormalities and provide additional sup-
corneal damage are ascertained by slit-lamp examination port for the diagnosis.

42
Inflammatory Myopathies

Multiple autoantibodies may be present. Antinuclear KEY POINTS


antibodies and/or rheumatoid factor are often present at
• SjOgren syndrome is an autoimmune exocrinopathy
high titers; thel.r coexpression is more suggestive of SJOgren
affecting salivary and lacrimal glands, with the most
syndrome than if only one is positive. The presence of anti-
common presentation being sicca (dryness of the eyes
Ro/SSA autoantibodies Is specific in the setting of primary
andtor mouth).
Sjtlgren syndrome; their presence is less specific for second-
ary SjOgren syndrome because these antibodies are also com- • Patients w1th SjOgren syndrome are at increased risk for
mon in systemic lupus erythematosus. The presence of non Hodgkin lymphoma.
anti-La/SSB autoantibodies is also characteristic of SJOgren • A diagnosis of SjOgren syndrome is typically triggered
syndrome, although these are not currently included in tor- by a complaint of sicca and requires objectiveeonfirma-
mal disease classification criteria. When Initial evalltatlon is t ion of exocrinopathy along with demonstration of
uninformative, a lip (minor salivary gland) biopsy should be autoimmunity.
considered. Charac teristic histologic features include aggre- • In patients with Sjogren syndrome, management of
gate foci (>50 cells/aggregate) of CD4 T ceJls, B cells, and sicca is centered on preservation of moisture and relief
plasma cells surrounding secretory ducts. of symptoms; extraglandular involvement is treated
The differential diagnosis of SjOgren syndrome typically with immunosuppression.
relates to the presence of s!cca and/or parotid and lacrimal
enlargement. Sarcoidosis, lgG4- related disease, granulomato-
sis with polyangiitls, lymphoma, and viruses such as mumps, Inflammatory Myopathies
hepatitis C. and HIV may be associated with similar findings.
Ocular and oral dryness may also occur with aging or in Overview
response to drugs such as anticholinergics. The inflammatory myopathies include polymyositis (PM), der-
matomyositis (DM), immune-mediated necrotizing myopathy
(IMNM), and inclusion body myositis (IBM). Although the
Management pathogenesis of these disorders is disparate, they aU include
Management of sicc~ is centered on preservation of mois- immune-mediated muscle inflammation and destruction.
ture and relief of symptoms. Eyes are treated with artificial
tears andior topical cyclosporine. The use of glasses with
side panels to prevent surface drying may be helpful, as may
Evaluation of the Patient with
be plugging of the lacrimal ducts to promote tear retention. Muscle Pain or Weakness
For oral dryness. meticulous dental care is essential. Sugar- The differential diagnosis of muscle disease Is broad and
free candles can stimulate saliva tlow. Artificial saliva may includes hereditary, environmental, infectious. and acquired
be used, but its effects may be too transient to be usefuL muscle disease (Table 24). The Inflammatory myopathies com-
Vaginal symptoms may be treated with topical lubricants or monly present with symmetric proximal muscle weakness
estrogen. Pharmacologic therapies Include the muscarinic without substantial muscle pain or tenderness. Serum cre-
agonists pilocarpine and cevimeline, although these may atine kinase is almost universally elevated in the inflammatory
not be well tolerated and are contraindicated in narrow- myopathies and is a marker of muscle damage. The presence
angle glaucoma. of pain or tenderness, however, should prompt considerations
Patients with extraglandularinvolvement are treated with of other causes of myopathy. Muscle pain is more common
immunosuppression. Evidence-based guidelines are limited, with overexertion, muscle cramps, and injury. Muscle tender-
and decisions are often empiric. Options include hydroxychlo- ness should prompt consideration of infectious, thyroid, or
roquine and methotrexate for arthritis, and cyclophospha- drug-induced myopathies. Medications and drugs are com-
mide and rituximab for severe or systemic central nervous mon causes of myopathy and should be considered In the dif-
system disease. Some of these agents may have benefit tor sicca ferential diagnosis of the inflammatory myopathies (Table 25).
as well. Weakness is often caused by conditions other than myo-
pathy. Cardiopulmonary disease, malignancy, depression, and
deconditionlng frequently lead to generalized weakness.
Prognosis Although uncommon, primary neurologic disorders such as
The prognosis for primary SjOgren syndrome is favorable, with spinal muscle atrophies, amyotrophic lateral sclerosis, and
no overall increase in mortality. Management of sicca reduces myasthenia gravis may cause weakness. These diseases are
the risk of corneal damage or loss of teeth. In rare instances, distinguished by involvement of muscle groups other than the
end-organ involvement can be Ufe-threatening. Secondary proximal muscles, nerve involvement on electromyography
SjOgren syndrome may be associated with significant morbid- (EMG), and a normal serum creatine kinase. Finally, the pres-
ity from the primary condition. An increased risk of lym- ence of exercise-induced weakness may point to a metabolic
phoma is present in both forms of the syndrome. etiology rather than an inflammatory myopathy.

43
Inflamm atory Myopathies

TABLE 24. Differential Diagnosis of Myopathy


Myopathy Common Examples Comments
Inflammatory myopathies Dermatomyos1t1s (DM); polymyositiS (PM); DM and PM· acute-subacute onset, symmetric,
immune-mediated necrotizing myopathy(IMNM); proximal; DM· pathognomonic rash; IMNM: acute
inclusion body myositis (IBM) onset, symmetric, proximal; IBM· insidious onset,
proximal and distal
Systemic rheumatologic Systemic lupus erythematosus; systemic sclerosis; Prominent extra muscular features typical of
disease mixed connective tissue disease underlying disorder
1 Muscular dystrophies Duchenne muscular dystrophy; Becker muscular Childhood onset; cardiomyopathy possible
dystrophy
Metabolic myopathies Glycogen storage diseases; camitine Exercise intolerance
palmitoyltransferase deficiency
Mitochondnal myopath1es Kearns·Sayre syndrome; Letgh syndrome Vanable age of presentation and features; 1solated
myopathy or wtth neurologic, muhisystem disease
Endocrine disorders Hypothyroidism; hyperthyroidism; adrenal Prominent myalgia
insufficiency
Infection mduced Bactenal: pyomyositis; viral· mfluenza; parasitic: Prom1nent myalgta; sometimes accompanied by
trichinosis fever

Asymptomatic elevations of serum creatine ktnase; chronic


muscle atrophy; acute, severe rhabdomyolysis wfth kidney failure
Antimalarials Can occur after prolonged use Infrequent elevation of serum creatine kinase (30%), muscle
weakness (50%); myopathic electromyogram findings (1 00%);
cardiomyopathy can occur
Cocaine Can occur after stngle use Asymptomatic elevations of serum creatine ktnase; acute,
severe rhabdomyolysis with kidney failure
Colchicine Usually months to years; increased risk with Proximal muscle weakness with elevations of serum creatine
coadministration of cytochrome P450 inhibitors kinase; mild sensory symptoms; reduced reflexes
Glucocorticoids Increased with long-term use Proximal muscle weakness in the absence of elevations of
serum creattne kinase
Statins Usually weeks to months but can occur at anytime; Elevations of serum creatine kinase with myalgia and
increased risk with preexisting neuromus<;ular weakness
disease, hypothyroidism, kidney failure, and/or
coadministration of cytochrome P450 inhibitors; may
also trigger immune-mediated necrotizing myopathy
Zidovudine Variable; may be more common after long-term use Elevations of serum creatine kinase with myalg1a and weakness

Epidemiology and Pathophysiology expression of autoantibodies In many patients. T- and B-celJ


proliferation and activation, as weU as increased cytol<ine
of Inflammatory Myopathies and chemoklne levels in muscle, also occur.
The annual incidence of Inflammatory myopathies is between
2 and 10 cases per million. Two peaks of onset occur, with the
KEY POINTS
first in chJldhood and the second In middle age. A female adult
predominance is approximately 2:1 with the exception of IBM. • The inflammatory myopathies commonly present with
which preferentlaUy affects men. symmetric proximal muscle weakness without substan-
Genetic factors contribute to the risk of' inflammatory tial muscle pain or tenderness; serum creatine kinase is
myopathies. The clearest association is with HLA DRB1.0301, almost universally e levated and Is a marker of muscle
but alleles vary In populations throughout the world. Except damage.
for IMNM, environmental factors have not been clearly impli- • Muscle tenderness should prompt consideration of
cated. Immune mechanisms that contribute include the Jnfectious, thyroid, or drug-induced myopathies.
presence ofT lymphocytic infiltrates in muscle tissue and the

44
Inflammatory Myopathies

Polymyositis Dermatomyositis
Clinical Manifestations Clinical Manifestations
Muscle Involvement In DM, muscle involvement is clinically similar to PM but is
The symptoms of PM usually progress slowly ove r months, almost un iversally accompanied by a unique constellation of
although a more rapid onset may occur. Progressive, symmet- skin findings.
ric, proximal muscle weakness is the classic and dominant
feature. Activities such as arising from a chair. stair c limbing, Cutaneous Involvement
and lifting objects above shoulder height are typically affected. Cutaneous involvement of OM can be virtually pat hognomic
Neck flexors a re also commonly involved. Muscle pain and and serve as an indicator of response to therapy (Table 26).
tenderness, it'prcsent, are mild. Muscle atrophy is a late mani- Most typical is the Gottron rash (Gottron papules and Gottron
festation. Weakness of respiratory muscles may also occur. stgn), erythematous to violaceous areas over the metacar-
pophalangeal and proximal interphalangeal joints (Figure 18).
CJ Cardiopulmonary Involvement
Interstitial lung disease, most commonly nonspecific interstitial
See MKSAP 18 Dermatology for more information.

pneumonitis, occurs in about lO't of patients with PM. It is par- Muscle Involvement
ticularly prornlnentin the subset of patients with antlsynthetase Similar to PM, the insidious onset of symmet ric proximal
syndrome (also seen in association with dermatomyositis). In muscle weakness is most common in OM. Some patients pre-
addition to interstitial lung disease and myositis, antisynthetase sent initially with rash and develop weakness later.
syndrome is characterized by che presence of Raynaud phenom- A small percentage of patients may present with skin rash
enon, nonerosive arthritis, and mechanic's hands, a dermato- only and never develop muscle involvement (amyopathic DM). At
Jogic manifestation characterized by rough. cracked. scaly skin least 6 months without evidence of muscle involvement is neces-
along the lateral aspects of the digits and pahns with horiZOntal sary before the diagnosis of a myopathic DM can be established.
lines resembling the weathered hands of a laborer (See MKSAP
18 Dermatology). It can occasionally present as rapidly progres-
Sive respiratory failure. Antisyntheta.se syndrome is associated
Cardiopulmonary Involvement C]
Cardiopulmonary involvement Is similar to that seen in PM,
with autoantibodies against antl-aminoacyl-tRNA :.ynthetases, but interstitial lung disease is more common in OM (20'¥.,-25%).
most commonly anti-Jo-1 antibodies. Arrhythmias. complete atrioventricular hlock, heart fajlure,
Cardiac Involvement includes conduction system abnor- myocardial infarction, and pericarditis have been reported. C1
malities that can result in arrhythmias and sudden death.
Pericarditis can occur, and pericardia! tamponade has been Gastrointestinal Involvement
reported. Patients are also at increased risk for myocardial Gastrointesti nal involvement in DM resembles that in PM.
infarction and heart failure, which represent an Important
cause of morbidity and mortality in PM.

Gastrointestinal Involvement
Immune-Mediated Necrotizing CJ
Myopathy
The striated muscle of the upper two thirds of the esophagus
IMNM occurs in a small subset of patients with intlammatory
can be affected by PM. leading to dysphagia, dysmotility, and
myopathy who demonstrate prominent myonecrosls on biopsy
increased risk of aspiration p neumonia. Cl

TABLE 26. Cutaneous Manifestations of Dermatomyositis


Cutaneous Sign Location Clinical Appearance
Gottron rash (Gottron Metacarpophalangeal and proximal Erythematous to violaceous papules; occasi onal scale; can
papules and Gottron sign) interphalangeal joints; occasionally on distal ulcerate; atrophic scars may occur
interphalangeal joints, elbows, knees
Heliotrop e rash Eyelids Subtle pink to deep purple or brown discoloration; may
be associated with edema of eyelids or periorbital edema
Vsign V ofneck Erythematous to violaceous papules to patches
Shawl sign Base of posterior neck to upp er back Erythematous to violaceous p apules to patches
Fixed erythema Malar area (may cross nasolabial folds); Erythematous to violaceous papules to patches
flanks of trunk
Nail area changes Periungual area; cuticles Periungual erythema; capillary dilatation and dropout;
cuticular hypertrophy; cuticular in farcts
Mechanic's hands Lateral aspects of digits and palms Hyperkeratotic fissuring of the palmar and lateral surfaces
of the fingers resembling the hands of a laborer

45
Inflammatory Myopathies

It from PM and DM. Muscle distribution is typically symmet-


ric. but asymmetry may occur. On physical examination. mus-
cle weakness and atrophy are often present in the hip flexors ,
quadriceps, fi nger flexors, and forearm flexors. Serum creatine
kinase levels are elevated but are lower than levels seen in PM
andDM.

Gastrointestinal Involvement
Up to half of patients with IBM have cricopharyngeal muscle
involvement, leading to dysphagia and increased risk of
aspiration.
KEY POINTS
F16 UR£ 1 8. Gottron rash. Erythema and mild scaling are seen most • Polymyositis and dermatomyositis are characterized by
prominently over the second and third metacarpophalangeal joints, and more progressive, symmetric, proximal muscle weakness,
subtle erythema is present ovecthe proximal interphalangeal joints of both hands. leading to decline in physical function; dermatomyositis
The cracked and thickened skinon the lateral surfaces of the second digit
is further defined by cutaneous involvement.
bilaterally is consistent with mechanic's hands.
• Gottron rash {Gortron papules and Gottron sign) is the
characteristic rash of dermatomyositis, and is defmcd as
without the inflammatory changes typical of the other intlam- erythematous to violaceous areas over the metacar-
Cl matory myopathies. An immune-mediated mechanism is sug-
CONl. gested by the presence of antibodies to signal recognition
pophalangeal and proximal mterphalangeaJ joints.
• Immune- mediated necrotizing myopathy is character-
particles (SRPs) or 3-hydroxy-3-methylglutaryl-coenzyme A ized by severe, rapidly progressive proximal muscle
(HMG-CoA) reductase. Although statin exposure may result in weakness, very high serum creatine kinase levels, and
antibodies to HMG-CoA reductase and IMNM, most patienrs lew extramuscular manifestations.
with stalin-induced myopathies have processes other than
• Inclusion body myositis is a uniformly insidious condi-
lMNM (see Table 25).
tion that involves both the proximal a nd distal muscles;
although typically symmetric, muscle distribution may
Clinical Manifestations be asymmetric.
Patienrs with !MNM experience severe, rapidly progressive
weakness. very high serum creatine kinase levels, and few (if
any) eXtramuscular manifestations. Inflammatory Myopathies
Muscle Involvement and Malignancy
The characteristic symptom of fMNM is severe proximal mus- There is an association between risk of malignancy and DM
cle weakness, making it difficult ro distinguish from PM and (3- to 12- fold that of age-matched populations) as well as, to a
DM. Rapidity of onset may provide a clue. In patients with lesser extent, PM. IMNM may occasionally be a paraneoplastlc
statln- triggered lMNM. the weakness may persist or progress syndrome, but ffiM is not strongly associated with malignancy.
even after statins are discontinued. The risk of malignancy is greatest in the 3 years following the
diagnosis of DM or PM. Common associated cancers are ovar-
Other Organ Invo lvemen t ian, lung, pancreas, stomach, colon, and lymphoma. Age- and
Cardiopulmonary involvement lniMNM is rare. Gastrointestinal sex- appropriate cancer screening is indicated at the time of
involvement has not been reported. CJ diagnosis of OM or PM. Many experts advocate CT of the chest,
abdomen, and pelvis, particularly in patients with OM.

Inclusion Body Myositis KEY POINT

IBM is a uniformly insidious condition and is usually easUy • Because of the malignancy risk associated with polymy- HVC
distinguishable from PM, DM, and lMNM because of its very ositis and dermatomyositis. age- and sex-appropriate
slow onset and pattern of muscle involvement. cancer screening is indicated at the time of diagnosis.

Clinical Manifestations
Muscle Involvement
Diagnosis
Patienrs typically report a history of preexisting weakness Muscle-Related Enzymes CJ
averaging 5 years. AJthough proximal muscle involvement is Elevations ofserum muscle enzymes are particularly useful in
present, distal muscle involvement also occurs, distinguishing differentiating intrinsic muscle disease from neurologic causes

46
Inflammatory Myopathies

of weakness. Elevated serum creatine klnase levels are the dermal-epidermal junction. perivascular lymphocytic infil-
CJ
CONT
most sensitive serum Indicator of muscle pathology. and levels trates. and increased dermal mucin. However, these findings
parallel the severity of Involvement. Aldolase can occasionally may be similar to those of systemic lupus erythematosus.
be elevated when serum creatine kinase levels are normal Diagnosis of amyopathic DM is made with skin biopsy
Serum aminotransferases and serum lactate dehydrogenase conftrmation ofDM in the setting of at least 6 months of nor-
may also be elevated but are nonspecific. CJ mal muscle strength, muscle enzymes, normal EMG, and, If
performed, normal muscle biopsy.
Autoantibodies
KEY POINTS
Various autoantibodies are used to help diagnose Inflamma-
tory myopathies. Some, such as antinuclear antibodies. Jack • Elevated serum creatine kinase levels are the most sen-
spectflclty. Others, often termed myosltls-spectflc antibodies, sitive serum indicator of muscle pathology. and levels
parallel the severity or involvement.
have more diagnostic and prognostic value (Table 27).
• Antibodies against aminoacyl- tRNA synthetases, par
Imaging Studies ticularly anti-Jo-1 (antl-histidyl-tRNA synthetase anti-
Although not required to diagnose inflammatory myopathies, bodies), have been associated with antisynthetase syn
MRT is potentially useful ln identitytng areas of muscle i nflam- drome (interstitial lung disease, Raynaud phenomenon,
mation as an appropriate site for muscle biopsy. MRl is also arthritis, and mechanic's hands).
helpful at Identifying muscle atrophy; follow-up with MRl may • In patients with inflammatory myopathies, MRI can be
provide a noninvasive technique to assess therapeutic efficacy. helpfullo locate a site tor muscle biopsy, IdentifY mus-
cle atrophy, and assess therapeutic efficacy.
Electromyography • Muscle biopsy is valuable in diagnosing inflammatory
EMG may show findings hlgh ly characteristic of actiVe myost- myopathies and for excluding metabolic myopathies.
tis, although no single fincling is pathognomonic. The proce- mitochondrial disease, and infection.
dure, however, is invasive and uncomfortable. It may also be
normal in the presence of active disease due to the patchy
distribution of muscle inflammation (that is, sampling error). Management
Muscle Biopsy
Glucocorticoids are the cornerstone of therapy for the in flam-
matory myopathies except for IBM. Prolonged exposure to
CJ
Muscle biopsy is valuable in diagnosing inflammatory myopa- high-dose glucocorticoids, particularly in the elderly, can be
thies and for excluding metabolic myopathies, mitochondrial associated With the appearance of steroid-induced myopathy
disease, and infection. In PM and DM, features inclttde inflam- that may cause new proximal muscle weakness In a patient
matory cell infiltrates, muscle fiber necrosis, degeneration, who had previously been improving; reduction in dose is
and regeneration. appropriate management.
Muscle biopsy findings in IBM include mononuclear cell Add!tlonal immunosuppressives are often required to con-
In filtrates wlth unique "rimmed" vacuoles wlthout muscle cell trol inflammation or to serve as glucoconicoid-sparlng agents;
necrosis; electron microscopy may show inclusion bodies. methotrexate and azathioprine are most frequently used. DM
and PM are most responsive to immunosuppression, although
Skin Biopsy DM is more responsive than PM. For patients with stalin-
Skin biopsy is useful in diagnosing DM. Light microscopy find- induced IMNM, statin discontinuation is mandatory but not
ings it1clude mild epidermal atrophy, vacuolar changes at the sufficient, and immunosuppressive treatment is required. IBM

Antmuclear DM;PM Seen in up to 80%, nonspecific


Anti-aminoacyl-tRNA synthetase; most commonly DM Antisynthetase syndrome (ILD, Raynaud phenomenon,
anti-Jo-1 (anti-histidyl tRNA synthetase) non erosive arthritis, mechanic's hands)
Anti-cytoplasmic 5'-nucleotidase lA 18M Insidious onset
Anti-HMG Co-A reductase IMNM Typically but not always in t he setting of stalin exposure; may
also be seen in patients with stalin exposure without IMNM
Anti-Mi 2 DM Seen in up to 30%; associated with acute onset, typical rash
Anti-signal recognition protein IMNM Severe, treatment-resistant disease and myofiber necrosis
but little to no inflammation on muscle biopsy

OM • detm.li1omyo$itis; IBM - inclusion body myositis: tLO • fntorstitial lung disease; IMNM -. immune-mediated necmtil:i"'g myopathy; PM :.. polymyositis.

47
Systemic Sclerosis

usually does not respond to immunosuppression, and them- ofl9 cases per million. It is more common among women and
Cl peutic be
trials should undertaken only In selected patients and black persons.
CONT. discontinued if benefit is not appreciated. Other immunosup- Vascular injury, vascular and visceral fibrosis, and innate
pressives, including mycophenolate mofetil and rltuximab. and adaptive immune activation with autoantibody production
have been used in refractory OM, PM. and IMNM. Intravenous play an Interactive role in SSe. One of the earliest SSe manif~ta­
immunoglobulin may be a reasonable alternative in OM. Cl tions ls Raynaud phenomenon, suggesting that vascular injury
Physical therapy may help maintain muscle function and precedes the fibrotic reaction. With vascular damage, the
should be used in all patients. endothelial cells release endothelin-l, a potent vasoconstrictor
that also induces vascular smooth muscle proliferation and
fibroblast activation. Platelet activation occurs concmrently
Prognosis with subseque11t release of growth factors that promote further
The prognosis In OM/PM varies from near-complete resolution vasoconstriction, fibroblast activation, and collagen production.
to severe, treatment-resistant disease with marked functional B-cell activation results in production ofinterleukln (IL)-
impairment and early mortality. Prolonged symptoms prior to 6, which directly stimulates fibroblasts as well as various
treatment, greater weakness at presentation, extensive multi- autoantibodies. The end result is increased collagen produc-
system disease, and the co-occurrence of malignancy confer a tion and deposition by activated scleroderma fibroblasts, along
poorer prognosis. The degree ofserum creatine kinase elevation with a progressive obliterative vasculopathy.
is not predictive of outcome. Myo~itis-specitlc antibodies may
have prognostic value. Patients with anti-Jo-1 antibodies have
worse outcomes (due to interstitial lung disease), whereas those Classification
with anti-Mi-2 antibodies often respond completely, despite a Although intended tor research studies, the 2013 American
rapidly progressive course of DM. College of Rheumatology/European League Against
Patients with lMNM frequently respond well to statln Rheumatism SSe classification criteria can be useful when a
discontinuation (if appropriate) and immunosuppression. patient presents with features suggesting SSe (Table 28).
Given the lack of effective treatment. patients with IBM Classically, SSe is divided into three subtypes based on the
typically demonstrate a slow and gradual loss offunction, with extent of skin involvement: limited cutaneous systemic sclero-
progression to wheelchair dependence within 10 to 15 years. sis (LcSSc), diffuse cutaneous systemic sclerosis (DcSSc), and
systemic sclerosis sine scleroderma (internal organ involve-
KEY POINTS
ment only). These subtypes have clinical and prognostic utility
• Initial treatment for most inflammatory myopathies because specific long-term complications may be more likely
consistsofglucocortfcoids, with additional immuno- In one subtype than another; however, overlap Is common.
suppressives (most commonly methotrexate and azathi- Importantly, LcSSc is less commonly accompanied by fibrosis
opline) often required to control inflammation or serve of internal organs, but is most commonly associated with pul-
as glucocorticoid-sparing agents. monary arterial hypertension and CREST (calcinosis, Raynaud
• Inclusion body myositis often does not respond to phenomenon, esophageal dysmotility, sclerodactyly, and tel-
immunosuppression. and therapeutic trials should be angiectasia) syndrome.
discontinued if benefit is not appreciated. Various disorders may present with skin thickening and
• In patients with dermatomyositis or polymyositis, pro- other manifestations that overlap with the findings of SSe;
longed symptoms prior to treatment. greater weakness these conditions should be considered in the differential diag-
at presentation. extensive multisystem disease. and the nosis of SSe (Table 29). Importantly, these conditions are not
co- occurrence of malignancy confer a poorer prognosis. typically associated with Raynaud phenomenon, and the
absence of Raynaud phenomenon in a patient with skin thick-
• Patients with inclusion body myositis typicaUy demon-
ening makes SSe unlikely.
strate a slow and gradual loss of function, with progres-
sion to wheelchair dependence within lO to 15 years. KEY 'OINTS
• Systemic sclerosis is a multiorgan disease characterized
by fibrosis and vasculopathy; the s kin is the principle
Systemic Sclerosis target, with Raynaud phenomenon being one of the
earliest manifestations.
Epidemiology and • Systemic sclerosis is divided into three subtypes based
Pathophysiology on the extent ofskin involvement: limited cutaneous
Systemic sclerosis (SSe) Is a multlorgan disease characterl?.ed systemic sclerosis. diffuse cutaneous systemic sclerosis.
by fibrosis and vasculopathy. The s l<i n is the principal target, and systemic sclerosis sine scleroderma.
but internal organs are also affected. SSe Is relatively rare, with
(Continued)
a prevalence of27S cases per million, and an annual incidence

48
Systemic Sclerosis

TABLE 28. American College of Rheumatology/European league Against Rheumatism ClassificatiOn Criteria
for Systemic Sclerosis
Manifestation Additional Manifestations Weight/Score•

Skin th1ckening of fingers of both hands extendmg 9


proximal to the MCPs
Skin thickening of the fingers (count higher score only) Puffy fingers 2
Sclerodactyly of fingers distal to the MCPs but proximal to PIPs 4
Fingertip les•ons Dig•tal tip ulcers 2
Fingertip pitting scars 3
Telang1ectasia 2
Abnormal nailfold capillaries 2
Pulmonary hypertension and/or interstitial lung Pulmonary hypertension 2
disease (maximum score is 2)
Interstitial lung disease 2
Raynaud phenomenon 3
SSe-related autoantibodies{maximum score is 3) Anticentromere 3
Anti-Scl-70 (.antitopoisomerase-1) 3
Anti-RNA polymerase Ill 3
MCP- met.>cupophalangeal; PIP • pto•imal inrerphalangeal; SSe • systemic sclerosis.

•A scote of 9 or more t.Jquates lo definite systemic scicr®fs.

From van den Ho.ogen F, Khanna 0, Frans-en J, John$Ot'l SR, Baro" M, Tyn.detl A, et ol. 2013 chusifi~tjon critetia for $Y$temic sderosis: an Atnericon CoHego oi Rheumatoiogy/
Eorope<on league Against Rheumatism Collabot ative Initiative. Ann Rheum Ois 201 3;721747-SS. doi; 10.1136fannrheumdis-2013·204424.1PMIO: 24092682) Copyright 2013,
American College of Rheumatology. Adapted with pe<missoon from 8MJ Publishing Group Ltd.

KEY P 0 I NT S (continued} feature is poikiloderma, in which areas of hyperpigmentation


mixed with hypopigmentation give the skin a salt-and-pepper
• The absence of Raynaud phenomenon in a patient with
appearance.
skin thickening makes systemic sclerosis unlikely.
Facial involvement can lead to limitation of the oral aper-
ture and difficulty eating. The face is aJso typically devoid of
wrinkles, causing patients to sometimes look younger than
Clinical Manifestations their age.
and Diagnosis Calcinosis occurs in approximately 25% of patients with
The diagnosis of SSe is dependent on the presence of specific SSe (Figure 20). Present in the hands, forearms. elbows. glu-
clinical findings and autoantibodies (present in 90'X,-9S% of teal region, and iliac crest, these deposits can be seen/felt and
patients). In patients with clinical features suggestiVe of SSe. are easily detected on radiograph.
autoantibody testing should be done (Table 30 ).
Musculoske letal Involvement
Cutaneous Involvement Joint involvement occurs in 12'1 to 65% of patients with SSe; it
The skin is the most common organ involved in SSe, with the has a hand/wrist predominance and is one of the few forms of
hands being universally affected. In LcSSc, the skin over the inflammatory arthritis that affects the distal Interphalangeal
fingers/hands, face, and neck is typically affected. In DcSSc. joints. Between l'>o and 5% of patients have a rheumatoid
skin Involvement Is more extensive and additionally includes arthritis-SSe overlap, with positive anti- cyclic citrullinated
the arms, trunk, and lower extremities. antibodies and classic rheumatoid arthritis manifestations
The earliest skin manifestations are often diffusely swol- along with those of SSe.
len firlgers/hands (sclerodactyly). Sktn thickening. especially Patients with SSe may develop aero-osteolysis, or resorp-
in LcSSc. may be subtle, and the inability to tent the skin over tion of the terminal bony tuft of fingers and less commonly the
the fingers may be an Important clue. With time, the skin toes, with prevalence as high as 20% to 2S'Yo Jn more severe
becomes thickened, atrophic, and lmmoblle; skin thickening disease (Flgure 21).
around the joints can lead to contractUies (FigUle 19). SSe-associated myositis occurs In IO'X, to lS'X. of patients.
Vascular complications of the skin include digitaJ Infarcts, MyaJgia and proximal muscle weakness are common symp-
subungual infections, and ischemic skin ulceration. Another toms. Serum creatine k1nase and/or serum aldolase levels are

49
Systemic Sclerosis

TABLE 29. Common Manifestations/Features of the Scleroderma Spectrum Dosorders


Disorder Manifestation/Feature Comments
Systemic Sclerosis

Diffuse cutaneous Distal and proximal skin thickening (chest, abdomen, Skin involvement is extensove and is commonly
systemic sclerosis arms prox1mal to wrists); commonly has visceral accompanied by mternal organ fibrosis and ILD
(DcSSc) organ involvement
Limited cutaneous Distal (face, neck, hands}, but not proximal, skin More likely to develop PAH and Raynaud
systemic sclerosis thickening; typically not accompanied by internal phenomenon early in the disease and more likely to
(lcSSc) organ fibrosis display features of the CREST syndrome
Systemic sclerosis Fobrosing organ Involvement without skin th1ckenong Difficult to doagnose; prognosis may be similar to
I
sine scleroderma LcSSc

Localized Scleroderma

Morphea Focal plaques of skin thickening, generally on the Systemic manifestations or Raynaud phenomenon is
trunk extremely rare
Linear scleroderma Streaks/lines of thickened skin Same as morphea

Scleroderma-like Conditions-

Eosonophilic fasciitis Orange peel induration (peau d'orange) of pro:.cimal Full-thickness skin biopsy demonstrates lymphocytes.
extremities with sparing of hands and fuce; peripheral plasma cells, and eosinophils infiltrat1ng the deep
eosinophilia; skin retraction over the superficial veins fascia; glucocorticoids are the mainstay oftreatment
may be more apparent with elevation of an affected limb
Nephrogenic Secondary to gadolinium in patients with kijdney Skeletal muscle fibrosis with contractu res and/or cardiac
systemic fibrosis disease; brawny, wood-like induration of extremities, muscle involvement can occur, with cardiomyopathy
sparing the digits and increased mortality; changes in use and formulation
of gadolinium have reduced incidence
Scleredema Indurated plaques/patches on back, shoulder girdle, Typically seen in long-standing diabetes mellitus
and neck
Scleromyxedema Waxy, yellow-red papules over thickened skin of face, Associated with paraproteinemia (lgGA.) and may
upper trunk, neck, and arms; deposition of mucin with therefore occur in the setting of multiple myeloma or
large numbers of stellate fibroblasts in the dermis AL amyloidosis; more frequent in men
Chronic graft.versus· Lichen planus-like skin lesions, or localized or Occurs most commonly after hematopoietic stem cell
host disease generalized skin thickenong transplantation; may occasoonally be seen after blood
transfusion In an ommunocompromtsed host
Drug and toxin Can produce scleroderma-like tissue changes Examples: bleomycin, docetaxef. pentazocine.
exposure L·tryptophan, organic solvents
CRl:ST • c.aJcinosis cuti,, R3ynaud phenomenon. esophageal dysmot1l1ty, sdaroda<.tyfy, and telangiectast.a; flO • tnterstit~f tung dfsease; PAH • pulmonary arterial hypertension.

•ScJeroderma·like skin change-s may atso occur as a manife.station of systemic endocrine, ~idru~y. or infilttative d isordets.
-

TABLE 30. Autoantibodies and Their Associations in Systemic Scferos1s


Autoantibody Clinical Aslsociations Comments

Antinuclear antibodies DcSSc; LcSSc Overall prevalence in SSe: 70%; not associated woth
specific manifestattons
Anticentromere (kinetochore proteins) LcSSc :t PAH Overall prevalence in SSe up to 30%; highly associated
(>90%) with CREST variant of LcSSc
Anti-Sci· 70 (DNA t opoisomerase-1) DcSSc; ILD Overall prevalence 10 SSe: up to 30%; highly assoctated
with DcSSc
Anti·RNA polymerase Ill DcSSc; scleroderma renal crisis Useful in DcSSc with negative anti-Sci- 70
Anti·U3-RNP (fibrillarih) DcSSc; PAH; myosotis Associated with poor outcome, more common in black men
Anti-PM-Scl Myositis Associated w ith overlap syndrome and polymyositis
Anti-Ku Myosotis Rare occurrence
Anti-Th/To LcSSc; PAH Rare occurrence

CREST• calcinosis, Roynaud phenomenon. esophogeal dy<motility, sc:leoodaetyly. •nd telangiectasia; DcSSc • cliffuso cut.>neous syst•mic sclerosis; ILO- intersti~l tung disease;
LcSSc- limited cu""'eous sys1emic sclerosls; PAH- pulmonary artenal hypertension; RNP • ribonucleoprotein; SSe• sylltemic scleroSI$.
L-------------~------------------------------------------~----------------------------

50
Systemic Sclerosis

Fl GURE 19. A35·year·old woman with diffuse cutaneous systemic sclerosis. fl CiU RE 21. Aposteroanterior radiograph ol the hands in a patient with
Note shortening of fingers and tight atrophic appearance of the skin. fh!S patient is systemic sclerosis and acro·osteolysis. Note the destruction of the distal phalanges,
unable to make d full fist particularly of the lirst and second digits, which will eventually result in clinical
shortening of the affected digits.

overlap between polymyositis and SSe. These patients usually


have less skin and gastrointestinal involvement, but have more
calcinosis and lung disease.
Tendon rubs can be felt or heard with a stethoscope
because fibrosis affects tendons/tendons sheaths, leading to
palpable and audible frlctlon.

Vascular Involvement
Raynaud phenomenon occurs in approximately 95% of patients
With SSe; it is the most common early manifestation of SSe,
typically occurring years before other changes. Initially, Raynaud
phenomenon is episodic, but with vascular fibrosis, the blood
supply becomes permanently restricted. Approximately 80% of
patients with the combination of Raynaud phenomenon, an
SSe-associated autoantibody, and nailfold capillary changes in
an SSe pattern (flgure 22), will develop SSe.
Digital ulcers occur in about IS% of patients with SSe,
typically on the extensor surfaces and tips ofthe fingers and toes
but also on the dorsum of hands and feet and even on the lower
extremities. Vascular changes in the extremities may be indica-
tive of involvement of the microcirculation of internal organs.

Gastrointestinal Involvement
Symptoms of esophageal dysmotility and reflux are common
FIGURE 20. Cakinosis seen In CREST (calcinosis cutis, Raynaud phenomenon, in all forms ofSSe and may offer an early due to the diagnosis
esophageal dysmotility, sclerodactyly, and telangiectasia) syndrome in limited when associated with other symptoms.
cutaneous systemic sclerosis. This patient has deposits of calcium in the Approximately 80% of patients with SSe have involve-
subcutaneous tissues arouncl the elbow. Calcinosis often occurs in the hands and ment of the lower two thirds (smooth muscle portion) ofthe
forearms but can also affect other locations such as the trunk or lower extremities.
esophagus. Manifesting primarily as symptoms of dysmotillty
and/or gastroesophageal reflux disease, esophageal involve-
elevated, and electromyogram demonstrates myopathic ment is also associated with an Increased risk of Barrett esoph-
changes. There is a strong association between SSe-associated agus and adenocarcinoma.
myositis and myocardial involvement. The presence of Gastric symptoms also result mainly from dysmotility.
anti-PM-Scl antibodies identifies a group of patients with an Patients describe early satiety and may report nausea and

51
Systemi c Sclerosis

use of moderate- to high-dose glucocorticoids, and the pres-


ence of anti-RNA polymerase Ill antibodies. Patients charac-
teristically present wit h ma nifestations of hypertensive
emergency, including headache, encephalopathy, seizures, and
hypertensive retinopathy. Rarely, a normotensive form ofscle-
roderma renal crisis can occur.
Laboratory tests demonstrate microangiopathic hemolytic
anemia with schistocytes on peripheral smear, thrombocyto-
penia, and proteinuria. Serum creatinine is typically elevated
and may remain so for some time after controlling blood
pressure. Abnormalities In the renin-angiotensin-aldosterone
system and endothelin 1 contribute to the pathophysiology.

Lung Involvement
LWlg Involvement In SSe includes pleuritis and pleural effu-
sions, Interstitial lung disease (ILD), pulmonary arterial
hypertension (PAH). bronchiolitis, pulmonary veno-occlusive
disease. respiratory muscle weakness, and skln involvement of
the trunk restricting chest wall movement. Patients are also at
increased risk of lung cancer.
Significant ILD occ urs in 50'X. of patients with DcSSc
(85% of patients with anti- Scl-70 antibodies) and 35% of
patients W1th LcSSc. and is the main cause of disease-asso-
ciated mortality. The most common pathology pattern Is
nonspeci.fic Interstitial pneumonitis followed by usual Inter-
stitial pneumonitis. In patients with SSe. men are more
likely than women to develop ILD, and black persons tend to
have more severe disease. An abnormal FVC early In the dis-
ease course (first 5 years) Is highly predictive of developing
more severe disease, as Is fibrosis of more than 20% of lung
volume on baseline high- resolution CT scan. There is a
strong association between ILD and GERD. but whether
FIGURE 2 2 . upillary loops in systemic sclerosis. Top, Early changes with loss GERD contributes to ILD (through aspiration) is uncertain.
of capillary loop densily (or drop out) with marked dilatation of the remaining ILD Is common enough that all patients should undergo
capillary loops. Bottom, late changes with extensive loss of capillary loops.
pulmonary function tests (PFTs) and high-resolution CT at
Courtesy of Mario Cutolo, MO.
the time of Initial SSe diagnosis, and PFTs with DLCO should
be repeated every 6 to 12 months Cor 5 years. A decline In the
vomiting as a result of delayed gastric emptying. Gastric antral FVC of tO% or the DLCO oflS% within 12 months should raise
vascular ectasia (GAVE). also known as watermelon stomach, concern for progression.
is most common in patients with DcSSc and those with anti- PAH has a prevalence of 10% in SSe. Patients present with
RNA polymerase liJ antibodies. Bleeding ectasias can occa- eKertlonal dyspnea, and with more advanced disease may have
sionally cause significant blood loss. chest pain and edema as the right ventricle begins to fail.
Small Intestine bacterial overgrowth is common and can Patients with SSe should undergo echocardiography annually
cause malabsorption and diarrhea. Diagnosis is established by and more frequently for new or concerning symptoms. PFTs
hydrogen breath test, although an empiric trial of an tibiotics can provide a clue to underlying PAH; an FVCIDLCO ratio of
is often utilized. Small intestine dysrnotility can result in ~1.6 suggests the diagnosis. Right heart catheterization Js
pseudo-obstruction. Large Intestine dysmotility may lead to required for accurate diagnosis.
constipation. and vascular ectasia can occur in the large intes-
tine (watermelon colon). Some patients may develo p fecal Cardiac Involvement
Incontinence. Cardiac involvement with SSe is more common in men.
Clinically significant pericarditis occurs in lO'X, of patients.
I:J Kidney Involvement
One of the most concerning manlfesta!lons of SSe is sclero-
Myocardial fibrosis Is a more significant process caused by
vascular vasospasm leading to myocardial ischemia and fibro-
derma renal crisis. which affects 5%of patients and was previ- sis. The result may be heart failure and arrhythmias. Mortality
ously the chief cause of mortality. Risk factors Include DcSSc, in those with myocardial fibrosis Is quite high. Cl

52
Systemic Sclerosis

KEY POINTS KEY P 0 I NT S (continued)


• The earliest skin manifestations of systemic sclerosis are • Patients with scleroderma renal crisis characteristically
often diffusely swollen fingers/hands (sclerodactyly); present with manifestations of hypertensive emergency:
with time, the skin becomes th•ckened. atrophic, and risk factors Include diffuse cutaneous systemic sclerosis.
immobile. use of moderate- to high-dose glucocorticoids, and the
• Raynaud phenomenon is the most common early mani- presence of anti-RNA polymerase III antibodies.
festation of systemic sclerosis. • Lung involvement In systemic sclerosis includes pulmonary
• Symptoms of esophageal dysmotility and reflux are arterial hypertension and interstitial lung disease (ILD); ILD
common in all forms ofsystemic sclerosis and may offer is currently the main cause of disease-associated mortality.
an early clue to rhe diagnosis when associated with
other symptoms.
Management CJ
Table 31 outlines treatment options for manifestations of
(Continued)
SSe.

Cutaneous manifestations Methotrexate, mycophenolate, cyclophosphamide, and rituximab have been shown to improve sktn
scores; ior pruritus, antihistamines can be tried; low-dose prednisone can also help; laser therapy can
used for telangiectasias.
Musculoskeletal NSA!Ds, intra-articular or low -dose prednisone, hydroxychloroquine, or methotrexate can be used for
manifestations arthralgia/arthritis; methotrexate, azathioprine, or mycophenolate can b e used for significant myositis.
Raynaud phenomenon Cold avoidance; gloves; central warmth
Calcium channel blockers
Sildenafil; losartan; prazosin
Topical nitrates; low-dose aspirin
Digital sympathectomy
Gastroesophageal reflux Avoid eating dose t o bed time; head of bed elevation
disease
H2 blockers; proton pump inhibitors
Surgical fundoplication
Gastrointestinal dysmotility Metoclopramide (avoid long-term use); domperidone (limited access program)
Erythromycin; cisapride(limited access program)
Gastric antral vascular ectasia Ablation laser therapy
Small tntestine bacterial Rotating antibiotics
overgrowth
If fat malabsorption, cholestyramine
Pseudo-obstruction A promotility agent such as domperidone can be tried
If resistant, octreotide may be useful
Consttpation Fiber; stool softeners
Polyethylene glycol
Scleroderma renal crisis ACE inhibitors (typically captopril)
Dialysis
Interstitial lung disease Mycophenolate mofetil
Cyclophosphamtde
Lung transplantation
Pulmonary arterial Supplemental oxygen; treat right-sided heart failure
hypertension
Prostanoids; endothelin receptor antagonists; phosphodiesterase-S Inhibitors; soluble guanylate
cyclase inhibitors
Combination therapy
Lung transplantation

53
Mixed Connective Tissue Disease

ACE Inhibitors (typically captopril) can be lifesaving in Pregnancy


CJ
CONT.
patients with scleroderma renal crisis and should be titrated to
Pregnant patients with SSe may experience hypertensive dis-
control blood pressure. Dialysis may be needed temporarily.
ACE inhibitor therapy should be continued In scleroderma ease. p reecla mpsia. pretem1 delivery. and low birth weights. In
renal crisis even in the presence of a rising serum creatinine geneml, pregnancy does not make SSe worse, and some fea-
and the need for dialysis, as late improvement may occur. In tures (for example, digital ulcers) may actually improve.
patients at high risk for developing scleroderma renal crisis, Contratndications to pregnancy in patients with SSe include
the prophylactic use of calcium channel blockers seems to PAH and severe restrictive lu ng disease (FVC of <1 L); in the
otTer some protection. However, prophylactic use of an ACE setting of PAH, the hemodynamic changes of pregnancy can
inhibitor has not been shown to otTer protection and may put the mother at considerable risk. See Principles of
Increase mortality. Therapeutics for details on medications and pregnancy.
Cyclophosphamide can be used for stabilizing ILD. but KEY POINT
the impact is modest and may not persist beyond 1 year. • Pregnant patients with systemic sclerosis (SSe) may
Recent evidence suggests that mycophenolate mofetil is as use- experience hypertensive disease. preeclampsia, preterm
ful for stablllzing ILD as cyclophosphamide in SSe but wtth delivery. and low birth weights; contra indications to
less toxicity. Mycophenolate mofetil can also be used for long- pregnancy in patients with SSe include pulmonary
term therapy, maklng It the preferred agent for most patients arterial hypertension and severe restrictive lung disease.
with SSe-associated ILD.
Rituxlmab is a promising agent in SSe; In case series and
one controlled trial, it has stabilized or slightly Improved lung
function and skin ulcers. Nintedanib (a tyrosine kinase inhibi- Mixed Connective
tor) and plrfenidone (a synthetic molecule) were recently
approved by the FDA for treatment of Idiopathic pulmonary Tissue Disease
fibrosis; results from clinical trials in SSe are pending.
Overview
Autologous hematopoietic stem cell transplantation has been
studied for several years ln patients with severe SSe; one third to Despite long-standing attempts to define rheumatologlc diseases
one half of patients had slgnJficant improvement In skln thick- according to precise classification, some patients express only
ening and antibody status. However, cost and potential toxicity nonspecific, nondlagnostlc signs and symptoms (undlfterenti-
limit the utility of stem cell transplantation at present. CJ ated connective tissue disease [UCfD)), whereas others experi-
ence features of more than one disease simultaneously (overlap
KEY POINT$ syndromes). Mixed connective tissue disease (MCfD) Is a specific
• Treatmen t options for systemic sclerosis are mainly overlap syndrome that includes clinical manifestations of at least
symptomatic and organ system specific. two of the tollowing: systemic lupus erythematosus (SLE), poly-
• ACE inhibitors can be lifesaving in patients with sclero- myositis, and systemic sclerosis. Positive anti-VI-ribonucleopro-
tein (RNP) antibodies are the primary laboratorY feature.
derma renal crisis and should be titrated to control blood
See Table 32 for a comparison of MCfD, UCfD, and over-
pressure even in the presence of rising senun creatinine.
lap syndromes.

Mixed Raynaud phenomenon; arthntis; Pos1tive anti·Ul-RNP Disease/organ involvement specific


connect1ve puffy fingers; sclerodactyly; serositis; antibodies
Anti-inflammatones for symptoms; DMARDs
tissue disease esophageal dysmotility; myositis;
Fulfills criteria for at and/or immunosuppressives for arthntis or other
interstitial lung disease; PAH
least two of the major organ disease
following: SSe, PM, and
Vasodilators for PAH and Raynaud phenomenon
SLE
PPI for esophageal disease
Undifferentiated Variable; most common include Insufficient criteria for Same as mixed connective tissue disease
connective Raynaud phenomenon, arthralgia. any specific connective
tissue disease skin rash. cytopenia, and serositis tissue disease
Overlap Variable; will have features sat1sfymg Fulfills critena for two Same as mixed connective tissue disease
syndrome two distinct autoimmune diseases distinct autoimmune
(such as SLE, RA. PM, and SSe) diseases
DMAAO • disoa,....mo~ilying anllrheumotic drug; PAH • pulmonary arteriol hypertension; PM - polymyositis; PPI - prolon pump inhibitor; RA- rheumatoid orthritis;
RNP • nbonucleop<or.>~n; SLE • >y$temic lupus erylh•nustosus; SSe- >y$temlc sclerosis,

54
Cryst a l Arthr opathies

Epidemiology and of excessive levels of uric acid (hyperuricem.ia). Gout is


increasingly common, with a U.S. prevalence of approximately
Pathophysiology 4'X.. Factors contributing to this increase include dietary
MCfD is rare, and 80'Jr. of patients are women. B-cell abnor- changes, increasing obesity, and an aging population (the
malities and an association between anti- Ul-RNP antibody elderly account for the sharpest rise in gout over the past few
titer and disease activity have been described. decades). Nearly all patients with gout have comorbidities,
including hypertension, coronary artery disease, hyperlipi·
demia, diabetes mellitus, and chronic kidney disease (CKD).
C] Clinical Manifestations Comorbidities frequently complicate treatment by posing rel-
and Diagnosis ative contraindications for appropriate therapeutics (for exam-
See Table 32 for details on the clinical manifestations and diag- ple, CKD for NSA!Ds or colchicine).
nosis of MCfD. Raynaud phenomenon is a particularly com- Men reach a steady-state serum urate level after p uberty,
mon feature. Pulmonary arterial hypertension is the major whereas premenopausal women are generally protected from
cause of death, but severe kidney disease is uncommon. hyperuricemia by estrogenic effects. It is rare for premenopau-
Although antinuclear antibodies, rheumatoid factor, and anti- sal women to develop gout, whereas men usually have their
cyclic citrulllnated peptide antibodies are often present, the fir~i gout attack in the third to ftfth decade.
presence of anti- double-stranded DNA or anti-Smith antibod-
ies make SLE a more likely diagnosis. Cl Pathophysiology
Uric acid Is the end producl of purine metabolism, and about
Management two thirds of urate production derives from cellular turnover
of nucleic acids (purine bases); the remainder is derived from
Treatment for MCfD Is directed at the specific diseases and dietary purine intake. A smaiJ proportion of patients overpro-
organ system involvement manifesting in the individual duce urate on an inherited metabolic basis. Unlike most mam-
patient (see Table 32). mals, humans do not express url<:ase, an enzyme that converts
urate into highly soluble allantoin. Urate therefore accumu-
Prognosis lates and can precipitate as crystals in joints and other tissues
If serum concentrations exceed the saturation point. At nonnal
Prognosis of' MCfD varies depending on the spectrum and body temperature, the saturation point of urate is 6.8 mg/dL
severity of organ involvement. Patients with Initially mild dis- (0.40 mmoi/L). At lower temperatures (tor example, in the
ease may develop new and more severe manifestations over extremities), the saturation point is lower.
decades, Importantly Including interstitial lung disease and Nearly all urate is filtered at the glomerulus, but approxi-
pulmonary arterial hypertension. As with many other sys- mately 90'X, is reabsorbed in the proximal tubule; in most
temic inflammatory disorders, patients are at increased risk of patients with gout, hereditary underexcretion of uric acid
atherosclerotic cardiovascular disease. contributes to hyperuricemia (Table 33). Not all patients with
UCfD usually remains mild and stable but over time may hyperuricemia develop gout; about 20% of the U.S. population
evolve into a more specific autoimmune disorder, most com- is hyperuricemic compared with the 4% prevalence of gout.
monlySLE. The factors that lead to the development of gout In hyperu rice-
KEY POINTS mia are unclear, but degree ofelevation in urate level is predic-
• Mixed connective tissue disease includes clinical mani- tive. A serum urate level greater than 10 mg/dl (0.59 mmol/L)
festations of at least two of the following diseases: sys- is associated with a 30'X, likelihood of an initial attack of gout
temic lupus erythematosus. polymyositis. and systemic within 5 years.
sclerosis; anti- UJ ·ribonucleoprotein antibodies are typ· Acute gouty arthritis is triggered when tissue mac-
ically present. rophages ingest u ric acid crystals, lead ing to t he generation of
interleukin (IL)-lf!. IL-lf! is an inflammatory cytokine that
• Treatment for mixed connective tissue disease is directed
causes locaJ vasodilation, triggers production of other intl.am-
<1t the specific diseases and organ systems involved.
matory <.'}'tokines (tumor necrosis factor <X and IL-6). and both
rec ruits and activates neutrophils. Complement activation on
the surface of uric acid crystals also promotes neutrophil
Crystal ArthroF,athies recru itment. Neutrophils are p rimarily responsible for the
symptoms and signs of an acute gout attack.
Gout
Epidemiology Clinical Manifestations
Gout is characterized by intermittent painful inflammatory Three req uirements generally must be fulfilled for the onset
joint attacks, in response to crystals formed as a consequence of clinically apparent gout: 1) hyperuricemia, usually

55
Crystal Arthropathies

TABLE 33. Causes of Hyperuricemia

Primary renal uric acod underexcre toon (hereditary, renal tu bular basis)
Chronic kidney d isease of any cause (secondary uric acid underexcretion)
Uric acid overproduction due to primary defect in punne metabolism· PRPP synthetase overactivoty; HPRT deficiency
Conditions of increased cell turnover leading to purine/urate generation: leukemia/lymphoma; p soriasis; hemolytic anemia;
polycythemia vera
Drug-onduced hyperuncemia (agents reducing rena l glomerular filtration and/or tubular urate excretion): thoazide and loop diuretics:
cydosporine; low-dose salicylates; ethambutol; pyrazinamide; lead ongestion/toxicity
Diet-induced hyp eruricemia (agents high in purines or induci ng puri ne/urate b iosynthesis): alcohol; shellfish; red meat; hogh-fructose
corn syrup-sweetene d b everages and foods; imp act on serum urate gen erally is limited in the absence of oth er causes
HPRT - hypoxanthine-guanine pho•phoribo•ylcnnsferase; PRPP • phosphoribosylpy1ophosphaoo.
--------------------------- ------------------------------------------------~

long-stan ding, 2) uric acid deposition in the joints and/or soft In chronic recurrent gout, patients experience increas-
tissues, and 3) a reaction to phagocytosed crystals that leads to ingly frequent and severe. and often polyarticular, arthri tic
an acute inflammatory event. a ttacks. These attacks may eventually evolve Into a persistent
chronic arthritis.
Acute Gouty Arthritis Other patients with long-standing disease develop topha-
Cl Initial acute gouty attacks are typically monoartlcular, and ceous gout. Tophi are solid chalky white masses of uric acid,
;:.so% offrrst attacks in men involve the first metatarsophalan- surrounded by inflammatory cells and a rind oftlbrous tissue.
geal (MTP) joint oft he great toe (podagra). lnflammatory states They are located around joints and in soft tissues. with a pre-
such as infection . surgery, a nd myocardial infarction can pro- dilection for the extensor surfaces of the elbows, the distal
voke gouty attacks, possibly through volume and pH changes. Achilles tendon, the fingers (usually from the proximal inter-
The hallmarks of an attack are pain, tenderness. swelling, red- phalangeal joints distally) (Flgure 23), and the cartilaginous
ness, and warmth of the affected area. Attacks typically begin portions of the ears. The size ranges from millimeters to sev-
at night, and peak within 12 to 24 hours. The pain is often so eral centimeters in diameter. Tophi a re deforming. can Inter-
exquisite that even the touch of a sheet on an affected toe Is fere with function, and directly erode bone. Ulceration of
unbearable. Untreated, most gout attacks self-resolve within overlying skin can occur, and accompanying infection can be
days to a few weeks, although with long-standing disease, difficult to treat because tophi are avascular.
attacks can persist for months. In men with established dis-
ease, attacks eventually affect joints other than the first MTPs.
Including the proximal feet, ankles, and knees. and later virtu-
Diagnosis
A diagnosis of acute gout should be entertained in any patient
m
ally any joint, including the spine. Postmenopausal women with an acute monoarticular or ollgoartlcular arthritis. The
may present differently. with initial attacks often involving differential diagnosis is listed in Table 34. Although designed
finger joints already affected by osteoarthritls. For those with primarily for research, gout classlftcauon criteria are available
more severe or long-standing disease, polyarticular attacks
may occur. Soft tissues can also be involved, manifesting as
acute bursitis, periarthritis, and gouty panniculitis and cellu-
litis, the last of which can be m isdiagnosed as a refractory
bacterial soft-tissue infection. CJ

Intercritical Gout
Jntercritical gout Is the period between gout attacks. Early In
the disease course, the inte rcritical period can be years,
although most p atients will have a second gout attack within
2 years. As the disease progresses, the intercrit!cal period can
progressively shorten. Even in the a bsence of acute arthritis,
uric acid crystals still reside within joints and soft tissues, and
low-grade Inflammation persists in the absence of clinically
apparent systemic disease.

Chronic Recurrent and Tophaceous Gout F16 URE 2 3. Fingers ol a patient with numerous bulky tophaceous deposits of
Chronic recurrent gout and tophaceous gout result from a monosodium uricacid crystals, a consequence of years of gout with uncontrolled
failure to recognize or adequately treat gout at an earlier stage. hyperuricemia.

56
Crystal Arthropathies

TABLE 34. Differential Diagnosis of Acute Gouty Arthritis


Condition Comments
Infectious arthritis Presentation may be identical to gout. Infectious arthntis 1S usually monoarticular b ut can be
polyarticular Onset may b e less acute than g out. Gout and Infectious arthntis can coex1st.
Acu te calcium pyrophosphate crystal Pseudogout is less likely to p resent in t he g reat to e, but acute p resentations may ot herwise
arthritis (pseudogout ) b e identical to gout. Synovial fluid analysis can distinguish these entities. Gout and
p seud ogout can coexist.
Basic calc1u m p hosphate depos1t1on Basic ca lcium phosphate deposition 1n articular cartilage and periart1culartissues. Because
of their small size, basic calcium p hosphate crystals are unlikely to be seen in synovial fluid
on light microscopy except as aggregates st ained with alizarin red.
Trauma Trauma can lead to local p ain and swelling, with o r without a fracture, and can also trigger
a gout attack.
Other forms of inflammatory arthritis Clinical context, affected joints, and pattern o f arthnt1s are all helpful 1n distinguishing
(e.g., reactive arthritis, rheumatoid arthritis, these entities Synovial fluid analysis is particularly important when th~e is diagnostic
psoriat1c arthritis, acute rheumat1c fever) uncertainty.

for guidance, including as an online calculator (http://


CJ
co ~.
goutclassificationcalculator.auckland.ac. nz).
The gold standard for the diagnosis of acute gout is the
identification of negatively birefringent, needle-like uric acid
crystals within neutrophils In fluid obtained by arthrocente-
sis. Negatively birefringent means that under polarized light,
the crystals appear yellow when parallel and blue when per-
pendicular to the polarizing axis. Arthrocentesis is not always
completed when gout is suspected, but when infection is a
concern, joint aspiration for Gram stain and culture is
mandatory.
Findings that support an acute gout diagnosis include
onset of symptoms over several hours, first MTP joint or mid-
foot/ankle involvement, and severe pain. The involved joint(s)
are warm, red, swollen, and very tender. Low-grade fever may
be present. Measuring serum urate at the ti me of attack is
often done but may not be helpful, because levels may drop
precipitously during periods of acute systemic inflammation
(owing to the uricosuric effects of circulating cytokines).
Nonetheless. an elevated serum urate level increases the likeli-
hood of gout. Checking the serum urate 2 weeks after acute
gout resolution gives a more accurate measure of baseline
level. C- reactive protein and erythrocyte sedimentation rate
Fl GURE 2 4 . Periarticular bony erosions, appearing as punched-out bone
are elevated but are nonspecific. Joint fluid will have an lesions with disruption of bone cortex (overhanging edges), represent areas of
inflammatory leukocyte count (above 2000/I!L (2.0 x 109 / L] tophaceous monosodium uric acid deposition and accompanying inflammation in
and occasionally >100,000/~tL (100 x 109 /L). with neutrophil a patient with severe, tong-standing. inadequately treated gout.
predominance). Importantly, the diagnosis of acute gout. even Courtesy of Elai ne Karis, MO.
when conflrmed by the presence of Intracellular crystals in
synovial fluid, does not definitively rule out a concomitant Management
infection. There are three components to treating gout: 1) treating acute
Radiographs are not helpful in diagnosing acute gouty attacks, 2) prophylaxis to prevent future attacks, and 3) urate-
arthritis, but the characteristic radiographic changes of estab- lowering therapy. See Principle of Therapeutics for details on
lished disease (punched-out lesions with overhangjng edges of the medications described in this section.
cortical bone) may help confirm a history of previously unrec-
ognized gout (Figu.re 24). Newer imaging modalities (ultra- Treatment of Acute Gouty Arthritis
sonography and dual-energy en can demonstrate uric acid Treatment of acute gout focuses on anti-inflammatory therapy;
deposition in joints and are being studied for their possible colchicine, NSA!Ds, and glucocorticoids are all reasonable
role in acute diagnosis. options. Treatment choice should be determined by potential

57
Crystal Arthropathies

CJ drug
CONT
interactions and patient comorbidlties. The simplest is
colchicine. mg at the Orst symptoms of a gout attack, foi-
1.2

ACR, allopurinol should be initiated at 100 mg/d and titrated
in 100-mg increments as needed, and for those with stage 4 or
lowed 1 hour later by a 0.6-mg dose. Colchlcme Is most effective 5 CKD. allopurinol should be initiated at SO mg/d and titrated
when used early in attacks (<24 hours after onset) and is less In SO-mg Increments as needed. An uncommon but serious
useful when the attack is well established. High-dose NSAID complication of allopurinol is a hypersensitive rash that may
therapy for 5 to 7 days Js effective, as are glucocort!colds in any progress to DRESS (drug reaction with eosinophilia and sys-
form -intra-articular injection, intramuscular depot injection temic symptoms) syndrome; the drug should therefore be
(for example, depo-methylprednisolone, 40-80 mg), or an oral discontinued In most (if not all) patients who develop a rash.
"burst" of prednisone (for example, 0.5 rng/ kg/d, for S days). Febuxostat Is equally or more efficacious than allopurinol. It Is
Most gout attacks respond to a week or less of therapy, less likely to cause hypersensitiVity reactions than allopurinol
although more severe attacks may require a longer course of treat- and does not require dose adjustment In mild to moderate
ment For patients with severe and refractory attacks, or with CKD, but is expensive. There appears to be an increased risk of
contraindlcations to other treatments, off-label use of lL-llnhibi- heart- related death With febuxostat compared to allopurinol;
tors (anakinra or canakinumab) can be considered. Gouty ceUuli- FDA- mandated safety studies are ongoing.
tis, if present, should be treated as any other acute gout attack. Cl The uricosuric agent probenecid is infrequently used because
it is less effective than xanthine oxidase inhibitors and should be
Urate-Lowering Therapy avoided in patients With CKD or nephrolithiasis. Combination
Guidelines conflict as to whether patients with gout should therapy With a xanthine oxidase inhibitor and probenecid (or the
receive dietary counseling to reduce serum urate levels. If rec- incidentally uricosuric agents Josartan and fenofibrate) may be
ommended, patients may limit intake ofshellfish, oily 1ish, red more effective than a xanthine oxidase inhibitor alone. A more
meat, high-fructose foods, and alcohol. Overly strict diets that elf~ uricosuric agent, lesinurad, was recently FDA approved as
are unlikely to be complied with are not helpful, however. add-on therapy to allopurinol or febuxostat; reversible l<idney
Weight loss (when appropriate) and increasing dairy intake Insufficiency may develop in some patients.
may also lower serum urate levels. In patients with hyperten- For patients with severe recurrent and/or tophaceous gout
sion, agents other than diuretics should be used; losartan, for intolerant or resistant to standard therapies, pegloticase Is an
example, has mild uricosuric effects. option. Infused every 2 weeks, pegloticase lowers serum urate
lt is important to note that two recently published guide- to nearly zero, but 30o/o to SO% of patients develop antibodies to
lines differ regarding the role of pharmacologic urate-lowering the drug within a month, rendering it ineffective and increas-
therapy in patients with gout. The 2016 American College of ing the likelihood of infusion reactions. Pegloticase should be
Physicians gUideline (http:l/annals.org/aim/article/ 2578528/ discontinued in any patient for whom it ceases to work. Cl
management-acute- recurrent-gout-c linical- practice-guide
line-from-american-college) notes a lack of evidence support- Prop hylaxis
ing a specific target level for urate lowering; this guideline When initiating urate-lowering therapy. mobilization of uric acid
stresses diS<.'tlssing che risks and benefits of urate-lowering crystals from the joints and soft tissues can provoke acute attacks.
therapy with patients and suggests a "treat to avoid symptoms" Accordingly. patients starting urate-lowering therapy should be
approach without specifically considering the serum urate lev- placed on anti-Inflammatory prophylaxis to prevent flares. Fbr
els. The 2016 European League Against Rheumatism (EULAR) patients without tophi, the ACR recommends continUing prophy-
recommendations support a •treat-to-target" approach (con- laxis for at least 6 months and 3 months after achieving the target
sistent with the 2012 ACR gout guidelines), reducing the serum serum urate level. ln patients with tophi, prophylaxis should be
Llrate level to less than 6.0 mg/dL (0.35 mmol/L) in patient'> continued for 6 months following achievement of the target serum
without tophi and less than 5.0 mg/dL (0.30 mmoi/L) in urate level and resolution of tophi. Most patients tolerate colchi-
patients with tophi. Both the ACR and EULAR recommend Cine, 0.6 mg (once or tWice daily); for those who do not, low-dose
urate-lowering therapy for patients With gout plus any of the NSAIDs or glucocorticoids are appropriate substitutes.
following: (1) ~stage 2 CKD; (2) ~2 acute attacks per year; (3) one
UY POINTS
or more tophi; or (4) uric acid nephrolithiasis.
Contrary to prior practice, urate-lowering therapy can be • The gold standard for the diagnosis of acute gout is the
CJ initiated during an acute attack if adequate anti-inflammatory ident ification of negatively birefringent, needle-like uric
acid crystals within neutrophils in fluid obtained by
therapy is concurrently started; doing so may improve compll-
a.nce in some patients. arthrocentesis.
Three classes of urate-lowering therapy are available: • Colchicine, NSAIDs. and glucocorticoids are all reasona-
xanthine oxidase inhibitors (reduce urate production). urico- ble options for treatment of acute gout; choice should
suric agents (decrease renal urate resorption). and peglotlcase be determined by potential drug interactions and
(a uricase). A xanthine oxidase inhibitor, either allopurinol or patient comorbidities.
febuxostat, is the recommended first-line therapy. Allopurinol (Continued)
is FDA approved for doses up to 800 mg/d. According to the

58
Crystal Art hropathies

KEY P 0 I N TS (COIItlnued)
HVC • The 2016 American College of Physicians g·uideline
notes a Jacl< of evidence supporting a specific target
level for umte lowering; the guideline stresses discuss-
Ing the risl<s and benefits of urate-lowering therapy
w ith patients and suggests a "treat to avoid symptoms"
approach without specifically conside ring the seru m
urate levels.
• The Americ-an College of Rheumatology and the
Europea n League Against Rheumatism both recom-
mend a "treat-to-target" approach. reducing the sen1m
urate level to less than 6.0 mg/dL (0.35 mmoi/L) in
patie nts without tophi and less tha n 5.0 mg/d L {0.30
mmol tLl in patients with tophi. FIGURE 2 5 . Cartilage calcification (choodrocalcinosis) of the knee. This
• Patients starting urate-lowering therapy should be placed radiograph shows linearly arranged calcific deposits in the articular cartilage (arrow}.
on antHntlammatory prophylaxis to prevent flares.
pophalangeal (MCP) joints, in descending order. Asymptomatic
CPPD is common in the elderly and in osteoarthritic joints, and
it may be a precursor of osteoarthritis with CPPD.
Calcium Pyrophosphate
Deposition
Deposition of calcium pyrophosphate (CPP) crystals in and on
Acute Calcium Pyrophosphate Crystal Arthritis
Acute CPP crystal arthritis (pseudogout) typically presents as a
CJ
cartilaginous surfaces can provoke acute inflammatory arthri- monoarticular inflammatory arthritis, characteriZed by sud-
tis that is clinically similar to acute gout. Calcium pyrophos- den onset of swelling, pain, loss of function, tenderness. and
phate deposition (CPPD) is less well characterized than gout, warmth of the affected joint, usually a knee or wrist. Similar to
but tour subgroups are described: asymptomatic CPPD; acute acute gout, attacks may be provoked by systemic Insults such
CPP crystal arthritis; chronic CPP crystal inflammatory arthri- as major surgery or acute illness. Attacks are usually milder
tis; and osteoarthritis with CPPD- than those of gout, but If untreated can persist for months.
Definitive diagnosis requires identification of CPP crystals
Epidemiology and Pathophysiology (along with leukocytes) In synovial fluid; in contrast to uric
CPPD primarily affects the elderly, and prior joint damage is a acid crystals, the CPP crystals are rhomboid shaped and posi-
significant risk factor. The risk tor cartilage calcification rively birefringent under polarized light. Radiographic evi-
(chondrocalcinosis) doubles tor every decade past 60 years, dence of cartilage calcification in elderly patients with acute
and nearly half of patients in their late 80s have CPPD. For monoarticular arthritis is suggestive but not diagnostic of
younger patients with CPPD, consideration of contributory acute CPP crystal arthritis. Cl
metabolic disease (hyperparathyroidism, hemochromatosis,
hypophosphatasia, hypomagnesemia) is warranted. Chronic Calcium Pyrophosphate Arthropathy
The pathophysiology of CPPD Is incompletely under- Chronic calcium pyrophosphate arthropathy may be present
stood. Pyrophosphate produced by chondrocytes likely pre- as two patterns: (1) chronic calcium pyrophosphate (CPP)
cipitates wlth calcium to form CPP crystals, which then crystal inflammatory arthritis and (2) osteoarthritis with cal-
activate inflammatory pathways resulting in an acute arthritic cium pyrophosphate deposition (CPPD). Chronic CPP crystal
attack. CPPD may also d rive osteoarthritis by inducing proin- inflammatory arthritis is a polyarthritis involving the wrists
flammatory activity In chondrocytes and synovial tlbroblasts, and MCP jolms (•pseudo-rheumatoid arthritis"); It is rare and
remlting in cartilage damage. Epidemiologic evidence suggests difficult to treat. Osteoartluitis with CPPD manifests as typical
that cartilage calcification might play a role In osteoarthritis of osteoarthritic findings involving joints not commonly associ-
the knees and wrists, but not at the hip. ated with osteoarthritis (such as shoulders or MCP joints);
radJographlc CPPD often precedes the onset of osteoarthritis.
Clinical Manifestations and Diagnosis suggesting a causal role for the CPP.
Asymptomatic Calclwn Pyrophosphate Deposition
Asymptomatic CPPD occurs with radiographic changes in the
absence of dlnical symptoms. Cartilage calclflcatlon appears as
Management I:J
Because there Is no known mechanism for dissolving or pre-
a linear opacity below the surfa<.-e of articular cartilage (FJgUre venting the formation of articular CPP crystals, treatment
25). It most commonly occurs in the knees, wrists (triangular aims at abrogating the inflammatory manifestations of the
fibrocartilage), pelvis (pubis symphysis), and metacar- dlsease (Table 3S). C1

59
Infectious Arthritis

TABLE 35. Management of Calcium Pyrophosphate Deposition


Clinical Presentation TNtatment/Comments

Cartilage calcification (chond roca lcinosis) No specific treatment


Acute calcium pyrophosphate crystal Local treatment: joint aspi ration, followed by intra-articular glucocorticoid injection,
arthritis (pseudogout} intramuscular glucocorticoid injection; joint immobilization; ice packs
Systemic treatment: NSAI Ds: colchicine; glucocorticoids (oral or parenteral)
Prophylaxis if recurrent attacks (three or more annual attacks): low-dose colchici ne or daily
NSAIDs(with gastroi ntestinal protection)
Chronic calcium pyrophosphate crystal Lo w -dose colchicine or da1ly low-dose NSAIDs (w ith gastrointestinal p rotection); low·
inflammatory arthritis dose glucocortico1ds
Osteoarthritis with calcium pyrophosphate Same treatment as osteoarthritis without CPPD (e.g., physical therapy; pain control; local
deposition (CPPD) glucocorticoids}

KEY POINTS
Infectious Arthritis
• Asymptomatic calcium pyrophosphate deposition is
characterized by radiographic changes. including carU- Diagnosis CJ
lage calcification (chondrocalcinosis). in the absence of Clinical Manifestations
clinical symptoms. Infectious arthritis typically presents with pain, swelling,
• The presentation of acute calcium pyrophosphate warmth, and erythema of the affected joint, accompanied by
crystal arthritis (pseudogout) resembles gout : how- fever and constitutional symptoms. Knee infection is com-
ever. attacks are usually milder. and crystals are mon, but any joint may be affected. Previously damaged joints
rhomboid shaped and positively birefringent under are at increased risk for involvement. Monoarthritis or new
polarized tight. inflammation of a single joint in a patient with well-controlled
• Chronic calcium pyrophosphate crystal inflammatory inflammatory arthritis should prompt evaluation for infectiOn.
arthritis often involves the wrists and metacar- Physical examination commonly shows functional loss of
pophalangeal joints ("pseudo- rheumatoid arthritis"). both active and passive range of motion in addition to signs of
inflammation. Skin examination can reveal signs suggesting
• Osteoarthritis with calcium pyrophosphate deposition
gonococcal infection (pustular skin lesions) or portals of entiy
causes typical osteoarth1itic findings involving joints
(scratches, bites, or thorns).
not commonly associated with osteoarthritis (shoul-
ders, metacarpophalangeal joints).
laboratory and Imaging Studies
In patients with suspected infectious arthritis, the most
Basic Calcium Phosphate important diagnostic procedure is expeditious arthrocentesis.
The synovial fluid should be sent for Gram stain, culture (bac-
Deposition terial and, when indicated, mycobacterial and/or fungal) , leu-
Basic calcium phosphate (BCP) deposition occurs in the elderly kocyte count, and crystal analysis. Synovial fluid levels of
and forms in the articular cartilage; in contrast to CPPD, BCP glucose, lactate dehydrogenase, and total protein do not aid in
crystals also often deposit perlarticularly. BCP is thought to diagnosis and should not be obtained. Depending upon the
rnrely cause inflammatory arthritis and periarthritis, most clinical picture, the fluid may be sent for polymerase chain
classically manifesting in elderly women as "Milwaukee shoul- reaction to detect Mycobacteria or Borrelia burgdorferi DNA.
der, • an inflammatory periarthritis and arthritis of the s houl- Bacterially infected synovial fluid is cloudy and less
der. leading to progressive destruction of the rotator cuff and viscous than that seen in noninflammatory arthritis.
glenohumernl joint. Diagnosis is usually clinical; imaging of Elevated synovial tluid leukocyte counts are typically greater
BCP crystals requires special stains and/or electron micros- than 50,000/J..IL (SO x 109 /L) and often exceed 100,000/!!L
copy and is rarely performed. (100 x 109 /L). with a polymorphonuclear cell predominance.
KEY POINT Such high leukocyte counts may occur in other conditions
• Basic calcium phosphate deposition can ra rely cause (for example, crystal arthropathies) but should be presumed
Milwaukee shoulder, an inflammatory arthritis and peri- infectious until proven otherwise. Notably, the presence of
arthritis of the shoulder occurring in elderly women crystals does not exclude the possibility of co-infection.
that leads to progressive destruction of the rotator cuff Patients with gonococcal, mycobacterial, or fungal infec-
and glenohumeral joint; diagnosis fs usually clinical. tions, injection drug users. or those who are immunocom-
promised may have lower synovial fluid leukocyte counts.

60
Infectious Arthritis

A positive Gram stain for bacteria should be considered


CJ definitive, but the sensitivity of the test is inadequate for a
CONT.
negative result to rule out infection when suspicion is high.
Synovial fluid cultures are usually positive in bacterial infec- Age >80 years
tions unless antibiotics were administered prior to arthro- Alcoholism
centesis. The peripheral blood leukocyte count, erythrocyte Cutaneous ulcers or skin Infections
sedimentation rate, and C-reactive protein are often ele-
Diabetes mellitus
vated; however, normal levels do not exclude the diagnosis,
End-stage kidney disease
and elevated levels may occur in inflammatory arthritis and
other conditions. History of intra-articular glucocorticoid injection
Blood cultures should always be drawn before antibiotic Injection drug use
administration. When gonococcal arthritis is suspected, uro- Low socioeconomic status
genital. rectal, and pharyngeal specimens should also be Malignancy
obtained tbr nucleic acid amplification testing.
Patients receiving immunosuppressive agents
Plain radiographs are usually normal early In the course
Preexisting arthritis or joint damage (e.g .• rheumatoid arthritis
of the infection. but baseline films are helpful to identify other or osteoarthritis}
cUseases or contiguous osteomyelitiS. Later radiographs (weeks
Prosthetic joint or recent joint surgery
to months) often show nonspeclflc changes. In advanced
Sickle cell disease
Infection. periosteal reaction, marginal or central erosions.
and subchondral bone destruction may be seen. Bony ankylo- Risk Factors for Gonococcal Arthritis
sis Is a late sequel. In joints that are difficult to evaluate clini-
Younger, sexually active patients
cally or have complex anaromfc structures. ultrasonography.
CT, and MRI can delineate the extent of the effusion and iden- Risk Factors for Lyme Arthritis
tify early bony changes. CJ Travel or residence in an endemic area
KEY POINTS Documented tick bite or erythema chronicum migrans
• Infectious arthritis typically presents with pain, swell- Risk Factors fOt' Mycobacterial or Fungal Arthritis
ing, wam1th. and erythema of the affected joint. accom-
, Patients with HIV infection or other immunosuppression
panied by fever and constitutional symptoms; monoar-
thritis or new inflammation ora single joint in a patient Patients receiving tumor necrosis factor CJ. inhibitors
with well-controlled inflammatory arthritis should Travel or residence in an endemic area
prompt evaluation for infection. Risk Factors for Vlra I Arthritis
• Diagnosis of infectious arthritis is confirmed by arthro-
Chronic viral infection (HIV, hepatitis 8, hepatitis C)
centesis and evaluation of the synovial tluid for Gram
Exp osure to infectious agent or vaccine (e,g., rubella)
stain, culture, leukocyte count, and crystal analysis.
Travel or residence in an endemic area
Exposure to affected children (parvovirus. rubella)

Causes
See MKSAP 18 Infectious Disease tor details on the specific Approximately 75% of adult nongonococcal infectious
infections and diseases discussed in this section. arthritis is caused by gram-positive cocci. with Staphylococcus

m Infection with Gram-Positive Organisms


Bacterial arthritis is a medical emergency requiring Jmmedi-
aureusbeing the most frequent microorganism in both native
and prosthetic joints. Staphylococcus epldermldls infeclions
occur more commonly in prosthetic than in native joints.
ate diagnosis and treatment. Risk factors include an lmmuno-
compromised state (including diabetes mellitus). injection lnfedion with Gram-Negative Organisms
drug use, joint surgery, and having a prosthetic joint. A recent Disseminated Gonococcal Infection
"dirty• skin break may be a contributing factor. Infections Neisseria gonorrhoeae most typically occurs in younger, sexu-
occur very rarely after invasive procedures such as arthrocen- ally active Individuals. Disseminated infection occurs in l'X> to
tesis or joint injection (Table 36). Most joint infections are 3% of patients infected with N. gonorrhoeae, with arthritis a
monoartlcular and derive from hematogenous seedlng of common feature.
synovium. Polyarticular infectious arthritis may occur In Disseminated gonococcal infection may present in one of
injection drug users. those with systemic inflammatory disor- two ways, although there may be overlap. The first is the
ders such as rheumatoid arthritis, and in patients with over- arthritis-dermatitis syndrome, a triad of tenosynovitis, der-
whelming sepsis. matitis (usually painless pustular or vesiculopustular lesions)

61
Infectious Arthritis

Cl (Figure 26), and polyarthralgia without frank arthritis. Fever,


chills, and malaise are common. Inflammation of multiple
B. burgdor{erl DNA can be detected by polymerase chain reac-
tion In synovial fluid.
CONT. tendons of the wrists. fingers, ankles, and toes distinguishes
this syndrome from other forms of infectious arthritis. The Mycobacterium tuberculosis
second presentation is a purulent arthritis. usually without The most common musculoskeletal manifestation of tubercu-
associated skin lesions or fever. Patients present with acute losis is vertebral osteomyelitis (Pott's disease). usually resulting
onset of mono or oligoarthritis; the knees, wrists, and ankles from the hematogenous spread of Mycobacterium tuberculosis
are most commonly involved. Into the cancellous bone tissue of the vertebral bodies. Patients
Patients with the arthritis-dermatitis syndrome are more usually have a primary pulmonary focus or extrapulmonary
likely to have positive blood cultures. whereas those with foci such as the lymph nodes. Predisposing factors for skeletal
purulent arthritis are more likely to have positive synovial tuberculosis Include previous tuberculosis infection, malnutri-
fluid cultures. Nucleic acid amplification testing should also be tion, alcoholism, diabetes mellitus. and H!V Infection. The
obtained on samples from genital, rectal, and pharyngeal sites. onset of symptoms is commonly insidious, and disease pro-
gression is slow, sometimes resulting in delayed diagnosis.
Nongonococcal Gram-Negative Organisms Peripheral arthritis may also occur and typically presents
Aerobic gram-negative bacilli are the predominant cause of with chronic pain in a single weight-bearing joint such as the
nongonococcal gram-negative joint infections. Predisposing knee, with only limited swelling. Concurrent pulmonary
factors Include injection drug use, advanced age. and an tuberculosis is present in a minority of these patients.
immunocompromised state. Pseudomonas aerugtnosa can be Musculoskeletal tuberculosis iniections are chronic and indo-
seen in Infection related to injection drug use. Patients with lent: laboratory indicators of Inflammation may be normal.
sickle cell anem.la may become infected with Salmonella. Synovial Ouid shows only nonspecific Inflammation, and radi-
Gram -negative anaerobes account for only 5% to 7% of bacte- ographic abnormalities may be delayed. Synovial biopsy is
rial arthritis cases, most commonly in prosthetic joint infec- usually necessary for diagnosis because the yield of synovial
tions and/or immunocompromlsed hosts. fluid smear and culture is Jess than SO%.

Lyme Arthritis Fungal Infections


Although arthralgia and myalgia often occur at the earlier Fungi are uncommon but important causes of bone and joint
stages of Lyme disease, Lyme arthritis is a late-stage manifesta- infections. Risk factors Include immunosuppression and exposure
tion. It Is typically monoarticular or oUgoarticular, most com- to the fungi in endemic areas. Travel and immigrn.tion have broad-
monly in the knee. It should be suspected in patients who may ened the risk of some fungal infections to non-endemic areas.
have had untreated or incompletely treated Lyme disease, Fungal joint infection may be indolent and more difficult to diag-
although not all patients will have a history compatible with nose than other bone and joint infections. Fungal causes of osteo-
prior Lyme disease. AU patients with Lyme arthritis, however, myelitis and arthritis Include coccidioidomycosis, blastomycosis,
should have positive enzyme-linked immunosorbent assay cryptoeoccosis. candidiasis, and sporotrichosis. Fungal artluitis is
(ELISA) and Western blot serologies; this is the major means of usually a result of hematogenous dissemination but can occur after
establishing the diagnosis of Lyme arthritis. Additionally, direct inoculation of the joint Diagnosis of some fungal infections
(for example. blastomycosis and sporotrichosis) can be made by
synovial fluld examination and culture, whereas serologic testing
.and synovial biopsy may be needed for other organisms (such as
Coccidioides). Synovial fluid leukocyte count<: vary. CJ

Viral Infections
HIV
HIV Infection is associated with several musculoskeletal dlsor-
deiS, includ.i ng painful articular syndrome, HIV-assoclated
arthritis, reactive arthritis, inlettlous arthritis, and diffuse
infiltrative lymphocytosis syndrome (OILS). Painful articular
syndrome consists of bone and joint pain, especially in t he
lower extremities in an asymmetric pattern, lasting less than 24
hours. HrV-associated arthritis is a nondestructive arthritis that
jnvolves joints of the lower extremity in an oligoarticular pat-
FIGURE 2 6 . Disseminated gonococcal infection can present as a febrile
tern and usually la:.ts less than 6 weeks. Reactive arthritis Is the
arthritis-dermatitis syndrome with migratory polyarthralgia that may evolve to
septic arthritis. tenosynovitis, and painless skin rash that may involve the palms primary torm of spondyloarthritls seen In patients with HlV
and soles. Skin lesions ca n vary from maculopapular to pustular, often with a infection and is likely due to a response to other sexually trans-
hemorrhagic component mitted or enteric infections, rJ.ther than the HIV itself. Reactive

62
Infectious Arthritis

anhrltls in HIY may take a chronic relapsing course and may Mosquito-Borne Viruses
be accumpanied by enthesopathy and mucocutaneous mani- Zika, dengue, 1\nd chikungunya viruses are transmitted by
festations. Psoriatic arthritis Is not more common but is often Aedes mosquitos m tropical areas. Zlka also can be vertically
more severe in patients with HIV infection, with disabling and sexually transmiued. and localized transmission in !he
enthesitis and joint erosion as well as axial involvement includ- southern United States Is now reported. All of these viruses
ing sacroiliilis. OILS Is a rare condition that resembles SjOgren can cause musculoskeletal symptoms In addition to fever,
syndrome but with CDS rather than CD<1 cell infiltration. rash, and headache. In chtkungunya, arthritis is a predomi-
nant feature: patients may experience synovial thickening and
Hepatitis tenosynovitis, often Involving the fingers and wrists. Zil<a may
Hepatitis B virus infection causes a self-limited arthritis in up to also manifest with a symmetric polyarthritis, whereas dengue
25% of Infected patlentl; during a prodromal stage prior to the tends to produce more arthralgia. The musculoskeletal symp-
onset of jaundice. Joint involvement can be sudden and severe, toms of Zika and dengue tend to subside within a couple of
with a pattern thut is usually symmetric, but c::m be migratory or weeks, whereas those ofchlkungunya usually continue longer,
additive. Hand and knee joints are most often affected; wrists, resembling rheumatoid arthritis (without rheumatoid factor
ankles, elbows, shoulders, and other large joints may be involved or anti· cyclic citrullinated peptide antibodies) in some cases.
as well. Morning stiffness is common. Fusiform swelling of the Diagnosis of these mosquito-borne diseases is based on clint-
small joints of the hand may be found on physical examination. cal features and serologic testing. Polymerase chain reaction
Patients with chronic hepatitis B vims infection may have recur- testing on blood samples is also available.
rent but self-limited poly.~rthralgia or polyarthritis; it is not
known to progress or cause joint damage Both hepatitis Band C Prosthetic Joint Infections [:J
v1rusinfection maybeaccomp:mied by the presence of rheuma- Bacterial infection complicates 1% to 3% of total knee and hip
toid factor, which can cause diagnostic confusion. replacements. higher rates are seen in immunocompromised
Acute hepatitis C virus infection can cause acute-onset patients and those with rheumatoid arthritis. Infection with
polyarthritis. Including the small hand joints, wriSts, shoul- gram positive organisms such as S. au reus is most common.
ders. knees, and hips. One third of pallents have an oligoar- Prosthetic joint infections are divided into early onset
thritis. Chronic hepatitis C V1rus infection is often associated (<3 months after placement). delayed (3 to 24 months postsur
with circulating Immune complexes, which may produce the gery). and late onset (>24 months after placement). Early and
clinical syndrome of mixed essential cryoglobulinemia (arthri- delayed infections are usually related to surgical contamina-
tis, glomerulonephritis, and vasculitis). tion at the time of the implantation, whereas late Infections
result from hematogenous seeding of the joint. Early and late
Parvovirus 819 prosthetic joint infections typically present with pain. warmth,
Up to 6Q'X, of adults with parvovirus Bl9 experience arthritis. effusion. and fever. Peripheral leukocyte count and inflamma-
It often presents acutely, is symmetric and polyarticular, and tory markers are usually elevated. Delayed infections. how-
typically involves the proximal small joints of the hands. ever, may be more difficult to recognize because symptoms are
Parvovirus Bl9 arthritis should be suspected when appropriate less severe. and they are often caused by less virulent microor-
clinical features are present in someone who has exposure to ganisms such as coagulase-negative staphylococci and other
children, such as teachers and caregivers. Acute parvovirus skin organisms. CJ
infection is diagnosed by detecting anti-parvovirus IgM anti-
bodies in the serum. Anti-parvovinJs lgG antibodies are highly KEY POINTS
prevalent In the general population and indicate prior Infec- • Approximately 7S'X, of adult nongonococcal infectious
tion. Joint symptoms may persist for weeks to months. arthritis is caused by gram-positive cocci. with
Staphylococcus au reus being the most frequent micro-
Rubella organism in both native and prosthetic joints.
Arthritis is uncommon In childhood rubella Infection. but up • Disseminated gonococc..ll infection may present as the
to 6Q'X, of adul ts with rubella develop joint symptoms, mainly arthritis dermatitis syndrome (tenosynovitis. dermatitis.
arthralgia. Joint Involvement is usually symmetric and migra- and poly.~rthralgla) or as a purulent arthritis 'vith pain
tory, with resolution of most symptoms within 2 weeks. The and swelling but usually without skin lesions or fever.
small joints oft he hands. wrists, elbows, ankles, and knees are
• Lyme arthritis is a late-stage manifestation of the
most commonly affected. TenOSYnOV1tis and carpal tunnel
disease and is typically monoarticular or oligoar-
SYndrome also occur. The pathogenesis of rubella arthritiS is
ticular. most commonly m the knee. and inflamma-
thought to be due to immune complexes and/or persistence of
tory: diagnoSIS should be suspected in patients who
the virus In synovial cells or joint macrophages. Diagnosis Is
may have had untreated or incompletely treated
usually made by detection of lgM antlrubella antibodies: the
Lyme disease.
virus may also be cultured from the nasopharynx or joint tis- (Concinued)
sues. Joint fluid findings are inflammatory.

63
Infectiou s Arthritis

KEY P 0 IN TS (continued)
• Mycobacterium tuberculosis arthritis typically presents
Management CJ
Jn patients with suspected infectious arthritis. blood and
as chronic pain in a single weight ·bearing joint such as synovial cultures must be obtained before treatment, but
the knee, with only limited swelling; concurrent pul- empiric antibiotic therapy should be started while awaiting
monary tuberculosis is present in a minority of patients. culture results (Table 37). In suspected bacterial arthritis,
• Early and late prosthetic joint Infections usually present the initial antimicrobial coverage should be broad and
wilh pain of the joint, warmth, effusion. and fever; account for host factors such as immunosuppression as well
delayed infections may be more dirtlcult to recognize as likely causative microorganisms and regional antibiotic
because they are often caused by less vim lent microor- sensitivity data. Antibiotics should initially be given paren-
ganisms, and symptoms may be less severe. terally. Given the high prevalence of methicillin-resistant

TABLE 37. Infectious Arthritis Treatment Based on the Suspected Pathogen


Likely or Identified Pathogen First-Line Therapy Second-Line Therapy Comments
Gram-Positive Cocci
If MRSA is a concern (risk factors or Vancomycin Cflndamycin; daptomycin;
known MRSAcarrier) Jinezolid
MSSA Nafcillin or cefazolin Narrow treatment to MSSA
coverage based on sensitivity
data.

Gram-Negative Ba cilli

Enteric gram-negative bacilli Third generation Fluoroquinolones


cephalosporin (e.g.,
ceftriaxone or cefotaxime)
Pseudomonas aervginosa Ceftazidime; cefepime; Carbapenems; aztreonam;
piperacillin-tazobactam fluoroquinolones

Gram-Negative Cocci

Neisseria gonorrhoeae IV ceftriaxone for at least Fluoroquinolones (only if In the absence of specific culture
7 days plus 1 gram of oral culture sensitivities confirm sensitiVIty data, "stepping down•
azithromycin x one dose susceptibility) to oral therapy of any type is no
longer recommended due to
increasing res1stance of
N. gonorrhoeae to commonly
used oral agents.

Gram Stain Unavailable or lncondusive

Likely pathogen depends on Vancomycin, or vancomycin Appropriate to start with broad


patient risk factors: consider MRSA, +third generation antibiotic coverage and narrow
and gram-negative organism if cephalosporin, or coverage if culture data
immunocompromised, at risk for antipseudomonal antibiotic become available.
gonococcal infection, or with joint if pseudomonas suspected
trauma; also consider community
patterns of mfection.
Borrelia burgdorferi (Lyme arthritis) Oral doxycycline or If inadequate response or
amoxicillin x 28 days concurrent neurologic findings,
IV ceftriaxone x 28 days.
Mycobacterium tuberculosis 3- to 4-drug treatment Duration may vary from 6 months
(e.g., isoniazid, or longer depending on drug
pyraz1namide, rifampin, regimen (shorter treatment if
ethambutol, streptomycin) rifampin 1s used).
Fungal infections Amphotericin 6, Prolonged treatment courses of
echinocandin, or azoles several months may be needed;
(fluconazole, itraconazole, maintenance therapy may be
voriconazole, posaconazole), required in high-risk patients.
depending on suspected
organism or culture data

rv • fn lta,.,nous; MRSA • melhloll•n·re•iS'.ant Staphylococcus aureus; MSSA • methi<:il!tl\-sen$itive Smphylococcus aureus.

64
Systemic Vasculitis

S. au reus, vancomycin is often initially used to cover gram- Treatment of prosthetic joint infections Is challenging
CJ
CONT.
positive cocci. The duration of treatment depends on the and requires early surgical consultation. Orthopedic
causative organism and patient response. but usually lasts 2 implants serve as a nidus for microorganisms, and the avas-
to 4 weeks. cularity of the infected hardware limits antibiotic pene-
An infected joint must also be adequately drained. trance. Many patients require removal of the orthopedic
Needle aspiration is an acceptable approach and should be device as part of a two-stage procedure, with reimplanta-
performed regularly (usually daily) as long as there is an effu- tlon of a new device after an appropriate course of intrave-
sion. Ultrasound guidance may improve the ability to fully nous antibiotics. Cl
drain the joint. Surgical drainage is an equally good alterna-
KEY POINTS
tive and is more inunediately definitive. Additionally. surgical
drainage is required for joints that are not easily accessible • In patients with suspected infectious arth ritiS. blood
for needle aspiration (for example, sternoclavicular, ster- and synovial cultu res must be obtained before treat-
nomanubrial, shoulder. and hip joints). ifthere is evidence of ment, but empiric antibiotic therapy should be started
soft- tissue extension of Infection, or If the clinical response while awaiting culture results.
to antimicrobial therapy is inadequate. The goal of surgery Is • In addition to antibiotic therapy, an Infected joint must
to remove aJJ purulent material and nonviable tissue and, in also be adequately drained by needle aspiration or sur-
some cases, to perform synovial biopsy or synovectomy. gical drainage.
Antibiotic treatment is recommended for all patients with • Many patients with prosthetic joint infections
Lyme arthritis. Approximately 90% of patients will respond to req ui re removal of the orthopedic device as part of a
a 28-day course of oral doxycycline, amoxicillin, or cefuroxime two-stage procedure. with reirnplantation of a new
axetil. Patients with incomplete responses may need treatment device after an appropriate course of intravenous
with a second course or a more aggressive drug regimen, usu- antibiotics.
ally intravenous ceftriaxone. Treatment beyond 1 month of
ceftriaxone offers no benefit and should not be employed.
Antibiotic-refractory Lyme arthritis occurs in SIO% of patients,
probably represents a progression to sterile autoimmune Systemic Vasculitis
arthritis. and responds to synovectomy or treatment with
disease-modifying antirheumatic drugs such as hydroxychlo- Overview
roquine or methotrexate. Vasculitis is inflammation of blood vessels, including the
Mycobacterial joint infections require at least 6 to capillaries, arteries, and veins. Clinical manifestations result
9 months of therapy. Treatment of arthritis associated with from tissue ischemia associated with the involved vessels.
viral infections Is largely supportive, although specific viral Vasculitis may be primary, secondary to an autoimmune
therapy is appropriate for HIV and hepatitis C infection: disease, or triggered by other causes (Table 38). Mimics of
Immunosuppressive therapy with glucocorticoids and rttuxi- vasculitis must also be considered in the differential diagno-
mab may be needed In refractory hepatitis C-related disease or sis (Table 39). Primary autoimmune vasculitis disorders are
severe mixed cryoglobulinemia. discussed i n this section.

TABLE 38. Causes of Secondary Vasculitis


Medications I

Comm on causes: ant•mlcrob1al agents (e.g., m ~nocydine, sulfadiazine); antit hyroid agents (mostly propylthiouracil [80%-90%),
methimazole, carb1m azole, and benzylthiouracil (1 0%-20%J); other cardiovascular drug s {hydralazine ); tumor necrosis factor a inhibitors
Rare causes: vaccines; antiepileptic agents; antiarrhythmic agents; diuretics; anticoagulant s; antineoplastic agents; hematop oietic
g rowth factors; NSAIDs; psychot ropic drugs; sym p athom im et ic agents; allopurinol; interferon alfa; levamisole {assoCiated w ith co caine)

Infections

Hepatitis A, B. and C Vtruses; HIV; bactenal end ocarditis; parvovirus B 19

Neoplasms

Ha1ry cellleukem •a {associated w1th polyarteritis nodosa); other hem atologic and sohd malignancies

Autoimmune Diseases

Systemic lupus erythematosus; rheumatoid arthritis; Sjogren syndrome; inflammatory myo pathies; systemic: sclerosis; relap sing
p olychondrltis, i nflammatory b owe l d isease, p rimary biliary Cirrho sis

65
Sys temic Vasculitis

Infection (sep sis, endocarditis, hepatttiS) Rash and/or musculoskeletal symptoms can occur
Drug toxicity/poisoning Cocaine, amphetamines, ephedra alkaloids, and p henylpropanolamine may produce
vasospasm, resulting 1n ischemia.
Co agulcpathy Thrombotic diseases (dissemmated mtravascular co agulation; antiphospholiptd
syndrome; thrombot iCthrombocytopentc purpura) can produce ischemic symptom s.
Malignancy Para neoplast ic vasculit is is rare. Any organ system may be affected, b ut the ski n and
nervous system are t he most common. Vasculitic symptoms may precede, occur
simultaneously with, or follow diagnosis of cancer. Lymphoma occasion ally may
involve the blood vessels and mimic vasculitis. Consider malignancy in patients with
incomp lete or no response to therapy for idiopathic vascu litis.
A trial myxoma Classic triad of symptoms 1s embolism. intracardiac obstruction leadmg to p ulmonary
congestion or heart failure, and constitutional symptoms (fatigue; weight loss; fever}.
Skrn les1ons can be identical to those seen in leukocytoclastlc vascu l1tis. Atrial
myxomas are rare but are the most common primary intracardiac t u mors Myxomas
can also occur tn other cardiac chambers.
Cholesterol em b oli Typically see n in patients with severe atherosclerosis. Embolization may occur after
abdominal trauma, aortic surgery, or angiography. May also occur after h eparin,
warfarin, or t hrombolytic therapy. Patients may have lived o reticularis, petechiae and
p urpuric lesions, and localized skin necrosis.
----

Large-Vessel Vasculitis (PMR) that may occur before, concurrent with, or following

CJ Giant Cell Arteritis diagnosis of GCA.


Physical examination may reveal scalp or temporal artery
Epidemiology and Pathophysiology tenderness and Induration. reduced pulses and bruits. or aor-
Giant cell arteritis (GCA; temporal arteritis) affects patients tic regurgitation and heart failure. laboratory flndl ngs may
over SO years of age (peak incidence between 70 and 80 years}. include elevated erythrocyte sedimentation rate (ESR) and/or
Most patients are women. GCA Is more common in white per- ( - reactive protein (CRP), but some have normal values.
sons; incidence ranges from 10 to 20 / 100,00 0 in Europe. An Nonspecific evidence of inflammation may Include anemia
association with HLA-DRB.04 has been identified. and thrombocytosis.
GCA is characterized by granulomatous inflammation of GCA is suspected on the basis of the clinical presenta-
affected vessels with infiltration of lymphocytes. mac- tion and is confirmed by temporal artery biopsy and /or
rophages, and multinucleated gian t ceUs. Involved vessels imaging of great vessels. New or atypical headache. jaw
Include the aorta, its major branches off the arch, and sec- claudication, or visual changes in a patient over the age of so
ondary branch vessels. including the external carotid, sub- years. especially with concurrent PMR. s hould raise suspi-
clavian, axillary, temporal. ophthalmic, ciliary, occipital, and cion. Temporal artery biopsy is diagnostic, but false-negative
vertebral arteries. The level of vessel involved dictates the results are common; bilateral temporal artery biopsy can
clinical symptoms. Increase the yield. Importantly, temporal artery biopsy will
remain abnormal for up to 2 weeks after initiation of gluco-
Clinical Manifestations and Diagnosis cortlcoids. Angiography is used to document and follow
Common GCA symptoms Include headache. scalp pain, and subcranlal disease.
temporal artery tenderness. Symptoms are frequently unilat-
eral but can be bilateral. Ach ing and fatigue with chewing Management
(jaw claudication) indicates ischemia of the muscles of masti- Suspected GCA must be treated immediately to prevent visual
cation. Fever, fatigue, and weight loss may be present. The loss. Prednisone, 1 mg/kg/d, Is recommended. Intravenous
most feared complication is ischemic optic neuropathy, which pulse methylprednisolone for 3 days is used for acute visual
can cause amaurosis fugax and blindness. Because blindness loss. but established blindness is usually Irreversible.
is usually permanent, early recognition and treatment of any Symptoms and Inflammatory markers usually respond rnpidly
visual change are critical. Subcranial dlsease Involving great to glucocorticoids; lack of response should prompt reconsid-
vessels in the chest occurs In 25% of cases, resulting in upper e ration of the diagnosis. High-dose prednisone is maintained
extremity claudication. Severe but uncommon complications for 2 to 4 weeks: after symptoms resolve and Inflammatory
include aortic aneurysm and dissection. Dilation of the aortic markers normalize, prednisone Is tapered by 10% to 20%every
root may cause aortic valve regurgitation and heart failure. Up 2 weeks. Once a dose oflO mg/d is reached, the taper Is slowed
to SO% of patients with GCA have polymyalgia rheumatica to 1 mg per month. Patients should be carefully monitored for

66
Systemic Vasculitis

symptom recurrence. ESR and /or CRP should be monitored common carotid. and renal arteries; the pulmonary arteries
Cl monthly but should not be the sole Indication for adjusting the
CONT.
may also be Involved. TA is rare (40/milllon in Japan and 4.7 to
glucocorticoid dose. Mild nares can be managed with increases 8/millton elsewhere). In contrast to GCA, TA predominantly
of prednisone by LO% to 20% and a slower tapering schedule. affects younger women. Arterial lesions ore often stenotic
Based on limited data, daily low-dose aspirin may help to ("pulseless disease"), and one third contain aneurysms.
reduce the risk of blindness and is recommended for those Histopathology is simllar to GCA.
without contraindication. Glucocorticoid-sparing immunosup-
pressives such as methotrexate are sometimes used. although
little data support their efficacy. The ll-6 inhibitor tocilizumab
Clinical Manifestations and Diagnosis
TA manJfestations include carotodynia, limb claudication,
c:J
was recently approved by the FDA for treatment of GCA. reduced pulses, bruits, and blood pressure discrepancies
The prognosis for properly treated GCA is good unless between the arms. Heart failure related to aortic Insuffi-
aortitis is present. GCA may recur. Cl ciency or coronary artery disease may occur. Neurologic
manifestations include transient ischemic attack, stroke.
Polymyalgia Rheumatica and mesenteric ischemia. As With GCA. laboratory studies
Epidemiology and Pathophysiology are nonspecific and reveal anemia as well as elevated ESR
Although not a vasculitis, polymYlllgia rheumatica (PMR) is an and CRP. Angiogram may demonstrate arterial stenosis or
Inflammatory disorder that frequently accompanies GCA. aneurysm (Figure 27).
PMR and GCA likely reflect the clinical spectrum of a single
disease process, although PMR occurs 3 to lO times more fre- Management
quently. Up to SO% of patients presenting with GCA have PMR, Primary treatment of TA is high-dose glucocorticoids (1 mg/
and 20'X.ofpatients presenting With PMR h.ave GCA symptoms kgtd) with a slow taper. Glucocorticoid-sparing medications
on quest!onJng. such as disease-modifying antirheumatic drugs are used but
without clear eVidence for efficacy. Angloplasty. graft place-
Clinical Manifestations and Diagnosis ment. and bypass may be necessary but should be avoided
PMR is associated with pain and stiffness of the neck, shoulder, during active inflammation. The leading cause of death is
and hip girdle. Pain and stiffness are worse after immobility; heart failure ; stroke and cardiovascular disease also contribute
1 hour or more of morning stiffness is common. Inflammation to morbidity. The 10-year survival rate is 90%. Cl
is periarticular (bllrsitis and tenosynovitis}. Synovitis in the
hands and feet occ2sionally o-ccurs. Constitutional symptoms
and laboratory findings resemble GCA, but temporal artery
biopsy should be performed only if GCA is suspected. Diagnosis
of PMR is made clinically. Differential diagnosis includes myo-
pathies, metabolic syndromes (thyroid and parathyroid), and
musculosl<eletal syndromes (capsulitis, cervical spondylosis. or
calcium pyrophosphate deposition).

Management
PMR responds dramatically to low-dose prednisone (12.5-
20 mg/d); lack of a rapid response should prompt considera-
tion of alternate diagnoses. Prednisone taper is initiated 1 to
2 months after symptom resolution and requires months to
years. Monitoring of recurrence is managed similarly to GCA.
For relapses, recent guidelines for the management of PMR
(developed by a collaborative effort of the American College of
Rheu rna to logy and the European League Against Rheumatism}
recommend increasing the prednisone to the last pre-relapse
dose at which the patient was doing well, followed by a grad-
ual reduction within 4 to 8 weeks back to the relapse dose.
Glucocorticoid-sparing therapies are the same as for GCA.
Prognosis Is good, although periodic recurrences are common.

Takayasu Arteritis FIG UR £ 2 7. Aortic angiogram from a patient with Takayasu arteritis. Note the
high-grade stenosis of the proximal right subclavia" artery(while arrow) as well as
Epidemiology and Pathophysiology the leh subclavian artery just below the origin of the leh vertebral artery (blaclc
Takayasu arteritis (TA) causes inflammation of the large ves- arrow). Incidentally noted is an anatomic variation with a common origin of the
sels, most commonly the aorta, followed by the subclavian, right brachiocephalic artery and the leh common carotid artery.

67
Systemic Vasculitis

K£Y POINTS increased interleukins. T celis, and macrophages have been


• Giant cell arteritis should be suspected in a patient over identified as potential contributors to vessel damage.
the age or SO years with new or atypical headache, Jaw
claudication, or visual changes. Clinical Manifestations and Diagnosis
PAN most commonly affects the skin, neurologic, and muscu-
HVC • Suspected giant cell arteritis should be treated immedi- loskeletal systems. It does not involve the lungs and rarely the
ately with prednisone because of the risk for visual loss; heart. Kidney Involvement is renovascular rather than glo-
diagnosis is confirmed with temporal artery biopsy. as merular. Cutaneous PAN is a variant confined to the skin. See
pathologic findmgs will persist for up to 2 weeks after Table 40 for the clinical and laboratory findings of PAN.
initiation of prednisone. The gold standard for diagnosis Is focal segmental pan-
HVC • Polymyalgia rheumatica is associated with pain and mural necrotizing Inflammation of a medium-:;ized vessel
profound stiffness of the neck, shoulder, and hip girdle; shown on biopsy. The biopsy is usually performed on an
it responds dramatic.:ally to low-dose prednisone. involved, easily accessible area, such as skin or a peripheral
• Takayasu arteritis occurs in younger women and causes nerve/muscle. PAN may also be diagnosed on angiogram; mes-
inflammation of the aorta and other major noncranial enteric or renal arteries show characteristic aneurysms and
vessel~; manifestations include carotodynIa, limb clau- stenosis, especially at branch points.
dication, reduced pulses, bruits. and blood pressure
discrepancies between the arms. Management C]
Glucocorticolds and cyclophosphamide are Indicated for
severe organ-threatening disease; glucocorticoids and disease-
Medium-Vessel Vasculitis modifying antirheumatic drugs are used for milder disease.
Polyarteritis Nodosa HBV-associated PAN is treate<:l with short-term glucocorti-
Epidemiology and Pathophysiology coids, antiviral medication. and plasmapheresis if necessary.
Polyarteritis nodosa (PAN) is a mre systemic necrotizing vascu- The S-year survival rate for treated PAN is 80%, and the relapse
litis that affects medium and occasionally small arteries. rate is 10'%. to 20%. CJ
Prevalence is 31/rnilllon but declining. PAN is more common
in men than women. Average age of onset Is SO years. Primary Angiitis of the Central Nervous System
Hepatitis B virus (HBV) infection has been strongly associ- Epidemiology and Pathophysiology
ated with PAN. However, the proportion of patients with HBV- Primary angiitis of the central nervous system (PACNS) is a
associated PAN has dedi ned from 36% to less than S')(, since the rare medium-vessel vasculitis of unknown cause that is con-
advent of the HBV vaccine; thus. most contemporary cases are fined to the central nervous system. Incidence is 2.4/100,000.
presumed autoimmune. Activated endothelial cells as well as Median age at onset is so years. There are three histologic

TABLE 40. Cltnical Features of Polyarteritis Nodosa


Organ System Symptom$ Frequency Comments

Constitutional Fever; malaise; wetghtloss 65%


M usculoskeleta I Arthralgia; myalgia SS%
Skin Purpura; nodules; necrotic ulcers 50%-60%
Neurologic Mononeuritis multiplex; peripheral 79% Wrist drop; foot drop
neuropathy
Kidney Hypertension; hematuria; proteinuria 40% Renal artery microaneurysms with tissue
infarct/hematoma; no glomerulonephritis
Gastrointestinal Mesenteric ischemia; intestinal perforation; 38% One third of gastrointestinal cases manifest as
pancreatitis; cholecystitis; appendicitis; acute abdomen
gastrointestinal bleeding
Testtcular Orchitis 17% Usually unilateral, due to testtcular artery
involvement
Other Sensorineural hearing loss Case reports Bilateral; symmetric; sudden onset; rapidly
progressive
Elevated erythrocyte sedimentatton rate tn
82%; elevated C-reactive protein; leukocytosis;
anemia; thrombocytosis; increased liver
chemistry tests tn 33%

68
Sys temic Vas culitis

presentations, all with patchy distribution: granulomatous II n I' 0 I NT S tcontlllwd)


(SS'X.), lymphocytic (28'X,), and necrotizing (14'X.).
• Kawasaki disease (KD) affects children many patients

CJ Oinical Manifestations and Diagnosis


Patients with PACNS usually present with gradual and pro
recover fully, but coronJry aneurysms may develop, and
adults who had KD in childhood may suffer long-term
cardiac sequelae.
gressive symptoms of headache, cognitive impainnent. neuro-
logic deficits, transient ischemic attacks, and strokes.
Laboratory studies are normal. Cerebrospinal fluid {CSF) is
abnormal In 90 'X,, with elevated protein, lymphocytic pleocy-
Small-Vessel Vasculitis
tosis, and occasional oligoclonal bands. MRI shows nonspe- ANCA·Associated Vasculitis
cific white and gray matter changes and infarcts. MR ANCA-associated vasculitis Includes three diseases character-
angiography and CT angiography have limited usefulness due ized by the presence of ANCA: granulomatosis with polyangii-
to poor resolution. Cerebral angiogram may demonstrate ves- tls, microscopic polyangiltls, and eosinophilic granulomatosis
sel "beading• (alternating dilations and stenoses} but has lim- with polyangiltls, along with ANCA-associated glomerulone-
ited sensitivity and specificity. Brain biopsy is the best test for phritis.
diagnosis. but the patchy distribution of findings results in a There are two types of vasculitis-associated ANCA:
so·x. false-negative rate. p-ANCA (perinuclear, directed against the neutrophil enzyme
Evaluation centers on ruling out other conditions. includ- myeloperoxidase} and c-ANCA (cytoplasmic, directed against
Ing Infection, malignancy, and reversible cerebral vasocon- the neutrophil proteinase 3). Perinuclear and cytoplasmlc
striction syndrome. Cl refer to patterns of Immunofluorescent staining; enzyme-
linked immunosorbent assays are used to confirm antibody
Management positivity.
PACNS is treated with high-dose glucocorticoids and cydo- ANCA may play a direct role in vessel damage by hyperac-
phosphamlde. Patients often have permanent dlsabillty from tivating already primed neutrophils, leading to vessel endothe-
neurologic damage, and the recurrence rate is 27't. lial inflammation and damage. The presence of granulomatous
inflammation in some forms of ANCA-associated vasculitis
Kawasaki Disease suggests a role for cell-mediated Immunity.
Kawasaki disease (KD} Is a medium-vessel vasculitis that See Table 41 for a comparison of the features of the three
affects children and Is very rare In adults. KD presents as fever, forms of ANCA-associated vasculitis.
rash, cervical lymphadenopathy, conjunctival congestion, and See MKSAP 18 Nephrology for details on lddney involve-
mucositis. Coronary vessel vasculitis, aneurysm formation, ment in ANCA-associated vasculitis.
a~d other cardiac complications (heart failure, pericarditis,
arrhythmias) may develop. Treatment is with intravenous Granulornat osJs with Polyangiltis
immunoglobulin and aspirin. Epidemiology and Pathophysiology
Many patients recover fully. However, coronary aneu- Granulomatosis with polyanglftis (CPA) Is the most common
rysms may develop, and adults w ho had KD in cl1ildhood may ANCA-assocfated vasculitis, with an incidence of7 to 12/mil-
suftcr long- term cardiac sequelae. Clu·onic low-dose aspirin is lion/year. It is more prevalent in Nordic countries and white
indicated for coronary artery abnormalilles. Clopidogrel may persons. Typical age of onset Is between 45 and 60 years.
be added for cases with multiple aneurisms; warfarin prophy-
laxis is recommended for giant aneurysms. Clfntca/ Manifestations and Diagnosis
CPA affects the upper and lower airways, kidneys, eyes. and
CJ
UY POINTS
ears. At least 50~ of patients have constitutional symptoms.
• Polyarteritis nodosa most commonly affects the skin. More than 95%of pallenls are ANCA positive, overwhelmingly
neurologic. and musculoskeletal systems; the gold (>90%) directed against proteinase 3 (anti- PR3 antibodies;
standard tor diagnosis is focal segmental panmural c-ANCA).
necrotizing inflammation of a medium-sized vessel on GPA has two forms: systemic and localized. Systemic is
biopsy. more common, Involves major organs, and is anti-PR3 posi-
• Patients with primary angiitis of the central nervous tive. Localized has more granulomatous inflammation, has
system usually present w1th gradual and progressive less vasculitis, and is less likely to be antl-PR3 positive. Patients
symptoms of headache, cognitive impairment. neuro- in the localized group are more likely to be younger and
logic deficits. trans1ent ischemic attacks. and strokes; female: have mainly ear, nose, and throat Involvement; and be
trearment consists of h tgh dose glucocorticolds and more prone to relapse. See Table 41 for CPA clinical features.
cyclophosphamide. In the setting of a classic clinical presentation and positive
(Continued)
c -ANCA/antl PR3. diagnosis of CPA is straightforward.
However, because of slgnlflcant risks of treatment, biopsy of

69
Systemic Vas culitis

TABLE 41. Clinical Features of ANCA-Associated Vasculitis D•seases


Granulomatosis with M icroscopic Polyanglltls Eosinophilic Granulomatosis
Polyangiitis with Polyangiitis I
ANCA c-ANCA (antiproteinase-3 p-ANCA (ant1myeloperox•dase p-ANCA (antimyeloperoxidase
antibodies) (>95%) antibodies) (50%-75%) antibodies) (-50%)
Vascular Histology Pauci-immune necrotizing Pauc•-immune nongranulomatous Pauci-immune necrotizing
granulomatous vasculitis necrotizing vasculitis granulomatous vasculitis with
eosinophilic infiltration of vessel
walls and tissues; extravascular
granulomas
Cardiac Pericard1t1s; myocarditis; Pancarditis, endomyocarditis,
conductiOn disorder (<1 0%) conduction disorder, heart fadure
{27%-47%)
Ears/Nose!Throat Crusting; rhinorrhea; sinusitis; Sinusitis; sensorineural hearing Nasal polyps; rhinitis; sinusitis
otitis media; chondritis of ears and loss {9%-30%) {prodromal)
nose with saddle nose deformity;
septal perforation (70%-1 00%)
Gastro1ntest1nal Ulceration. perforat1on (5%-11%) Abdominal pain, bleeding Abdomfnaf pa1n, bleeding
(30%·58%)
Kidney Pauci·immune necrotizing Pauci-immune necrotizing Pauci-immune necrotizing
glomerulonephritis (40%-100%) glomerulonephritis (80%-1 00%) glomerulonephritis (25%)
Lung Alveolar hemorrhage, nodules; Alveolar hemorrhage; pulmonary Asthma {prodromal, >90%);
tracheallsubglottlc stenosis Infiltrates; pulmonary fibrosis nodular opacities; infiltrates
(50%-90%) (25%-55%) (25%-86%)
Ocular Scleritis; episderitis; retinal
vasculitis; retro-orbital
pseudotumor; dacryoadenitis
(14%-60%)
Skin Palpable purpura; nodules. Palpable purpura; livedo Palpable purpura; nodules (60%)
pyoderma gangrenosum; mucosal ret1cularis; nodules; necrotic skin
ulcerations ( 1Q%.50%) ulcers (30%-60%)
Neurologic Mononeuritis multiplex, sensorimotor Distal symmetric polyneuropathy; Sensorimotor peripheral
peripheral neuropathy (33%) mononeuritis multiplex (37%-72%) neuropathy, mononeuritis
multiplex (70%)
Central nervous system Central nervous system
involvement ( pachymeningitis) pachymeningitis; cerebral
{<5%) hemorrhage; infarcts (<20%)

CJ involved tissue is usually recommended. Histopathology of


most tissues demonstrates paucHmmune necrotizing granu·
Relapses are common (>50% 5 years after initial remis-
sion) and may respond better to rituximab than to cyclophos-
CONT. lomatous vasculitis; pauci-immune necrotizing glomerulone- phamide.
phritis without granulomas is seen on kidney biopsy. Kidney failure and infection are the main causes of
mortality. CJ
Management
For induction of remission In severe organ-threatening or life- Microscopic Polyangiitis
threatening dlsease. treatment of CPA consists of high-dose Epidemiology and Pathophysiology
glucocortlcoids plus cyclophosphamide or rituximab, followed The incidence of microscopic polyangiitis (MPA) is estimated
by maintenance therapy with azathioprine. mycophenolate at 2.7 to 94/millio n/year in Europe and lower elsewhere.
mofetil, or rituximab for at least l2 to 24 months after stable Average age at onset is between 50 to 60 years with a predilec-
remission bas been achieved. Clucororticoids alone are insuf- tion of men over women (1.8:1). ln contrast to GPA, ANCA are
ficient to control CPA. Patients with nonsevere fonns of CPA less prevalent (50%-75%) and tends to be directed against
(such as arthropathy or upper airway disease) without organ- myeloperoxidase (MPO) rather than PR3.
threatening dJsease can be treated with glucocorttcoids plus
either methotrexate or mycophenolate mofetil: such patients Clinical Manifestations and Diagnosis
should be carefully monitored for treatment failure or the Uke CPA. MPA characteristically affects the lungs and kldneys. Cl
development of renal or other organ-threatening disease. neces- along with other organ systems. See Table 41 for the clinical
sitating the more aggressive regimen. Using these approaches, features of MPA. Diagnosis is suspected based upon typical
CPA mortality has declined from 90% to around 10%. clinical findings and positive ANCA. although negative ANCA

70
Systemic Vasculitis

does not rule out the diagnosis. The diagnostic gold standard is
CJ
CONl
a biopsy demonstrating nongranulomatous necrotizing pauci-
U.Y POINTS
• Granulomatosis with polyang1itls typically affects the
immune vasculitis ofsmall vessels or pauci-immune necrotiz- upper and lower airways, k1dneys. eyes. and ears:
ing crescentic glomerulonephritis in the kidney. Absence of
induction of remiSSIOn in severe organ-threatening or
granulomas distinguishes MPA from GPA. life-threatemng disease consists of high-dose glucocor-
tlcoids plus cyclophosphamide or rltuxlmab, followed
Management by maintenance thernpy with azathioprine, mycophe·
Uke GPA, MPA treatment requires high-dose glucocorticoids
notate mofetll, or rltuxlmab.
plus either cyclophosphamide or rituximab. followed by main-
tenance therapy with azathioprine, mycophenolate mofetil, or • \.iicroscopic polyangiitls commonly atlects the lungs
rttuxlmab. Cl and kidneys; treatment requires high-dose glucocorti-
Prognosis Is worse in the setting of pulmonary hemor- colds plus either cyclophosphamide or riluximab. tal-
rh.age or rnpldly progressive glomerulonephritis. Surv1val With lowed by maintenance therapy with <Wlthioprine,
treatment Is 82% at 1 year and 76% at 5 years. mycophenolate mofetil, or rituximab.
• Eosinophilic granulomatosis with polyangiitis is associ
Eosinophilic Granulomatosis with Polyangiitis ated with asthma, nasal polyps, rhinitls, sinusitis, atopy,
Epidemiology and Pathophysiology peripheral and tissue eosinophilia. migr.ttory pulmo-
Eosinophilic granulomatosis with polyanglitis (EGPA) is the nary infiltrates, and mononeuritis multiplex; treatment
rarest ANCA-assoclated vasculitis, With an Incidence of 0.11 to consists of glucocortlcoids for mild disease, with cyclo-
2.66/million/year and a prevalence oflO to 14/million (France). phosphamide 3dded for more severe disease.
EGPA has no predisposition for gender or ethnici ty. In addition
to neutrophil activation, eosinophil infiltration, activation, Immune Complex-Mediated Vasculitis
and degranulation participate in the pathogenesis. Immune complexes form from cross-linking of multiple anti-
gens and ant1bodies. If not cleared, immune complexes deposit
Clinical Manifestations and Diagnosis
CJ The typical patient with EGPA has a history of asthma (96%-
in tissue, leading to complement and neutrophil activation
with inflammation and tissue damage. Although any tissue or
100~). nasal polyps, rhinitis. sinusitis, and/or atopy. A pro- organ may be affected, the classic flndlng Is invariably In the
dromal phase (months to years) consisting of arthralgia, skin. Inflammation and erythrocyte extravasation from
myalgia, malaise, fever, and weight loss may occur. An Involved vessels reswt tn nonblanching palpable purpura, usu-
eosinophilic phase with Increased peripheral and tissue ally In dependent areas (Figure 28). Leukocytoclaslic vasculitis
eosinophilia follows , with migratory pulmonary lnfl!trates refers to disintegration of nuclei (nuclear dust) of dead neutro-
and, less commonly, endomyocardial infiltration and gas- ph.ils along with tlbrinold necrosis of the vessel wall.
trointestinal disease. The subsequent acute vasculltic phase
includes mononeuritis multiplex or peripheral sensorimo- Cryoglobulinemlc Vasculitis
tor neuropathy (70%), kidney (25%), and skin Involvement Cryoglobullns can cause Immune complex-mediated small-
(60'X.) . Paradoxically, the vasculitis phase is often associated vessel vasculitis. There are three types of cryoglobulins;
with Improvement of asthma. See Table 41 for the clinical
features of EGPA.
Laboratory findings show peripheral eosinophilia of
more than 10%, or more than 1500/~L (1.5 x 109 /L). Only
50 % of patients have a positive ANCA, mostly directed
against MPO.
Diagnosis is based upon typical clinical findings, eosino-
philia, and biopsy demonstrating fibrinoid necrosis and eosin-
ophUic infLltration of vessel walls. as well as extravascular
granuloma formation.

Management
In EGPA, glucocorticoids alone may be sufficient for mild dis-
ease without major organ Involvement. With kidney, gastroin-
testinal, cardiac, or neurologic involvement, cyclophosphamide
F IGUIE 28. Palpable purpura Is the dassic ra.sh of any small·vessel, immune
is indicated.
complex-mediated vasculitis.The lesions are nonblanching and represent
Mortality for EGPA is the lowest of all the forms of ANCA- extravasations of blood from damaged vessels. Purpuric lesions are typically more
assoclated vasculitis. The 5-year survival is 97%, and the prominent on the lower extremities, a consequence of the superimposed effect of
relapse rate is 28%. C1 gravity on hydrostatic pressure.

71
Systemic Vasculitis

discussion here is limited to types fl and lfl ("mixed" types). Both as abdominal pain or bleeding (65%). arthritis and arthralgia
are polyclonal, but type II cryoglobulins include a monoclonal (63'X,). and glomerulonephritis (4Q'X,) may be present.
lgM rheumatoid factor, where-c~s type 111 cryoglobullns include a Although rare, life-threatening pulmonary hemorrhage may
polydonal !gM rheumatoid factor. The ability of rheumatoid fac- occur.
tor to directly bind other antibodies facilitates the formation of There are no specific laboratory tests for diagnosis: serum
immune complexes even in the absence of persistent antigen. lgA may be elevated but is not sensitive or specific. Diagnosis
See MKSA P 18 Hematology and Oncology for details on cryoglo- is conflrmed with biopsy. Skin biopsy demonstrates leukocy-
bulins and the di.tlhenrialion from cold agglutinin disease. toclastic vasculitis with heavy deposits of lgA and complement
on immunofluorescent staining. Renal histology is identical to
Epidemiology and Pathophysiology lgA nephropathy.
Mixed cryoglobulinemia accounts tor 85% to 90'~ of all cases;
90~. of mixed cases are related to hepatitis C virus (HCV) infec- Management
tion, which can cause both type ll and type Ill cryoglobulinemia. Although lgA vasculitis in children tends to be self- limited,
Autoimmune diseases such as systemic lupus erythematosus adults are more likely to develop severe persistent disease,
and SjOgren syndrome cause type Ill cryoglobulinemia. Onset is especially nephropathy, and may require glucocorticoids and
usually in the fifth dec-ade, and women slightly outnumber men. cyclophosphamide. Cl

Hypersensitivity Vasculitis
Cl Clinical Manifestations and Diagnosis
Cutaneous symptoms (palpable purpura. Raynaud phenome- Epidemiology and Pathophysiology
non, ulcers, necrosis, and livedo reticularis) predominate in Hypersensitivity vasculitis is a small-vessel vasculitis medi-
70% to 90% of patients with mixed cryoglobulinemia, but any ated by immune complex deposition confined to the skin. lt
organ may be lnvolved. Peripheral neuropathy (60%), arthritis may be triggered by an antigen such as a drug or infection; in
(40%), and glomerulonephritis (40%) are corrunon. In addition SO'X. of cases, the antigen is unknown.
to cryoglobulins, a low C4 complement and positive rheuma-
toid factor are present. A false-negative cryoglobulin result Clinical Manifestations and Diagnosis
may occur if the serum sample is not malntained at 37.0 oc The most common presentation of hypersensitivity vasculitis
(98.6 °F) due to ex vivo cryoprecipitation at room temperature. Is palpable purpura in dependent regions, developing 7 to 10
HCV Infection associated with mixed cryoglobulinemia days after exposure to a triggering antigen; lesions appear in
may go unrecogni:?..ed for many years before the development "crops" and resolve over a few weeks after the antigen is
of vasculitis. It is therefore important to test for HCV infeCtion removed. Internal organs are unaffected. Sk1n biopsy with
in patients with cryoglobulinemia. immunofluorescence demonstrates leukocytoclastic vasculitis
without heavy lgA deposits. Evaluation should be guided by
Management clinical signs and symptoms, and may only require a complete
When possible, treatment of the underlying cause of cryoglo- blood count, basic chemistries, and urinalysis.
bulinemia Is the first priority. For HCV- related disease, antivi-
ral medication is the primary therapy. For severe or refractory Management
disease, the vasculitis must be independently addressed. Removal of the antigen (if Identified) and supportive care are
GJucocorticoids and cyclophosphamide have been used in the usually sufficient. Resolution within a month is the rule. If
past; plasmapheresis and rituximab (provided there is no symptoms persist or recur, anti-inflammatorles, topical or
hepatitis B virus infection) have demonstJ:ated efficacy and low-dose systemic glucocorticoids, colchicine, or dapsone
may carry less toxicity. Cl may be helpful.
KEY POINTS
IgA Vasculitis
See MKSAP 18 Nephrology for information on lgA nephropa- • Mixed cryoglobulinemia is associated with cutaneous
thy and on kidney Involvement ln IgA vasculitis. symptoms, peripheral neuropathy, arthritis. and glo-
merulonephritis, with 90'X, of cases related to hepatitis
Epidemiology and Pathophysiology C virus infection: treatment of the underlying disorder
IgA vasculitis (Henoch-Schonleln purpura) is a common vas- causing cryoglobulinemia is required.
culitis of childhood that occurs rarely in adults. Estimated • lgA vasculitis (Henoch-SchOnlein purpura) is character-
incidence In adults is 14/million/year. Onset is often preceded ized by palpable purpura and abdominal pain/bleeding
by a viral or streptococcal upper respiratory infection. and occurs mainly in children: adult disease is rare but
more likely to be severe and/or persistent ahd to require
19'1 Clinical Manifestations and Diagnosis glucocorticoids and cyclophosphamide.
IAI Patients with lgA vasculitis typically present with a palpable (Continued)
purpura in dependent areas. Gastrointestinal symptoms such

72
Othe r Rheumatolog ic Diseases

KE Y P 0 I NTS {conUnued) Other clinical manifestations include venous thrombosis that


• The most common presentation of hypersensitivity vas may affect the large veins, including the vena cava and the
culitis is palpable purpura, developing 7 to 10 days after dural venous sinuses. Central nervous system (CNS) manifes-
exposure to a triggering antigen; removal of the antigen tations include brainstem lesions and aseptic meningitis; the
and supportive care are usually sufficient management. most common CNS symptoms are headache and diplopia.
Inflammatory arthritis (usually in the knees), skin lesions, and
gastrointestinal inflammation/ulceration indistinguishable
from Inflammatory bowel disease also occur. Diagnostic crite-
Other Rheumatologic ria are listed in Table 42. CJ
Behc;et syndrome is most common In Asia, with the high-
Diseases est reported prevalence Jn Turkey; It Is much Jess common in
North America. It is slightly more prevalent in men than
Beh~et Syndrome women, and men usually have more severe disease. HLA-BSl
" ' Behc;et syndrome is a systemic disease associated with inflam- is strongly associated with the disease and is seen in patients
1.1.1 matory intlltration of multiple organs. as well as small-vessel with varied ethnicity.
vasculitis and large-vessel vasculopathy. The characteristic Treatment for mild disease includes colchicine and low-
clinical features are recurrent painful oral and genital mucosal dose glucocorticoids. The phosphodiesterase-4 inhibitor apre-
ulcerations (Figure 29) and inflammatory eye disease (panu- milast appears to be benefidal for oral ulcers. High-dose
veitis, retinal vasculitis). A characteristic clinical finding is glucocortlcoids and lmmunosuppressives such as cydosporlne
pathergy, an inflamed papular or pustular response to local and azathioprine are used for more severe disease. Interferon
skin injury that is clinically defined by similar lesions appear- alia and tumor necrosis factor (TN F)-a inhibitors may be used
ing 48 hours after skin prick with a sterile needle (Figure 30). for refractory disease. Interleukin (IL)- 1 and IL-6 inhibitors
have been reported to be beneficial.
KH POINT
• Behc;et syndrome is characterized by recurrent painful
oral and genital mucosal ulcerations, inflammatory eye
disease, and pathergy.

Relapsing Polychondritis
Relapsing polychondritis {RP) is a rare autoimmune disease
characteri1.ed by inflammation and damage of cartilaginous
tissues. Anti-type II collagen antibodies have been identified,
but a role in pathogenesis has not been established. RP can be
primary or associated with other autoimmune diseases. par-
fIGURE 2 9. Recurrent, painful genital mucosal ulcerations are a characteristic ticularly ANCA-assoclated vasculitis, Beh~t syndrome. and
clinical feature of Beh~et syndrome. In women, they typically appear on the vulva, antlphospholipid syndrome. Onset usually occurs between the
as shown. and may heal with disfiguring scaring. :ages of 40 and so years.

TABLE 42. International Criteria for Beh~et Disease


Manifestations Points"
Oral ulcers 2
Genital ulcers 2
Pathergy
Skin lesions (pseudofolliculitis; skin ulcers;
eryth ema nodosum)
Eye lesions (anterior uveitis; posterior uveitts; 2
retinal vasculitis)
Central nervous system lesions
Vascular lesions (arterial thrombosis; large
Fr GURE 3 0. Pathergy assodated with Beh~et syndrome, characterized by vein thrombosis; phlebitis)
pustular-appearing skin lesions occunlng 48 hours after skin pricking with a sterile •Four o< mo(e p ointi are noQded for the dfagnosis of Beh~et syndrome.
needle.

73
Other Rhe umatologic Diseases

Tissues affected include the cartilaginous portions of the TABLE 43 . Yamaguchi Criteroa for the Diagnosts
external and middle ear (90'7.), nose (60'X,), tracheobronchial of Adult·Onset St1ll D1sease
tree (50%), and joints (6S'X,). In nammation of noncartilaginous Major Criteria• Approximate Frequency
connective tissue also occurs, particularly uveitis and keratitis
Daily spiking fever to 39.0 oc 99%
(65%). Cochlear and/or vestibular dysfunction may rarely (102.2 °f}
occur (10%). Elevated inflammatory markers, anemia, antinu-
Arthralgia/arthritis >2 weeks 85%
clear antibodies, and other autoantibodies are often observed
but are nonspecific. Cartilage biopsy, which is necessary only Nonpruritic salmon-colored 85%
macular/maculopapular rash
with atypical presentations in which there is substantial diag- on trunk or extrem1t1es
nostic uncertainty, typically demonstrates inflammatory cell Leukocyte count>10,000/!JL 90%
infiltration of the perichondrium. cr can demonstrate bron- (10x 109/L), >80%
chial thickening, strictures, malacia, and air trapping. neutropnils
Therapy is guided by organ involvement and disease Minor Criteria Approximate Freque ncy
severity. Mild diSease may respond to NSAIDs, colchicine, or
dapsone. Glucocorticoids and disease-modifying antirheu- Sore throat 66%
matic drt~gs (DMARDs) such as c.:yclosporine, azathioprine, or Lymphadenopathy and/or 65%/50%
splenomegaly
methotrexate are used for more severe disease. Biologic drugs
such as TNF-cx inhibitors have been reported as effective but Elevated AST, ALT, o r LDH 70%
not on a consistent basis. Surgery (stenting, dilation, extirpa- Negative ANA and RF -
tion, reconstruction) for airway lesions may be necessary. ALT • alanine aminotnnsfer..ue. ANA • antinuclear antibodies; AST • aspartate
amonotransierase: lOH • lactato dehydrogenase; RF • rheumatoid factor.
KEY POINT
•():agnosis requir@S five critenil with at feast two major etiteria indvded.
• Relapsing polychondritis is characterized by inflamma- Adapted with pormis.sion f<om Yamaguchi M. Ohra A. Tsunematsu T. Kasukawa R.
tion and damage of cartilaginous tissues; tissues most Milv$him;, v. Kashiwagi H. et al. Preliminary criteria (or classifl.cation of adult Still's
disease. J Rhovmatol. 1992 Mar;19(3):424-30. (PMID: 1578458) Copyright1992,
commonly affected include the cartilaginous portions the Journal of Rheumatology.
of the external and middle ear. nose, tracheobronchial
tree. and joints.
Autoinflammatory Diseases
Autolnflammatory diseases (periodic fever syndromes) are a
Adult-Onset Still Disease category of rare monogenic conditions characterized by
Adult-onset Still disease (AOSD) Is a rare disorder that affects episodic and/or persistent inflammation ln the absence of
CJ multiple organ systems. Symptoms and signs include daily antigenlcally driven autoimmunity. Pathogenesis involves
high spiking fever, evanescent salmon-colored rash on the dysregulation of the innate immune system. The innate
trunk and extremities, arthritis, lymphadenopathy, and leuko- immune system represents a process separate and distinct
cytosis. It is most commonly diagnosed in young adults from the antigen- based immune response modulated by
(median age, 36 years) but has been described in older age lymphocytes. Rather than adaptively responding to anti-
groups. Some reports indicate a predilection for women. The gens, the innate immune system responds to stimuli that it
cause is unknown, but AOSD appears to be a disease of auto- "innately knows" to mandate a response. Examples of such
inflammation rather than autoimmunity. stimuli are "pathogen-associated molecular patterns• such
See Table 43 for diagnostic criteria. Diagnosis is clinical, as bacterial cell wall proteins and "damage-associated
and other infectious, neoplastic, and rheumatologic diseases molecular patterns~ or markers of cellular damage such as
must be excluded. Rare complications include hemophago- released uric acid. This process is largely regulated by
cytic lymphohlstiocytosis, myocardltis, shock, multiple organ "inflammasomes," large protein structures that recognize
failme, disseminated intravascular coagulation, thrombotic these stimuli. Autolnflamrnatory diseases Involve inherited
microangiopathy, and fulminant hepatitis. Cl or spontaneous mutation of genes that encode for inflam-
Treatment includes NSA!Ds, glucocortlcoids. DMARDs masomes, including components of the lL-lP-activatlng
such as methotrex-ate, and Il-l Inhibitors. TNF-cx and lL-6 infl ammasome and the TN F-a receptor pathway.
inhibitors may be beneficial, but more studies are needed to See Table 44 for the features and treatment of select auto-
establish their efficacy. Most patients respond to therapy, and Inflammatory diseases.
prognosiS of AOSD with treatment Is good.
lEY POINT
KEY POINT • Autoinflammatory diseases are characterized by epi-
• Adult-onset Sti II disease is characterized by daily high sodic and/or persistent inflammation and involve
spiking fever. evanescent salmon-colored rash. arthritis, inherited or spontaneous mutation of genes that encode
lymphadenopathy, and leukocytosis. for inflammasomes.

74
Other Rheumatologic Diseases

TABLE 44. Features and Treatment of Select Autoinflammatory Diseases


FMP TRAPS FCAst> MWSb NOMID/ Schnitzler
CINCAI> Syndrome
Inheritance AR AD AD AD AD Acquired
Gen e MEFV TNFRSF1A NLRP3 NLRP3 NLRP3 Unknown
Prote1n Pyron TNF receptor Cryopy.-in Cryopyrin Cryopynn MGUS (hallmark but
type 1 not proven causal)
Age at Onset 65% <10 years 50% <1 0 years <1 year of age Childhood Neonatal Adult(51 ± 10years)
of age, 90% of age; up to period
<20years of fifth decade
age
Ethnoc1ty Mediterranean All European European All Caucasian/European
Clinical
Manifestations
Attack Durat1on 12-72 n Days to weeks 12-24 h 1-2 days Continuous Daily
Abdominal Pain; serositls Pain; serosi1is Nausea Pain
Pleuritis Common Common Rare Rare
Musculoskeletal Monoarthritis Large j oints Arthralg ia Arthralgia; Epiphyseal Arthralgia; arthritis;
of lower oligoarthritis overgrowth; bon e pain
extremities contractures
Rash Erysipeloid on Migratory with Cold· Urticaria-like Urticaria-like Urticarial rash;
lo wer legs underlying induced ; neutrophilic
myalgia urticaria-like
Other High risk for Conjunctivitis; Conjunct1V1t1s; Sensorineural Sensorineural LeukocytOSIS;
amyloidosis periorbital headache; deafness; deafness; lymphadenopathy;
edema amyloid OS IS; conjunctivitis; aseptiC hepatospleno·
sensorineural amyloidosis meningitis; megaly; MGUS
deafness mental (usually lgM kappa);
retardation; may later develop
amyloidosis Waldenstrom
macroglobulinemia
Treatment Colchicine Glucocorticoids; IL- 1 ~ IL-1 ~ IL-1 P IL-1Raorll·1P
TN F-a inhibitors inhibitio n inhibition inhibition inhibition
AO • c.utosomal dom)nant; AR • auto-somal recessive; CINCA • chrook inf.antlle n eurologic.. cutaneous, articular syndrom e; FCAS • fam ili.al cold avtotnft~mm atory syndrome;
FMF • fam<liel Med<terranean fever; IL •lnterleukin; IL-lRo • lnterievkin-1 re<:eptor antogo~>tst; MGUS • monoclonalgammopathy of uncertoin significance; MWS • Muckle-Well$
syndrome; NOMIO • neonatol-onset multisynem lnHammltOI)I disease; TNF • tvmor neCto~is factor, TRAPS • tumor necrosis factor receptor-associated periodic syndrome.
•FMF i5 the 1110st common autolnftarnrnatory disease.
~The CJyopyrin-.associJted penodic iyndromas (CAPS) indude f~milial cold iutoinfl~mmatoty Sytldrotn<), Muckl&-Wefls syndtoma, and neonaral-onset multlsys.tem •nflammatory
d isease (chrome infantile n e\lrologic, c:uuaneovs, articular synd rome).
------

Sarcoidosis Typical radiographic findings include cystic or sclerotic lesions


and a lacy pattern of multiple lesions (Figure 31). Sarcoid myo-
Sarcoidosis is an inflammatory disease of unknown cause pathy Is histologically present in 25% to 75% of patients but is
cl1aracterized by fonnation of noncaseating granulomas in symptomatic in only 0.5% to 5%. The most common clinical
multiple organs and tissues. Sarcoidosis most commonly presentation is chronic proxlmal weakness with development
affects the lungs, which is discussed in MKSAP 18 Pulmonary of atrophy or contractures. Biopsy may show inflammatory
and Critical Care Medicine. Rheumatologic manifestations are muscle disease or destruction of muscle by granuloma forma-
discussed in this section. tion. Glucocorticoids. DMARDs, and TNF-a inhibitors may be
In patients with sarcoidosis, a periarthritis involving mul- used to treat arthritis, bone disease, and myopathy.
tiple joints may occur, typically affecting the ankles but also Rarely, sarcoidosis can cause Heerfordt syndrome (uveo-
potentially the knees. wrists, and small joints of the hands and parotid fever), a combination of fever, uveitis, and parotitis
feet Erosive damage. deformlty. and dactylitis may develop. with or without cmnial nerve VII palsy. This can lead to sicca
Bone involvement has been reported in 1% to 15% of patients: symptoms and can mimic SjOgren syndrome.
it most commonly accompanies multiorgan disease and is LOfgren synd rome is a sarcoidosis triad of bilateral hllar
associated with a chronic disease course and poorer prognosis. lymphadenopathy. erythema nodosum. and migratory

75
O ther Rheumatologic Disease s

KIV P 0 IH TS (continued)
• LOfgren syndrome is a sarcoidosis triad of bilateral hilar HVC
lymphadenopathy, erythema nodos um. and migratory
polyarth ralgia; the presence of all three findings has a
9S'Yv diagnostic specificity for sarcoidosis.

Genetic Diseases of Connective


Tissue
Mutations in genes encoding for collagen, fibrillln, and
other components of connective tissue can lead to poorly
functional skin an d integument. Common clinical find-
ings include skin and joint laxity, skeletal fractures. and
cardiovascular complications related to involvement of
the aorta and cardiac valves. The three most common syn-
dromes are Ehlers-Danlos syndrome (EDS), Marfan syn-
drome (MFS), and osteogenesis lmpertecta (OT). See
Table 45 for a summary of clinical findings and manage-
ment of these syndromes.
Vascular monitoring is necessary in MFS and several
types of EDS. Patients with MFS should undergo echocar-
diography to assess the aortic root and ascending aorta at
the time of diagnosis and at 6 months to ensure stability.
FoUow-up studies should be completed at least annually;
MRl and/or CT are often used to further delineate risk. The
best means of vascular surveillance in vascular EDS is
unclear, and all patients should undergo expert consulta-
tion. See MKSAP 18 Cardiovascular Medicine for more
information.
KEY POINTS
• Common clinical findings of genetic diseases of connec-
tive tissue include s kin and joint laxity, skeletal frac-
tures, and cardiovascular complications related to
involvement of the aorta and cardiac valves.
• Regular vascular monitoring is required for patients
with Marfan syndrome and several types of Ehlers-
fl GU RE 31. Hand radiograph showing the typical cystic lesions(red arrow) Danlos syndrome.
and a lacy pattern (green arrow) characteristic of sarcoidosis.

polyarthralgia. Fever is frequently present. When all three lgG4-Related Disease


CJ parts of the triad are present, there Is a 95% diagnostic speci- Seve.ral previously ill-defined illnesses have been brought
under the banner of lgG4-related disease {fgG4-RD) (Table 46).
CONT. ficity for sarcoidosis, obviating the need for biopsy. LOfgren
syndrome has a good prognosis and usually remits in 2 to 16 Most of these conditions are characterized by infiltration and
weeks. The goal of treatment is symptom reduction; NSAIDs tumefaction of the affected tissue with resultant organ enlarge-
are usually adequate. but low-dose glucocorticoids, colchi- ment, fibrosis, and dysfunction. Patients commonly present
cine, and hydroxychloroqulne can all be helpful. m with a sentinel o.rgan enlargement, but careful evaluation often
reveals more extensive disease.
KEY POINTS
Currently, JgG4 is hypothesized not to be pathologic, but
• Rheumatologic manifestations ofsarcoidosis include instead to act as a blocking antibody in patients with atopic
arthritis involving multiple joints that may develop ero- disease, preventing allergens from 1ntemcting with lgE on
sive damage, deformity, and dactylitis; bone disease; mast cell surfaces. Consistent with this model, lgG4 elevations
and myopathy. accompany loss of clinical responses to allergens in atopic
(Continued)
individuals.

76
Other Rheumatologic: Diseases

TABLE 45. Findings and Management of Genetic Diseases of Connecttve Tissue


Ehlers·Danlos Syndrome Marfan Osteogenesis
Syndrome lmperfecta
Hypermobility Classic Kyphoscoliotic Vascular
Genetic Testing Gene is COLSA1; PLOD1 COL3A1 FBN1 COL
Available unknown COL5A2
Inheritance AD AD AR AD AD ADorAR
Clinical Findings
Musculoskeletal Joint laxity; joint Hypermobility; Hypermobility; Hypermobility; Hypermobility; Bone
instabil ity; joint dislocations; progressive joint tall stature; fractures;
musculoskeletal pes planus scoliosis; dislocations dolichosteno· short stature
pain; early marfanoid melia•;
osteoarthritis habitus arachnodactyly;
pectus
excavatum/
carinatum;
scoliosis
Skin Easy bruising; Smooth, velvety; Easy bruising; Thin; Hyperextensible
mild laxity easy bruising; hyperextensible translucent;
hyperextensible; easy bruising
striae atrophicae
{widened stretch
marks)
Cardiovascular No risk of organ Mitral/tricuspid Medium-size Arterial Aortic
rupture or valve prolapse; arterial rupture; rupture; aneurysm/
dissection aortic root mitral/tricuspid aneurysm; dissection; mitral
dilation {rare); valve prolapse dissection valve prolapse
arterial rupture
{rare)
Other Muscular Muscular Organ rupture Myopia {most Dentinogene·
hypotonia; hypotonia; {uterus, bowel, common feature sis
delayed motor scleral fragility rarely spleen, [ 60%)); ectopia imperfectab;
development with risk of liver); lentis; high hearing loss;
globe rupture; pneumothorax; arched palate; b lue sclerae
restrictive lung gingival pneumothorax;
disease; recession blue sclerae
recurrent
pneumonia;
heart failure
Management" Joint Joint protection; Joint Joint Annual Bisphospho·
protection; echo/vascular protection; protection; ophthalmic nates;
supportive care monitoring; echo/vascular echo/vascular examination; audiology
preconception monitoring; monitoring; periodic aortic assessments;
counseling preconception preconception arch imaging; joint and
counseling; counseling P-blockersd; joint bone
physical protection; protection;
therapy; pregnancy dental
bracing counseling/ evaluations
monitoring
AD • autosom al dominant; AR • autosomal receuive.

·'Dolichos-~et~omelia describos whe:n thlf! oxtrernit~s are disproportionately 1ong for the size of the trunk.

I "Dentinogenesis imperfe<ta is defHled as discolored (blue/gray or y&llowlbrown). tronslucent. weak toetk pron~ to br.,4kage.
I "Genelic consultation is •ppropriate for all listed conditron•.
w rate of aortopathy.

Most patients with lgG4-RD are men over the age of tree disease. Systemic symptoms such as fever or weight
SO years; history of atopic disease is common. Clinical signs loss are uncommon.
Include painless enlargement of lymph nodes or the thy- lgG4-RO can affect vascular structures directly, including
roid, parotid. or submandibular glands; proptosis with the carotid, pulmonary, coronary, and iliac arteries as well as
orbital pseudo-tumor; back or chest pain from aortic the aorta, In particular the thoracic aorta. The affected vessels
involvement; and abdominal pain from pancreatic or biliary can develop aneurysms, dissection. and stenosis. Aperiarteritis

77
Bibliography

TABLE 46. Select Conditions Associated with Bibliography


lgG4-Related Disease Approoch to the Patient wltll Rheumatologlc Disease
Condition Organ Involved Brown AK. How to lnreqll-et plain r~c.liogr-Jphs in clinical pmctit-e. Be.~t Prdct
Res Clln Rhcumlltol. 2013:27:249-69. (PMID: 23731934] doi:I0.10J6/j.
Hypertrophic Dura mater berh.2013.03.004
pachymenongitts Castro C. Gourley M. Diagnostic testing and lntc11m~tation of tests for autolm·
munlty. J Allergy Clin tmmunol. 2010;125:5238-47. fPMif): 20061009]
Lymphocytic Pituitary gland dol:10.1016/j.jocl.2009.09 .011
hypophysitis (A)Iglazler Cl, Sutej PC. LaborJtory resting ln the rheunwtlcdise~ses: a prJcll-
IdiopathiC orbital Periorbital olten with involvement rol re-view. South Med J. 2005;98:J85 · 9l. [I'M IDe 15759949]
inflammatory disease of the ocular adnexal tissues Courtney P. noherty M. Joint aspiration and Injection and synovial fluid
analysts. !lest Pract Hes Clin Rheumatol. 2013 Apr;27(2):137-M. IPMID:
Mikulicz disease Parotid glands 23731929]
Dacryoaden 1tis Lacrimal glands l~gnocco A, Ceccarelli F. Perrkone C G-Jtumel<~t<l A, nnucd A, Ricci 1':, et nl.
The use ofmusculoskelerat ullrnsoufld in a rheumatology outpatient d.IJJic.
Kuttner tumor Submandibular glands Med Ultrason. 2014:16:332 .s.[I'MIO: 25463887]
Riedel thyroiditis Thyroid Man A. Shojanta K. Phoon C. ?.tl J. de B:ldyn MH, PI D. et al An evaluation or auto-
Immune antibody testing patterns In a Carwdian he:llth region and an evalua-
Inflammatory aortitis/ Aorta/arteries tlon of a labordtory at~,•orlthm aimat at K'"Ciucing unnett:<S<uy t~ing. Clln
vasculitis RheumaUJI. 2013;32:601 -8. IPMII): 2'J29"l519) dol:10.1007/sl0067·012-2111-y
Qaseem A, Alguire P, Dallas P. Feinberg LIZ, Flr~erald FT, llorwitch C. eta!.
Autormmune pancreatitis Pancreas Appropriate u se of screenJng and diagnostic tests to foster high-value, cost-
Ormond disease Periaortic mass conscious care. Ann Intern Med. 2012;156:147-9 . [PMtD: 22250146)
(retroperitoneal fibrosis) doi:10.7326/0003-4319-156-2·20120IJ70-000IJ
Yazdany J, Sclunajuk G, Robbins M, Dalkh 0, BeaU A. Yelln E. et al; Amt:rican
Tubuloint.erstitial Kidneys College of Rheumatology Core Membership Group. Choosing wisely: the
nephropathy Amet"lcan College of Rheunutology's top S list of things physicians and
patients should question. Arthritis Care Res (Hoboken). 2013;65:329-39.
Sclerosing cholangitis Biliary tract [PMID: 23436818) llof:IO.I002/ncr.2J930

Principles ofTherapeutlc:s
primarily affecting the infrarenal aorta in conjunction with Cheng L. Xiong Y. Qin CZ, Zhung W, Chen XP. ll J, et al. HL\-8'58:01 is
strongly assoctatell with allopurinol-Induced se\ll!re cutaneous adverse
retroperitoneal fibrosis may also develop. reactions In Han Chinese patients: a multlcentre retrospective case-control
Diagnosis is by tissue biopsy. which demonstrates a dense cllnlc:ul study [Letter!. Br J Oermatol. 2015;173:555-8. (PMID: 26104483]
doi:IO.Illl/bjd.13668
lymphoplasmacytic infiltrate, CD4- positive T cells and plasma
da Costa BR, Reichenbach S. Keller N, Nartey L. Wandel S, JonJ P, et nl.
cells in germinal centers, lgG4-staining plasma cells. storiform Effectiveness of non-steroidal antH.nflan1 matory drugs for the treatment of
fibrOSis, obliterative phlebitis or arteritis, and tissue eosinophilia. pain in knee and hip osteoarthritis: a ne!Work meta-analysis. t.ant~t.
2017:39(}.e21-e.'l3. [PMif): 28699S9S] doi:l0.1016/S0140·6736(17)317~4-0
lgG4 levels are elevated in the serum in 70% to 80'll. of
Hochberg MC, Altma n RO. April KT, Bt::nkh~!ti M, Guyatt G, McGowan J. et al:
patients; therefore, a normal serum level does not rule out the Amerkan (',allege of Rheumatology. American College of Rheul!llllology
disease. A ratio of the lgG4level to totallgG of more than 8% is 2012 recommendations for the use of nonpharmacologic and pharmaco ·
logic Lhernpies in osteclllrthritls of the hand, hip, and knee. ArthritlsOlre
suggestive of the diagnosis. PET scan can identify involvement Res (Hoboken). 2012:64:465-7~. lPMJO, 22563Sa9)
that is not clinically apparent and can be used to document l lwnng VG, Snag K. The satecy of low dose glucocorl!colds In rheumatic dis-
response. eases: results from observational studies. Neurohnmunomodulation.
2015:22:72-82. [PMID: 252282301 doi:IO.ll$9/000362727
Initial treatment is prednisone, 0.5-0.6 mg/kg/d, for 2 J<avammgh A, Mease PJ, Gomt7.-RelnolJ, Adebajo AO, Wollen haupt J, Gladman
to 4 weeks with a slow taper over 3 to 6 months; some DO. et at. Tre-d.t ment of psoriatic ar!hrttls In a phase 3 randomi.sed, placebo-
controlled trial w ith apremilast, an or,ll phosphod!esterase4 inhibitor. Ann
experts advocate subsequent low-dose (5 mg/d} therapy for Rheum Di~. 2014:73:1020-6. [PMID, 24595547] doi:IO.ll36/nnnrheum·
several years. Rituximab has shown signitkant benefit and dis-2013-205056
appears to affect lgG4 production more than other JgG sub- E<roon FP. vJn der Burg LR, Ramlro s, undewe RB. Buchbinder H, Falzon l. ctt
ul. Non-steroidal anti-lnfl~mmatory drugs (NSAIDs) for axial spondytoar-
types. Methotrexate may be a useful agent to maintain thritls (ankylosing spondylitis and non-radiographic axial spondyloarthri-
remission In patients treated with prednisone or rituximab. tls). Cochmne Database Syst Rev. 201S:COOJ09S2. (PMIO: 261861731
doi:10.1002/14651858.C00!0952.pub:.!
Azathioprine and mycophenolate mofetil have also been
McGrory BJ, Weber Kl. JevsevJr OS, Sevarlno K. Surgk:ul management of
used as glucocorticoid-sparing agents. Treatment response osteoarthritis of the knee: evidence-based guideline. J Am Acad Onho1>
may be Influenced by the amount of fibrosis present prior to Surg. 2016;24:c87-93. [PMID: 27355286] dol:I0.543S/JAAOS·D-16·001S9
therapy initiation. Mcinnes 18, Mease PJ, Kirkham B. K>lvanaugh A, Rltchlin CT, Rahman P. et al;
FUTURE 2 Study Group. Secuklnumab, a human antHnterleuklo-l7A
See MKSAP 18 Gastroenterology a nd Hepatology for infor- monoclonnl antibody, tn patient~ with psoria!lc arthritis (FUTURE 2): n
mation on lgG4-related pancreatitis. rJndomL>;C(I. double-blind. placebo-controlled. phase 3 trial. Lancet.
20tS:3l:l6:1l37·16. [1'11410: 26135703] doi:IO.l016/S0110·6736(1S)61134-5
KEY POINT Mletenz TJ, Xiao C. Callahan u: Selr-management of arthritis symptoms by
complementary a nd a lttrnative medlc:ine movemc.nt therapl~. J Altern
• The conditions comprising igG4- related disease are Complement Med. 2016:22:401 7. [PMID: 27058260] doi:IO.I089/
~cm.2015.0222
characterized by infiltration and tumefaction of the
Mok CC. Ylng KY. Ylm CW. Siu Yl~ Tong KH, lo CH. et >~1. Tacmfimu.~ versus
affected tissue with resultant organ enlargement, fibro- mycophcnolate mofetlf for hlductlon thempy of lupus nephritis: a ran-
sis. and dysfunction; diagnosis is by tissue biopsy. domlsed conrrolled trial a nd long-tem1 follow-up. Ann Rheum Dis.
2016:75:30-6. (PMID: 2S.SS03.19l doi:10.1136fannrh~'t1mdls-2014-206456

78
Bibliography

Omolr MA. Alahmadl A, Johnson SR. S~lety and effectiveness or mycopht·no- Hochberg MC, Allman RD. April 1'0. llenkhalti M. Guyalt G. McGowan J. et ~1:
late in :;ystcmlc sclerosis. A ~-ystemntlc revl~w. PLoS One. 201S;IO.co-124205. Americ-.m College of Rheum;\tology. llmerlcan College or Rheumatology
(PMID: 2593:1090] doi:l0.1371/jomml.pone.Ol2'120!> 2012 recommencJ;ttinns for the use of nonpharmacologlc ami pharmnco-
Singh JA. $aag l<C. Bridges SI.Jr. Akl EA.Ilannuru RR, Sullivan MC..:. cr al.:/015 l<>glc lhernpi~ In osteoorlhrlti~ of the hand. hlp. and l<n~>e. Arthrills O.u·e
flmerican COllege of Rheumatology guideline ror I he treatment ot' r11euma- Res (Hobol<en). 20f2:64:465·7~ . 1PMID: 12563589)
told arthritis. Arthrl!is Rhcumntol. 2016;68:1 · 26. [PMJI): 265~5940] MeA lindon TE. llnnnum RR. Sulllwn MC. flrden 1\K, llercnbaum 1·: Brcrmij-
dol: 10.1002/art.39480 7.einslrn SM. et al. OARS! guidelines for the non-surgical n\lln<Jgement of
van der Coes MC. Jambs JW. Bljlsma JW. The value of glucocorticoid co-lherapy knee osteoarthritis. Osteo~rthritis Curtilage. 2014;22:363·88. [t'MIO:
In dlffercnr rheumatic diseases-posillvc Hnd ooversc effects. Arthrilfs Res 24462671] doh10.1016fj.joc.t.2014.01.003
Ther. 2014:16 Suppi2:S2. li'MID: 25606693] dol :10.11 86/ar~686 McAIIn<lnn TE. LuValley MP. Harvey WF, Price Lt.. Driban JB. Zhang M, et al.
Wang c. Schmid CH. Iversen MO. Harvey WF, Fielding RA, Oriban JB. er al. Eflect of lntr~-~rt1cular triamcinolone vs saltr1e on knee cartilage volume
\.omparnlfve effectiveness of tal chi versus physical theropy ihr knee osteo- and pain in patienls with knee oSleoarthritls: a rJndomtzed clinical trial.
arthritis: n rnndomtred trial. Ann lnlem Med. 2016:165:77·86. (PMID: JAM/I. 2017:317:1%7·1975. [PMID: 28510679] doi:lO.lOOlljama 2017.5283
27183035} dui:I0.7326/M 15·2143 M;lChMo GC, Mnher CG, Ferreiro PH. f>inheiro MB. l.ln CW, Day RO. et nl.
Efricacy and safety of pm·JcetamCl1 for spinal pain ~nd osteoarthrilis: sys-
Rheumatoid Arthritis tcm~tlc review and meta-analysis of rnndomlsed placebo controlled trials.
Alerah.a D, Neogl ·1; Silman AJ, Funovfts J, Felson or. Bingham co 3rd, et ol. 13MJ. 2015:3SO:hl22S. (!>MID: 25828856] dol:10.1136ibmj.hl225
2010 Rheumatoid arthritis classillcation criteria, an Ameri""n College or Messier SP, Mihalko Sl, Leg-dult C. Miller GO. Nicklas BJ. DcVItn P. et al. Effects
Rheumatology/European Le.ilgue Against Rh~umotlsm collahomtive initia- or Intensive diet Jnd exercise on k.nee jOint loads, inflammation. and cUn!-
tiVe. Arthritis Rheum. 2010;62:2569-81. [PMIO: 20872595( doi:10.1002/ C!I outcomes among ove~-eight and obese adults with knee osteoarthritis:
art.27S84 the fl)EA randomized clinical trial. JAMA. 2013;310:1263 ·73. fPMJf):
Bakker MF. Jacobs JW, Welsing PM. Vcrst3ppen SM. Tekstm J. 1bn F.. et ul: 240650!:!1 dot: 10.1001 /j:lma.2013.277669
Utrecht Rheumatoid Arthritis Cohort Sludy Group. Low-dose prednisone Sfemlenluk RAC. Harris lA, AgorltsasT, Poolman RW. Srignardello-retersen R.
Inclusion in a methotrexate-based, tight control strategy for early rheuma- van de Velde s. et at. Arthroscopic surgery lor degenerotivc knee artbrttls
to!<l arthritis: a randomized trtal. Ann Intern Med. 2012:1S6:329·39.(PMID: and mentscal re~rs: a cllnicnl proctice guideline. BMJ. 2017:3S7:jf982.
22393128J dol:10.7326t0003-4819-156 5-201203060-00004 [PMID: 28490431] doi:lO.Il:i6/bmj.j1982
Moreland I.W. O'Dell JR. Paulus HE. Curtis JR. Bathon JM. StClair EW. et al: Thortund JB. Juhl Cll. Roos EM. l..ohmander LS. Arthm,;coplc surgery for
TEAR lnoestig;uors. A rnndomi?.ed comporntive eflecllveness study of ornl degenerative knee: $ystematie t-eview and meta-analysis of heneflts and
triple the111py versus etllnem:pt plus methotrexate in early aggres.~ive rheu· harms. SMJ. 20!5:350:112747. [PMIO: 260800451 dol:I0.11361bmj.h2747
matoid arthritis: the treatment of Early Aggressive Rheumatoid Arthrttls
Trial. Arthritis Rheum. 2012;64:2824-35. [f>MIO: 225084681 doi:10.1002/ Wang C. Schmid CH. Iversen MD, HalVe)' WF, Fielding RA. Drthan JB. et al.
art.34498 Comparative effectiveness oftai chi ve!'lius physlClliherapy for knee osteo-
arthritis: a rnndomized triol. Ann Intern Med. 2016:165:77-86. [P?-110:
Scher JU. Littman OR. Abromson SB. Mlcrnbiome in Inflammatory A•~hritis 271830351 doi:I0.7326/MIS·2143
and I Iuman Rheumatic Diseases. Arthritis Rheummol. 2016;68:35·45.
(PMlD: 26331579 l dol: 10.J002fart. 39259 Ftbromyalgla
SChiff MH. Jl'lffe JS, FreuMii<:h B. l·lead-m-head. rondC>mised. crossover Adnms Ell, Mcl:lroy IIJ, Udall M, Masters £:.1. Mann RM. Schaefer CP. et at.
studyororol versus subcutaneous methotrexate In patients with rheumatoid Progression o r flbromyalgla: results fron\ a 2-year observational Obromy.JI-
arthritls: dnrg-exposure lhnitalions of oral methotre)(at<! at doses -IS mg gia ond chronic pain study In the US. J Pain Res. 2016:9:325·36. (PMIO:
may be overcome with subcutaneous adminlstrotion. Ann Rheum Dis. 273303251 doi:l0.2147/JPR.S10004~
2014;73:1549-51. LPMID: 24728329] doi:IO.ll36/annrheumdls- 2014
205228 llernardy K. Klose P. Busch AJ. Choy EH. HAuser W. Cognitive behavioural
therapies for llbromyalgla. Cochrane Outab~se Syst Rev. 20!3:C0009796.
Singh JA, Saag KG, BridgesSLJr. Akl EA. ll:mnuru RR. Su1livan MC. er nl. 2015 [PMID: 24018611) doi:I0.1002/14651858.CD009796.pub2
Ameriean COII~ofRheumatologygu1deline for the t~tmentorrheuma­
told arthritis. AnhTit1s Rheumntol. 2016;68:1-26. (PMJO: 26S4S940J C..:lauw OJ. ~lbromy:~lgia: a clinical review. JAMA. 2014;311:1547·55. [PMID:
doi:10.J002/~rt.39480 24737367} dof:lO.t001/jama.20J4 .3266
Smolen JS. Alet:lha D. Mcinnes fB. Rheumatoid arthrltis.l.ancet. 2016:388:2023- Macfarlane GJ, Kronisch C, Dean LE. At~cni F. Hauser W. Flu IS E. et ~1. EULAR
2038. [PMIO: 27156434} dol:l0.10J6/SOH0-6736(16)30173-8 revised recommendations for the management of flbromyalgla. Ann
Rheum Dis. 2017:76:318-328. [PMIO: 27377815] doi:10.1136/onnrheum-
Souhrler M. Barber Chamoux N, Tatar Z. Couden: M. Dubost JJ. Mathieu S. dls-201(>·109724
Cardiovascular risk ln rheumatoid arthritis. Joint Bone Spine. 2014;8t:298-
302. [PMIO: 24880'190] dol:10.1016/j.Jbspln.2014.01.009 QuelrO'~ LP. Worldwide epidemiology uf llbromyalgia. Curr Pain I lead ache
Rep. 2013;17:356. (PMII): 238010091 doi:10.1007/sll916·013-0356· S
Stoffer MA. Schoels MM, Smolen JS, Aletaha 0, Brcedveld FC. Burmester C,
et al. Evldence for treating rheumatoid arthritis to target: results of a sys- Wolfe F. Clauw DJ. Fllzcharles MA, Goldenberg 01, KG~ RS. Mease P. era!. The
tematic literature se.uch update. Ann Rheum Dis. 2016:75:1D-2.2. (PMID: Ameclean ('~lllege of Rheumatology preliminary diagnostic crileria for
25990290] doi;!O.lt36/annrheumuis·20JS-207526 flbromy.Jigla and measurement of symptom severity. Arthrttl~ care Res
(lloboken). 2010:62:600- IO.(PMID: 20461783] doi:l0.1002/ocr.20140
Osteoartlultls
Bannuru RR, Schmid CH, Kent DM. Vnyshrot EE. Wong Jrl, McAlindon TE. Spondylonrthrltls
COmparative ellecllveness or pharmacologic interventions fnr knee osteo- Asquith M. Rosenbaum JT. Th<! interacllon between host J(enetlcs and the
arthritis: a sy:,1ematlc review and nttwork melll-unalysls. Ann Intern Med. mrcrobi<>me In the pathogenesis <>f ;-pondyloarth ropathles. Curr Opin
2015;162:46·~-lPMIO: 25560713J dol:t0.73261M14-1231 Rhcumatol. 2016:18:105·12. [PMID: 27152700] dol:10.109?fBOR.
da Costa SR. Reichenbach S. Keller N. Nart~y L Wandel S. Joni P, et al. 0000000000000299
t::lfecllvene.o;.~ of non-steroidal anti-inflammatory drugs for the treatment uf Coates L.C, Kavnnaugh /1, Mease Pl, Soriano ER. Laurd Acosta ·Fclquer M.
pain In knee ond hlp osteoarthritis: ll network meta -analysis. Lancet. Armstrong AW. et 111. Group for Rest.-arch a nd Assessment of Psoriasis and
20l7;390:e21-e33. [PMID: 28699595) doi:10.10t6/S0140-6736(17)317H-O l'soriutlc Arthritis 2015 treatment recommendations for psoriatic arthritis.
Derry S. Moore Ri\. Rahb!e R. Topi<'al NSAJf)s for chronic musculoskeletal pain Arthritis Rheumatol. 2016:68:1060·71. (PMID: 26749174] doi:IO.l002/
In adults. COchrnne l>atahase Syst Rev. 2012:C0007~00. (PMIO: 22972108] art.39S73
doi:10.IOO:lil4651858.CD007400.puh2 Colla R, COrrudo A. Cancatone FP. Rheumatologk nnd extmlntt'Stinal manifes-
Fernandes!.. Hagen Kll, Bijlsma JW, Andrwssen 0, Christensen r, Conagh.an tations ofinllammalory bowel dlseas~. Ann Med. 2016:48:577-SS.S.(PMIO:
PG. et al; European League Ag;ltnsr Rheummlsm (EULARJ . F.ULAR recum- 273100961
mendatlons for Ihe non-phnrmacological core management of hlp and Coss<:e I. Smolen JS. Ramlro S. de Wtt M. Cutolo M, Ooug:.~dos M. et ~1.
knee osteoarthritis. Ann Rheum Dis. 20!3;72:1125 35. [I'MID: 23S9SI.f2] l':uropean U.>as:ue Against Rheumatism (f:lULAR) recommendations lhrthe
doi:IO.II36/annrheumdis-2012·202711S mamlj,'l'ment of psorl:ttlc arthritis with ph.1rmU('tlloglcal themples: 2015
Fr-~nsen M, Mt:Conncll s. H~rmer AR. Van dcr F.sch M. Slmic M. llenncll K L upr.lllte. Ann Rh.,um Dis. 2016;75:499 ·510. fPMIO: 266442321 doi:IO.tl36/
l;'xerclse f'or osteoarthritis of the knee. Cochrune Database Syst Rev. annrheumdls·:l015-208337
201S:I:CD004376 . [PMID: 25569281! doi:l0.1002114651858.CD004..'~76. llusnl ME. Comorbidllies In psorlulic arthritis. Rheum Dis Clln North Am .
pub3 2015:41:677 98. [PMIJ..): 26'176226] doi:IO.l016ij.rdc.201S.07.008

79
Bibliography

Mease PJ. Biologic lhci'Jpy ror psoriatic arthritis. Rheum Dis (..'tin North Am. Vlvlno 1'13. Carsons S!i, r-oulks G. Daniels TF. Parke fl. Bnmnan MT, et al. New
2015:11:7:13·;~8. (!'MID: 2&176229] dol:10.1016fJ.rdc.201S.07.0!0 treatment guidelines lOr SjOgren's disease. Rheum Dis Clin North Am.
N~polltnnu M, C.aso r., Scarpa R. Megna M, Patrl fl. llalatu N, ct al. Ps01iatfc 2016:42:.<;:ll-51. [PMID: 27431353] doi:JO.l016/).rdc.20J6.0:J.<ll0
nrthriHs and psorl~sis: diflcrcntinl diagnosis. Clln Rheumatol. 2016;35:1893·
lnRammatory Myopathies
190l.IPMIIJ: 2.7156076] dol:10 .1007/sl0067 016 ·3295 · 9
Basharat P. Chrlstophcr-Stinc L Immune-mediated necrcti7.ing myop.1Thy:
!'ilc:pcr J. f>oddubnyy D. New evidence on the management ofspondyloarthri· update on diagnosis :1nd management. Curr Rheum.atol Rep. 2015:17:72.
tis. Nat Rev Rheumotol. 2016;12:282-95. LPM!D: 27052-1891 dol:l0.1038f (PMIO: 26515574] doi:JO. I007fSll926· 015-0S48· 6
nrrhenm.2016A2
lundberg IE, Miller f.W, Tj:lmiund fl. Bnttai M. Diagnosis and classiOcation of
Soriuno ER. M~nagement of psnriatic ~rlhrilis: lmditional disease- modifying idiopathic inflammatory myoparhles. J Intern Med. 2016:280:39·.'il . [!'MID:
rheumatic ogcntsand targeted small molecules. Rheum DisCI in North Am.
27320359] doi:10.1111/joim.l2S24
2015;41:711·22. [!'MID: 264762281 dol:10.1016/j.rdc.201.~.07.012
Mahler M. Millc:r FW. Fritzler MJ. ldlopathlc lnOammatory mytlpathiesand the
Stolwljk C. wn Onnu M, Boonen A. van Tubergen A. Global prevalence of spon· anth-ynthetase syndrome: a comprehensive review. Autolmmun Rev.
dyloarth11tls: a systematic review and meta-regression analysis. Arthritis C.are 20H:I3:367· 7i.(PMIO: :.!44241901 doi:10.1016/j.llutrev.2014.01.022
Rt'S (I Joboken). 2016:68:1320·31. [PMID: 267134321 dol:10.1002/ocr.22831
Mammen AL Nt-cmtlzinp,myopathics: beyond slat ins. Curr Opin Rheum~ to!.
Tnurog JD, Chh~l>nl A. Colbert RA. Ankylosing spondylitis nnd oxlal spondy- 201~:26:679·83. (PMID: 252031171 doi:10.1097/BOR.0000000000000106
loanhrltis. N £ngl J Med. 2016;374:2563-74. (PMIO: 27355535] dol:l0.1056/
Nt:JMra14C6182 Mammen AL Stntin-associated autoimmune myopathy. N Eng! J Med.
2016;374:661·9.lPMIO: 26886523) dot:l0.1056/Nrr.JMral515t61
Ward MM, De<xihar A, Akl EA, Lui A. F.rrnann J, GeMI~r LS, et aJ. American
Q~JegeofRkumatologyiSpondylilisAssod:ltlonofArnetica/Spondyk>arthritl~ Needham M. Mnstaglln fL. Sporudic inclusion body myositis: ~ review of
Rest"arch <1nd Treatment Network 201 S recommendations for the trentment of recent clinical advances and current ~pproaches to di3gnosls nnd treat-
~ nl.-ylosing spondylitis and non radiographic axl~l spondyhlOirthritis. Arthritis ment. Clln Neurophyslol. 2016:127:17M·73.li'MIO: :16n8717] dol:l0.1016/j.
Rhcumatol. 2016;68:282-98.(1'MID: 2&1019911 doi:IO.I002fart.39298 cllnph.201S.I2.011
Ql3ng Jl<. Kim Wl!. BaJbergenow A, Alhusa~n R. Risk ol'maUgnancy In der-
Systemic Lupus Erythematosus matomyositis nnd polymyosltls. J Cutnn Med Surg. 2017;21:131-136. [PMID:
Abele~ AM, flbclcs M. The clinical
utility of a positive anllnuclear Jntlbody test 27.134779] doi:IO.I177f!20347541666560i
result. Am J Med. 20l3:J26:342 · 8. (PMIO: 23395534] dol:tO.I016fJ. Tleu l. Lundberg IE. Umaye V. Idiopathic lnOarnmatory myositis. !lest Pr~ct
am1med.2012.09.014 Res Clln Rheumatol. 2016;30:149-6R. (PMIO: 27-121222) doi:l0.1016/j.
Donley MA, Jayne 0, Ginzlcr EM, lsen~.rg D, Olsen NJ, Wofsy D. et al; fiLMS berh.2016.0-1.007
Gmup. Mycophenolate versus azathioprine as maintenance therapy for
Systemic Sclerosis
lupus nephritis. N Engl J Med. 2011 ;36S:1886·9S. [PM!O: 220876801
doi:10.1056fl-o'l'.JMoal014460 Cappelli L. Wigley FM. Management of Rnynaud phenomenon :tnd dlgiml
ulcers in scleroderma. Rheum Dis Clin North Ant. 201S;41:419··38.LPMIO:
Hahn 13H. McMahon MA. Wilkinson A, W:lllace WO. O~ikh 01, F'h7.gcr.lld JD.
26210127) dol:l0.1016fj.rdc.201S.04 .00S
c:t al: American College or Rheumatology. American College: of
Rheumatology guidelines for screening. treatment, and management or Hudson M. Scleroderma renal crisis. Curr Opin Rheununol. 2015:27:549-54.
lupus nepluitls. Arthritis Care Rei (Hoboken). 2012.64:797-808. (PMID: [PMID; 263~Vnl cloi:l0.10971'SOIW000000000000221
22556106] doi:IO.l002/acr.21664 Marder W. Uttlejohn EA. Somers F.C. Pregnancy and autoimmune conne(tlve
llallk>glu S. Carlloglu A, Akdeniz D, Kamaslan Y. Kosar A. Pibromyalgtn in tissue dtse.1~. Best Pract Res Clln Rheurnatol. 2016;30:63-80. [PMIO:
patlenlli wllh other rheum~tlc dlseaS<..o:;: prev,tlcnce and relationship wfrh 274212171 dol: t 0.1016fj.berh.2016.05.002
diseas~ activity. Rhcumatol Int. 2014:34:1275-80. (PMID: 24589726] McCmy 0, Mayes MD. Upd<tte on systemic sclerosis. Curr Allergy Asthma Rep.
doi:l0.1007/s00296-0l4-2972-8 2015;15:25. (PMID: 26139334] doi:10.1007/s11882-015·0526-0
l.azz.amnl MG. Daii'Arn F. F'tedl M, Nnlli C. Reggla R. Lojacono A. et at. A com- Silver KC, Silver RM. Management ofsystemic· sclerosis-associated interstitial
prehensive review oflhe clinicalappro<~ch tc> pregnancy and syst~mlc lupus lung disease. Rheum Ols CUn North A111. :t015;41:439-.57. (!'MID: 26210128}
erythematosus. J flutoimmun. 2016;7~:10tH17. [PMID: 27377453] doi:10.t016fj.rdc.201S.04.006
dol: 10.1016/j.jaut.2016.06.016
Valen7.u~la fl. Nandagopal S, Steen VD. Chung L Monttorlng nnd diagnostic
Miner JJ. Kim AH. Cardiac: manifestations or systemic lupus erythematosu.~. approoches for pulmonary a rterial hypertension in patients with systemic
Rheum nisClin North Am. 2014:40:51-60. [PMID: 24268009]dol:l0.1016/j. sclerosis. Rheum Dis Clin North Am. 1015;41:489·506. [PMID: 26210131]
rclc.2013.10.003 dol:10. 1016/j.rdc.2015.04.009
Petri M. Orbal AM, flhmxin GS, Gordon C. Men! II JT, Fortin PR, et al. Deriv-ation van den lloogen F. Khanna 0, Pr.msen J,Johnson SR. Barcn M, Tyndall A. era!.
and validation of the Systemic Lupus International Collaborntlng Clinics 2013 clussllic:ltion criteria for systemic sclerosi~: an American college or
classlflcatlon criteria for systemic lupus erythematosus. Arthritis Rheum. rheumatology/European le~gue ag~inst rheumatism collaborative Initia-
20U;64:2677-86. [PMrD: 225S3077J do!:10.1002/:~rt.34473 tive. Ann Rheum Dis. 2013;72:1747-55. (PMID: 24092682] dol:l0.1136/ann·
Singh JA. Hossain A. K(>th A. Oliveira A. Mudano AS. Grossman J. et a!. rheumdls-2013· 20~42~
'lfeatments for lupus nephritis: a b)'Stcmatic review and network m&.lo nal-
ysts. J Rheumatol. 2016;43:1801 1815. (PMID: 27585688) Mixed Connective Tissue Dlsea~
wn Vol!enhoven RF, Mosca M. Bertslas C. Isenberg D. Kuhn A, Le.slmm K. Mosca M. Tan I C. \lagnani S, Carli L llombardiert S. The diagnosis and classlfl·
cr al. Treat-to-target In ~ystemic lupus erythematosus: recommendations cntton of undifferentiated connt.><.1:ive tissue dise<~ses. J Autoimmun.
from an intcmatlonnl task force. Ann Rheum Dis. 2014;'73:958-67. (PM!O: 2014;48· 49:50·2. [PMiD: 245188551 dol :l0.1016/j.jaut.2014.01.019
21739325) doi:l0.1136fanmheumdls-2013· 20Sl39 Tan! c. Carll L. Vagnanl S. Talarico R. Baldini C. Mosca M. et al. The dlagnusL~
and classification of mixed connective tissue disease. J Autolmmun.
Sjllgren Syndromt 2014:48·49:46-9.[PMID: 2-1-1613871 dol:10. 1016/1.j3ut.2014.01.00R
Drsons SE. Vlvino FB. Parke A. Drteron N. Sankar V. Rrasington R. et al. Crystal Arthropath ies
Treatm~nt guidelines for rheumatolngic manifestations of SjOgren's syn-
drome: use of bkllogic agents. management of fatigue, and inflammatory Abhishek A. Galcium pyropho.~phote deposition disease: a review or epide-
musculoskelecal paln. Arthritis care Res (Hoboken). 2017;69:517-527. miologic findings. Curr Opln Rheumatol. 2016:28:133-9. (PMID: 26626724)
[PMJl): 27390247] doi:10.1002/acr.22968 doi: 10.1097/BOR.0000000000000246
F'oufks CN. f.oMot SL. Donshlk PC. etal. Clinical guidelines for management Hill EM, Sky K, Sit M. Colla mer fl. Higgs J. Does stnrtl ng allopurtnol prolong
ofdry eye associated wtthS}l>grendlsease. Ocul Sur( 2015Apr:13(2):118-32. acute treated gout? A ra ndoml7.ed cllnlt~dl trial. J Cltn Rhcumatul.
[PMiD: 25881996] 2015;21:120-S. [!>MID: 25807090] doi:I0.1097/RHU.0000000000000235
Shlhoskl Cll, Shlbosld SC. ~ror R, Criswell LA, la~toulle M. Llermnn TM, et Khanna PP. Gladue HS. Singh MK. f.lttCeru!d 10. Rae S. Prakash S. et al.
al; International Sj(lgren's Syndrome Criteria Working Group. 2016 Treatment of ncute gout: a systematic revii!!W. S<!min Arthrtrls Rheum.
flmcrlcan CollegeorRheumatulogy/European League flg:>inst Rheumatism 201-1:4-1:31 · 8. [PMiD: 2-16SOml doi:I0.10t61j.semarthril.2014.02.003
dasslfleatinn criteria for primary SjOgren's >yndmme: a consensus and Lipsky PE. Calabrese LH. Kavnn~ugh A. Sundy JS, Wright D, Wolfson M, eta!.
data-driven methodolom' involving three international patient tuhorts. Pegloticase lmmunogcnlclly: the relarionshlp between efficacy and anti·
Ann Rheum Dis. 2017;76:9·16. [PMID: 27189466] dol:10.1136/annrlleumdls· body development In patients treated for refractory chronlcgour. Arthritis
2016·210571 Res Thcr. 2014:16:R60. [PMID: 2~588936.1 dol:lO.l186/ar4497

80
Bibliography

Neogi T. Jansen TL. Dalberh N, Fr.msenJ. Schumacher fIR. Beremlsen D, et ~1. Greco A, De VIrgilio 1\, Rizzo MI. Gallo A. Magliufo G. Fuscont M, et al.
2015 Gout Clossification Criteria: un 1\merlcan College or Rheumatology/ Microscopic polyungiit1s: advances in diagnostic and therapeutic
European Le-Jgue Agalnsr RheutHatlsm colluhomttve lnltlnllve. Arthrtlls approaches. Autoimmun Rev. 2015;1-1:837-44. [PM ID: 25992801}
Rheumatol. 201~:67:25$7·68. [PMIO: 263S2H?:il doi:IO.J002/art.39254 doi .10.10t6/].nutrev.20l~.os.oos
Qaseem A, Harris RP, l'orctea MA; Clinical Guidelines Olmmittee or the Greco A. RizroM I. OeVIrgtfloA, Gallo A. Fusconl M. RuoppoloG, et al. Churg-
American College of PhYlOICI~ns. Man~enl<!nt of Acute and Recurrent Gout: Strauss syndrome. J\utofnunun Rev. 2015:14:341-8. (PMID: 25500434]
A Clinical Practice Guideline From the AmeriC'•n College of Physicians. dol: 10.101 6/].autrev.2014 .12.004
Ann Intern Med. 2017:166:58-68. [PMIO: 27802508]doi:l0.7326/M16-0570 Micheletti RC, Werth VP. Sm3ll vesselwsculit1~ of the skin. R~um Dis Clin
Richette P. Doherly M. Pascual £. llarskova V, Becce F. C<lstafiedo· Sanabrla J, Not1h Am.2015:41:21 · 32, vii.[PMIO: 2S399937)doi:10.1016fj.rdc.2014.09.006
et al. 2016 updated £ULAR evldencc- b3sed re<.~>mmendations lor the man- Patel RM. Shulman ST. Kaw~~ki disease: a comprehensive revic:w of lreat-
~gement of gout. Ann Rheum Ols. 2017:76:29-42. [PMID: 27~57514] ment oplions. J Clio Pharm Ther. 2015;10:620·5. [PMIO: 26547265}
doi:10.U36/annrheumdis-20l6-209707 doi: 10.1111/jcpr.12334
Rosenthal AK, Ryan LM. Calcium Pyrophosphate Deposition Ulsease. N Engl J Rodriguez-Pia A, Monach !'A. Primary angiitis or the cenrral11ervous system in
Mecl. 2016;37<1:2575-84. [!'MID: 27355536) dol:10.1056fNEJMra1SJI117 adults and children. Rheum Dls Clln North 1\m. 2015;41:47-62, viii. !PMID:
van Ourme CM, Wechnlcl<ar MD. Buchl:>inder R, Schlesinger N. van der Heijde 25399939] doi:IO.JOJ6fj.rdc.2014.09.004
D. 1...3nd~ RB. Non-slellllcl.1l onli-inllammutory drugs for acute gout. Stone JH, Tuckwcll K. Dim<maco S. l<learman M, A ringer M. Blockmans U. et
Cochrnne Database Syst Rev. 2014:CDOIOI20. [PMIO: 25225849) al. Trial of to(.:lllz.umab In giant-cell arteritis. N Engl J Med. 2017;3n:317-
doi:l0.1002/146Sl858.C0010120.pub2 328. (PMID: 28745999} doi:lO.l056/NEJMoa1613849
Infectious Arthritis Wey-and CM. Gorom.y 11. Giant-cell arteritis and polymyalgta rheumatica
A1vlkolr SL Steere AC. Diagnosis and tre~tment of Lyme arthritis. lnfc<.:t Ois [Letrer]. J\1 Engl J Med. 2014;371ll653. [PMJI): 2533775'1] doi:J0.\056/
Clln North t11n. 2015;29:269-80. [PMID: 25999223) dol:l0.1016/j. NEJMc1409206
i<.lc.2015.02.004 )'.lees M. Watts RA. llojem~ IM. Cid MC. Crestanl R, Hauser T, et al . EULA!U
Kapadia BH. Berg RA. {}Jiey JA, Fritz J, Bhave A. Mont MA Periprosthetlc joint EAA-f:DrA recommendations tor the management of ANCA-associared
Infection. Lancet. 2016:387:386-394. (PMID: 261357021 dol:JO.l016/S0140- vasculitis. Ann Rheum Dis. 2016;75:1563·94. [P~HD: 27338n6]dol:l0.1136/
6736(14)61798·0 annrheumdls-2016·20913~

Osmon OR, Oerburl El'. Aerendl AR, t.ew D. Zlmmerll W. SteckclhergJM. et al: Other Rheumatologic Diseases
lnrecl!ous Diseases Society of America. Uiagnosls and munagement of Cerraud-Valentin M. Jamliloux Y, fwaz J. Sevt! P. Adult-onset Still's disease.
prosthetic joint infectlon: clinical prnclice guidelines by the Infectlous Autolmmun Rev. 2014;13:708 · 22. [PMlO: 24657513) doi:lO.l0161].
Diseases Society of America. Clln Infect Ois. 2013;56:el-e25. [PMID: autrev.20H.Ol.OS8
23223583] dol:10.t093/ddfds803
International Team for the Revision or the international Crtterla for Behc;et's
Shnl'ff KA. Richards RP. 'lownes JM. Clinical management of septic archrltis. Disease (!TR-lCBO). The !ntm1alional Criteria for l!dlt;efs Disease (ICBO):
Curr Rheumatoi Rep. 2013;15:332. (PMil): 23591823] dol:10.1007/sll926· a collaborative sludy of'Il countries on the sensllivily and specitlclly of the
013-0332-4 new criteria. J Eur Acad Dem1atol VenereoL 2014;28:338-47. [PMIU:
Wang OA. Tamb}'dh I'A. 5eptlc arthritiS In immunocompetent and lrnmuno- 23141863] doi:lO.Ullfjdv.J2107
suppres!;ed hosts. Oest i>rJct Res Clin Rhtumatol. 2015:29:275-89. [PMID: Koook s. SarcoidosiS: a rheumatologist's perspective. Ther Adv Musculoskelet
26362744] dol:l0.10l6/j.herh.201S.OS.008 Dis. 2015:?:196-205.[PMlD: 26425148] doi:IO.Ilnti759720XISS91310
Systemic Vasculilis Mahajan VS, Mattoo H. Deshpande V. Pfllai SS, Stone JH. lgG4 -related dl~Jse.
Cacoub P, Comarmond C. Demont F, Savey L, Saadoun D. Cryoglobulinemia Annu Rev Pathol. 2014;9:315·47. (PMiD: 24111912] doi:IO.ll46fannurev·
v.Jsculitis. Am J Med. 2015;128:950-S. [PMfD: 25837517) dol:l0.1016/j. pathol-012513· 104708
amjmed.2015.02.017 O'Regan A. Berman JS. SaocoldosJs. Ann Intern Mcd. 2012:1S6:ITC5-l, ITCS-2.
Comarmond C. C~t'Oub P. Granulomatosis wlth poi,YlJngUtls (Wegener): clini- l'fCS-3. iTCS-4. iTCS-S,lTCS-6, ITCS-7.1TCS· 8, iTCS-9, ITCS-tO,ITCS-11,
cal aspects and treatment. Autolmmun Rev. 20t4;13:1121-5. LPMIO: ITCS· I2, ITCS-13, 1TCS·I4, ITCS· IS; quiz JTCS-16. [PMIO: 22547486}
<51'19391) doi:IO.IOI6/j.autrev.2011.08.017 doi:I0.7326/0003-4819· 156·9·20120SOI0·01005
de Souza AW. de Carvalho JF. Oiagnostlc and classification criteria ot'Takayasu Peruglno CA. Wallace ZS. Meyersohn N. Oliveira G, Stone JR. Stone JH. 1...3rge
arteritis. J Autoimmun. 2014;48-49:79-83. [PMIO: 214613811 dol:10.1016/j. vessel lnvolwment by lgG4-related disease. Medicine <nalt1more).
jaut.2014.01.012 2016:9S:e3344. [1'M10: 27128181] dol:l0.1091/MD.0000000000003344
De Vlrgllio A. Greco A. MaRiiulo G. Gallo A. Ruoppolo G. Conte M. et al. RubarteiU A. AutoinOammutory disease.•. lnm1unol Lett. 2014;161:226-30.
Polyarteritis nodo.sa: a contemporary overview. Autoimmun Rev. [PMIO: 24452074] d<>l:l0.1016/j.lmlet.2013.12.013
2016;1S:S64-70. [PMIO: 26884100] dol:10.10l6fj.autrev.2016.02.<ll5 Russo RA, Brogan PA. Monogenic autoinOammatorydiSeases. Rheumatology
Dejaco C. Singh YP, Pere! P. Hutchings A. Camelllno D. Mackie S. et al; (Oxford) 2014;53:1927-39. {I'MID: 2483tOS6] doi:l0.1093/rbeumatology/
European l.ellgue Against Rheumatism. 2015 Recommendations for the keul70
managemenr of polymyalgia rheumatlca: a European l.eague Against VItale A. Sota J, Rlgante 0, t.opalco G. Molinaro E Messina M. et 91. Relapsing
Rheumatism/American Olllege of Rheumatology collabor-Jtlve inltlalive. polychondrltls: 3n upd~te on pathogenesis, clinical features, di&gJ10Stic
Ann Rheum !)Is. 2015:74:1799·807. [PMID: 26359488] doi:JO.J136/ann- tool~. and therapeutic perspec!l~s. Curr Rh~umatol Rep. 2016;18:3.lPMID:
rheumdls-2015-207492 26711694] dol:l0.1007fsl1926-015-0S49-5

81
Rheumatology Self-Assessment Test
This self-assessment test contains one-best-answer multiple-choice questions. Please read these directions carefully
before answering the questions. Answers, critiques, and bibliographies immediately follow these multiple-choice
q uestlons. The American College of Physicians (ACP) is accredited by the Accreditation Council for Contln ulng Med-
ical Education (ACCME) to provide continuing medical education for physicians.

The American College of Physicians designates MKSAP 18 Rheumatology for a maximum of 22 AMA PRA Category 1
Credits™. Physicians should claim only the credit commensurate with the extent of their participation in the activity.

Successful completion of the CME activity, which includes participation Jn the evaluation component, enables the
participant to earn up to 22 medical knowledge MOC points in the American Board of Internal Medicine's Mainte-
nance of Certification (MOC) program. ft is the CME activity provider's responsibility to submit participant comple-
tion information to ACCME for the purpose of grantjng MOC credit.

Earn Instantaneous CME Credits or MOC Points Online


Print subscribers can enter their answers online to earn instantaneous CME credits or MOC points. You can submit
your answers using online answer sheets that are provided at mksap.acponline.org, where a record of your MKSAP
18 credits will be available. To earn CMEcredits orto apply for MOC points, you need to answer all of the questions in
a test and eam a score ofa tleast so·~ correct (number ofcorrect answers divided by the total number of questions).
Please note that if you are applying for MOC points, you must also enter your birth date and ABTM candidate number.

Take either of the following approaches:


• Use the printed answer sheet at the back of this book to record your answers. Go to mksap.acponline.org,
access the appropriate online answer sheet, transcribe your answers, and submit your test for instantaneous
CME credits or MOC points. 1here is no additional fee for this service.
• Go to mksap.acponline.org, access the appropriate online answer sheet, directly enter your answers, and
submit your test for instantaneous CME credits or MOC points. 1here is no additional fee for this service.

Earn CME Credits or MOC Points by Mail or Fax


Pay a $20 processing fee per answer sheet and submit the prfnted answer sheet at the back of this book by mail or
fax, as instructed on the answer sheet. Make sure you calculate your score and enter your birth date and ABIM can-
didate number, and fax the answer sheet to 21.5-351-2799 or mail the answer sheet to Member and Customer Service,
American College of Physicians, 190 N. Independence Mall West, Philadelphia, PA 19106-1572, using the courtesy
envelope provided in your MKSAP 18 slipcase. You will need your 10-digit order number and 8-digit ACP JD number,
which are printed on your packing slip. Please allow 4 to 6 weeks for your score report to beemailed back to you. Be
sure to include your email address for a response.

If you do not have a 10-digit order number and 8-digit ACP 10 number, or if you need help creating a usemame and
password to access the MKSAP 18 online answer sheets, go to mksap.acponline.org or email custserv@acponline.org.

CME credits and MOC points are available from the publication date of July 31, 2018, until July 31, 2021. You may
submit your answer sheet or enter your answers on line at any time during this period.

83
Directions
Each of the numbered ttems is followed by lettered answers. Select the ONE lettered answer that 1S BEST 1n each case

CJ item 1 (C) Microscopic polyangiitis


A 72 year old man is evaluated in the emergency department (0) Rheumatoid vasculitis
after falling when his leg gave way as he tried to arise from (E) 1hromboanglitis obliterans
bed. He has left htp pain. with the inability to stand and pain at
rest. He was recently diagnosed with lymphoma. for which he
is receiving chemotherapy. History is significant for a lett hip Item 3
replacement 7 years ago for osteoarthritis. His chemotherapy A 78-year· old woman is evaluated for a 2-year history of
regimen consists of rttuxlmab plus hyperfractlonated cyclo- gout with progressively more frequent and severe attacks.
phosphamide, vincristine, doxorubicin. and dexamethasone. She currently has pain and swelling in the right second fin-
On physical examination, temperature is 38.2 °C ger. History is also significant for hypertension, chronic kid-
(100.8 °F): other vital signs are no rmal. 1he right upper ney disease, nephrolithiasis, and type 2 diabetes mellitus.
chest Is implanted with a venous ae<.-ess port. Warmth and Medications are lisinopril, furosemide, metformin, and t he
tenderness around the lett hip are noted. Pain in the groin is maximal dose offebuxostat; she Is allergic to allopurinol.
noted.1here Is limitation of motion in all d irections on both On physical examination, vital s igns are normal. The
active and passive range of motion of the left hip. lhere are joint findings are shown.
no other joint abnormalities.
Laboratory studies show an erythrocyte sedimentation
rate of 73 mm/h, a leukocyte count of 13,400/J..IL (13.4 ><
109 /L). and a serum urate level of8.2 mg/dL (0.48 mmoi/L).
Left hip radiographs show peri prosthetic lucency.
Which of the following Is t he most likely diagnosis?
(A) Gout flare
(8) Hemarthrosis
(C) Hip dislocation
(0) Prosthetic joint infection

CJ Ite m 2
A 55-year-old man is hospitalized for acute respiratory fail-
ure requiring intubation and mechanical ventilation. He Laboratory studies show an erythrocyte sedimenta-
also has increasing leg swelling with nonheallngskin ulcers tion rate of 76 mm/h, a serum creatinine level ofl.S mg/dL
on the legs for t he past 4 weeks, and pain with swelling of (132.6 j..lmoi/L), and a serum ura te level of 6.3 mg/dL
the bilateral wrists and finger joints for the past 6 weeks. He (0.37 mmol/L).
quit smoking IS years ago. History is otherwise un remark-
able. Prior to hospitalization he was taking no medications. Whic h of the following is the most appropriate treatment?
On physical examination, temperature is 37.2 oc
(A) Add probenecid
(99.0 °F), blood pressure Is 150/95 mm Hg. pulse rate Is 110/
min. respiration rate is 20/min (ventilator set rate. 14/min). (8) Stop febuxostat; begin pegloticase infusions
and oxygen saturation is 92% on Fl02 of 40%. 1here is blood (C) Stop lisinoprll; begin losartan
in the endotracheal tube. Lung crackles are hea rd bilater- (D) Continue current treatment
ally. Swelling of the wrists, metacarpophalangeal joints,
and proximal interphalangeal joints is noted. Two necrotic
ulcers on the left leg and one necrotic ulcer on the right leg Item 4
are present. There is 2+ pitting edema of the legs. A 30-year-old man is evaluated for arthritis. Three weeks
Laboratory studies show normal C3, C4. and rheuma- ago he noticed the abrupt onset of dysuria without a dis-
toid factor: negative antinuclear antibodies: positive ANCA charge. Two weeks later. he developed the acute onset of
with a perinuclear pattern: and 3+ protein on urinalysis. war mth, pain, and swelling in the left knee. Two days later,
Blood and sputum cultures are negative. pain and swelling developed over the left heel. He also noted
Chest radiograph shows diffuse bilateral Infiltrates. the onset of stinging and redness of the right eye. He takes
Biopsy of a skin ulcer shows nongranulomatous, necrotiz- no medications. The dysuria and eye symptoms have since
ing small-vessel vasculitis with immunofluorescence nega- resolved.
tive for immune complexes. On physical examination, vital signs are normal. A
Which of the following Is t he most likely diagnosis? moderate effusion of the left knee iS present with warmth
and pain with range of motion. Diffuse swelling. warmth,
(A) Granulomatosis with polyangiitls erythema, and tendemess are present at the insertion of t he
(B) JgA vasculitis left Achilles tendon at the calcaneus. 1here is no rash.

85
Self-Assessme nt Test

Radiographs of the left knee show a joint effusion, no that arise from the pylorus. There Is no evidence of a hiatal
bony abnormalities, and normal joint space. hernia or other abnormal findings.
Aspiration of the left knee is performed; synovial fluid
Which of the following is the most likely diagnosis?
analysis shows a leukocyte count of 5000/JJL (5.0 x 109 /L)
with 65'%. neutrophlls and 35% mononuclear cells. negative (A) Cameron lesions
Gram stain and cultures, and no crystals. (B) Dieulafoy lesions
Which of the following Is the most appropriate d iagnostic (C) Gastric antral vascular ectasia
test to perform nex1? (D) Portal hypertensive gastropathy

(A) Chlamydia nucleic acid amplification urine testing


(B) C-reactlve protein
(C) HLA-B27 ~~ 1
A 76-year old man Is evaluated for fever and a swollen.
Cl
(D) Interferon -gamma release assay
painful left knee. He was hospitalized 7 days ago for heart
failure and appropriately treated. However. he developed
fevers up to 38.0 •c (100.4 "F). On examination. lungs were
Item 5 clear to auscultation. The left knee was hot and swollen;
A 32-year-old woman is evaluated for a 15-year history of 60 mLofturbld fluid was drained from the knee. Gram stain
low back pain. 1he pain Is worse with rest, improves with of the synovial fluid was negative: microscopy revealed
movement, and can awaken her during the night. Family needle shaped intracellular crystals. Gout was diagnosed
history is notable for three paternal uncles with back prob- and the knee was drained again. followed by an injection
lems. She takes naproxen twice daily with some relief. of 80 mg of methylpredmsolone. without Improvement.
On physical examination, vital signs are normal. Joint Intravenous methylprednisolone. 60 mg/d for 3 days, did
examination does not reveal any warmth, erythema. or not Improve the knee or the fevers. History is also significant
swelling. Tenderness over the sacroiliac joints bilaterally for hypertension and gout. Other medications are furose-
and reduction m the range of motion of the lumbar spine mide. llsinoprll. metoprolol, subcutaneous heparin, and
are noted. morphine as needed.
Laboratory studies a re notable for an erythrocyte sedi- On physical examination today. temperature is 38.0 •c
mentation rate of27 mm/h. (100.4 •F), blood pressure 1s 148/92 mm Hg, pulserate is 116/
A plain anteroposterior radiograph of the pelvis shows min, and oxygen saturation Is 97% on ambient air. The left
fusion of the sacroiliac joints. knee is warm, swollen, and tender.
Which of the following is the most appropriate diagnostic Blood. urine. and synovial fluid cultures are negative.
test to perform next? Which or the following is the most appropriate treatment
forthekn~?
(A) ANCA
(B) Anti-cyclic cltrullinated peptide antibodies (A) Anaklnra
(C) Antinuclear antibodies (B) Colchicine
(D) HLA- B27 antigen (C} Ibuprofen
(E) No additional testing (D) Vancomycin

C] ltem 6 Item 8
A 43-year-old man is evaluated in the emergency depart- A 55-year-old woman is evaluated for an 18-month his ·
ment for gastrointestinal bleeding. He reports loose. da rk tory of increasingly severe knee pain with the inability
stools of 2 weeks' duration. He has an 8-year history of to arise when seated on the noor. She does not have pain
diffuse cutaneous systemic sclerosis (DcSSc) complicated by at rest or nocturnal pain. Medications are celecoxib and
Raynaud phenomenon and gastroesophageal retluxdisease. omeprazole.
He does not dri nk alcohol or take NSAIDs. His only medica- On physical examination. vital signs are normal. Bony
tion is omeprazole. hypertrophy of both knees is presenr. There is no warmth,
On physical examination, temperature is normal. erythema, or swelling of the joints.
blood pressure IS 100/60 mm Hg. pulse rate is 80/min. Plain anteroposterior knee radiographs show medial
respiration rate is 16/mln, and oxygen saturation is 94'X. joint space narrowing, peaking of the tibial spines, and
breathing ambient air. Skin changes associated with DcSSc osteophytes; there are no erosions o r osteopenla.
are noted from the hands to the elbows In theupperextrem-
ities and from the feet to the knees In the lower extremities. Whkh or the following is the most appropriate manage-
lhe abdomen Is nontcnder to palpation. The remainder of ment?
the physical examination is normal. (A) Glucosamine supplements
Laboratory studies show a normal chemistry panel and
a hematocrit level of26'X.. (B) Knee replacement surgery
Upper gastrointestinal endoscopy demonstrates linear (C) Physical therapy
ectatic vessels resembling the stripes found on a watermelon (D) Prednisone

86
Self-Assessment Test

Item 9 second digits and palms; and pain and swelling of the sec-
o nd and fourth proximal interphalangeal joints bilaterally.
A 25-year-old man is evaluated for the gradual onset of
bilateral low back pain without radiation to the lower She does not take any medications.
On physical examination, temperature is 37.6 oc
extremities daily, with Increasing severity over the past
(99.7 °F), pulse rate is 95/min, and respiration rate iS 20/min;
year. 1he pain now awakens him during the night 2 to
blood pressure and oxygen saturation are normal. 'The tips
3 times per week, with mornjng stiffness lasting more than
of the digits are cool without discoloration. Erythema of
an hour. He has Improvement with exercise and no improve-
ment at rest. He takes ibuprofen with some improvement. the malar area, forehead, and chin Is present. Pulmonary
examination reveals crackles at the lung bases. There is no
On physical examination, vital signs are normal. Lim-
ited lateral bending bilaterally and a reduction in forward weakness.
flexion at the lumbar spine are noted.TI1e remainder of the Skin findings are shown.
examination is normal.
An anteroposterior plain radiograph of the pelvis and
sacroiliac joints is unremarkable.
Which of the following is the most appropriate diagnostic
test to perform next?
(A) Bone scan
(B) CT of the lumbar spine
(C) MRI of the sacroiliac joints
(D) Radiography of the hip joints

Item 10
A 75-year-old man is evaluated for a 5-year history of hand
joint pain and morning stl1fness lasting a few minutes. He
walks about a mile every day and plays tennis 3 days a week; Laboratory studies:
however, the pain has started to limit him from gripping Creatine kinase 115 U/L
a tennis racket. He also has noticed fatigue for the past Antinuclear antibodies Titer: 1:1280
6 months. He reports no other constitutional symptoms. Anti-double-stranded DNA antibodies Negative
shortness of breath, or rash. History is also significant for Anti- Jo-1 antibodies Positive
hypertension and diet-controlled type 2 diabetes mellitus. Anti-Smith antibodies Negative
Medications are losartan, acetaminophen as needed, and
aspirin, 81 mg/d. Which of the following is the most Likely diagnosis?
On physical examination, vital signs are normal. Bony
enlargement of multiple proximal and distal interphalan- (A) Antisynthetase syndrome
geal joints of the hands is noted. The remainder of the phys- (B) Sjogren syndrome
ical examination is normal. (C) Systemic lupus erythematosus
Laboratory studies show an erythrocyte sedimentation (D) Systemic sclerosis
rate of22 mm/h, a hematocrit level of 4l'X,, and a C-reactive
protein level of 0. 9 mg/dL (9.0 mg/L).
Hand radiographs show joint-space narrowing and Item 12
osteophytesofmultlple proximal and distal interphalangeal A 33-year-old woman is evaluated for concerns about the
joints, and similar changes are seen in both first carpometa- development of rheumatoid arthritis (RA). She has some
carpal joints; there are no erosions. mild joint discomfort in the hands at the end of the day. She
Which of the following is the most appropriate next step in is a software engineer and has been smoking ftve cigarettes
management? per day for the past 2 years. She takes no medications. Her
identical twin sister was recently diagnosed with seroposi-
(A) Anti-cyclic citrullinated peptide antibodies tive RA, and her maternal grandmother had RA.
(B) Antinuclear antibodies On physical examination, vital signs are normal. There
(C) Rheumatoid factor is no swelling or tenderness in the joints of the hands,
(D) Serum urate level wrists. elbows, knees, or feet.
(E) No further testing Which of the following is the most appropriate preventiVe
measu,re for this patient at risk for developing RA?

Item 11 (A) Avoid tood/drinks containing high-fructose com


syrup
A 47-year-old woman is evaluated for an onset over 2 to
3 weeks or low-grade. intermittent fever; blanching of the (B) Begin hydroxychloroqufne
second and third right fingertips In response to cold expo- (C) Begin problotics
sure; cracking and peeling of the skin on the sides of the (D) Smoking cessation

87
Self-Assessment Test

CJ Item 13 (C) Irritable bowel syndrome


(D) Small intestinal bacterial overgrowth
A 26-year-old man is hospitalized for a !-week history of
dry cough and progressive shortness of breath. He has a
2-year history of systemic lupus erythematosus (SLE); dis-
ease manifestations have been pleuropericarditis, polyar- Item 15
thritis. leukopenia. and nephritis. He currently reports no A SO-year-old man is evaluated for a left lower extremity
fever. sputum production. or hemoptysis, as well as no other ulcer. He has a 15-year history of worsening arthritis for
SLE manifestations. His disease has been djfficult to control which he has never been evaluated. He takes ibuprofen as
and has needed multiple tapering doses of prednisone. He needed.
currently takes 15 mg/d of prednisone: other medications On physical examination, vital signs are normal. The
are hydroxychloroquine and azathioprine. spleen tip is palpable. There is swelling of multiple small
On physical examination, temperature is 36.4 oc joints at the hands. knees, and metatarsophalangeal (MTP)
(97.6 °F). blood pressure is 150/100 mm Hg, pulse rate is joints. There is ulnar deviation andsubluxationofthe meta-
98/mln, respiration rate is 30/min. and oxygen saturation carpophalangeal joints. Subcutaneous nodules are present
is 84% breathing ambient air. Dirfuse crackles are heard on at the elbows bilaterally. There is a 2- x 2-cm shallow ulcer
auscultation of the chest. Diffuse tenderness and swelling of at the medial left lower extremity just above the ankle.
multiple small joints of the hands are present. The remaln -
Laboratory studies:
derofthe physical examination is normal.
Hematocrit 33'.t
laboratory studies: Leukocyte count 2100/)J.L (2.1 x 109 /L), with 900
Erythrocyte sedimentation 68mm/h neutrophils
rate Platelet count 276,000/J.IL (276 x 109 /L)
Hematocrit 42'Yo Urinalysis Normal
Leukocyte count 5500/)J.L (5.5 x 109/L) with
normal differential Which of the following is the most likely diagnosis?
Complements (C3 and C4) Low (A) AA amyloidosis
C-reactive protein Normal (B) Felty syndrome
Creatinine 0.6 mg/dL(S3)J.moi/L)
Procalcltonin Normal (C) Sarcoidosis
(D) Systemic lupus erythematosus
Chest radiograph and cr scan of the chest show diffuse
infiltrates in both lungs with ground glass opacities In multiple
Jobes. Bronchoalveolar lavage shows only increased leuko-
cytes with lymphocytic predominance. Cultures are pending. Item 16 CJ
A 49-year-old man is evaluated a 3-monrh history of right
Which of the following is the most likely diagnosis? knee swelling without significant pain. He does a lot of
(A) Acute lupus pneumonitis physical activity as a park ranger in Michigan but does not
(B) Acute pulmonary hypertension recall trauma. He likes to walk for exercise and does not
have significant pain when he walks. He has only minimal
(C) Community-acquired pneumonia
discomfort in the knee and stiffness when he squats. He
(D) Diffuse alveolar hemorrhage reports no swelling or pain in other joints and no fever. rash,
(E) Shrinking lung syndrome or other symptoms. He takes no medications.
On physical examination, vital signs are normal.
Examination of the right knee shows a large effusion with-
out erythema. but with minimal warmth and tenderness;
Item 14 full range of active motion is noted. lhe remainder of the
A 45-year-old woman is evaluated for a 4-month history musculoskeletal and physical examination is normal.
of diarrhea and a 4.5-kg (10-lb) weight loss. She reports Knee radiographs confirm a large joint effusion In the
explosive episodes of loose stools that follow most meals. right knee: no other abnormalities are seen.
1here is a feeling ofbloating and p3ln with the episodes. She
has a 10-year history of diffuse cutaneous systemic scle- Which of the following tests is most likely to confirm the
rosis (DcSSc) complicated by Raynaud phenomenon with diagnosis?
occasional digital ulceration and gastroesophageal reflux (A) HLA-B27 haplotype testing
disease. Her only medication is omeprazole. (B) MRI of the right knee
On physical examination, vital signs are normal. BMJ
is 21. Skin changes associated with DcSSc involve the (C) Serologic testing for Borrelia burgdorferi
arms, forearms, and hands. The abdominal examination (D) Synovial fluid analysis for crystals
reveals active and loud bowel sounds but is otherwise
unremarkable.
Which of the following is the most likely diagnosis? Item 1 7 CJ
A 55-year-old man is evaluated in the emergency depart-
(A) Carcinoid syndrome ment for a !- month history of low back pain and lower
(B) Chronic mesenteric Ischemia extremity swelling during the past 2 weeks. History is

88
Self-Assessment Test

Cl levothyroxine.
significant
5
a goiter that was surgically removed
for
years ago; pathology revealed extensive fibrosis. He takes
and numbness and weakness of the right foot; left testicular
pain for 1 week; and a painful rash on his legs for 2 days. He
CONT. was diagnosed 2 months ago with hypertension, for which
On physical examination. temperature is normal. he takes hydrochlorothiazide.
blood pressure is 165/90 mm Hg, pulse rate is 88/min. and On physical examination, temperature is 37.2 oc
respiration rate is 18/mln. A surgical scar over the anterior (99.0 °F'l, and blood pressure is 165/90 mm Hg. The left tes-
neck is noted. lhere is 2+ lower extremity edema to the mid ticle is tender. Small necrotic ulcers are noted on the legs.
ca If bilaterally. The rema lnder of the examination is normal. Numbness of the right lateral ankle and calfls noted. as well
Laboratory studies show a hematocrit level of 34'~. a as weakness of right foot plantar flexion.
serum creatinine level of 2.1 mg/dL (185.6 j.lmol/L), and a
normal urinalysis. Laboratory studies:
Abdominal ultrasound shows btlateral hydronephro- Erythrocyte sedimentation rate lOOmm/h
Hemoglobin 10 g/dL (!00 g/L)
sis. Noncontrast CT of the abdomen and pelvis shows a
Leukocyte count 13,000/J.!L (13x 10'1/L)
soft-tissue mass surrounding the infrarenal aorta without
Platelet count 430,000/J.lL (430 x 109/L)
any significant lymphadenopathy: the ureters are encased
within the mass. Creatinine 1.7 mg/dL (150.3 J.l!TlOl/L)
ANCA Negative
Wh ich of the following is the most Ukely diAgnosis? Urinalysis Normal
(A) Germ cell tumor Renal angiogram shows microaneurysms of the renal
(B)lgG4-associated retroperitoneal fibrosis arteries. A deep skin biopsy (deep dermis and subcutis)
(C) Lymphoma
shows panmural inflammation with necrosis of a medium-
sized artery.
(D) Malignant peritoneal mesothelioma
Which ofthe following is the most llkeJy diagnosis?

Item 18 (A) Giant cell arteritis


(B) lgA vascull tis
A 42-year-old woman is evaluated during a follow-up visit
for Raynaud phenomenon of 6 months' duration. She also (C) Microscopic polyangiitis
has gastroesophageal reflux disease. Her only medication is (D) Polyarteritis nodosa
ranitidine.
On physical examination, vital signs are normal. There
is no sclerodactyly or digital pitting at the fingertips. The Item 20
remainder of the physical examination is normal. An 85-year-old woman is evaluated for left knee pain of
Laboratory studies are significant for positive antinu- moderate intensity that occurs with ambulation and at
clear antibodies (titer: 1:640) in a centromere pattern. with night. Acetaminophen provides no relief. History is signifi-
a strongly positive anti centromere Bant! body level. cant for coronary artery disease. hypertension, and stage 3
The nail fold capillaries are shown. chronic kidney disease. Medications are aspirin, lisinopril,
metoprolol. and vitamin D.
On physical examination, blood pressure is 152/
88 mm Hg; other vital signs are normal. The left knee
demonstrates crepitus and decreased passive range of
motion; a small effusion is present, and there Is no warmth
or tenderness.
Joint aspiration of the left knee shows a leukocyte
count ofllOO/IJL (1.1 x 109 /L).
Whlch of the following is the most appropriate treatment
for long-term symptom control?
(A) Intra-articular glucocorticoid injections
(B) Low-dose prednisone
Which of the following is the most likely diagnosis? (C) Naproxen
(D) Oxycodone
(A) Dermatomyositis (E) Topical diclofenac
(B) Limited cutaneous systemic sclerosis
(C) Mixed connective tissue disease
(D) Systemic lupus erythematosus Item 21
A 79-year-old man Is evaluated for a 2-month history of
progressive malaise and weakness, aching bilateral shoul-
Item 19 ders and hips, and stiffness for 2 hours in the morning and
A 65-year-old man is evaluated for a 1-month history of after immobility. He recently noted aching in his }aw when
progressive malaise, myalgia. a 3.6-kg (8.0...Ib) weight Joss. chewing. He also reports new left-sided headaches. Last

89
Self-Assessment Test

week he had an episode of diplopia lasting 1 minute. He has Laboratory studies show positive rheumatoid factor,
hypertension, for which he takes hydrochlorothiazide. high-titer antinuclear antibodies, and high-titer anti- Ro/
On physical examination, vital signs are normal. Ten- SSA an tibodies.
derness and :;light swelling over the left temple are pres- A Schirmer test for ocular wetting is diminished at
ent. Painful and limited range of motion of both hips and 3 mm. Chest radiograph is normal. Radiographs of the
shoulders is noted. 1he remainder of the examination is hands show no erosions.
unremarkable.
Laboratory studies show an erythrocyte sedimentation Which of the following is the most appropriate treatment
rate of85 mm/h. at this time?
(A) Artificial tears and sugar-free candies
Which of the foUowing is the most appropriate initial
management? (B) Methotrexate
(C) Pilocarpine
(A) Cfofthe head
(0) Rituximab
(B) low-dose aspirin
(E) Topical oph thai m ic NSAID drops
(C) Methotrexate
(D) Prednisone
(E) Temporal artery biopsy Item 24
A 45-year-old woman is evaluated for a 2-day history of
deep boring pain in the right eye. She also describes eye
Item 22 redness and photophobia but no recent trauma to the eye.
A 67-year-old man is evaluated for a 1-year history of weak- She has a 10-year history of rheumatoid arthritis, treated
ness, with increased tripping on curbs and dHflcul ty with with etnnercept.
handwriting, which is not as neat as in the past. He reports On physical examination, vital signs are normal.
no muscle cramping. He takes no medications. Diffuse right eye redness is noted, and there is pain on
On physical examination, vital signs are normal. There extra-ocular movement testing. Gentle pressure over the
is no rash. There is symmetric weakness of the forearm and eye with the lid closed results in pain. There is no sclero-
thigh muscles. Reduced grip strength and reduced wrist malacia in either eye. There is diminished visual acuity of
and finger flexion are noted. Reflexes are normal. There is the right eye. Limited range of motion of the right wrist
atrophy of the muscles of the foream1s and interosseous is noted. There Is no swelling of the joints of the upper or
muscles of the right hand greater rhan rhe left. There are no lower extremities.
fasciculations.
Laboratory studies show a serum creatine ki nase Which of the following Is the most likely diagnosis?
level of 1150 U/L and a normal thyroid-stimulating hor- (A) Conjunctivitis
mone level; antinuclear and anti- Jo-1 antibody testing is (B) Episcleritis
negative.
Electromyogram and nerve conduction studies show (C) Scleritis
myopathic changes in the proximal and distal muscles of (D) Subconjunctival hemorrhage
the extremities, as well as some neurogenic changes.
Which of the follOWing is the most likely diagnosis?
Item 25 C]
(A) Amyotrophic lateral sclerosis A 26-year-old woman is evaluated in the emergency
(B) Inclusion body myositis department for an acute onset of fatigue, chiUs. and joint
(C) Mitocl10ndrial myopathy pain in the fingers of the left hand, left wrist, and right
(D) Polymyositis ankle during the past 3 days. There is no travel history. She
is currently ta king Ibuprofen for her sym ptoms; she also
takes an oral con traceptive piU.
Item 23 On physical examination. temperature is 39.0 •c
(102.2 °f). blood pressure Is 114/72 rnm Hg, and pulse rate
A 66-year-old woman is evaluated after developing flve is 106/mln. Fusiform enlargement of the second and fourth
dental caries over the past year. She cannot eat crackers digits ofthe left hand with pain on extension is noted. Teno-
without accompanying water. She has lost two teeth due to synovitis over the dorsum of the left wrist and ar the right
caries. She also reports scratchy and itchy eyes for 2 years ankle is present.
and Intermittent joint pain, particularly of the small hand A p ainless lesion on the palm is shown (see top of next
joints, for 1year. She takes ibuprofen as needed for the joint page).
pain, which provides relief.
On physical examination, vital signs are normal. Two Which ofthe following is the most Ukely dJagnosis?
molars are missing, and there Is no saiLvary pooling below (A) Chi kungunya infection
the tongue. Bilateral parotid and lacrin1al enlargement is
present. Mild tenderness without swelling of the second (B) Disseminated gonococcal Infection
through fourth metacarpophalangeal joints bilaterally is (C) Reactive arthritis
noted. (D) Sarcoidosis

90
Self-Assessment Test

On physical examination, temperature is 37.5 oc


(99.5 °F), blood pressure is normal but systolic pressure is
16 mm Hg less on the right arm than the left arm, pulse rate
is 90/ min, respircltion rate is 20/mln, and oxygen satura-
tion is 93% breathing ambient air. There is no rash. There
is no temporal tenderness A bruit Is heard over the right
supraclavicular fossa. The right raclial pulse is reduced. A
diastolic decrescendo murmur is heard in the upper right
sternal border. Blbasilar crackles are heard. Painful range of
motion is noted in the shoulders and hips. No joint swelling
is present.
Transthoracic echocardiogram shows aortic valve
regurgitation with normalleaJlets, dilated aortic root, and
dilated left ventricle.
Which of the following is the most Ukely diagnosis?
(A) Kawasaki disease
(B) Polyarteritis nodosa
(C) Subcranial giant cell arteritis
(D) Takayasu arteritis
ITEM 25

Item 28
CJ Item 26 A 43-year-old woman Is evaluated for a 3-day history of left
knee pain and swelling. Similar episodes have occurred in
A 32-year-old woman Is evaluated for a recent episode of
transient left monocular blindness. She noted dimness of either knee as well as the left w rist during the past 2 years.
vision in the left eye that came on suddenly and persisted She also reports a sense of generalized weakness, and has
for 15 minutes and then resolved completely. She had no experienced constipation and vague abdominal discom-
accompanying headache or other symptoms. She has an fort during t he past year. Because of her constipation, her
8-year history of systemic lupus erythematosus. which thyroid-stimulating hormone level was recently measured
initially manifested as photosensitivity, discoid rash, and and found to be normal. She takes no medications.
arthritis. She has responded well to treatment and has been On physical examination, vital signs are normal. The
doing well without active dlsease for the past year. History left knee Is warm with a moderate- to Large-sl:zed e.ffusion,
is significant for recurrent tlrst trimester pregnancy loss and decreased range of motion is noted. Proximal muscle
attributed to positive antlphosphollpid and anticardiolipin strengt h is normal.
antibodies. Medications are hydroxychloroqulne, predni- Radiographs show a thin white line a t the chond ral
sone, aspirin, and a daily multivitamin. surfaces of both knees and at the pubic symphysis.
On physical examination, vital signs are normal. Car- Join t asp iration of th e left knee shows a leukocyte count
diac rhythm Is normal. A 2/6 holosystollc murmur Is heard of35,000/~L (35 x 109/L), With 90% neutrophtls; polarizing
at the apex with radiation toward the ax!lla. Temporal and microscopy show s numerous positively birefringent rhom-
carotid artery pulsations are normal; there is no scalp ten- boid crystals within neutropltils.
derness or vascular bruits. The remainder of the physical Which of the following laboratory studies will most likely
examination, inclucling ophthalmologic examination, is identifY the cause of this patient's symptoms?
normal.
(A) Anti-cyclic cltrulll nated peptide antlbcxUes
Which of the following Is the most likely cause of her visual
(B) Serum calcium
symptom?
(C) Serum creatin e kinase
(A) Bacterial endocarditis (D) Serum urate
(8) Carotid artery stenosis
(C) Giant cell arteritis
(D) Ubman-Sacks endocardJtis Item 2 9 CJ
A 66-year-old man Is hospitalized for progressive dyspnea

CJ Item 2 7 for the past 10 days. He was diagnosed with granulomatosis


with polyangiltis 5 years ago; at that time, cyclophosphamide
A 73-year-old woman is hospitaliZed for symptoms of heart was Initiated, with resolutlon of all symptoms. He was sub-
failure, with progression occurring over the past 2 weeks. sequently switched to maintenance azathioprine therapy.
She also reports a 6 week history of arm aching that is Today he was started on Intravenous methylprednisolone.
worse with lifting and reaching, hip aching, morning stiff- On physical examination, temperature is 37.8 •c
ness, fever, and malaise. She reports no headache or jaw (100.0 °F), blood pressure Is 140/85 mm Hg, pulse rate Is
claudJcatlon. 100/ min, respiration rate is 25/mln, and oxygen saturation

91
Self-Assessment Test

is 90'Jf. on of oxygen by nasal cannula. Bilater-.lf crackles


2L prednisone was weaned from 15 to 10 mg/d 2 months ago,
Cl are
CONT.
heard in the tower lung fields. then to 8 mg/d (current dose) 1 month ago. She remained
asymptomatic until 2 weeks ago. She says that her current
Laboratory studJes:
symptoms are just as bad as when she was first diagnosed.
Erythrocyte 90mm/h
On physical examination. vital signs are normal; blood
sedimentation rate
12 g/ dL (120 g/ L)
pressure is identical in both arms. 1here Is no temporal
Hemoglobin
tenderness or induration. Painful range of motion of both
Leukocyte count with Nonnal
differential shoulders and hips is noted.
Platelet count 450,000 (450 X 109 / L) Which of the following is the most appropriate management?
ANCA Positive with a cytoplasmic
pattern and a titer of 1:160: (A) Prednisone, IO mg/d
positive proteinase 3 antibodies (B) Prednisone, 30 mg/d
Urinalysis Nonnal (C) Prednisone, 60 mg/d
Chest radiograph shows hazy opacification and a few (D) Prednisone, 20 mg/d, and methotrexate
nodules in the mid and lower lung zones. Sputum and bron-
choalveolar lavage. Gram stain, and cultures are negative.
Blood cultures are negative. I t em 32 I::::J
In addJtion to discontinuing azathioprine, which of the A 50-year-old woman Is hospitalized for right ann weak-
following is the most appropriate treatment? ness and altered mental status. Over the past 4 months, she
has had gradual onset of headaches that have progressively
(A) Cyclophosphamide worsened, and her family reports that the patient has had
(B) Etanercept cognitive problems over the past 1 to 2 weeks. Today she
(C) Methotrexate developed right arm weakness. She has no other pertinent
history and takes no medications.
(D) RitUXimab
On physical examination. the patient Is not oriented w
place or date. Vital signs are nonnal. The patient has 4+/5
strength of the right upper extremity and achieves 16 of 30
Item 30 points on the Mini- Mental State Examination: the remain-
A 47-year-old woman is evaluated for a.6-month history der oft he neurologic examination is normal.
of disequilibrium. She often feels "dizzy" and "ofl'- bal- Laboratory studies show a nonnal erythrocyte sedi-
ance," which has led to several falls. She does not expe- mentation rate, complete blood count with differential, and
rience vertigo. She has a 10-year history offibromyalgia, comprehensive metabol!c panel; a negative ANCA; and a
and has experienced a 60% improvement in pain and negative urinalysis.
function with graded aerobic exercise. duloxetine, and Cerebrospinal fluid analysis shows a leukocyte count
pregabalin. of 12/~L (12 x t04 fl; all lymphocytes) and a protein level of
On a focused physical examination, vital signs are nor- 70 mg/d.L (700 mg/ L); Gram stain and cultures are negative.
mal. Neurologic examination, including strength testing, MRI of the brain shows diffuse nonspecific white mat-
sensory examination (light touch, proprioception), tandem ter changes and a low attenuation area In the right frontal
walking. and Romberg testing, is normal. lobe. Cerebral angiogram demonstrates multiple areas of
Laboratory studies show normal serum folate, thyroid- vessel dilation and stenosis. Brain biopsy shows granuloma-
stimulating hormone, and vitamin B12 levels; hemoglobin tous vessel inflammation.
A1c level Is 5.0%.
In addition to high-dose glucocorticoids, which of the
Which of the following is the most appropriate next step In following is the most appropriate management?
management?
(A) Adalimurnab
(A) Discontinue duloxetlne (B) Cyclophosphamide
(B) Discontinue pregabalin (C) Methotrexate
(C) Measure methylmalonic acid and homocysteine levels (D) Rituximab
(D) Order an MRI of the bra!n
(E) Schedule vestibular rehabilitation
Item 33
A 27-year-otd woman is evaluated for a sudden onset ofjoint
Item 31 pain and significant stiffness in her fingers. wrists, knees,
A 75-ye.ar-old woman is evaluated for a 2-week history of and ankles for the past 3 days. She had a brief fever with
gradually increasing pain in both shouldeTS and hips; the muscle aches a few days ago and a faint, pink rash on her
pain radiates down both arms to the elbows and down both arms and legs at the same time. She reports no other symp-
hamstrings to the knees. She reports no headache, jaw clau- toms. She takes no medications. She works at a daycare.
dication. or vision changes. She was diagnosed with poly- On physical examination, temperature Is 37.2 oc
myalgia rheumatica 3 months ago. She started prednisone, (99.0 °F); other vital signs are normal. There are no rashes.
15 mg/d, with immediate and complete rellef of symptoms; hair loss, or oral ulcers. There is mild warmth and tenderness

92
Self-Assessment Test

of the first through flfth proximal interphalangeal and


metacarpophalangeal joints bilaterally as well as the wrists.
Item 36 C]
A 35-year-old woman is hospitalized for left-sided pleuritic
Warmth and tenderness of the knees and ankles are also chest pain and dyspnea that began 1 day ago. Four weeks ago,
noted, with pain on range of motion. she began to experience fever once or twice a day. pharyngi-
Which of the following will most likely conflrm the diag- tis. intermittent rash on the trunk and proximal extremities
nosis? that occurs with the fever. severe joint pain, and myalgia. She
gave birth 10 weeks ago to a healthy female infant.
(A) Antinuclear antlbooies On physical examination. temperature is 39.0 •c
(B) HIV testing (l 02.2 •r). pulse rate is 90/mln. and respiration rate is 22/
(C) Parvovlrus Bl9 testing min. Enlarged cervical lymph nodes, hepatomegaly. and
(D) Rheumatoid factor
splenomegaly are present. A pleural friction rub is heard.
A pink maculopapular rash is present on the trunk. Ten-
derness and swelling of the wrists, knees, and ankles are
Item 34 noted.
A 36-year-old woman is evaluated for a 2-month history of Laboratoey studies:
morning stiffness in the hands. WJists, knees, and feet lasting Erythrocyte sedimentation 90mm/h
90 minutes. She recently noted difficulty making a fist in the rate
morning and feels like she is walking on pebbles when she Hemoglobin 10 g/dL (100 gi L)
flrst gets out of bed. She feels better after a hot shower and Leukocyte count 20,000/JiL (20 x 109 /L). 90%
activity. Her only medication is ibuprofen, which is helpful. neutrophils
On physical examination, vital signs are normal. There Is Alanine aminotransferase 80U/ L
tenderness and swelling of the second, third, and fifth meta- Aspartate aminotransferase 70 U/L
carpophalangeal joints bilaterally, the second through fourth Ferritin 6000 ng/ mL (6000 f.lg/L)
proximal Interphalangeal joints of both hands, the right wlist, Urinalysis Normal
the left knee, and the second through fltlh metatar.;ophalangeal Chest radiograph shows a small left-sided pleural
joints bilaterally. The remainder of the examination is normal. effusion.
Which of the following is the most appropriate diagnostic Which of the foUowing is the most Ukely diagnosis?
test to perform next?
(A} Adult-onset Still disease
(A) Anti-cyclic citrullinated peptide antibodies
(8) Cryoglobulinemic vasculitis
(B) HLA-B27
(C) Lymphoma
(C) Parvovlrus lgG antibodies
(D) Microscopic polyanglitis
(D) Serum urate
(E) Systemic lupus erythematosus
(E) Thyroid-stimulating hormone

Item 35 Item 37
A 73-year-old woman Is evaluated for a 10-year history A 49-year-old woman is evaluated for recently worsening
of osteoarthritis affecting multiple joints over the years. joint symptoms. She has a 13-year history of Crohn disease
including the distal joints of her fingers, bases of the characterized by four to six stools daily and mild crampy
thumbs, knees, and cervical and lower lumbar spines. She abdominal pain. She also has a 1-year history of arthri-
has chronic dally pain In at least one joint. She has tried tis. She currently has pain in the left knee, right ankle,
nonpharmacologic measures, and she had minimal bene- and two joints of the right foot; diffuse swelling involving
fit from intra-articular glucocorticoid and hyaluronic acid the left third toe; and 30 minutes of morning stiffness.
injections to her knees. She was recently diagnosed with She has been treated with various NSAIDs, which seem
peptic ulcer disease. History is also significant for coronary to worsen her bowel disease. She has tried, in succession,
artery disease, diabetes mellitus, and hypertension. Medi- azathiopline, mesal amine, and methotrexate without nota-
cations are enalaprll, carvedllol, metformin, atorvastatin, ble Improvement In her symptoms. She currently Is taking
pantoprazole, and low-dose aspirin. methotrexate.
On physical examination. vital signs are normal. On physical examination, vital signs are normal. The
Heberden nodes and squaring ofthe bUateralcarpometacarpal left knee has a small effusion. Dactylitis of the left third toe
joints are present. Crepitus and limited extension of the cervi- is present. The right ankle and the right second and third
cal spine are noted. Bilateral knee varus deformity and bony metatarsophalangeal joints are tender to palpation.
enlargement are present. w1th crepitus on range of motion.
Which of the following Is the most appropriate long-term
Which of the following Is the most appropriate treatment? treatment?
(A) Duloxetlne (A) Adallmumab
(B) Gabapentin (B) Intra-articular glucocorticoid injections
(C) Ibuprofen (C) Prednisone
(D) Topical capsaicin (D) Rituximab

93
Self-Assessment Test

I tem 38
A 64-year-old man is evaluated for a 9- month history of a
swollen and painful right knee without trauma or injury.
The knee has been drained twice; synovial fluid leukocyte
counts were between 15,000/).IL to 20,000/f.ll. (15-20 x
109 / L), predominantly lymphocytes. All stains and cultures
have been negative. He also reports a low-grade fever in
the evening. He has no history of skin rash or tick bites.
He had a positive tuberculin skin test 20 years ago and was
treated with isoniazid for an unknown duration. He takes
no medications.
On physical examlnatlon, vital signs are normal. The
right knee is tender and has a moderate effusion, with
reduced passive range of motion. The remainder of the
physical examination is normal.
Laboratory studies show an erythrocyte sedimentation
rate of 48 mm/h and negative Lyme disease serologies.
Chest radiograph shows three small calcified granu-
lomas in the lefl upper lobe. Plain radiograph of the right
knee shows joint space narrowing, osteopenia, and small
sinus tracts.
ITEM 39
Which of the following is the most appropriate diagnostic
test to perform next?
(A) Angiotensin -converting enzyme level Item 40 C]
(B) CT of the chest A 21-yea r-old woman is evaluated for a 2-week history of
(C) HLA-B27 testing
worsening rash and arthritis as well as intermittent low-
grade fever. She has a 5-year history of systemic lupus ery-
(D) Synovial biopsy thematosus (SLE). She has been doing well without active
clisease for the past 3 years and has been compliant with
hydroxychloroquine.
Item 39 On physical examination. temperature Is 37.8 oc
A 59-year-old woman is evaluated for a 5-year history of (100.0 °F), and blood pressure Is 150/86 mm Hg; other vital
bilateral hand pain and stiffness ofseveral hand joints, with signs are normal. A malar rash is present. DiiTuse tenderness
enlargement and <:rool<edness developing at some of the and swelling of multiple small joints of the hands are pres-
hand joints over the past few years. The stifihess is pres- ent. New dependent edema is present.
ent tn the morning and lasts about 30 minutes. One area Laboratory studies show low C3 and C4 complement
of enlargemen t, at the right fourth distal interphalangeal levels, a serum creatinine level of1.8 mg/dL (159.11J.moi/L),
(DIP) joint, has occasionally become abruptly swollen, red, a urine protein-creatinine ratio of 3200 mg/g, and active
and tender, and is currently Inflamed. Family history Is urine sediment on microscopic examination..
significant tor deformed and painful, arthritic hand joints Which of the following laboratory studies should be done
in her mother. next?
On physical examination , vital signs are no rmal.
There are no rashes or nail changes. Grossly deformed (A) Anti-double-stranded DNA antibodies
digits on both hands with bony enlarge ment and devia - (B) Antinuclear antibodies
tion at the right second, left second and third, and bilat- (C) Anti-Ro/SSA and anti-La /SSB antibodies
eral fourth DIP joints are noted. A tender, erythematous (D) Anti-Smith antibodies
bony nodule is present over the rlghr fourth DIP joint.
Significan t squaring of the right carpometacarpal joint is (E) Anti- Ul-ribonudeoprotein antibodies
also present. The remainder of the musculoske letal exam -
ination is nonnal.
Laboratory studies show normal erythrocyte sedimen- Item 41 CJ
tation rate, C-reactlve protein, serum urate, rheumatoid A 64 -year-old woman Is evaluated for worsening leg weak-
factor. and anti-cyclic citrullinated peptide antibodies. ness. She was evaluated 2 weeks ago for a 3-week history
A hand radiograph is show n (see top of next column). of lower extremity weakness; laboratory studies showed a
serum creatine kinase level of23,000 U/L. History is notable
Which of tbe following Js the most likely diagnosis? for hypercholesterolemia treated with lovastatin, which
(A) Erosive osteoarthritis was discontinued at the initial visit.
On p hysical exam ination, vital signs are normaL
(B) Gout
There is no rash. Examination shows 3/5 strength of the
(C) Psoriatic arthritis quadrice ps and hamstrings (4/S strength 2 weeks ago);
(D) Rheumatoid arthritis 4/5 strength in the deltoids. biceps, and triceps; and S/5

94
Self-Assessment Test

CJ strength of the neck flexors. neck extensors. and distal


upper and lower extremities. lhe remainder of the exam-
!nation is normal.
CONT.
Laboratory studies:
Hemoglobin 9.3 g/dL (93 g/ L)
5600/j.IL (5.6 X 109 /L)
Leukocyte count
Current laboratory studies show an erythrocyte sed- Mean corpuscular volume 111fL
imentation rate of 25 mmlh and a serum creatine kinase Platelet count 330,000/j.IL (330 X 109/ L)
level of20,876 U/L.
Which of the following is the most likely cause of the
In addition to a muscle biopsy, which of the following is anemia?
most likely to establish the diagnosis?
(A) Inflammation
(A) Anti-cyclic citrull!nated peptide antibodies (B) Iron deficiency
(8) Anti-cytosollc 5'- nucleotldase lA antibodies (C) Methotrexate
(C) Anti-hlstidyl tRNA synthetase antibodies (D) Tocllir.umab
CD) Anti-I-IMG Co-A reductase antibodies

Item 42
Item 4 4
A 55-year-old man is evaluated for a swollen and painful
CJ
A 32-year-old woman is evaluated for a 2-year history of right knee of 2 days' duration. One week ago, he injured
dry eyes and dry mouth. She wakes up in the morning with both knees when he fell on the concrete pavement. He pre·
a feeling like something is in her eyes. She cannot eat dry viously had two episodes of acute gout affecting the right
foods without consuming large amounts of water. She has first toe. He takes no medications.
noticed a change in the shape of her face; it has gotten more On physical examination, temperature is 38.1 oc
round. She previously saw an ophthalmologist who found (100.6 °F): other vital signs are normal. lhere are super-
abnormal ocular surtace staining consistent with dry eyes. ficial scrapes on the front of both knees. The right knee is
She uses artificial tears as needed. swollen and tender and has a moderate eff'usion with red-
On physical examination, vital signs are normal. There ness: restriction ofboth passive and active range of motion
is no pooled saliva under the tongue. Bilateral parotid and due to pain is noted. lhe remainder of the examination is
lacrimal enlargement is present. lhe remainder of the normal.
examination is normal. Plain radiograph of the right knee shows mild joint-
Antinuclear antibodies. anti· Ro/SSA antibodies, space narrowing. an effusion. and soft-tissue swelling.
and rheumatoid factor are negative. Screening for HIV Which of the following is most likely to establish the
and hepatitis B and C is negative. Serum lgG levels are diagnosis?
normal.
Chest radiograph is normal. (A) Aspiration of the right knee
(B) Blood cultures
Which of the following is the most appropriate diagnostic
test to perform next? (C) Erythrocyte sedimentation rate
(D) MRI of the right knee
(A) CT of the chest
(B) Lip biopsy
(C) Parotid biopsy Item 45
(D) Schirmer test A 65-year-old woman is evaluated during a follow-up visit
(E) Sialography for giant cell arteritis. Prior to confirmation of the diagnosis
she was started on 60 mg of prednisone daily and low-<lose
aspirin. The diagnosis was subsequently con finned with a
temporal artery biopsy. She Is otherwise healthy and takes
Item 43
no additional medications.
A 67-year-old woman is evaluated fbr a 3-year history On physical examination, vital signs are normal. There
of severe rheumatoid arthritis. She had an inadequate is a healing Incision over the tight temporal artery.
response to methotrexate and low-dose prednisone. She Dual-energy x-ray absorptiometry (DEXA) was per-
responded well to the addition of lntliximab, but eventually formed 6 months ago. Based upon the results of the Fracture
the drug lost effect and she required a change in biologic Risk Assessment Tool, the patient was classified as medium
therapy. She has done well with toclllzumab and meth- risk (lO'X.-20%) for a major osteoporotic fracture in the next
otrexate over the past year. She notes several months of 10 years.
prominent fatigue. History is also significant for type 2 dia-
betes mellitus, hypertension, and hyperlipidemia. Current Which of the following Is the most appropriate next step In
medications are methotrexate, folic add, tocilizumab, basal management?
Insulin, llsinoprll, metoprolol, atorvastatln, ibuprofen, and (A) Begin alendronate
omeprazole.
(B) Begin terlparatide
On physical examination, vital signs are normal.
Joint examination reveals no swollen or tender joints. The (C) Repeat DEXA
remainder ofthe physical examination is normal. (D) No additional testing or therapy

95
Self-Assessment Test

Item 46 (C) Pes anserine bw-sitis


A 48-year-old woman Is evaluated for 3 months of progres- {D) Quadriceps tendonitis
sive exertional dyspnea. She was diagnosed with limited
cutaneous systemic sclerosis 4 years ago. She also has Ray-
naud phenomenon <l11d gastroesophageal reflux disease. Item 49
Medications are nifedipine and omeprazole. A 48-year-old woman is evaluated for a 2-month history of
On phystcal examination, vital signs are normal. increased dyspnea, wheezing, and nonproductive cough.
Oxygen saturation is 9S'X, at rest, breathing ambient air. She also reports intermittent pain and swelling in the wrists
Scattered telangiectasias are present on the face, nose, and and knees for the past 6 months. She was diagnosed with
hands. Sclerodactyly at the fingers and tightness of the skin scleritis of the left eye I month ago that improved witl1 glu-
at the necl< are noted. There is increased intensity of the pul- cocorticoid drops. She also had two episodes of right pinna
monic sound and a widened split S2 on cardiac examination. pain, redness, and swelling. Medications are prednisolone
Lungs are clear to auscultation. acetate ophthalmic and ibuprofen as needed for joint pain.
Electrocardiogram and chest radiograph are normal. On physical examination, vital signs are normal. There
is no rash. Sclerae are normal. Hearing Is normal. The tra-
Which of the following Is the most likely cause of her chea is tender. On lung auscultation, wheezing is heard in
symptom? both tung fields with no crackles or rubs. Swelling of the
(A) Interstitial lung disease wrists and knees Is present.
(B) Myocardial fibrosis The ear findings are shown.
(C) Pulmonary arterial hypertension
(D) Venous thromboembolic disease

Item 47
A 67-year-old man is evaluated tor as-year history of right
knee pain, with morning stiffness lasting 20 minutes.
On physical examination, vital signs are normal. Lim-
itation of flexion and extension of the right knee is noted,
with bony enlargement and tenderness to palpation along
the medial and lateral joint Unes. Crepitus is noted upon
range of motion of the right knee. Bony enlargement of a
few proximal interphalangeal and distal interphalangeal
joints on both hands is present.
Which of the following i.s the most appropriate initial imag-
ing study?
(A) cr
(B) MRI Laboratory studies show an erythrocyte sedimenta-
(C) Radiography tion rate of 60 mm/h and a nom1al complete blood count
(D) Ultrasonography with differential except for a hemoglobin level of 11 g/dL
(110 g/L).
Chest radiograph is normal. CT of the chest reveals
Item 48 bronchial thickening with strictures; there Is air trapping
distal to the strictures.
A so-year-old woman is evaluated for a 1-month history of
pain around the lower part of the inner left knee. The pain Which of the following Is the most likely diagnosis?
is aggravated by ascending stairs and rising from a seated
position. She has a history of knee osteoarthritis, for which (A) Cogan syndrome
she takes acetaminophen. Her new pain is different in loca- {B) Polyarteritis nodosa
tion a11d Intensity compared with her prior pain. She has no (C) Relapsing polychondritis
other pertinent personal or family history. (D) Systemic lupus erythematosus
On physical examination, vital signs are nonnal. Tender-
ness to palpation and slight swelling over the proximal medial
tibia about 6 ern below the anteromedlal joint margin of the Item 50
knee are noted. Examination of the knee shows no crepitus, A 28-year-old woman requests a referral to an obstetrician.
minimal pain along the medial joint line, and no pain on val- She has been attempting to conceive, and her menstrual
gus stress; anterior and posterior drawer signs are negative. period ts now 3 weeks late. Her home pregnancy test was
Which of the following ls the most Ukely diagnosis? positive. She has a 2-year history of seropositive rheuma-
toid arthritis. Her only medication is hydroxychloroqulne;
(A) Medial collateral ligament tear methotrexate was stopped 4 months ago in anticipation of
(B) Meniscal tear conception.

96
Self-Assessment Test

On physical examination, vital signs are normal. Swell- occiput, trapezius, elbows, and hips bilaterally.lhere is no
Ing of the second and third metacarpophalangeal joints of joint swelling.
the right hand Is noted.
Pregnancy test is positive. Which of the following is the most appropriate treatment?

Which of the following is the most appropriate treatment (A) Add meloxicam
at this time? (B) Add pregabalin
(C) Discontinue duloxetine; start gabapentin
(A) Add etanercept
(D) Discontinue duloxetine; start sertraline
(B) Add letlunomide
(C) Discontinue hydroxychloroquine
(D) No change In therapy Item 53
A 44-year-old woman Is evaluated forrecent-onsetdyspnea
on exertion and a nonproductive cough. She has a 1-year
C] ltem 51 history of diffuse cutaneous systemic sclerosis. She takes no
A 68-year-old man is evaluated in the emergency depart- medications.
ment for a 4-week history of severe lower left leg pain, On physical examination, vital signs are normal. Oxy-
swelling. and redness. The pain is localized to the shin and gen saturation Is 96% breathing ambient air. Skin thick-
ankle. He was previously diagnosed with cellulitis, but no ening is noted on the face, neck, anterior chest, upper
improvement was seen with sequential courses of cepha- extremities to the mid forearm bilaterally, and lower
lextn and clindamycin. History is also significant for gout extremities to the mid calf bilaterally. tieart examination
and atrial fibrillation. Medications are allopurinol. colchi- is normal. There are fine crackles at the bases of both lungs
cine, and apixaban. on auscultation.
On physical examination. vital signs are normal. The Anti- Scl-70 antibodies are strongly positive.
left lower shin is erythematous, warm. and tender to pal-
pation; the erythema is not sharply demarcated. The left Pulmonary function tests:
ankle is swollen. warm. and tender. There is no ulceration or Today 6MonthsAgo
skin breakdown over lower legs or feet. Solid masses at the f.VC 85% of predicted 96% of predicted
extensor surfaces of both elbows are present. DJ.CO 65% of predicted 75% of predicted
Laboratory studies show an erythrocyte sedimenta- tiigh- resolution cr scan of the chest s hows lower lobe
tion rate of90 mm/h and a serum urate level of 6.8 mg/dL ground -glass opacities in a pattern consistent with nonspe-
(0.40 mmoi/L). Blood cultures performed l week ago were cific interstitial pneumonitis.
negative.
Plain radiographs of the left shin and ankle show no Which of the following is the most appropriate treatment?
changes s uggestive of osteomyelitis. (A) Cyclophosphamide
Aspiration of the left ankle yields no fluid .
(B) High-dose glucocorticoids
Which of the following is the most appropriate next step in (C) Methotrexate
management? (D) Mycophenolate mofetll
(A) Indomethacin
(B) Intravenous clindamycin
Item 54
(C) MRI of the lower leg
(D) Prednisone A 30-year-old woman Is evaluated for a 3-month history
of pain and swelling of the proximal interphalangeal (PIP)
(E) Surgical debridement joints. She has difficulty performing tasks such as gripping
or typing. Naproxen provides minimal relief.
On physical examination, vital signs are normal. There
Item 52 are no rashes or oral ulcers. SWelling and tenderness oft he
A 47-year-old woman is evaluated during a follow-up visit PIP joints bilaterally are noted. Pain occurs with flexion of
for a 2-year history of symptoms diagnosed 4 months ago as the ftngers.1here is blanching coloration of the fingers and
fibromyalgla. She reports ongoing widespread pain, fatigue, cyanosis at the fingertips. Nailfold examination shows a
and difllcultyconcentrating. She does not havettouble falling few dilated capillary loops. There are no cardiopulmonary
asleep, but her sleep is nonrestorative. She was seen3 months friction rubs. The remainder ofthe examination Is normal.
ago, and was given duloxetine and instructed to exercise. The Laboratory studies performed 1 month ago showed
medication produced a modest benefit, but she continues to the following: a positive antinuclear antibody titer of 1:320
have difficulty working as a housekeeper. She can only exer- with a speckled pattern; negative anti-double-stranded
cise to a limited extent without experiencing disabling pain DNA antibodies, anti-Smith antibodies, anti- Ul- ribonuc-
for the next several days. On a 9-point pain scale, her pain leoprotein antibodies, anli-Ro/SSA antibodies, anti-La/SSB
was formerly an 8; after initiation of duloxettne, It decreased antibodies, anti-cyclic citrullinated peptide antibodies, and
to as. rheumatoid factor; and normal complete blood count with
On physical examination, vital signs are normal. differential, serum creatinine, and urinalysis.
1here is tenderness between the shoulder blades and at the Radiographs of the hands are normal.

97
Self-Assessment Test

Which of the following is the most appropriate management? Which of the following is the most likely diagnosis?
(A) Add hydroxychloroquine (A) Acute calcium pyrophosphate crystal arthritis
(B) Discontinue n aproxen; begin ibuprofen (B) Gout
(C) Obtain plain radiography of the feet {C) Infectious arthritis
(D) Repeat anti-double-stranded DNA antibodies (D) Palindromic rheumatism
(E) Repeat antinuclear antibodies

Item 55 I t em 57 CJ
A 78-year-old woman is evaluated for knee pain that occurs A 48-year-old man is evaluated for a 3-week history ofslowly
when walking, ascending and descending stairs, and getting progressive dyspnea on exertion. left-sided chest pain, and
up from a seated positlon. She was diagnosed with bilateral fever. He has a 20-year history of anti-cyclic citrullinated
knee osteoarthritis 10 years ago. She has tried acetaminophen, peptide antibody- positive rheumatoid arthritis. Prior to
up to 3000 mg/ d in divided doses, Without significant relief. starting a tumor necrosis factor a inhibitor. his tuberculin
PreVIous examinations have documeoted bony hyper- skin test was nonreactive. Medications are methotrexate,
trophy and crepitus on range of motion of the knees with etanercept, and folic acid. Despite these medications, he has
weakness of the quadriceps. morning stiffness for 1 hour and has swelling of the joints in
She is scheduled to begin physical therapy. Topical his hands. wrists. and feet.
d!clofenac is prescribed. On physical examLnation, temperature is 37.8 oc
(100 °F), blood pressure is 148/84 mm Hg, pulse rate is
The patient should be warned about which of the foJiowing
100/ min, and respiration rate Is 24/rnin. Breath sounds are
possible side effects?
absent at the right base, with dullness on percussion. Joint
(A) Eye and mucous membra ne Irritation changes typical of rheumatoid arthritis are present, along
(B) Localized lipoatrophy with swelling and tenderness of eight joints. The remainder
of the examination is normal.
(C) Locall?.ed skin hypopigmentat!on
Chest radiograph shows a moderate right-sided pleural
(D) Localized skin rash effusion.
Pleural fluid analysis following a thoracentesis shows
Item 56 a leukocyte count of3SOOIJ.!L (3.5 x 109 / L) with 4% neutro-
A 78-year-old woman Is evaluated for a 2-week history of phils, 87% lymphocytes, and 9% monocytes; a pleural fluid
pain and swelling in the left wrist. In the past year, she expe- glucose level of 6.0 mg/dL (0.3 mmol / L); a pH of 7 .2; and
rienced two similar episodes, one in the left knee and one in a pleural fluid lactate dehydrogenase level of 900 U/ L. A
the left wrist, each resolVIng after 3 weel<s. pleural fluid adenosine deaminase measurement is low, at
On physical examination, th e patient appears well. 30 LJ/ L.
Vital signs are normal. 1he left wrist is swollen, warm. and
Which of the following is the most likely cause of the
tender, w ith decreased range of motion.
pleural eO'uslon?
Laboratory studies show an erythrocyte sed imenta-
tion rate of 53 mm/h and a serum urate level of 3.8 mg/dL (A) Malignancy
(0.22 mmol/L). (B) Para pneumonic effusion
A p lain radiograph of the left wrist Is shown. (C) Rheumatoid pleuritis
(D) Tuberculosis

Item 5 8 CJ
A 36-year-old woman is hospitalized for acute kidney injury
and hypertension. She has an 18-month history of diffuse
cutaneous systemic sclerosis. She was well and taking no
medications prior to hospitalization.
On physical examination, temperature is 36.6 •c
(97.8 °F), blood pressure is 240/130 mm Hg, pulse rate is
100/mln, and respiration rate Is l8/min. Cardiac exam-
ination reveals a prominent S4 • Pulmonary auscultation
reveals bibasilar crackles. Cutaneous examination reveals
sclerodactyly of both hands as weU as skin induration of the
forearms and anterior chest.
Laboratory studies:
Hemoglobin 8.0 g/dl (80 g/L)
Platelet count 90,000/jJL (90 x IOq/L)
Creatinine 4.9 mg/dL (433.1 ).Ulloi/L)
Urinalysis 2+ protein; a few hyaline casts

98
Self-Assessment Tes t

Peripheral blood smear shows schistocytes and and both wrists. 1here are no periarticular erosions in the
Cl decreased
CONT.
platelets. wrists or hands.
Which of the following Is the most likely diagnosis? Which of the following Is the most appropriate treatment?
(A) Disseminated intravascular coagulation (A) Adalimumab
(B) Hemolytic uremic syndrome (B) Allopurinol
(C) Scleroderma renal crisis {C) Low-dose prednisone
(D) Thrombotic thrombocytopenic purpura (D) Methotrexate
{E) NSAlD therapy

C] ltem 59
A 31-year-old man is evaluated In the emergency depart-
ment for fever and red. painful eyes. He reports a 1- month
Item 6 1 C]
history of Intermittent painful oral and genital ulcers, knee A 63-year-old man is evaluated for a 2-month history of
pain, and fever. and 1 week of bilateral eye redness. pain. progressive lower extremity weakness. He reports increased
and photophobia. difficulty arising from a chair, climbing stairs, and reaching
On physical examination . temperatu re is 38.3 oc overhead. He was diagnosed with polymyositis 6 months
(101.0 •f), and pulse rate is 100/mln. Slit lamp examination ago: prednisone and azathioprine were initiated at that
reveals white cells in the anterior chambers of both eyes. time, with clinical improvement. He currently takes pred-
l here are aphthous ulcers on the tongue and one aphthous nisone, 20 mg/d, and azathiop rine. 100 mg/d.
ulcer on the scrotum. Swelling and wa rmth are noted In On physical examination. vital signs are normal. The
both knees. lhe heart, lung, abdominal, and neurologic patient cannot arise from the chair without using his arms
examinations are normal. to push off and can squat only by using his hands to hold
on to the chair. He has cushingoid features and scattered
Which of the following is the most likely diagnosis? ecchymoses on the forearms.1here is no rash.
Laboratory studies show a serum creatine kinase level
{A) Beh~et syndrome
ofl60 U/L.
{B) Crohn disease
(C) Sarcoidosis Which of the following is the most appropriate manage-
ment?
(D) Systemic lupus erythematosus
(A) Orderelectromyography
(B) Order muscle biopsy
Item 60
{C) Reduce azathioprine dose
An 82-year-old woman Is evaluated for a 3-year history
of chronic discomfort and stiffness in the hands, wrists, (D) Reduce prednisone dose
and shoulders. Some of these joints intermittently become
acutely swollen and warm. She currently has 3 days of
severe leftwrist pain and swelling. She previously had been Item 62
diagnosed with seronegative rheumatoid arthritis; there A 26-year-old woman is evaluated for a 4-week history of
was no improvement with sulfasalazine or intravenous progressivedyspneaon exertion. She has experienced malaise
infliximab, both of which were discontinued. Short courses and myalgia for the past 6 months, noting that her arms ache
of prednisone have alleviated the acute attacks of arthri- when she does physical activity with them. She was previ-
tis. History is also significant for hypertension and stage 3 ously healthy. She reports no rashes, headache or jaw claudi-
chronic kidney disease. Medications are hydrochlorothia- cation, gastrointestinal symptoms, or neurologic symptoms.
7.ide. metoprolol, and losartan. On physical examination, blood pressure is 120/60 mm
On physical examination, vital signs are normal. Hg in the right arm and 95/50 In the left arm, pulse rote is
Decreased range of motion of both wrists and a few meta- 80/min, and respiration rate is 18/mln. There is no rash. A
carpophalangeal (MCP) joints is noted, and shoulder abduc- reduced radial pulse in the left upper extremity is noted. A
tion is limited to 9 0 degrees passlvety. The left wrist is grade 2/6 decrescendo diastolic murmur at the left sternal
swollen and tende r. There are no subcutaneous nodules at border is heard. lhe lungs are clear. lhere is no synovi tis.
the elbows. hands, or heels. Strength is normal.
Laboratory studies: Laboratory studies:
Erythrocyte sedimentation 51 mm/h Complete blood count Normal, except for anemia
rate Erythrocyte sedimentation 90mm /h
Urate 4.3 mg/dL (0.25 mmoi/L) ra te
Rheumatoid factor 15 U/mL (IS kU/L) Hemoglobin 10 g/dL (100 giL)
Anti-cyclic citrullinated Negative Creatinine Normal
peptide antibodies Antin uclear antibodies Negative
Urinalysis Negative
Radiographs show osteoarthritic changes at the sec-
ond through fourth MCP joints bilaterally, both wrists, and Transthoracic echocardiogram shows mild to moderate
glenohumeral joints. Chondrocalcinosis is seen in the knees aortic valve regurgitation, dilated aortic root, normal valve

99
Self-Assessment Test

leaflets, left ventricle dilation, and normal left ventricular as a rash starting in the feet and spreading up to the thighs
ejection fraction. and buttocks. She feels generally achy In the joints and mus-
cles. She reports no diarrhea, vomiting. or nausea. She was
Which oft he following is the most likely diagnosis?
previously healthy. She takes no medications.
(A) Giant cell arteritis On physical examination, vital signs are normal. A pal-
(B) lgA vasculitis pable purpuric rash is noted on the feet. legs. and buttocks.
There is peri-umbilical tenderness to palpation.
(C) Kawasaki disease Stool is positive for occult blood.
(D) Polyarteritis nodosa
(E) Takayasu arteritis Laboratory studies:
Complete blood count with differentia I Normal
Erythrocyte sedimentation rate 70mm /h
Antinuclear antlbodles Negative
Item 63 ANCA Negative
A 25-year-old woman Is evaluated for a 2-month history of Utinalysis Normal
joint pain and swelling involving the hands, knees, and feet.
On physical examination, vital signs are normal. Swell- CT of the abdomen shows a short segment of small
ing is noted at the second and third metacarpophalangeal bowel thickening and edema.
joints of both hands. the left wrist, both knees, and the sec- Which of the following is the most appropriate test to esl2b-
ond through fifth metatarsophalangeal joints of both feet. lisb the diagnosis?
laboratory studies are positive for rheumatoid factor
and anti-cyclic citrulllnated peptide antibodies. (A) Kidney biopsy
Radiographs oft he hands and feet are normal. (B) Mesenteric angiography

In addition to methotrexate and short-term prednisone, (C) Serum lgA levels


which of the following should be Initiated? (D) Skin biopsy with Immunofluorescence

(A) Folic acid


(B) Niacin Item 66
(C) Thiamine A 53-year-old man Is evaluated for as-year history of recur-
(D) VItamin C rent gout attacks Involving the base of the great toes, mid
feet, and ankles. Episodes are becoming more frequent and
severe. History is also significant for hypertension and stage
CJ Item 64 3 chronic kidney disease. Medications are lisinopril and
metoprolol. The patient is of Thai descent.
A 55-year-old man is evaluated for a 3-week history of pro-
On physical examination, vital signs are normal. 1here
gressive joint pain. swelling of the ankles, and occasional
dry cough. He also reports a 2-day history of low-grade are no tophi or swollen joints.
fever and painful red lumps on his shins. He has no other Laboratory studies show a serum urate level of 9.2 mg/
dL (0.54 mmol/L).
significan t history and takes no medications.
On physical examination. temperature is 37.8 •c Which of the following Is the most appropriate next step In
(100.0 •f): other vital signs are normal. TI1e chest is clear to management?
auscultation. Swelling and warmth of the ankles are noted.
1here are three raised. erythematous, and indurated subcu- (A) Begin allopurinol
taneous nodules on the right anterior shin and one on the (B) Begin probenecid
left anterior shin. (C) Measure antinuclear antibodies
laboratory studies show a normal com plete blood (D) Order HlA-B ·ssOl allele testing
cou nt and an erythrocyte sedimentation rate of70 mm/h.
Chest radiograph shows bilateral hilar adenopathy but
is otherwise normal. Item 67
Which of the following is the most appropriate manage- A 45-year-old man is evaluated for a 3-month history of
ment? swollen and painful hands and toes of both feet, without
trauma or Injury. He also has morning stiffness of the small
(A) Biopsy of the hilar node joints of the hands and feet lasting 2 hours. He takes no
(B) Biopsy of a sh in lesion medications.
(C) Rheumatoid factor testing O n physical examination, vital signs are normal. Dif-
fuse tenderness and swelling are noted in the third and
(D) Synovial fluid cultures
fourth toes of the right foot, fourth toe of the left foot, and
(E) No further testing third and fourth digits of the left hand. 1he remainder of the
joint examination is normal.

m It em 65
A 30-year-o ld woman is evaluated in the emergency depart-
1he appearance of the hand digits is shown (see top of
next page).
Plain radiographs of the feet show joint-space narrow-
ment for worsening abdominal pain over past 2 days as well Ing and soft-tissue swelling of the digits without erosions.

100
Self-Assessment Test

Item 69
A 43-year-old man is evaluated for knee, hip, shoulder, and
arm stiffness, worsening over the past year. He also has pain
in the arms, legs, neck, and torso, both on and between the
joints. He states that his discomfort is continuous. He also
reports chronic fatigue, lack ofsleep, a band- like headache,
and difficulty concentrating. He also has Intermittent con-
stlpation and diarrhea. He takes acetaminophen and ibu-
profen as needed tor pain without muc h benefit.
On physical examination, the patienrappears tired and
anxious, with no acute distress. Vital signs are normal. He
has minimal tenderness on palpation of multiple muscle
groups. Full range of motion of all joints Without crepitus is
noted. There Is no joint swelling or rash.
laboratory studies, Including complete blood count,
erythrocyte sedimentation rate, serum creatine kinase, and
thyroid-stimulating hormone. are within normallimlts.
Which of the following is the most Jikely diagnosis?
(A) Fibromyalgi:J
(B) Hypothyroidism
(C) Osteoarthritis
(D) Rheumatoid arthritis

ITEM 67 Item 70
A 25-year-old woman is hospitalized for a 4-week history
CJ
Which of the following disorders ls most likely to be associ- ofswelling of the legs, weight gain. and shortness of breath
ated with this patient's presentation? on exertion. She was diagnosed wllh systemic lupus erythe-
matosus 1 year ago when she presented with polyarthritis,
(A) Infectious arthritis
rash, and alopecia. She was initially treated with hydroxy-
(B) Rheumatoid arthritis chloroquine and prednisone with a good response.
(C) Spondyloarthritis On physical examination, blood pressure is 142/96 mm
(D) Systemic lupus erythematosus Hg: other vital signs are normal. There is pitting edema of
the lower extremities extending to the knees. The remainder
of the physical examination is normal.
CJ Item 68 Laboratory studies:
A 54-year-old man is evaluated for a 5- month history of mid Erythrocyte sedimentation 68 mm / h
lower back pain. The pain radiates from the center of his rate
back to his flanks and abdomen. He has intermittent numb- Hematocrit 38%
ness In the areas of the pain. He also reports a 2.3-kg (S-Ib) Complements (C3 and C4) low
weight loss over the past 3 months. He emigrated from India Creatinine 1.0 mg/dL (88.4 ~mol/ L)
to the United States20 years ago and travels to India yearly. Anti-Smith antiboclies Positive
He has tried Ibuprofen for the pain without relief and takes Anti-double-stranded DNA Positive
no other medications. antiboclies
On physical examination, temperature is 38.2 °C Urinalysis 3+ protein: no erythrocytes:
(100.8 oF); other vital signs are normal. Examination of no leukocytes: no casts
the spine reveals kyphosis. tenderness to palpation. and Urine protein 6000 mg/24 h
paraspinal muscle tightness around no. Decreased sensa-
tion to light touch along the TIO dermatome is noted. lhe patient Is started on prednisone, along with diuret-
Chest radiograph shows calcified granuloma in the right ics and an ACE Inhibitor.
upper lobe.1hora<.ic spine radiograph shows marked kypho- Kidney biopsy results show class V (membranous}
sis and erosive changes and collapse of the anterior portions lupus nephritis with absent chronicity and mild activity.
of the TlO and Ttl vertebral bodies. destruction oft he TIO-ll
Which of the following is the most appropriate treatment of
disk space, and surrounding fusiform paravertebral swelling. the kidney disease?
Which ofthe following is the most likely diagnosis? (A) Adalimumab
(A) Herniated intervertebral disk (B) Belimumab
(B) Multiple myeloma (C) Cyclophosphamide
(C) Osteoporosis (D) Methotrexate
(D) Tuberculous vertebral osteomyelitis (E) Mycophenolate mofetil

101
Self-Assessm ent Test

Item 71 more persistent in the past 4 months. He reports no daily


A 45-year-old woman is evaluated for increased pain and pain but has knee stiffness for lO minutes ln the morning
swelling in her left knee and right ankle. She has a 5-year and when he sits for an extended period of time. He reports
history of psoriatic arthritis and psoriasis treated with no swelling and no knee buckling or locking. History is
methotrexate, which previously Improved her joint and significant for gastroesophageal reflux disease and peptic
skin symptoms. She also takes folic acid. ulcer disease diagnosed 6 months ago now treated with
On physical examination, vital signs are normal. There omeprazole. He prefers not to take medications for his knee
is an erythematous. flaky patch on the left eloow. Nail pit- symptoms.
ting is present. The left knee has a moderate effusion and On physical examination, vital signs are normal. BMl
is warm without erythema, with discomfort on range of is 24. Crepitus and medial joint line tenderness to palpation
motion. The right ankle is swollen anteriorly, wlth discom- are noted. 1here is no redness, effusion. or signs of knee
fort on range of motion in flexion and extension. Instability.
Laboratory studies show an erythrocyte sedimentation Left knee radiograph shows mild medial joint space
rate of 30 mm/h. narrowing and spiking of the tibial spines.
Plain radiographs of the left knee show mild medial Which of the following Is the most appropriate manage-
and lateral joint space narrowing. ment?
Aspiration of the left knee shows a leukocyte count of
7500/J.Ll (7.5 X 109/L), with SO'X, neutrophils. (A) Ibuprofen
(B) Intra-articular glucocorticolds
Whlch of the following Is the most efficacious medication to
add to this patient's treatment regimen? (C) Left knee MR!
(D) Physical therapy
(A) Abatacept
(B) Hydroxychloroqulne
(C) lntliximab Item 74
(D) Rituximab A 21-year-old woman is evaluated for an 8-week history
of fatigue and low-grade fever. Last week she developed a
facial rash. She takes a multivitamin.
On physical examination, vital signs are normal. 1he
Item 72 remainder of the examination is normal.
A 24-year-old man Is evaluated for increasing left buttock The appearance of the face is shown.
pain that worsens over the course of the day. It is improved
by exercise and does not Improve with rest. 1he pain had
a gradual onset 1 year ago without a clear precipitant.
At that time, radiographs were unremarkable, and phys-
ical therapy provided no benefit. He has stopped exercis-
ing because of the increasing discomfort. Family history
includes ankylosing spondylitis in his father. 1he patient
takes no medications.
On physical examination, vital signs are normal. Nor-
mal ra nge of motion of the lumbar spine is noted. 1here is
no reduction In lateral bending and no limitation of cervical
spine motion in any plane. There are no warm, erythema-
tous, or swollen joints.
Laboratory studies show the presence of HLA-827;
normal complete blood count and C-reactive protein; and
negative rheumatoid factor and anti- cyclic citrullinated
peptide antibodies. Which of the following is the most likely diagnosis for the
Radiographs of the lumbar spine are normal. rash?
Which of the following Is the most appropriate treat- (A) Acute cutaneous lupus erythe matosus
ment? (B) Erysipelas
(A) Dlclofenac (C) Rosacea
(B) Etanercept (D) Seborrheic dermatit is
(C) Methotrexate (E) Subacute cutaneous lupus erythematosus
(D) Sulfasa lazine
Item 75 C]
A 53-year-old woman is evalu<tted for intermittent sen-
Item 73 sory Joss In the hands and an occasional shock-like sensa-
A 58-year-old man Is evaluated for a 3-year history of left tion from the neck down the back with neck flexion. She
knee stiffness, which was intermittent but has become has not noted any weakness. She has a 15-year history of

102
Self-Assessment Test

rheumatoid arthritis. Medications are methotrexate, etaner-


Cl cept,
CONT.
and folic acid.
On physical examination, vital signs are normal. Ulnar
does not recall how or if he was treated for the disease. He
has medical records from his pediatrician who has provided
ongoing care until now.
deviation of the metacarpophalangeal joints on both hands Physical examination, including vital signs, is normal.
is present without active synovitis. Neck flexion triggers her
symptoms. lhere is no muscle atrophy at the hands. Tinel Which of the following disease complications Is most
and Phalen signs at the wrist are negative. Reflexes, strength, Important in determining the nature of his sports evalua·
and sensation ofthe upper and lower extremities are normal. t ion and prognosis?

Which of the following is the most appropriate diagnostic (A) Bundle branch block
test to perform next? (B) Coronary artery aneurysm
(C) Myocarditis
(A) Electrodiagnostic studies of the upper extremities
(D) Pericarditis
(B) Flexion/extension radiography of the cervical spine
(C) Serum vitamin B11 level
(D) Thyroid-stimulating hormone level Item 78
A 38-year-old woman seeks advice regarding preventa-
I:J Item 76
tive measures for developing osteoarthritis. One year ago,
she started an exercise regimen of running approximately
A 36 -year~ld woman is brought to the emergency depart- 20 miles per week. She has no joint pain when running. His-
ment by her family for mental status changes. For the past tory is significant for polycystic ovary syndrome. Family his-
2 days, the family has noticed forgetfulness, confusion, and tory is notable for both parents having knee replacemencstbr
alteration of personality. She has a S-yearhistoryofsystemic osteoarthritis. Her only medication is an oral contraceptive.
lupus erythematosus (SLE), which has been quiescent. One On physical examination, vital signs are normal. BMI is
week ago, she developed fatigue. a diffuse erythematous rash. 31. Musculoskeletal examination is normal.
and polyarthritis. There is no history of focal deficits, seizures,
or neck stiffness. She does not have a fever. History is also sig- Which of the following is the most appropriate preventive
nificant for hypertension. Medications are losartan, hydroxy- measure for this patient?
chloroquine, prednisone, 5 mgtd, and a dally multivitamin. (A) Begin chondroitin sulfate supplements
On physical examination, the patient Is alert, awake, (B) Begin vitamin D
and follows commands but is not oriented to place and lime.
She is afebrile, and vital signs are normal. Neurologic exam- (C) Recommend weight loss
Ination other than mental status is normal, with no motor (D) Stop running
or sensory deficits. There Is no neck stiffness. The remainder
of the examination is normal.
Item 79
Labor.rtory studies:
Erythrocyte sedimentation 42mm/h
A 32-year-old woman is evaluated in the emergency depart-
Cl
ment for a 2-day history of headache and vomiting. She
rate
was diagnosed with Raynaud phenomenon I year ago and
Complements (C3 and C4) Low
C-reactive protein 0.2 mg/dL (2.0 mg/L) gastroesophageal reflux disease 6 months ago. Her only
medication is omeprazole.
Creatinine 1.4 mg/dL (123.8 J.tmoi/L)
On physical examination, temperature is 38.0 oc
Antinuclear antibodies Titer: 1:320
(100.4 •f), blood pressure is 240/140 mm Hg, pulse rare is
Cerebrospinal fluid analysis Normal cell count and glu-
cose; protein, 85 mg/dL 88/min, respiration rate is 16/min, and oxygen saturation
(850 mg/L) is 96% breathing ambient air. Skin findings are digital pit-
ting at the ends of the fingers. thickening of the skin over
cr scan of the brain is normal. MRl with MR angiogram the fingers and dorsum of the hands. and thickening with
of the brain shows multiple small punctate white matter poikilodermic changes over the skin of the anterior chest.
changes in periventricular areas. There is no evidence of Laboratory studies:
vasculitis or blood vessel narrowing.
Hematocrit 32%
Which of the following is the most likely diagnosis? Platelet count 75,000/J.IL (75 x 109 /L)
Creatinine 1.5 mgtdL (132.6~-tmoi/L)
(A) Acute bacterial meningitis Urinalysis 2+ protein; no blood
(B) Neuropsychiatric systemic lupus erythematosus
Peripheral blood smear shows diminished platelet
(C) Status epilepticus
numbers and schistocytes.
(D) Steroid-induced psychosis
Which of the following is the most appropriate treatment?

Item 77 (A) Captoprll


(B) Cyclophosphamide
An 18-year-old man Is seen for a physical examination
required to join the ice hockey team at his university. He (C) Methylprednisolone
was diagnosed with Kawasaki disease at age 5 years; he (D) Nitroprusside

103
Self-Assessment Test

Item 80 to touch, with a large effusion: range of motion is limited.


A 32-year-old woman Is evaluated during a follow-up visit especially flexion. Examination ofthe other joints is normal.
for a 3-month history of arthralgia affecting the hands, Laboratory studies show an erythrocyte sedimenta-
wrists, knees, and feet. At her initial visit 3 weeks ago, tion rate of 53 mmth and a leukocyte <:ount or t2.000ilJ.L
joint examination demonstrated nine tender and six swol- (12 X 109 / L).
len joints; prednisone,lO mg/d, was initiated at that time. Which of the following is the most appropriate next step in
She now reports some benefit from the prednisone for management?
morning stiffness and joint pain. Family history is nota-
ble for her mother with seropositive erosive rheumatoid (A) Perform arthrocentesis
arthritis. (B) Start colchicine
laboratory studies from 3 weeks ago showed an eryth- (C) Start prednisone
rocyte sedimentation rnte of 38 mm/h, and high levels of (D) Start sulfasalazine
rheumatoid factor and anti-cyclic cHrull!nated peptide
antibodies.
On physical examination today, vital signs are normal. Item 83
1he second and third proximal interphalangeal joints on A 52-year-old man is evaluated for an episodic rash on the
the right hand and the second metacarpophalangeal joint legs for the past month; diffuse arthralgia and Raynaud
on the left hand are tender and swollen. Her clinical dis- phenomenon for2 months; and burning pain and tingling of
ease activity Index score is 12, indicating moderate disease the feettbr 3 months. He has a remote history of intravenous
activity. drug use. He takes no medications.
Radiographs oft he hands and feet are normal. On physical examination, vital signs are nonnal. Cya-
Which of the following is the most appropriate treatment nosis of the fingertips is noted. There Is no sclerodactyly,
at this time? dilated capillary loops of fingernail beds, or digital pitting.
Palpable purpura is present on the lower legs. Reduced
(A) Increase prednisone sensation to pinprick of the soles of the feet is noted. lhe
(B) Initiate Ibuprofen remainder of the examination is normal.
(C) Initiate methotrexate Laboratory studies:
(D) Initiate mycophenolate mofetil Erythrocytesedimentation rnte 70mm/h
C3 Normal
C4 Low
Item 81 Rheumatoid factor Positive
A 34-year-old woman is evaluated for a 6-month history Antinuclear antibodies Negative
of gradually increasing pain in the left groin with some Cryoglobulins Positive
radiation to the left buttock, particularly with stair climb- Urinalysis 1+ blood; 2+ protein
ing. She has a 2-year history of lupus nephritis. Medica- Chest radiograph is normal. Skin biopsy shows leuko-
tions are hydroxychloroquine, mycophenolate mofetil, cytoclastic vasculitis.
and prednisone.
On physical examination, vital signs are normal. Mus- Which of the foUowing is the most appropriate test to
culoskeletal examination reveals weakness of the left hip perfonn next?
flexors. Decreased passive range of motion of the left hip in (A) Anticentromereantibodies
both external and internal rotation with pain is noted.
(B) Anti-cycllccltrullinated peptide antibodies
Which of the foDowing Is the most appropriate Initial test to (C) Anti-Jo-1 antibodies
evaluate the patient's hip pain? (D) Anti-Ul-ribonucleoprotein antibodies
(A) Dual-energy x-ray absorptlometry (E) Hepatitis Cantibodles
(B) MRI of the left hip
(C) Plain radiography of the left hip Item 84
(D) Ultrasonography A 30-year-old man is evaluated fora 7-year history of inter-
mittent pain and swelling in multiple fingers and toes, along

CJ Item 82
with 1 hour of morning stiffness. History is notable for an
episode of swelling of the left second toe 2 years ago. He
A 65-year-old man is evaluated in the emergency depart- has been treated with multiple NSAIDs over the years, with
ment for left knee pain and swelling of 10 days' duration. ibuprofen being the most efficacious.
He reports no other joint pain. He has a 15-year history of On physical examination, vital signs are normal. Mild
rheumatoid arthritis, which has been well controlled with soft-tissue swelling oft he fourth and fifth distalinterphalan-
methotrexate and adalimumab. He began tal<ing over-the- geal (DIP) joints bilaterally is present. Tenderness is present
counter naproxen 8 days ago, without Improvement. with metacarpophalangeal joint squeeze on the left. There
On physical examination, temperature Is 38.1 <>C is no rash.
(100.6 °F), blood pressure Is 106/68 mm Hg, and pulse rate 1he patient's nail findings are shown (see top of next
is 97/min. The left knee Is erythematous, warm, and tender page).

104
Self-Assessment Test

Which of the following tests is most appropriate to perform


next to assess this patienr s kidney disease?
(A) Antiphosphollpid antibodies
(8) CT of the abdomen and pelvis
(C) Kidney biopsy
(D) Renal arteriography
(E) Serum and urine protein electrophoresis

Item 86
A 71 -year-old woman Is evaluated for a 3-year history of
rheumatoid arthritis. She reports burning and tingling sen-
sations In her feet and legs that are worse at night, without
exacerbation with position or activity. She has not noted
bnck pain, lower extremity weakness, or bowel or bladder
dysfunction. Medications are methotrexate. hydroxychlo-
roqulne, low-dose prednisone, and folic acid. Her rheuma-
ITEM 84 toid arthritis was moderately active 6 months ago, at which
time leflunomide was added with good response.
Onphyslcalexamlnation, vitalslgnsarenonnal. Decreased
Laboratory studies show a serum urate level of6.7 mg/ sensation to light touch and vibration is limited to the feet. Joint
dL (0.40 mmol!L); rheumatoid factor and anti-cyclic citrul- examJnation reveals one tender and two swoUen metacarpo-
linated peptide antibodies are negative. phalangeal joints. Strength testing is Intact throughout, and
Plain radiographs of the hands show pencil-in-<:up reflexes are Intact lhere are no skin rashes or lesions.
deformities of the DIP joints of the third and fourth digits Laboratory studies show norma I complete blood count,
bilaterally; there is no periarticular ostcopenia. serum creatinine, blood glucose. and hemoglobin A 1c values.
Which of the following Is the most Ukely diagnosis? Which of the following is the most appropriale next·slep in
manageme nt?
(A) Gouty arthritis
(B) Psoriatic arthritis (A) Begingabapentln
(C) Reactive arthritis (8) Increase prednisone
(D) Rheumatoid arthritis (C) Stop leflunomlde
(D) Stop methotrexate

[:J Item 85
Item 87
A 32-year-old woman is evaluated loran 8-week history of
fatigue and low-grade fever. She also reports swelling and An 83-year-old woman is evaluated for a 15-year history
tenderness of the hand joints, along with morning stiffness of gout. Attacks were initially limited to the tlrst meta-
lasting 2 hours. Over the past 2 weeks, she has been taking tarsophalangeal joints and occurred once or twice a year.
naproxen with relief. Last week, she developed swelling of She now has recurrent attacks, occurring every 3 months.
the legs and gained 4.5 kg (JO lb). Her feet, ankles, knees, wrists. elbows, and hands have
On physical examination. blood pressure is 152/96 been affected. Attacks last for 1 to 2 weeks and have been
mm Hg: other vital signs are normal. A malar rash is pres- treated with NSA lOs. History is also significant for recurrent
ent Active tenderness and swelling of multiple joints of nephrolithiasis, stage 3 chronic kidney disease, and hyper-
the hands are noted. There Is pitting edema of the lower tension. Medications are allopurinol, 400 mg/d; colchicine,
extremities. The remainder of the physical examination is 0.6 mg/d; and lisinopril. She tolerates her medications well.
normal. On physical examination. vital signs are normal. There
are palpable masses at the olecranon processes and white
Labo ratory studies: nodules on a few distal and proximal interphalangeal joints
Erythrocyte sedimentation 88 mm/h o f both hands.
rate Laboratory studies show a serum creatinine level of
Hematocrit 38% 1-0 mg/dL (88.4 f.tmol/U and a serum urate level ofS.8 mg/dL
Complements (C3, C4) Low (0.34 mmol/L).
Creatinine 1.1 mg/dL (97.2 ~mol/ U
Antinuclear antibodies Tlter: 1:320 Which of the foUowing Is the most appropriate next step in
Anti-Smith antibodies Positive management?
Anti-double-stranded DNA Positive
(A) Add probenecid
antibodies
Urinalysis 3+ protein; no erythrocytes; (B) Discontinue allopurinol; begin peglotlcase infusions
no leukocytes; no casts (C) Increase aUopurinol
Urine protein 4000mg/24 h (D) Increase colchicine

105
Self-Assessment Test

Item 88 Methotrexate and leflunomide were tried, but she was not
A 40-year-old woman Is evaluated for a 12-year history of able to tolerate them. Her kidney disease is well controlled
Inflammatory arthritis, puffy lingers, Raynaud phenome- without any active nephritis.
non, serositis, and gastroesophageal reflux disease. On physical examination, vital signs are normal. Sev-
On physical examination, vital signs arc normal. The eral hyperkeratotic, dyspigmented, discoid plaques are
fingers are puffy. and cyanosis of the fingertips is noted. present on the scalp. Multiple joints of the hands are tender
Cardiac examination is normal. The lungs are clear. There is and swollen. 1he remainder of the examination is normal.
no friction rub heard on examination. Uiboratory studJes:
laboratory studies show positive anti-Ul-ribonucleo- Erythrocyte sedimentation rate 56mm/h
protein antibodies. Hematocrit 38'l(,
Which of the following compllcadons of this patient's Complements (C3, C4) Low
disease is a major cause of early mortality? Creatinine 1.5 mg/dL (132.6j.J.moi/L)
Anti- double-stranded DNA Elevated
(A) Central nervous system vasculitis antibodies
(B) Pericarditis Urinalysis Normal
(C) Pulmonary arterial hypertension Chest radiograph is normal.
(D) Rapidly progressive glomerulonephritis
Which of the following Is the most appropriate therapeutic
opdon to consider Jn this patient?
Item 89 (A) Abatacept
A 75-year-old woman is evaluated for a 15-year history of (B) Adalimumab
right knee pain, which recently has become more constant, (C) Belimumab
with morning stiffness lasting 15 minutes. She participates (D) Secukinumab
in a home exercise program. Her knee pain persists despite
treatment with acetaminophen. She could not tolerate top·
ical capsaicin or diclofenac. History is also significant ror
atrial fibrillation and hypertension. Medications are dab- Item 91
igatran and diltiazem. She would like to Improve her knee A 74-year-old man is evaluated for pain and stiffness in the
pain, because she is going on vacation in 2 weeks and will mid and lower spine that has progressively worsened over
be walking a lot. the past 10 years. The pain is worse with physical activity.
On physical examination, vital signs are normal. Exam- He notes recent difficulty when bending to pick something
Ination of the right knee reveals crepitus upon movement, up from the floor. He takes no medications.
bony hypertrophy presenting as varus deformity. and no On physical examination. vital signs are nonnal. No
effusion or warmth. rash or nail changes are seen. Limited range of motion and
Weight-bearing knee radiographs show moderate pain on motion of thoracic and lumbar spine are noted.
medial joint space narrowing, osteophytes at the medial There is no peripheral joint swelling or tenderness and no
femoral and tibial joint margins, and subchondral sclerosis sacroiliac tenderness. 1he FABER test of the hip ls normal.
at the medial tibia. Laboratory studies show a normal erythrocyte sedi·
mentation rate.
Which of the following is the most appropriate treatment? Thoracolumbar spine radiographs reveal bridging ossi-
(A) Alendronate fication on the right side along the anterolateral aspects of
(B) Arthroscopic debridement and lavage the vertebral bodies ofT9-U. Radiographs of the sacroiliac
joints are normal.
(C) Intra-articular glucocorticoid injection
(D) Pregabalin Which of the following is the most Ukely diagnosis?
(E) Tramadol (A) Ankyloslng spondylitis
(B) Calcium pyrophosphate deposition disease
(C) Diffuse idiopathic skeletal hyperostosis
Item 90
(D) Psoriatic arthritis
A 26-year-old woman is evaluated for a rash on her scalp
and polyarthritis associated with systemic lupus erythe-
matosus (SLE). She was diagnosed with SLE 8 years ago
with an initial presentation of photosensitivity, malar and Item 92 C]
discoid rashes, pleuropericarditis. and arthritis. She was A 78-year-old man is evaluated in the emergency depart-
initially treated with hydroxychloroqulne, azathioprine, ment. Five days ago he was diagnosed with community-
and glucocorticolds w ith a good response, but she has had acquired pneumonia at an urgent care center for which
intermittent flare-ups of arthritis and rash. She developed he was prescribed clarithromycin. Yesterday he developed
lupus nephritis 3 years ago and had a 6-mont h course of generalized weakness and diarrhea. History is also significant
cyclophosphamide followed by mycophenolate mofetil for hypertension and gout. He has a history of allopurinol-
(MMF). She is currently on MMF, hydroxychloroquine, and induced drug rash. Current medications are lisinopril. col-
prednisone but continues to have joint a nd skin disease. chicine. clarlthromycin. a nd febuxostat.

106
Self-Assessme nt Tes t

On physical examination. temperature is 36.9 •c


CJ
com.
(98.4 "F). blood pressure Is l04/ 60 mm Hg, pulse rate is 112/
min, respiration rate is18/min. and oxygen saturation is
Which of the following is the most likely diagnosis for the rash?
(A) Acute cutaneous lupus erythematosus
98% breathing ambient air. There is no rash. The oropharynx (B) DermatomyosttJs
appears dry. The chest is clear to auscultation. Tophi are (C) DtscoJd lupus erythematous
noted at the olecranon processes. There are no swollen or (D) Subacute cutaneous lupus erythematous
tender joints. Neurologic examination is normal.
Laboratory studJes:
Absolute neutrophil count 1300/~-tl (1.3 X 109 / L)
Item 94
Hemoglobin 9.8 g/dL (98 giL) A 49-year-old man is evaluated for a 10-year history of inter-
Leukocyte count 2700/JlL (2.7 X lOq!L) mittent low back paln with 30 to 60 minutes of morning
Platelet count 96,000/f.IL (96 x W / L) stiffness, both of which improve with exercise. The bad< pain
Reticulocyte count 1.2% can awaken him from sleep. He also reports occasional pain
(corrected) in the neck, shoulder blades, and buttocks on one side or the
Creatine kinase 8433 U/ L other. History is notable for anterior uveitis d iagnosed more
Creatinine 1.7 mg/dL (150.1 j.tmoi!L) than 30 years ago, the symptoms of which recurred every 3
Urinalysis 4+ blood; 0-1 erythrocytes/ to 4 years but have been subsequently controlled by topical
hpf; 0-2 ieukocytes/ hpf giucocorticoids. Current medications are as-needed NSA1Ds
and ophthalmic glucocorticoid drops.
There are no schistocytes seen on peripheral blood On physical examination, vital signs are normal. Lumbar
smear. spJne range of motion is restricted for torward flexion. There are
Which of the following is the most appropriate next step in no warm, erythematous, or swollen joints. There is no spinal or
management? sacroiliac tenderness. Conjunctivae are without injection.

(A) Begin high-dose glucocortlcoids Which of the following is the m ost appropriate test to per-
(B) Begin plasma exchange
fonn next?
(C) Stop colchicine and clarithromycln {A) Bone scan
(D) Stop febuxostat (B) CT of the sacroiliac joints
{C) MRI of the lumbar spine
(D) Radiography of the sacroiliac joints
Item 93
A 33-year-o ld woman is evaluated for a 4- week history
of recurrent wo rsening joint pain and a new rash on the Item 95
chest and upper back. She reports sun exposure from A 48-year-old man is evaluated during a routine follow-up
spend ing time at the beach. She has no other sym ptoms. visit for gout dJagnosed 2 months ago, at which time he
She has no other medical problems and takes no medi- started allopurinol and colchidne. Since then, he has had
cations. three acute attacks separately involving the left wrist, right
On physical examination, vital signs are normal. Dif- knee, and left foot He has no current joint pain. He also
fuse tenderness of multiple small joints of the hands is reports having dJarrhea two w three times daily over the
noted. The remainder of the examination is normal. past 6 weeks. History is also significant for type 2 diabetes
'The appearance oft he rash is show n. me!I it us, for which he takes insulin.
On physical examination. vital signs are normal. Mus-
culoskeletal examination is normal.
Laboratory studies show a serum creatinine level of
0.8 m g/dL (70 7 J.J.moi/L) and a serum urate level of 5.5 mg/
dL (0.32 mmol!L).
Which ofthe following is the most appropriate next step to
d ecrease the frequency of gout attacks?
(A) Add prednisone
(B) Add probenecid
(C) Discontin ue colchicine; begi n meloxicam
(D) Increase allopurinol
(E) Increase colchicine

Item 96
A 28-year-old woman is evaluated for a 3-week history
of isolated left groin pain. The pain is worse with weight
bearing. She has a 10-year history of systemic lupus

107
Self-Assessment Test

erythematosus (SLE) and lupus nephritis. She has also Pelvic and hip radiographs are nom1al.
received high-dose glucocorticoids multiple times in the
past 10 years. She is currently doing well without any active WhJch of the following Is the most appropriate test to
SLE manifestations. Medications are hydroxychloroquine perform next in this patient?
and low-dose pred nisone.
(A) CT of the hip
On physical examination, vital signs are nonnal. The
lefr groin has slight discomfort with hip movement and (B) MRl of the hlp
weight bearing but is non tender. There is no active synovi- (C) Radionudide bone scan
tis. The remainder ofthe examination is normal. (D) Single photon emission CT

108
Answers and Critiques

[::::J Item 1 Answer: D I:] Item 2 Answer: C


Educational Objective: Diagnose prosthetic joint Educational Objective: Diagnose microscopic poly:mgi.ltis.
infection.
The most likely diagnosis is microscopic polyangiitis (MPA).
Prosthetic Joint infection is the most likely diagnosis. This which characteristically affects the lungs and kidneys. along
patient has a prosthetic hip and is currently undergoing with other organ systems. Diagnosis is suspected based upon
chemotherapy. Malignancy and Immunosuppression are typical cllnlcal findings and positive ANCA. The diagnostic gold
risk factors for prosthetic joint infection, and thls patlenc standard is a biopsy demonstrating nongranuJomatous necrotiz-
also has a central venous catheter tha t increases his ing pauci-immune vasculitis of small vessels or pauci- immune
risk for bacteremia. The elevated leukocyte count and necrotizing crescentic glomerulonephritis in the kidney. This
erythrocyte sedimentation rate, along with periprosthetic patient has pulmonruy hemorrhage, paucHmmune smaU-ves-
lucency on radiographs. are all suggestive of prosthetic sel necrotiZing vasculitis of the skin, proteinuria, inflammatory
joint Infection. Prosthetic joint infec tions are divided arthritis, and a positive p-ANCA (directed against myeloperoxi-
into early onset (<3 months after placement). delayed (3 dase)- all consistent with the diagnosis ofMPA.
to 24 months postsurgery) , and late onset (>24 months Granulomatosis with polyangUtis (GPA) can cause the
after placement). Early and delayed infections are usu- same lung, skin, joint, and kidney findings but Is usually
ally related to surgical contamination at the time of the associated with a positive c-ANCA (directed against protein-
implantation, whereas late Infec tions result from hema- ase-3). Furthermore, GPA causes granulomatous in11amma-
togenous seeding of the joint. Early and late prosthetic tlon; absence of granulomas distinguishes MPA from GPA.
joint infections typically present with pain. warmth, lgA vasculitis (Henoch-Schonlein purpura) can affect
effusion, and fever. Staphylococcus aureus Is a leading lungs. skin, joints, and kidneys but it Is an immune complex-
causative agent in prosthetic joint Infections; treatment mediated vasculitis with deposition of lgA containing
involves prompt intravenous antibiotics and sometimes immune complexes identified on biopsy. which is not pres-
removal of the prosthesis. ent in this patient
Gout flare may also cause joint pain and loss of mobil- Rheumatoid vasculitis usually occurs in long-standing
ity. as well as fever, elevated leukocyte count, and inflam- disease with positive rheumatoid factor and is Immune com-
matory markers. However, this patient has no prior history plex mediated, none of which is present in this patient.
of gout, and the hip is an uncommon place for a gout flare, Thromboangiitis obliterans is a smoking-related vas-
especially an initial flare. Most initial gout flares affect the culopathy affecting small to medium vessels with inflam-
foot and/or ankle joints. mation and thrombosis of vessels In upper and lower limbs,
Hemarthrosis causes bleeding into a joint and is leading to reduced pulses and gangrenous ulcers. This diag-
characterized by pain, swelling, warmth. and impaired nosis Is unlikely because this patient no longer smokes, and
mobility. It is diagnosed by synovial fluid aspiration. This has involvement of internal organs in addition to the legs.
patient does not have a history or findings suggestive of
UY POINT
a bleeding diathesis to put him at risk for a traumatic
hemarthrosis. • The diagnostic gold standard of microscopic polyangi-
A patient with hip dislocation may present in a similar itis Is a biopsy demonstrating nongra nulomatous
manner as one with prosthetic joint infection, namely pain necrotizing pauci-tmmune vasculitis of small vessels
and difficulty moving the joint; however, a dislocation would or pauci-immune necrotizing crescentic glomentlo-
be evident on radiographs. nephritis in the kidney.
KEY POINT
Bibliography
• Both early and late prosthetic joint infections are typi- Kallenberg CG. The diagnosis and classification ofm lcroscoplc poJyanglitls.
cally characterized by pain, warmth, effusion, and J Autotmmun. 2014:-18-49:90-3. (PMID: 2H61388) doi:10.10161j.)aut.
2014.01.023
fever; treatment involves prompt intravenous antibi-
otics and possible removal of the prosthesis.
Item 3 Answer: B
BibUogr:aphy Educational Objective: Treat severe tophaceous gout.
Os1non DR, Berbarl liP. llercndt AR, Lew D. Zimmer!! W. Steckelberg JM.
et al; Infectious Diseases SOciety of America. Diagnosis and rrolnagemem In addition to stopping febuxostat, the most appropriate
of prosthetic joint Infection: diD leal practice guidelines by the lntecllous
Diseases SOciety of America. Clln Infect Dis. 2013:S6:el·e25. [PMID: treatment is pegloticase Infusions. This patient has severe
23223583] doi:lO.l093/cid/ciS803 recurrent and tophaceous gout that has been resistant to

109
Answers and Critiques

standard therapies, including febux.ostal. Pegloticase Is an trac:homatis is the most common agent causing urethritis
intravenously administered porcine-derived uricase (Infused that Is associated with reactive arthritis. This patient has the
every 2 weeks), which reduces serum urate to nearly zero triad of conjtmctivitis, urethritis, and arthritis seen in a sub·
within hours of administration. lf anti-drug antibodies do set of patients with chlamydia! reactive arthritis. Typically,
not fonn, tophi may resolve over the course of months. Other affected men experience dysuria and note a urethral dis-
urnte-Iowering therapies should be discontinued with initia- charge, although less frequently than in arthritis related to
tion of pegloti<.'3se because they can mask the development of gonococcal infection. Gonococcal arthritis, however, is not
antibodies that mnniiest as rising serum urate levels. Patients assodated with conjunctivitis and is often accompanied by
starling pegloticase should be placed on prophylaxis to pre- vesicopustular sl<ln lesions and tenosynovitis. Reactive a rthri-
vent acute gout attacks; colchicine, prednisone, or NSAlDs ris typically occurs 3 to 6 weeks alter the infectious trigger,
are appropriate. In this case, glucocorticolds and NSAJDs with a latency range of 2 weeks to 6 months. In a significant
should be avoided because of the concomitant type 2 diabetes minorily of cases, the triggering infection may go unrecog-
mellitus and chronic kidney disease; therefore, low-dose col- nized. Although lnt'ection has usually resolved by the time
chicine is the appropriate prophylactic agent of arthritis onset in patients with reactive arthritis, nucleic
It is important to note that two recently published guide- ac.id amplification urine testing for C. trochornatis should be
lines differ regarding the role of pharmacologic urate-lowering performed because some individuals may have asymptomatic
therapy in patients with gout The 2016 American College of persi~1ent infection or carringe of this organism.
Physicians guideline (http:/lannals.org/aim/article/2578528/ C-reactlve protein would not be of value In establishing
management-acute-recurrent-gout-dlnical-practice-guide- the diagnosis. The patient has an acute inflammatory proce;s
llne-from-american-college) notes a lack of evidence sup- In the joints based on his history, physical examination, and
porting a specific target level for urate lowering; this guideline synovial fluid analysis. C-reactive protein would not lend
stresses discussing the risks and benefits of urate-lowering further diagnostic information.
therapy with patients and suggests a "treat to avoid symptoms" HLA-827 antigen may be present in those with reactive
approach without specifically considering the serum urate lev- arthritis, but it has little diagnostic specificity in this disor-
els. The 2016 European League Against Rheumatism (EULAR) der. The presence of l·!LA-B27 antigen would neither rule in
recommendations support a "treat-to-target" approach (con- nor rule out reactive arthritiS.
sistent with the 2012ACRgoutguidelines), reducing the serum The interferon-gamma release assay <.-an indicate prior
urate level to less than 6.0 mg/dL (0.35 mmoi/L) ln patients exposure to tuberculosis. 'lhe clinical picture In this scenario
without tophi and less than 5.0 mg/dL (0.30 mmoi/L} in is not that of tuberculous arthritis, which tends to be manoa r-
patients with tophi. The pntient's serum urate level is 6.3 mgt ticular, most frequently affects the hlp, has a course that is sub-
dt (0.37 mmoi/L) on febuxostat therapy, and she continues to acute with progressive pain and loss of function over weeks to
have symptoms; therefore, escalation of treatment is appropri- months, and is not often associated with features of inflamma-
ate according to both the ACP and EULAR guidelines. tory synovitis on physical examim1tion (warmth, erythema).
Probenecid is not appropriate tor those with an estimated
KEY POINT
glomerular fllrratlon rate of less than 60 mUmin/ 1.73 ml or
with a history ofl<ldney stones; this patient has both. • Nucleic acid amplification urine testing is the appro-
Losartan has a modest uricosuric effect, but not signif- priate diagnostic test tor suspected chlamydia! reac-
icant enough that it would be recommended in a patient tive arthritis.
with severe tophaceous gout, especially one who remains
symptomatic with an elevated serum urate level despite Bibliography
febux:ostat therapy. Olrter JD. Hudson 1\P. Recent advances and future directions In under-
sl~ nding and treutlng Chlamydia-Induced reactl~~e llrthritis. Expert Rev
KEY POINT Clin lmmunol. 2017:13!197-206. [PMID! 27627462] doi:JO.I0801t7446
66X.2017.!233816
• Pegloticase may be considered for patients with severe
recurrent and/or tophaceous gout that is intolerant or
Item 5 Answer: E
resistant to standard therapies.
Educational Objective: Diagnose ankyiOSing spondylitis.
Bib liography
No additional tests are necessary. Dlfierent criteria have
Sltn<ly IS, Baraf TIS. Yood RA, Edwards NL, GuUem?'t-Urena SR. Treadwell
El, et al. Efficacy and rolerJblllty of peglorlcase lor the treatment or been proposed tor the diagnosis of ankylosing spondylitis.
chronic goulln patients refrnt'!Ory to conventionultre-Jlment: two rnnd- Common requirements include the presence of inflamma-
omlzed controlled trials. JAMA. 2011:306:71l-:.l0. [PMID: :.!16'16852)
doi:lO.JOOI/jama.20Jl.l169 tory back pain tor 3 or more months in a person younger
than age 45 years, limited lumbar spine motion, elevated
Item 4 Answer: A inflammatory markers, and evidence of bilateral sacrolliitls
on imaging. The patient has a long history of inflammatory
Educational Objective: Diagnose reactive arthritis.
back pain (Improves with exercise, worsens with sleep or
The most appropriate diagnostic test to perform next Is nudeic inactivity), loss ofrangeofmotion of the lumbar spine, and
acid amplification urtne testing for chlamydia. Chlamydia radiographs showing fusion of the sacroiliac joints, one of

110
Answers and Critiques

the typical features of ankylosing spondylitis. It is import- may have Cameron lesions. 1here is no evidence of a hiatal
ant to establish the diagnosis of ankylosing spondylitis to hernia, making this diagnosis unlikely.
assess the risk of further joint fusion and deformity in this Dieulafoy lesions are submucosal arterioles that inter-
patient. mitte ntly protrude through the mucosa and cause hem-
ANCA is present in some forms of medium-vessel orrhage. Dieulafoy lesions are of unknown etiology and
vasculitis. In the absence of cutaneous or internal organ account for about 2% of all causes of acute upper gastroin-
manifestations suggestive of vasculitis (sinus, pulmonary, testinal bleeding and are located in the proximal stomach
l<idney, cutaneous, and ophthalmologic abnormalities), along the lesser curvature. When bleeding they appear as
it is not appropriate as the next step in this patient's an isolated pumping arteriole in the absence of a mass or
evaluation. uloer. The patient's findings do not match those of Dieulafoy
Anti-cyclic citrullinated peptide antibodies have spec- lesions.
ifkity for the diagnosis of rheumatoid arthritis and are Portal hypertensive gastropathy (PHG) commonly
appropriately ordered when patients present with bilaterally occurs with advanced cirrhosis and has a characteristic
symmetric Inflammatory arthritis of the small joints of the mosaic appearance on endoscopy, most often seen In the
hands. These antibodies will not establish the diagnosis of body and fundus. It can be confused with GAVE.1his patient
ankylosing spondylitis. has no history of cirrhosis or findings to suggest chronic liver
Testing for antinuclear antibodies should be carried out disease, making PHG an unlikely diagnosis.
when there Is suspicion tor the presence of systemic lupus
KEY POINT
erythematosus. 1his patient does not have rash, sun sensitiv-
ity, alopecia, oral ulcers, pleuropericarditis, l<idney disease, • Patients with diffuse cutaneous systemic sclerosis are
or laboratory abnormalities that would raise suspicion for at risk for acute and chronic gastrointestinal bleeding
lupus. secondary to gastric antral vascular ectasia.
The presence of HLA-827 antigen is not a diagnostic cri-
terion for ankylosingspondylttls. However, such testing may Bibliography
be particularly helpful in patients with Inflammatory bacl< Fuccto 1- Mussctto A. lalcl7.il L Eu~ebi Ll·l. Rnr.mlt F. Diagnosis o~nd m~n­
ugemenl uf ~sene unt1-al v-Jscular Cl'tasin. World J Gastruintest l::ndusc.
pain and other manifestations of anl<ylosing spondylitis but 2013:5:6- I:J. [PM ID: 233:l0048J dui:I 0.4253/wjge. vs.\1.6
without evidence ofsacrolliitis on imaging.
kEY POINT
• A diagnosis of ankylosing spondylitis can be made in
ltem7 Answer: A
Educational Objective: Treat refractory acute gout.
Cl
a patient younger than age 45 years with symptoms of Anakinra is t he most appropriate treatmen t for this
Inflammatory back pain for 3 months or more and patient's knee. He has a persistent fever due to a severe
bilateral sacroiliitis on imaging. acu te gouty attack. Synovial fluid analysis permits defin-
itive diagnosis and can rule out other conditions. Under
Bibliography polarized light. monosodium urate crystals are needle
Taurog m. Chhabra A. Colbert RA. Ankylos>ng spondylitis nnd Mia) spun- shaped and negatively birefringent Whereas extracellular
dyloarthrlti~. N Engl J Med. 2016:374 :2S63·7~. [PMID: 273SSS3SI
dol: 10.1056/NEJMra 1406182 crystals confirm a c hronic gout diagnosis, crystals within
neutrophils define active, gout-Induced inflammation.
[:::J Item 6 Answer: C Even in the setting of acute gout, Gram stain and cultures
must be obtained to exclude infection because acute gout
Educational Objective: Diagnose gastric antral vascular
and joint Infection occasionally coexist. 1his patient has
ectasia associated wit h diffuse cutaneous syste mic
not responded favorably to glucocorticoid therapy, which
sclerosis.
characterizes some seve re acute episodes. In a case such
1he most likely diagnosis is gastric antral vascular ecta- as this. an lnterleukln- 1 inhibitor such as anaklnra should
sia (GAVE) associated with systemic sclerosis. GAVE Is the be p rovided as a reliable (although expensive) off -label
proliferation of blood vessels typically In the antrum of the treatment.
stomach; on endoscopy, it has the appearance of water- Colchic ine is unlike ly to be effective for an established
melon stripes (watermelon stomach). Approxi mately 60% attack of greater t han 12 to 24 hours' d uration. It is also
of patients w ith GAVE have an unde rlying autoimmune unlikely to be useful when intravenous and intra-articular
disease; t he remainder have portal hypertension secondary glucocorticolds have been ineffective.
to hepatic cirrhosis. GAVE ca.n be a source of both acute and 1he absence of a response to intra-articular or intrave-
chronic gastrointestinal bleeding. First-line therapy Is argon nous glucocorticolds necessitates an alternative treatment;
plasma coagulation or laser coagulation. however, NSAIDs would not be as effective as t he two pre-
Cameron lesions are erosions found o n the crest of gas- viou sly provided fa iled alternatives. Additionally. NSAIDs
tric folds within a large hiatal he rnia and are thought to be have numerous side effects that migh t be even more likely to
caused by mechanical trauma as the hiatal hern ia slides up occur during critical illness. and they are relatively contra-
and down. Up to 5% of patients with known h iatal hernias indicated in a patient with heart fa ilure.

111
Answers and Critiques

rr1 Blood. urine. and synovial fluid cultures have all been UY POINT
1.1.1 negative. the pneumonia Is resolving, and there Is no clinical • Physical therapy can be prescribed at any palm in the
CONI. evidence of infect ion. Vancomycin for methicillin -resistant
course of osteoarthritis instead of medication, as a
staphylococcus is unnecessary.
supplement to medication that does not adequately
KEY POINT reduce pain, or prior to surgery to increase strength
• For patients with severe and refractory goury attacks and potentially Influence surgical outcomes.
or with comratndlcatlons to other treatments. off
label use oflnterleukln llnhibitors (anakinra or Bibliography
Hochberg MC. Altman RO, April KT, Denkhaltl M, Cuyntt C. M~Cowan J,
canaklnumab) can be considered. et nl; American College of Rheumntology. Amerlcun College or
RheUITilllnlogy 2012 rt'mmm~nd~tloM ror the U'<C uf nonpharmnt~lluglc
:tnd phmnacoiO)IIC: thel"olpies in osteoorthrllis ol' th~ hand, hlp, and kn«.
Bibi.Jography Arthrirls CaJ'e Res (Hoboken) 2012;64:46S-74. [PMIO: 2'2563589]
Thucrtn~r n: Ooll NK, c~rtn~r E. An~klnlll forth~ tre~trnmt of acute: severe
gout In crltlc:llly Ill patients. Semlll Arthr111s Rheum. 2015;45.81 5
[PMIO: 25795473) dol: tO t0161].scmarthrit.2015 02.006 Answer: C
ltem9
Educational Objective: Diagnose an.kylosing spondylitis
usingMRl
Item 8 Answer: C
Educational Objective: Treat osteoarthritis with The most appropriate diagnostic test to perform next Is MRI
physical therapy. of the sacrolliac joints. This patient has symptoms suggestive
of back pain due to intlanunation. including young age of
Physical therapy Is appropriate for this patient whose his- onset, gradual onset, pain during the night. morning stiff-
tory, physical examination, and radiographic findings are ness, Improvement with motion, no history of trauma, and no
consistent w ith the diagnosis ofosteoarthnlls (OA). Physical improvemen t with rest A slngle anteroposterior pelvis plain
therapy Is an effective intervention for the management of radiograph to view the sacroiliac joints is an appropriate first
pain and reduced functioning due to OA, with numerous diagnostic step in this setting, which may reveal joint space
guidelines supporting exerctse as an appropriate interven- widening (early) or narrowing (late), erosions, sclerosis, and
tion for all patients with OA. Evidence Is most robust for ankylosis, and can establish the diagnosis of ankylosing spon-
knee OA. Many patients become sedentary due to their dylitis. However, p lain radiographs may be normal early ln the
symptoms, and physical therapy Is often a useful starting course of disease, as seen in this patient. MRI of the sacroUiac
point to transition patients to participation in a regular joints can then be utilized, which is more sensitive for detecting
exercise program. Physical therapy can be prescribed at any early spine and sacroiliac joint intlammatlon. The finding of
point In the course of the disense instead of medication. as n bone marrow edema on STIR or T2-weighted images with fat
supplement to medication thnt does not adequately reduce suppression is not specific for ankyloslng spondylitis but does
pain, or prior to su rgery to Increase strength and potentially suggest actiVe inflammation, particularly if found in charac-
Influence surgical outcomes. teristic periarticular and subchondral locations. MRI can also
Glucosamine supplements are the most widely used Identify soft-tissue evidence of Inflammation in the entheses,
over-the-cou nter products worldwide for OA. Many ran- bursae, and tendons. as well as subtle structural abnormalities.
domJzed controlled trials have found that pain, function . and Bone scan Is sensitive for the detection of osteoblast
radiographic progression Improve overall at a level equivalent activity and is the study of choice for detecting bone metas
to the effects of placebo In patients taking glucosamJne. tases from various forms of cancer. However, bone scan
Knee replacement surgery is effective for treatment of is not used in the diagnosis of any form of inflammatory
pain and disability but is not appropriate In a patient who arthritis because It lacks specificity.
has not been through a trial of physical therapy, particularly cr of the Lumbar spine can be useful in the diagnosis of
a patient who Is not regularly exercising. Assessment of the ankylosingspondylitis when radiography oft he sacroiliac jolnts
ability to carry out activities of dally IMng, as well as occu- is nom1nl. However, lt is not routinely used in the evaluation of
pational and recreational activities, will be important in the patients for arthritis due to the high dose of radlatlon required.
decision to seek surgery, and a physical therapist can be par- Hlp pathology frequently results In anterior groin paln
ticu larly helpful to the prtmary care provider 1n carrying out and would be unlikely to cause back pain. Therefore, radiog-
this assessment. ConslderaUons for surgical referral include raphy of the hip joints is not likely to aid in the diagnosis and
the presence of pain at rest or pain that awakens the patient Is not Indicated for this patient.
in the middle of the night, both of wh ich are less likely to
respond to analgesics, exercise, or physical m odalities. ICIY POINT
Prednisone Is an oral glucocorticoid that can be used as • M Rl is more senslrive than rndiography for detecting
a n adjunctive medication In the treatment of some forms of early spine and sacroiliac JOint inflammation and may
inJlammatory arthritis. However, insufficient evidence exists be Indicated in the evaluation of suspected spondy·
to support the use of oral glucocorticoids In the treatment loarthritls if radiographs are normal.
ofOA.

112
Answ e rs a nd Critiques

BibUography phenomenon, n onerosive inflammatory arthritis, constitu-


Sieper J. Poddubnyy 0 . Axial srondylo<~rthrtris. Ulncer. 2017. (PMID: tional findings such as low-grade fever, and mechanic's hands.
281109UlJ doi:IO.HH6/SOl40· 6736(l6)31S91 -1
Mechanic's hands Is a dermatologlc manifestation unique to
antisynthetase syndrome and is characterized by hyperker-
Item 10 A nswer: E atotic sl<in along the ulnar aspect of the thumb and radial
aspects of the dlgits, most commonly on the index and rruddJe
Educational Objective: Avoid serologic testing in patients
fingers, and involvement of the palms. Unlike eczema, there
with low pretest prolnbility of conne<:tive tissue disease.
is no pruritus, vesicles, or hand dominance. Antisynthetase
No further testing is currently needed in this 75-year~ld man syndrome can occur in patients with dermatomyositis or poly-
with no clinical findings suggestive of a connective tissue myositis; about one third of patients with dermatomyositis
disease. His symproms of hand joint pain are noninflammatory belong to this subset cha ractertzed by a relatively abrupt onset
(pain with use, no warmth or swelling, and only minimal mom- and features. Antinuclear antibodies (ANA) are often positive
ing stiffness), and the distribution and findings are consistent in. patients with antlsynthetase syndrome, but anti-amino-
with a diagnosis of osteoarthritis. Furthem10re, the radlogt-aphs acyl -tRNA synthetases antibodies, including the subset anti-
confirm these findings and do not show changes consistent Jo-1 antibodies, are more specific for the diagnosis.
with an Inflammatory rheumatologic disorder (such as erosive In patients with Sj()gren syndrome, constitutional find-
joint disease seen in rheumatoid arthritis and psoriatic arthritis, ings and inflammatory arthritis are common, but Raynaud
or tophi seen in gout). Therefore, no further testing is indicated phenomenon is tess so. Rash and mechanic's hands are not
Positive antinuclear antibodies (ANA) in a patient with characteristics. Furthermore, dry eyes and dry mouth are
nonspecific symptoms are difficult to interpret. As with the most common presenting symptoms, along with positive
all tests, the positive predictive value of ANA rests upon anti-Ro/SSA and anti-La /SSB antibodies.
the pretest probability of disease. In the presence of 0 or Fever, Raynaud phenomenon, rash, inflammatory
1 clinical manifestations of systemic lupus erythematosus arthritis, and positive ANA are seen in systemic lupus
(SLE), a positive ANA is associated with a very low posttest erythematosus. However, an abntpt onset rarely occurs,
probability ofSLE. The American College of Rheumatology's mechanic's hands are not a teature, and anti-Jo-1 antibodies
Choosing Wisely list currently recommends against testing are not characteristic.
ANA and ANA subserologtes without a clinical suspicion of In patients with systemic sclerosis, fever and inflam-
Immune-mediated disease. The clln1cal suspicion for a con- matory arthritis are far less likely. Furthermore, although
nective tissue disorder such as SLE or rheumatoid artluitis Raynaud phenomenon is very common in systemic sclero-
is low in this patient, and ANA, rheumatoid factor, or anti- sis, the findings of either localized or diffuse skin tighten -
cyclic cltrullinated peptide antibodies should not be tested. ing and thickening, including sclerod actyly, are absent in
Osteoarthritis, psoriatic arthritis, and gout can involve ells- this patient. Mechanic' s hands is not a feature of systemic
tal interphalangeal joints. However, this patient does not have sclerosis, and anti -Jo -1 antibodies are not characteristic.
the rash seen in psoriatic arthritiS or symptoms of crystal-in-
KEY POINT
duced arthrltis such as recurrent acute attacks and/or topha-
ceous deposits. Serum urate levels are helpful in a patient with • Antisynthetase syndrome is characterized by intersti-
symptoms suggestive of gout but are unlikely to helpful here. tial lu ng disease. myositis. Raynaud phenomenon,
nonerosive inflammatory arthritis. constitutional find-
KEY POINT
ings such as low-grade fever, and mechanic's hands;
• The positive predictive value of antinuclear antibody anti-a minoacyl-tRNA synthetases antibodies. such as
(ANA) tests is determined by the pretest probability of anti-Jo-1, are highly suggestive of the diagnosis.
disease: in the presence ofO or 1 clinical manifestations
of systemic lupus erythematosus (SLE), a positive ANA Blbllograpby
is associated with a very low posttest probability ofSLE. L.ega JC, P.lblen N, Reynaud Q, Durleu I. Durupt S, Outertre M, et al. The
clinical phenotype associated w ith myositis-specific ond associated
3Ut<>anllbodies: a meta-nnotysis revisiting U11~ so-called antlsynthetase
Bibliography syndrome. Auto lmmun Rev. 2014:13:863·9l . [PM ID: 24704867]
R<>llblns M. Dnfkh 0, Beall A. Yelin E. et al: American
V117.d~ny J. Schmajuk G, doi:10.1016/j.autrev.2014.03 .001
College of Rheumatology Core Membership Croup. Choosing wisely: the
American College of Rheumatology's top 5 list of things ph)ISlcians and
patients should question. Arthrl!1s Care Res (Hoboken). 2013;65,329-39. Item 12 Answer: D
[PMID: 23436818] doi:IO.l002/acr.21930
Educational Objective: Prevent rheumatoid arthritis
with smoking cessation.
It em 11 Answer: A
The most preventive measure for thls patient at risk for
Educational Objective: Diagnose antisynthetase
developing rheumatoid arthritis (RA) is smoking cessation.
syndrome.
Sm oking has been d efinitively identified as a risk factor for
The most likely diagnosis is antisynthetase syndrome, which the development of RA, especially in genetically susceptible
is characteri1..ed by interstitial lung disease, myositis, Raynaud individuals. This woman has a strong family history of RA,

113
Answers and Critiques

including an identical twin sister who was recently diagnosed. This patient has no fever or sputum production, and
Smoking increases the risk for RA 2 to 21 times depend- testing shows a normal leukocyte count and differential,
ing on the presence of one or two copies of the shared epi- normal C-reactive protein (CRP). and normal procalcltonin,
tope in the H LA makeup of the individual. Data suggest that making infection less likely. CRP Is a marker of inflam-
if she stops smoking now, the risk for developing RA will mation, and levels are significantly increased ln bacterial
decline each year that she does not smoke and will eventually pneumonia but frequently normal or only slightly elevated
be 30'X, lower than if she con tinued to smoke. but will not in SLE flares. Procalcitonin is produced by cells as a response
reach the level of someone who has never smoked for up to to bacterial toxins. which result in serum procalcltonln ele-
20 years. There is additional evidence that if she stops smoking vations in bacterial infections.
now and does develop RA, future mortality will be favorably The patient reports no hemoptysis, and lack of ane-
affected, as will her disease activity. Smoking stimulates the mia and absence of erythrocytes on bronchoalveolar lavage
production of enzymes that modify arginine to citrulline, and exclude a diagnosis of diffuse alveolar hemorrhage.
the modified proteins containing citrulline are targets for the Shrinking lung syndrome presents with chronic insid-
immune system in susceptible lndividuals. This patient's hand ious shortness of breath with low lung volumes possibly
symptoms seem more mechanical in nature (that is, end of the related to diaphragmatic dysfunction. This patient's acute
day) rather than Inflammatory (prolonged morn! ng stiffness). presentation and diffuse Infiltrates on imaging is not consis-
High-fructose corn syrup has been identified as a possi- tent with thiS syndrome.
ble risk factor for hyperuricemia and gout but not RA.
KEY POINT
The use of hydroxychloroquine to prevent the develop-
ment of RA in susceptible individuals is unproven. A clinical • Lupus pneumonitis is a rare but severe presen tiltion of
trial will soon evaluate the impact of hydroxychloroquine in systemic lupus erythematosus characteril.ed by s hortness
patients positive for anti- cyclic citrullinated peptide anti- of breath, hypoxia. and d iffuse pulmonary infiltrates.
bodies, but it is premature to recommend this intervention
without supporting evidence. Bibliography
To date, probiotics have not been shown to modifY risk Mittoo S. r-euCO. Pulmonary man ifestations of systemic 1\lpus erythemato-
s us. Semin Resplr Crit Care Med. 2014:35:249·54. [PMIO: 2~668539]
forRA doi:10.10SS/s-0034-1371537
KEY POINT
Item 14 Answer: D
• Smoking is a risk factor for the development of rheum a
toid arthritis, especially in genetically susceptible individ- Educational Objective: Diagnose small intestinal
uals; all patients should be counseled to quit smoking. bacterial overgrowth associated with systemic sclerosis.
The most lil<ely diagnosis is small intestinal bacterial over-
Bibliogrnphy growth (SlBO) associated with systemic sclerosis (SSe). More
m G!nseppe 0. Or.;lul N. AlFredsson L Askling J. Wolk /1. Cigarette smoking than 70% of patients with SSe have clinical gastrointestinal
and smoking cessation in relation to risk ofrheumatold arthriti~ in women.
Arthlitls ResTher. 2013;1S:RS6. [PMID: Z36078lS) do~!O.U8613r42:18 involvement. Gastrointestinal motility is compromised in 40%
to 90% of patients with systemic sclerosis, especially those
[::J Item 13 Answer: A with diffuse disease. Because of the decrease in motility of
the small bowel, bacterial overgrowth occurs and leads to
Educatio na l Objective: Diagnose acute lupus
the symptoms described in this patient history. including
pnewnonJtls.
diarrhea, bloating, and pain, and can lead to malabsorption.
Acute lupus pneumonitis is the most likely diagnosis ln this Patients with SSe can also develop chronic pancreatic Insuffi-
patient who has difficult-to-control systemic lupus erythema- ciency and develop symptoms similar to SIBO, which must be
tosus (SlE) requiring aggressive therapy with prednisone and considered in the differential diagnosis. Diagnosis ofSffiO can
azathioprine. Lupus pneumonitis is a rare but severe presen- be confirmed with glucose hydrogen breath testing or jejunal
tation of SLE characterized by shortness of breath, hypoxia. asplrate cultures. Treatment is with rotating antibodies to try
and diffuse pulmonary infiltrates. Other major entitles in the to reduce the overgrowth using agents with both aerobic and
differential diagnosis include infection and diffuse alveolar anaerobic coverage. Probiotics may have some benefit in such
hemorrhage. He has clear evidence of active SLE with poly- patients. It is important to screen such patients for nutritional
arthritis, a high erythrocyte sed.Jmentatlon rate, hypocom- deficiencies.
plementemla. and d iffuse pulmonary infiltrates on radiologic Carcinoid tumors are neuroendocrine tumors arising
studies. as well as lymphocytic predominance on broncho- from the aerodigestlve tract. Although most neuroendocrine
alveolar lavage. These findings are very suggestive of lupus tumors are hormonally nonfunctionlng, a few produce sero-
pneumonitis. It usually requires rapid and aggressive therapy tonin and are responsible for the clinical manifestations of
with glucocorticoids and/or Immunosuppressive agents. the carcinoid syndrome characterized by diarrhea and facial
Pulmonary hypertension does not present with d iffuse flushing. The diarrhea associated with the carcinoid syn-
infiltrates with ground glass opacities, as seen In this patient, drome is secretory, and symptoms are not confined to peri-
and It cannot explain the bronchoalveolar lavage findings. ods of food intake; nocturnal diarrhea Is a clinical hallmark.

114
Answers a nd Critiques

Chronic mesenteric ischemia is almost always associ- extremities and (rarely) splenomegaly. Sarcoidosis does not
ated with atherosclerotic disease. Symptoms consist of post- cause leukopenia or cutaneous ulcers and Is an unlikely
prandial pain within 60 minutes after meals, which results cause of this patient's findings.
in fear of eating an d weight loss. It Is not associated with Joint involvement occurs in 90% of patients with sys-
postprandial bloating or explosive diarrhea. temic lupus erythematosus (SLE). with inflammatory poly-
Irritable bowel syndrome is a symptom complex char- arthralgia the most common presentation. Frank arthritis
acterized by abdominal pain and a ltered bowel habits. Diag- occurs in 40% of patients with SLE. Both small and large
nosis is establlshed by fulfilling specific diagnostic criteria, joints can be affected. Persistent periarticular inflamma-
Including recurrent abdominal pain or discomfort at least tion can damage soft- tissue structures that support joints,
3 days per month in the last 3 months associated with two or resulting In reducible subluxation of the digits, swan neck
more of the following : impro\lement with defecation; onset deformities. and ulnar deviation (Jaccoud arthropathy). All
associated With a change in frequency of stool; and onset three bone marrow cell lines can be affected In SLE. Leu-
associated with a change in form (appearance) of stool. The kopenia occurs in SO% of patients, with lymphopenia pre-
presence of •red flags" such as anemia, fever, or weight loss dominating. Subcutaneous nodules are not usually found In
strongly suggests the presence of an alternative diagnosis. SLE.ln addition, up to 90% of patients withSLE have skin
(acute, subacute, or chronic lupus) or mucous membrane
KEY POINT
Involvement. The presence of splenomegaly and subcutane-
• In patients with systemic sclerosis, gastrointestinal ous nodules and absence of skin rash argue against the diag-
dysmotility can result in small intestinal bacterial nosis of SLE. In addition, serologies would help distinguish
overgrowth with resultant chronic diarrhea and RAfromSLE.
malabsorption.
UY POINT
BlbHography • Felty syndrome consists of the triad oflong-standing
Gyger 0. Baron M. Systemic sclerosis: gastrointestinal disease and its man- aggressive rheumatoid arthritis, neutropenia, and
agement. Rheum Dis Cltn North Am. 2015:41:459-73. (PMID: 26210129) splenomegaly and is associated with the risk for seri-
doi:J0.1016/j.ldc.2015.04.007
ous infections. lower extremity ulcers. lymphoma,
and vasculitis.
Item 1 5 Answ er: B
Bibliography
Educational Objective: Diagnose Felty syndrome. Owlla MB. Newman K, Akhtar! M. Felty's syndrome, Insights and updates.
Open Rheumatol J. 2014 Dec 31; 8:129-36. (PMCIO: PMC42964nl dol:
The most likely diagnosis is Felty syndrome, which consists 10.217411874312901408010129.
of the triad of long-standing aggressive r heumatoid arthri-
tis (RA). neutropenia, and splenomegaly. Patients with Felty
syndrome are almost always seropositive. 1hls patient has the
typical findings oflong-standingRA, including imolvement of
Item 16 Answer: C
Educational Objective: Diagnose Lyme arthritis.
Cl
multiple small joints ofthehandsand feet, joint subluxation,
and subcutaneous nodules. Felty syndrome Is associated with The most likely diagnosis is Lyme arthritis. which can be con-
the dsk for serious infections, lower extremity ulcers, lym- firmed by serologic testing for detection of Borrelia bu rgdor-
phoma, and vasculitis. Treatment of Felty syndrome consists ferl-specific antibodies using a two-tiered approach: enzyme-
of more aggressive therapy for the underlying RA. linked lmmunosorbent assay (El..JSA) foUowed by Western
AA amyloid results from accumulation of the AA pro- blot.1his patient is a park ranger in an area endemic for Lyme
tein, an acute phase reactant seen in chronic inflammatory disease. thus the diagnosis should be strongly suspected,
diseases. Although AA amyloidosis can be associated with especially given a m onoarthritis of the knee that is not overly
hepatosplenomegaly, the most common organ involved in painful. which is typical for Lyme arthritis. Lyme arthritis is
AA amyloidosis is th e k idney and is evidenced by heavy pro- a late manifestation of Lyme disease; after the first month c:l
teinuria. Hematologic abnormalities may include anemia as Infection. at least SilO bands should be present on Western blot
the result of chronic k idney disease and thrombocytopenia testing for lgG antibodies to different B. bwgdorferiproteins. It
secondary to splenomegaly, but neutropenia is unusu al. AA is not uncommon for patients to be unaware of or experience
amyloidosis cannot account for this patient's joint findings manifestations of early stages of d isease, such as erythema
or lower extremity u lcer. migrans or constitutional symptoms.
Sarcoidosis most commonly affects the lungs. Severe Positi\le HI.A-B27 haplotype may be seen in patien ts
sarcoidosis can uncom monly be associated with a chronic who have spondyloarthritis such as ankylosing spondylitis.
arthritis that ls typically polyarticular, Involving the shoul- psoriatic arthritis. or reactive arthritis. Although these con-
ders, hands, wrists, knees, and ankles, and often coexists d itions commonly involve the peripheral joints. including
with lung and cutaneous sarcoidosis. An entire digit may be the knees, th e arthritis is usually painful and ollgoartlcular.
affected, leading to dactylitis. Sarcoidosis can also be associ- not monoarticular. AdditionaUy, In flammatory symptoms
ated with slightly tender subcutaneous n odules of the upper such as morning stiffness are prominent.

115
Answe rs an d Critiques

MRl of the knee can be useful to diagnose meniscal, mass In the absence of lymphadenopathy, and neither germ
CJ ligamentous, and other soft- tissue abnormalities, as well cell tumor nor lymphoma can account for the fibrous goiter.
CONT. as synovitis and effusion. It is particularly useful to localize Mesothelioma Is a highly lethal disease that most com-
knee pathology in the clinical setting of trauma, which has monly Involves the pleura but also the peritoneum. The most
not occurred in this patient. Degenerative meniscal tears due frequently noted symptoms are abdominal pain, weight loss,
to knee osteoarthritis may be seen on MRl, without a clinical and Increase in abdominal girth due to ascites. Malignant
history of trauma, but this patient does not have clinical or peritoneal mesothelioma does not present as a retroper-
radiographic symptoms/signs of osteoarthritis. itoneal mass and does not cause urinary obstruction or a
In crystal arthropathies, metabolic abnormalities pro- fibrous thyroid goiter.
mote the formation and deposition of crystals that stimulate
KEY POINT
inflammation. Symptoms are typically of an acute painful
arthritis, which is not present in this patient. Synovial fluid • lgG4-related disease is characterized by JgG4-
analysis for crystals Is therefore not indicated. producing plasma cell infiltration and tumefaction of
the affected tissue with resultant organ enlargement,
KEY POINT
fibrosis, and dysfunction.
• Chronic monoarticuJar arthritis with a large effusion
and stiffness but minimal pain is characteristic of Bibliography
Lyme arthritis; diagnosis is made by serologic testing Khosroshahl A, ~rruthm MN. Stone JH. Shlnagare S. Salnanl N, Hasserjlan
(enzyme-linked immunosorbent assay followed, if RP. et al. Rethinking Ormond's disease: "idiopathic" retroperitoneal
fibrosis In rtte era of lgG4 -related disease. M"edfclne (Baltimore).
positive, by Western blot). 2013:92:82-91. [PMIO: 23429355] dol:IO.I097/MD.Ob013e318289610f

Bibliography
Arvikar SL. Steere AC. Diagnosis and treatment of Lyme arthritis. Infect Dis
a1n North Am. 2015;29:269-80. (PMID: 25999223] doi:lO.l016/J. Item 18 Answer: 8
idc.2015.02.004 Educatio na l Objective : Diagnose limited cutaneous
systemic sclerosis.

CJ Item 17 Answer: B The most likely diagnosis is limited cutaneous systemic


sclerosis (LcSSc). This patient has Raynaud phenomenon
Educational Objedive: Diagnose lgG4-assoclated
and abnormal nallfold capillary changes that suggest that
retroperitoneal Hbrosls.
the Raynaud disease is part of the features of an underlying
The most llkely diagnosis Is lgG4-associated retroperitoneal connective tissue disease. Raynaud phenomenon (sequen-
fibrosis. Most of the conditions that are Wlder the banner tial white, blue, and red color changes in the digits precip-
of IgG4-related disease (lgG4-RD) are characterized by Itated by co1d or stress) occurs in almost all patients with
IgG4-producing plasma cell Infiltration and tumefaction of systemic sclerosis (SSe). Raynaud phenomenon in LcSSc is
the affected tissue with resultant organ enlargement, fibrosis, Initially transient and reversible; later, structural changes
and dysfunction. Patients commonly present with a sentinel develop within small blood vessels, resulting in perma-
organ enlargement, but careful evaluation often reveals more nently impaired flow that produces acrocyanosis, digital
extensive disease. Clinical signs include painless enlargement pitting. and/or ulcerations. Office-based examination of the
of lymph nodes or the thyroid, parotid, or submandibular nailfold capJllaries of patients with SSe using a dermato-
glands; proptosis with orbital pseudo-tumor; back or chest srope or ophthalmoscope reveals both capillary destruction
pain from aortic involvement; and abdominal pain from pan- and dilated capillary loops, which can distinguish SSe from
creatic or biliary tree disease. This patient presents with a primary Raynaud. Early changes in naUfold capillaries in
classic picture of retroperitoneal fibrosis with back pain and the setting of a connective tissue disease include dilata-
kidney failure from the periaortic mass that Is large enough tion. In the sclerodenna spectrum disorders, nailfolds can
to encase the ureters resulting In obstructive uropathy and demonstrate capillary dropout and then more disorganized
kidney Injury. In a recent study, almost 60% of patients with loops as the vasculopathy progresses. Elevated antinuclear
retroperitoneal fibrosis were foWld to have lgG4-RD. Some antlboclles In a centromere pattern occur In 20% to 40% of
patients present with an inflammatory aortitis as well. The patients with SSe and have a 90% specificity for the disease,
patient's history of a fibrotic thyroid gland Is also a feature of and tn particular for LcSSc. The presence of anticentromere
lgG4-RD. antibodies also increases the risk for developing pulmonary
Young men with germ cell tumor can present with arterial hypertension (PAH). Thls patient will need to be
bulky retroperitoneal or mediastinal lymphadenopathy. A monitored for progression, including PAH screening.
testicula r mass may not always be present. Similarly. involve- Anticentromere B antibodies are not seen In patients
ment of retroperitoneal, mesenteric, and pelvic nodes Is with dennatomyosJtts o r mixed connective tissue disease,
common In many types of non-Hodgkln lymphoma and neither of which Js the likely diagnosis in thls patient. The
can lead to urinary tract obstruction. It is u ncommon for pathognomonic cutaneous features of dermatomyositis are
either of these conditions to present as a retroperitoneal the heliotrope rash and Gottron papules, which are absent

116
Answers and Critiques
---~---

in this patient, as is another key feature, proximal muscle lgA vascul!tls can cause testicular Involvement,
weakness. mononeuritis, and rash (palpable purpura), but it affects
Mixed connective tissue disease (MCTD) is an overlap small vessels and causes glomerulonephritis rather than
syndrome that includes features of systemic lupus erythe- renovascular involvement.
matosus (SLE), SSe, and/or polymyositis In the presence of Microscopic polyangiitis (MPA) is an ANCA-assoclated
antt-Ul-ribonucleoprotein antibodies. More than 50% of vasculitis affecting small vessels. It can cause neurologic
patients with MCTD have hand edema and synovitis at dis- involvement and skin rash, but kidney involvement Is due
ease onset, which is not present in this patient to glomerulonephritis rather than renal artery vasculitis.
Anticentromere B antibodies are rarely seen in SLE (4% Furthermore, MPA ls associated with a positive ANCA with
or less), and this patient does not have any findings asso- perinuclear pattern, directed against myeloperoxidase. This
ciated with this disorder. SLE should be considered in any patient does not have a positive ANCA or glomerulonephri-
patient who presents with unexplained multisystem disease. tis, making MPA an unlikely diagnosis
The most common early SLE manifestations include con-
UV POINT
stitutional symptoms (fever. weight loss, or severe fatigue),
arthralgia/arthritis, and skin disease, all of which are absent • fulyarteritls nodosa ls a vasculitis affecting medium-
ln this patient. sized arteries and Is characteriZed by constitutional
and neurologic symptoms. skin rashes. and kidney
KEV POINT
involvement that Is renovascular rather than glomeru-
• Elevated antinuclear antibodies In a centromere pat- lar in origin.
tern have a 90% specificity for systemic sclerosis. and
in particular for limited cutaneous systemic sclerosis. BlbHography
De Vlrglllo A, Greco A, Magllulo G. Gallo A, Ruoppolo G. Conte M, et al
Bibliography Polyarteritis nodosa: a contempomry ovelView. Autolmmun Rev.
2016;15:564-70. (PMJD: 26884100] dol:l0.1016/j.autrev.2016.02.01S
Mehm S. Wall<er J, P'oittem>n K. Fritzler MJ. Autoantibodies In sysremJc scle-
rosis. Autolmmun Rev. 2013;12:340·54. [PMID: 22743034) dol:l0.1016/
j. autteV.20J2.0S.Oll
&tem 20 Answer: E
EducationaJ Objective: Treat osteoa.rthrids In an elderly
Item 19 Answer: 0 patient using a topical NSAID.
Educational Objective: Diagnose polyarteritis nodosa
Topical diclofenac is the most appropriate treatment for
The most likely diagnosis Is polyarteritis nodosa (PAN), a long-term symptom control for this elderly patient with
vasculitis affecting medium-sized arteries. This entity may knee osteoarthritis (OA). Acetaminophen has not provided
occur in the setting of chronic hepatitis B virus infection, relief; furthermore, the efficacy of acetaminophen for OA
HIV Infection, and haizy cell leukemia. 1he most com- is increasingly being questioned because recent controlled
mon symptoms are constitutional, Including fever, mal- tr1als and meta-analyses demonstrated no benefit from
aise, and weight loss, and neurologic symptoms such as the drug, even at high doses. NSA1Ds are available In oral,
mononeuritis multiplex. Skin rashes, including purpura topical, and intravenous forms. Due to efficacy and cost-
and necrotic ulcers, occur in more than half of patients. effectiveness, oral preparations are usually first-line NSAID
Kidney involvement manifests as hypertension due to therapy In patients without contraindications to treatment.
renal artery vasculitis with renal infarction, not glomer- In OA, topical NSAlDs are considered to provide similar
ulonephritis. Orchitis, an uncommon manifestation, is pain relief as oral medications with fewer gastrointestinal
usually unilateral and due to testicular artery involve- effects. Furthermore, the American College of Rheuma-
ment. Mesenteric vasculitis may cause abdominal pain, tology currently recommends topical NSAIDs rather than
peribrat!on, and bleeding. This patient has all of these oral NSAIDs for patients aged 75 years or older. However,
findings except for mesenteric involvement. DiagnosiS they are associated with more skin reactions and are sig-
of PAN is best established by demonstrating necrotizing nificantly more expensive than oral NSAIDs. 1his elderly
arteritis In biopsy specimens or finding characteristic woman with unilateral knee OA ls an ideal candidate for
medium-sized artery aneurysms and stenoses on imaging topical NSAIDs-she has just one area of musculoskele-
studies of the mesenteric or renal arteries. 1his patient's tal pain and has severn! relative contratndfcatlons to oral
renal angiogram confirms the presence of vasculitis and NSAIDs (age, chronic kidney disease, coronary artery dis-
skin biopsy confirms medium-vessel vasculitis, consistent ease, and aspirin intake).
with the diagnosis of PAN. When a single symptomatic joint is present, injection
Giant ceU arteritis causes myalgia and elevated eryth- directly Into the Joint may deliver medication to the affected
rocyte sedimentation rate but involves large vessels usuauy site while mintm12ing the potential for systemic effects.
in the head and neck area or great vessels of the chest. Intra-articular InJections may be used along with or in place
not medium-sized vessels. Furthermore, it usually does not of oral or topical analgesics. Intra-articular glucocortlcolds
Involve nerves or kidneys. are associated with short-term benefit with few side effects.

117
Glucocorticoid Injection alone would not likely proVIde presentation Is not that of stroke. Stroke cannot explain the
long-term benefit for this patient. patient's malaise, weakness, aching, and tenderness over the
Low-dose oral prednisone is associated with mod- left temporal area.
est reductions In short-term knee pain and may increase 1here are Insufficient data from randomized controlled
a 6-minute walk distance in older patients with moder- trials to recommend dally low-dose aspirin to prevent GCA
ate-to-severe knee OA. However, long-term use Is associated complications, and this therapy would be Inappropriate
with slgnlflcant complications, Including an increased risk without concomitant high-dose prednisone.
for diabetes mellitus, osteoporosis, osteonecrosis, weight Methotrexate may be helpful as adJunctive therapy
gain, fluid retention, hypertension, cardiovascular disease, for GCA and PMR, especially for those patients who have
striae and bruising, and glaucoma and cataracts. relapses and are unable to taper glucocorticoids. It is not
Oral naproxen and other oral NSAIDs are relatively appropriate initial therapy for GCA, especially as it will not
contraindicated given the patient's hypertension, chronic take effect for several weeks, and this patient requires Imme-
kidney disease. and coronary artery disease. diate treatment to prevent vision loss.
Traditional opiates may rarely be warranted to control Temporal artery biopsy Is the gold standard for diag-
pain in patients with OA who have not responded to other nosing GCA; at least 1 em Is required in order to reduce the
agents or are poor candidates for other interventions to treat false-negative rate because of skip lesions. However, gluco-
painful joints, such as surgery. Opiold analgesics are Inap- corticoid therapy should not be delayed in order to establish
propriate for this patient because other treatment options the diagnosis of GCA. If necessary. temporal artery biopsy
are available. In addition, opiotds have limited effectiveness can be obtained 1 to 2 weeks after glucocorticoids have been
for chronic Joint pain and are associated with substantiaJ initiated, and will still reveal diagnostic abnormalities.
side effects in the elderly. especially Increased fall risk, cog-
UY POIJIT
nitive changes, and constipation.
• Hlgll-dose prednisone must be initiated lmmediately
lEY POIIT
in patients with signs and symptoms that are highly
• Topical NSATDs are beneficial for patients at high risk suggestive of giant cell arteritis to prevent irreversible
for tox!dty from oral NSAIDs. VIsual loss.

Bibliography Blblfogmphy
Rannou F. Pelletier JP, Martef·Pelle!ler J. Efficacy and safety of topical Butrgerdt F. Dejaco C. Matteson El, Dasgupta B. Polymyalgla rheumatlca
NSA!Ds In the management ofosteoarthritis: Evlden~ from real-life set- and giant ceiJ arter!US: a systematic review. lAMA. 2016;315:2442··58.
ting trials and surveys. SemJn Arthritis Rheum. 2016;45:Sl8-21. {PMIO: [PM£0: 27299619) dol:10 .100lljama.2016.S444
26806189) doi:IO.l0!6/j.sernarthrlt.201S.ll.007

Item 22 Answer: 8
Item 21 Answer: 0 Educational Objective: Diagnose Inclusion body
Educational Objective: Treat giant ceU arteritis. myositis.
The most appropriate treatment Is prednisone, 60 mg/d. This The most likely diagnosis Is Inclusion body myositis (IBM), an
patient has signs and symptoms of giant cell arteritis (GCA), inilammatory myopathy that can in~lve both the proximal
Including headache, jaw claudication, VIsual changes, and an and distal muscles; although typically symmetric, muscle
elevated erythrocyte sedimentation rate, as well as symptoms of distribution may be asymmetric. Sporadic IBM Is the most
polymyaJgla rheumatica (PMR). GCA is the most common prl- common musde disease in elderly populations, is more com-
maryvasculitis. Median age of onset is70 years; GCA In patients mon In men than women, and typically develops after age so
under age so is rare. GCA characteristically affects the second- to years and does not appear to aifect Individuals younger than
fifth-order branches of the aorta Affected arteries Include the 4S years old. Serum creatine kinase levels are typically less
external carotids, temporal arteries, and ciliary and ophthal- than 10 to 12 times the upper limit of normal. Its Insidious
mic arteries. Subclavian and brachlal arteries can be affected. onset and distal muscle involvement help to distinguish IBM
Uncommonly. intracranial arteritis may occur. Up to SO% of from the other inflammatory myopathies. 1he absence of rash
patients with GCA have PMR that may occur prior to, concur- helps to exdude dermatomyositis. Electrom}Ugram may show
rent with, or following diagnosis of GCA. It Is imperative that myopathic, neurogenic, or mixed changes with both short
high-dose prednisone be Initiated without delay to prevent irre- and long duration motor unit potentials and spontaneous
ve-sfble vtsualloss from GCA; some advocate Intravenous pulse activity. Diagnosis of IBM Is conflnned by muscle biopsy
methylprednisolone forpatlentswlth slgnlflcant visual changes. showing muscle fibers containing multiple rimmed vacuoles.
Prednisone should generally be adminisrered for approximately Distinguishing IBM from polymyositis Is Important because
1 month (or untiJ resolution ofsigns and symptoms), with sub- IBM is usually not treated with Immunosuppressive therapy
sequent dose reduction at a rate of about 10% every few weeks. and bas an overall poor response to treatment.
CT of the head would be helpful In assessing for Amyotrophic lateral sclerosis (ALS) is a progressive,
stroke-induced vision changes, but this patient's clinical neurodegeneratlve disease that can have an Insidious

118
Answers and Critiques

onset causing muscle weakness. Upper and lower motor help to increase saliva flow. Regular intake or rinsing \vtth
neuron signs may be seen, including spasticity, abnormal small sips of water Is also recommended. Artificial saliva
gait, increased reflexes, and difficulty with coordination substitutes would be considered next If these approaches
(including manipulation of small objects), tripping, and are insufficient.
falling. Findings may lnJtlally be proximal or distal and Methotrexate Is a nonblologlc disease-modifYing anti-
asymmetric. ALS can be distinguished from IBM by the rheumatic drug occasionally used In SjOgren syndrome for
presence of fasciculatlons, and ALS does not raise serum arthritis. Because the patient's joint disease is unlikely to
creatine kinase levels. result In future bony damage and Is responsive to ibuprofen,
Mitochondrial myopathy can present in childhood, no additional arthritis management is needed at this time.
adolescence, or young adulthood, and a family history of There is no documented indication for methotrexate to treat
mitochondrial disease Is often eVident. When myopathy pre- sicca symptoms associated with SjOgren syndrome.
dornJnates, the patient generally presents with proximal Pilocarpine Is a muscarinic agonist that may stimulate
muscle weakness. However, exercise intolerance, painful saliva secretion. It is often poorly tolerated and would not be
and sometimes severe muscle cramping, and eye symptoms considered unless basic measures to treat oral dryness are
such as ptosis and diplopia are common. Serum creatine lnsutfident.
kinase levels are usually within the normal range, but serum Rltuximab depletes B cells and has shown early prom-
lactatt' levels may be elevated. ise for the management of SjOgren syndrome. Early studies
Polymyositis occurs more frequently in women than suggest a possible benefit for treating slcca, but It is generally
in men and has Its age of onset before 50 more often than reserved for patients with systemic Sjogren complications or
IBM. It generally presents with proximal, rather than distal, severe disease. At the present tlme, its use Is not warranted
muscle weakness, and serum creatine kinase levels can be In this patient.
substantially higher than those seen in IBM, often greater Topical ophthalmic NSAID drops are absolutely contra-
than 10 to 50 times normal. indicated in SjOgren syndrome because they carry risk for
severecomeal damage.
KEY POINT
• Inclusion body myositis OBM) is an inflammatory KEY POIIT
myopathy that can Involve both the proximal and dis- • lnitlal management of SjOgren syndrome typically
tal muscles with typically symmetric muscle distribu- imolves treatment of slcca symptoms by restoring
tion; its insidious onset and distal muscle involvement moisture of the eyes and mouth.
help to distinguish IBM from the other inflammatory
myopathies. BibUogapby
Baer AN, V&lltt B. SJOgren Syndrome and Other causes of Slt'Ol ln Older
Adults. Clln Ger!atr Med. 2017;33:87-103. [PMID: 27886700] doU0.1016/J.
BibiJography cger.2016.08.007
Needham M. Mastaglla FL. Spo.radlc lnduslon body reyosltis: a review of
recent clintcat adwn~ and current approaches 10 diagnosis and treat-
ment. Clln Neurophyslol. 2016;127:1764-73. (PMIO: 267787171 dol :
10.1016/J.cllnph.201S.l2.01l Item 24 Answer: C
Educational Objective: Diagnose sderltls in a patient
with rheumatoid arthritis.
Item 23 Answer: A
1he most likely diagnosis is scleritis In this patient with rheu-
Educational Objective: Treat slcca symptoms assoclated
matoid arthritis (RA). RA Is one oft he most common diseases
with Sjtlgreo syndrome
associated with sclerttis. Typical features include eye pam,
Artlfidal tears and sugar-free candies are the most appro- pain with gentle palpation of the globe, and photophobia. 1he
priate treatment at this time. 1his patient meets diagnostic deep scleral vessels are Involved and may lead to scleroma-
criteria for Sjogren syndrome, with objective documentation lacia, which Is characteriZed by thin.ning of the sclera and Is
of slcca in the presence of antl-Ro/SSA antibodies, antinu- seen as a dark area in thewhlte sclera. Scleromalacia may lead
clear antibodies, and rhewnatold factor. Her dental carries to perforation of the sclera called scleromalacia perforans.
and missing teeth are almost certainly due to her dry mouth, Scleritis can be vision-threatening and lead to blindness; it Is
as evidenced by lack of sall\'al)' pooling below the tongue. therefore important to urgently refer the patient to an oph-
She has arthralgia but no definite arthritis and, consistent thalmologist for care.
with SjOgren joint involvement, no evidence of erosive joint Conjunctivitis also causes a red eye. Typically, the
disease on radiograph. 1 he Immediate treatment goal of sicca underlying vessels are visible, a discharge may be seen, and
is the re-establishment of wetting of the eyes and mouth. often there is mattering of the eye In the morning. The eye
For the eyes, topical application of artificial tears Is usually may feel Irritated but there Is no pain or loss of visual acuity.
the first strategy, potentially followed by addition of topical ln general, conjwtctivitls Is a diagnosis of exclusion. The
cyclosporine or llfitegrast If artificial tears prove insufficient. presence of pain and decreased visual acuity exclude con-
For oral dzyness, sugar-free candy or chewing gum can junctivitis in this patient.

119
~nswers a~~ Crit iq ues

Episclerltis is an abrupt inflammation of the super- secondary to travel to endemic countries. In addition to fever,
ficial vessels of the episclera, a thin membrane that lies maculopapular rash, and headache, patients may experience
just beneath the conjunctiva. The cause Is often unclear; synovitis and tenosynovitis of the fingers and wrists that is
rarely, it is associated with systemic rheumatologlc dis- usually symmetric and may resemble rheumatoid arthri-
ease. Patients with episcleritis frequently present with- tis. The presence of a pustular rash and lack of travel to an
out pain or decreased Visual acuity. On examination, the endemic area makes this diagnOSis unlikely.
inflammation appears localized. White sclera can be seen Reactive arthritis is an acute, asymmetric arthritis that
between superficial dilated blood vessels. Episcleritis typ- typically follows chlamydia! urethritis or infectious gastro-
ically resolves spontaneously. The presence of pain, diffuse enteritis within 6 weeks. It mainly affects lower extremity
redness, and decreased visual acuity make episderitJs an joints. Enthesitis, particularly at the Achilles tendon, can
unlikely diagnosis. also occur. Patients also often have conjunctivitis, circinate
Subconjunctival hemorrhage is a common disorder and balanitis. or keratoderma blenorrhagicum. A pustular or
typically benign In origin. It is caused by painless bleeding vesiculopustular rash is not part of the reactive arthritis
into the superficial portion of the eye. Examination reveals syndrome.
a blotchy redness (from extravascular blood) that is typi- Acute sarcoid arthritis presents most often as part
cally confined to one area of the conjunctiva. Subconjunc- of LOfgren syndrome, which is characterized by the triad
tival hemorrhage is painless and not associated with loss of hilar adenopathy, acute arthritis, and erythema nodo-
of vision. Most cases resolve Within several weeks Without sum. The acute polyarthritis is mostly oligoarticular, with
intervention. The patient's findings are not compatible with symmetric involvement most commonly involving the
subconjunctival hemorrhage. ankles or other lower extremity joints. This is in contrast to
DGI, which is usually asymmetric and involves the upper
lEY POINT
extremity joints. Sarcoidosis may involve the skin, but
• Rheumatoid arthritis is one of the most common dis- lesions may be papular, nodular, plaque-like, or indurated
eases associated with scleritis, which can be vision- and deep.
threatening and lead to thinning of the sclera and
KEY POINT
perforation.
• The arthritis-dermatitis syndrome of disseminated
BibUography gonococcal infection due to Neisseria gonorrhoeae is
Artlfonl M. Rothschild PR. Bn!zln A. Cuillevln 1.. Puechal X. Ocular Inflam- characterized by a triad of tenosynovitis. dermatitis.
matory discoses associated with rheumatoid arthritis. Nat Rev and polyarthraJgia without frank arthritis; fever,
Rheumatol. 2014;10:108·16. (PMID: 24323074] doi:IO.l038/nrrheum.
2013.185 chills, and malaise are common.

Bibliography
Garcia-Arias M, Balsa A, Mola EM. &pUc arthriUs. Best Pract Res Clin
I:Jitem 25 Answer: B Rheumatol. 2011;25:407- 21. (PM!D: 221002891 dol:l0.1016/j.berll.
Educational Objective: Diagnose disseminated 201).02.001
gonococc;~J Infection.

The most likely diagnosis is disseminated gonococcal infec- Item 26 Answer:


tion (DGl). This patient has the arthritis-dermatitis syn-
D
Educational O bjective: Diagnose Ubman-Sacks
CJ
drome of DGI due to Neisseria gonorrhoeae. This syndrome
endocarditis.
is characterized by a triad of tenosynovitis, dermatitis (usu-
ally painless pustular or vesiculopustular lesions), and poly- Libman-Sacks endocarditis (nonbacterial thrombotic
arthralgia without frank arthritis. Fever, chills, and mal- endocarditis) is the most likely cause of this patient's tran-
aise are common. Inflammation of multiple tendons of the sient monocular blindness. She has an 8-year history of
wrists, fingers, ankles, and toes distinguishes this syndrome systemic lupus erythematosus (SLE) and positive antlphos-
from other forms of infectious arthritis. This presentation of phollpid antibodies (anticardiolipin antibodies plus lupus
DGf is associated with positive blood cultures and character- anticoagulant) with recurrent pregnancy loss. She meets
istic skin lesions. Diagnosis is established by blood and syno- the crlteria for antiphospholipid syndrome. Patients who
vial fluid cultures, as well as pharynx, cervix, urethra, and have SLE with positive antiphospholipid antibodies are at a
rectum cultures. and/or nucleic acid amplification testing. high risk for developing valvular dysfunction/thickening,
Testing patients and their sexual partners for both gonorrhea and in some cases manifesting as Ubman-Sacks endocar-
and chlamydia Is critical. Initial therapy for DGI Is usually ditis. A recent study confirmed this significant association
with ceftrtaxone plus either azithromycln or doxycycline, between valvular heart disease and anti phospholipid anti-
and Is rapidly effective. body positivity. It was also found that the highest risk was
Chlkungunya virus Is an alphavirus endemic to Asia seen ln double-positive anti phospholipid antibodies/lupus
and Africa, where It is transmitted by mosquito vectors; anticoagulant patients, as is the case in this patient. Libman-
however, cases are Increasingly seen in the United States Sacks endocarditis may a.ffect 11% or more of patients

120
Answers and Critiq ues

with SLE and has no relationship to disease activity. 1he Kawasaki disease initially presents in childhood, affects
Cl condition is associated with large verrucous lesions near medium-s ized (coronary) vessels, and can lead to heart fail -
CONT. the edge of the valve. most often the mitral valve. Typical ure due to myocardial damage years after the inflammation
lesions consist of immune complexes. mononuclear cells. from vasculitis has resolved. Kawasaki disease does not have
and fibrin and platelet thrombi. Libman-Sacks endocar- an onset In late adulthood.
ditis Is usually asymptomatic but can be responsible for Polyarteritis nodosa is a medium-vessel vasculilis that
numerous complications, including embolic stroke, or in rarely affects the heart, and when it does, involves coronary
this case a transient ischemic attack in the territory of vessels, not large vessels; this patient has a large-vessel vas-
the ophthalmic artery, peripheral emboli. and infective culitis affecli ng the aorta.
endocarditis. In Takayasu arteritis (TA), the affected arteries are
Bacterial endocarditis can be a source of cardiogenic primarily the aorta (ascending, descending thoracic, and
emboli. but 90'X.ofpatients with infective endocarditis have abdominal aorta) and its major branches. In contrast to
fever. and often other constitutional symptoms, which are GCA. TA Is rare (2 cases/million patient-years) and mainly
absent in this patient. affects young women (9:1 ratio) with a typical age at onset
Patients with SLE are prone to develop premature between 15 and 25 years. Histologically, the pathophysio-
atherosclerosis (and can be a cause of stroke or transient logic processes ofTA and GCA are similar, with infiltration
ischemia attack), but this patient has no findings of ath- ofT cells, macrophages. and giant cells in the vessel wall.
erosclerosis involving the carotid arte ry; furthermore, This patient's age effectively excludes TA as the cause ofher
this diagnosis cannot account for the patient's heart symptoms.
murmur.
K£Y POINT
Transient monocular blindness can be a manifestation
of giant cell arteritis, but this diagnosis is extremely rare in • Giant cell arteritis can affect the great vessels of the
individuals less than 40 years old. chest causing upper extremity claudication and/or
aortitis; aortitis may lead to aortic root dilation, aortic
KEY POINT
regurgitation. and heart failure.
• Patients with systemic lupus erythematosus and posi -
tive antiphosphollpid antibodies are at a high risk for Blbllography
developing valvular dysfunction/thickening, in some Conzala-Gay MA. Plna T. Giant cell arteritis and polymyalgla rheurnallca:
cases manifesting as Libman Sacks endocarditis. an update. Curr Rheumatol Rep. 2015:17:6. [PMID: 25618572.1 doi:IO.l007/
sll926· 014-0480- l

BibUography
Vl...ero F. Com:alez·Echavani C. Rulz-Estevez B. Maderuelo l, Rulz-trasto=
C. Prevalence and pmllctors of valvular heart disease In paUents wllh Item 28 Answer: 8
systernlc lupus erythematosus. Autotnunun Rev. 2016:15:1134-1140. Educationa l Objective: Diagnose a secondary cause of
!PMID: 276391571 dol:10.10161).autrev.2016.09.007
acute calcium pyrophosphate crystal arthritis.
A serum calcium measurement is most likely to iden-
C] ltem 27 Answer: C tify the cause of this patient's symptoms. This young
woman has recurrent attacks of acute calcium pyro-
Educational Objective: Diagnose subcranial giant cell
phosphate (CPP) crystal arthritis (pseudogout). Clues to
arteritis.
the diagnosis Include acute and recurrent self-l!mlted
1he most likely diagnosis is giant cell arteritis (GCA}, a monoarticular arthritis, chondrocalcinos!s on radiograph,
large-vessel vasculitis that most commonly affects ves- and, most telllngly, identification of CPP crystals under
sels in the head and neck area (cranial GCA), resulting polarizing microscopy. This condition usually affects the
in jaw claudication, headaches (from involvement of the elderly; therefore, evidence of acute CPP crystal arthri-
superficial temporal artery). and visual changes (due to tis in a young person should always prompt an Investi-
Involvement of ophthalmic arteries). Polymyalgia rheu- gation for secondary causes. Secondary causes include
matica (PMR). although not a vasculitis, is an inflamma- hyperparathyroidism, hypothyroidism, hypophosphatasia,
tory disorder that frequently accompanies GCA. Symptoms hypomagnesemia, and hemochromatosis. This patient has
of PMR (shoulder and hip girdle pain and stiffness) may recent-onset signs and symptoms suggestive of hyper-
precede, accompany. or follow the diagnosis of GCA. and calcemia and hyperpara thyroidism, including abdomi-
may be the only symptom of silent or subclinical GCA. As nal discomfort, constipation, and weakness. lherefore.
in this case. GCA less commonly affects the great vessels of measuring the serum calcium Is appropriate (and serum
the chest (subcranial GCA), causing upper extremity clau- parathyroid hormone, If hypercalcemia Is present),
dication and/or aortitis. as seen in this patient. Aortlrls may because this patient appears to have CPP deposition due
lead to aortic root dilation. aortic regurgitation. and heart to hyperparathyroidism.
failure, as seen in this patient; this manifestation is usually Anti-cyclic citrullinated peptide antibodies are use-
a late feature of GCA. ful In the diagnosis of rheumatoid arthritis (RA). RA

121
A nsw e rs and Critiques

typically presents with the Insidious onset of symmetric prednisone at 6 months was observed In 67% of ritux-
arthrit is in the hands a nd feet along with morning stiff- imab-treated patients compared with 42% of cyclophos-
ness. Monoarthrltls Is a rare presenting feature of RA, but phamide patients (P= 0.01; NNT = 4). Ritux.imab was also as
it is usually followed by development of the symmetric effective as cyclophosphamide in the treatment of patients
arthritis. with kidney disease or alveolar hemorrhage. There were no
Checking serum creatine kinase would not be a first- significant differences between the treatment groups with
line test for t his patient. Although muscle disease is associ- respect to rates of adverse events.
ated with weakness, myositis would not account for many In trials of etanercept treatment for GPA, patients man-
of the other manifestations, including the monoarticular ifested an increased risk for the development of solid malig-
arthritis, radiographic cbondrocalcinosls, and synovial fluid nancies, especially if they had previously been treated with
crystals. cytotoxic drugs. Therefore. etanercept is not considered first-
Serum urate would be an appropriate laboratory Une treatment for GPA.
study to measure if gout was s uspected, but the synovial Methotrexate Is Inadequate as Induction therapy for
fluid analysis is consistent with acute CPP crystal arthritis severe disease: it can be used alone either as maintenance
rather than gout, in which negatively birefringent crys- therapy after induction, or for mild and limited disease.
tals are seen . Gout also more typically affects the meta-
UY POINT
tarsophalangeal joints, the mid foot, ankles, and knees,
although the finger joints may be affected in postmeno- • Ritwd.mab is appropriate treatment for patients who
pausal women. experience a relapse of granulomatosis with polyangiitis.

KEY POINT
Bibliography
• Evidence of acute calcium pyrophosphate crystal Comarmond C, Cacoub P. Granulomatosis wltb polyangfltis (Wegener):
arthritis (pseudogout) in a young person should clinical aspects and treatment. Autoimnum Rev. 2014;13:U21-S. [PMID:
25149391} d oi:10.1016/j.autrev.2014.08.017
always prompt an investigation for secondary causes
such as hyperparathyroidism, hypothyroidism,
hypophosphatasia, hypomagnesemia, and hemo- Item 30 Answ er: 8
chromatosis.
Educational Objective; Manage disequilibrium caused
by pregabalin.
Bibliography
Rosenthal AK, Ryan LM. Calcium Pyrophosphate Deposltton Disease. The most appropriate next step in management is to dis-
N Engl J Me(t 2016;374:257S-84. [PM!D: 27355536] dol:l0.1056/
NE1Mra1Sllll7 continue pregabalin. Disequilibrium is an unsteadiness,
or sense of imbalance, with standing or walking. Patie nts
may experience dlsequillbrium as a side effect of medica-
CJitem 29 Answer: D tion use. This patient has fibromyalgia, which Is charac-
terized by widespread pain, fatigue, disturbed sleep, and
Educational Objective: Treat relapsed granulomatosis
cognitive dysfunction . Education, exercise, and psycho-
with polyangiitis.
social support are cornerstones of treatment, although
Rituximab Is the most appropriate treatment for this pharmacotherapy is often warranted. Pregabaltn can be
patient who has experienced a relapse of granulomato- effective for patients with fl.bromyalgia, but use is often
sis with polyangiitls (CPA), manifesting with pulmonary limited by side effects, including weight gain, periph-
Inflammation and nodules despite maintenance treatment eral edema, lethargy, and espec ially dizziness (31o/o of all
with azathioprine. GPA is a rare systemic autoimmune patie nts in trials versus 9% on placebo). The patient has
disease of unknown cause that leads to vasculitis and fa llen several Urnes: therefore, Jt is imperative to remove
granulomatosis of small- to medium-sized blood ves- any possible offending agent, and pregabalin Is the most
sels. Without treatment, most patients dle within I year. likely culprit among her medications. Once the troubling
With treatment, most patients achieve remission within s ide effects resolve off medlcatlon, restarting at a lower
6 months. Relapses of GPA are common, occurring in more dose could be considered.
than SO% of patients within 5 years, and may occur In dif- Duloxetine can cause di.z:z.i.ness but not nearly as fre-
ferent organs from the initial presentation. Both r!tuximab quently as pregabalin (10% of patients In controlled trials
and cyclophosphamide are efficacious for initial induction versus 6% on placebo). Because pregabalin iS the more likely
therapy In patients with severe GPA. The choice of which cause, it should be stopped first; ifpossible, only one variable
therapy to use depends upon the Individual patient pre- should be changed at a time.
sentation and comorbid conditions. There is no need to measure methylmalonic acid and
The RAVE (Rituximab versus Cyclophosphamide for homocysteine levels 1n this patient. Although vitamin B12
ANCA-Associated Vasculitis) trial demonstrated that rit- deficiency can cause peripheral neuropathy, the normal B12
uxlmab Is superior to cyclophosphamide in the subgroup level and neurologic examination make this a very Wllikely
of patients with relapse. In this study, remission without diagnosis.

122
Ans we rs and Critique s

Given the high likelihood that pregabalin is causing Methotrexate con be added as a glucocorticoid-sparing
the diZ7lness as well as the nonnal neurolog1c examination, agent for patients who cannot be successfu lly weaned off
MRI of the brain is not indicated at this time. It wou ld be prednisone or who are experiencing significant glucocorti-
most useful if a space-occupying lesion were suspected, coid toxicity: thts patient has only had one flare-up and may
but there are no neurologic symptoms or signs referable to do well after a modest Increase followed by a more gradual
the central nervous system. If the patient were to develop taper of prednisone, making the addition of methotrexate
neurologic signs or If symptoms did not Improve with dis- premature at this time.
continuation of the pregabalin, this decision would need to
KEY POINT
be reevaluated.
Vestibular rehabilitation may be appropriate as a means • A relapse ofpolymyalgla rheumatica should be treated
of adapting to long- lasting or permanent vestibular dam- with an increase In p redntsone to the last pre-relapse
age. It Includes exercises and balance training Intended to dose at which the patient was doing well, followed by
stabilize galt and decrease symptoms and falls. It is not an a gradual reduction within 4 to 8 weeks back to the
appropriate first step In this case, however. because drug relapse dose.
discontinuation alone Is likely to result In Improvement of
her symptoms. Bibliography
OejJJco C. Singh YP, Ptrel P. Hulchlngs A. Camelllno 0 , Mackie S, tl al:
KEY POINT E uro~an l..~aguc A~lnst Rhtumallsm. 2015 Recommendallons for the
mana~ment of poly myalgia rheumarlca: a l::uro~an League Against
• Use of pregaballn Is often limited by side effects. Rheumarlsm/Amerlcan Coli~ ofRhtumatOI<l!Q' COIIabor~tive lnlllati~.
including we1ght gain, peripheral edema, lethargy, Ann Rheum Dis 201S;7H799·807. fPMID: 26359~88) doi:IO.ll361
annrheunl<lit-2015-207'192
and dizziness: discontinuation may be needed to
manage the side effects

Bibliography
DenyS. Cording M. WlfTen PJ. l.aw S. PhllllpsT. Moore RA Pn.>gaballn for pain
Item 32 Answer: B
Educational Objective: Treat primary angiitis of the
Cl
In tibromydlgt~ In :ldult'<. Cochr.mt Oat:lbast Syst Rev. 2016;9:CD011790. central nervous system .
fPM 10: 27684492]
In atldilion to high-dose glut'OCOrticotds, tyclophosphamlde
is the most appropriate treatment for this patient who has
Item 31 Answer: A primary angiitis of the central nervous system. Patients typi-
cally present with gradual progressive neurologic symptoms
Educational Objective: Treat relapsed poly myalgia
such as headaches, cognitive Impairment. and ot her neuro-
rheumatica.
logic deficits such as strokes. Laboratory studies, including
Prednisone. 10 mgfd, Is the most appropriate treatment for eryt hrocyte sedimentation rate. are typically nonnal. but
this patient with a relapse ofpolymyalgia rheumatica (PMR) 90% of patients have abnonnal cerebrospinal fluid with lym-
after her prednisone dose was weaned to 8 mg/d. Patients phocytic pleocytosis and elevated total protein . MRI. MR
with PMR experience symmetric pain and stiffness In the angiogram, or cr angiogram often demonstrates nonspecific
shoulder, neck, and hlp regions, typically without synovi- findi ngs; cerebral angiogram sometimes reveals beading. or
tis. PMR Is a clinical diagnosis based on the characteristic alternating stenosis and d ilation of vessels consistent with
symptoms In a patient older than 50 years and is supported vasculltls. Brain biopsy is fa lsely negative in SO% of patients
by an elevated erythrocyte sedimentation rate and/or C- due to patchy distribution of abnonnalities, but when pos-
reactive protein. Recent guidelines for the management of itive will show granulomatous vasculitis. Due to the rarity
PMR (developed by a collaborative effort of the American of th is vasculitis, treatment is based on expert opin ion and
College of Rheumatology and the European league Against retrospective stud ies. High-dose glucocorttcoids and cyclo-
Rheumatism) recommend Increasing the prednisone to phospham ide have been reported to be effective In inducing
the last pre-relapse dose at which the patient was doing remission and, given the severity of this patient's presenta-
well, followed by a gradual reduction within 4 to 8 weeks tion, is the most appropriate choice. Glucocorticoids could be
back to the relapse dose. lhis paUent was asymptomatic on weaned over 3 to 6 months or longer. depending upon patient
10 mg/d, prior to reducing to h er current relapse dose; t here- response to treatment. Cyclophospham ide could be discon-
fore, an increase to 10 mg/d Is appropriate. After the flare-up tinued atler 3 to 6 months. and a maintenance drug such as
subsides, a slow taper of\ mg every 4 weeks may be better azath ioprine or mycophenolate mofetll could be started.
tolerated then larger increments over 2- to 4-week periods. Anecdotal reports indicate that rituximab and tumor
Prednisone, 30 mgfd, would also help treat the flare-up, necrosis laclor a in hib itors (such as adallmu mabl may
but this higher dose ls probably unnecessary, and a lower be effective: however. u n til more reports and studies are
dose should be attempted first. available, t hese would not be th e first choice of treatmen t.
Predn isone, 60 mg/d, Is not a typical dose used for PMR, Methotrexate Is not Indicated for this patient because
and in the absence of concomitant giant cell arterit is is not it does not penetrate the centra l nervous system well and
needed to treat a PMR flare-up. would not be effective.

123
Answers and Critiques

UY POINT Bibliography
'tt>ung NS. nrown I<E. Porvuvlrus 1119. N !;ngl J Meet. 2004:350,586·97.
• Cyclophosphamide With hlgh-doseglucocorticoids is [PMIO: 117621861
appropriate treatment for primary angiitis of the cen-
tral nervous system.
Item 34 Answer: A
Educational Objective: Diagnose rheumatoid arthritis
Bibliography
with anti-cyclic cltrullbtated peptide antibodies.
Rodr!guez-1'13 A, Munoch P". Primary angiitis of ch~ cencral nerwus system
In adults and ch ildren. Rheum Dis Oin North Am. 2015:41:47-62, viii.
[1'\110: 1S399939)doi:IO.IOI6/j.rdc.2014.09.004
Anti cyclic citrulllnated peptide (CCP) antibody testing
is appropriate. lhis patient has a symmetric inflamma-
tory polyarthritis involving the small joints of the hands
and feet, which Is highly suggestive of rheumatoid arthri-
Item 33 Answer: C tis (RA). Laboratory studies, Including rheumatoid factor,
Educational Objective: Diagnose parvovirus 819. anti-CCP antibodies. and Inflammatory markers, can assist
in confirming the diagnosis, with anli-CCP antibodies the
Parvovlrus 819 testing will most likely confrrm the diagno- most helpful. Anti-CCP antibodies have a sensitivity of
sis. Parvovlrus 819, which causes the childhood condition 70'11,, similar to that of rheumatoid factor, but they have a
known as tlfth disease (erythema lnfectlosum), Is a DNA much higher specificity than rheumatoid factor (95%) and
virus with a tropism for erythrocyte precursors. Adults are more predictive than rheumatoid factor for erosive dis-
usually contract the virus from children; individuals at ease, making this test very useful to diagnose RA. lhe dual
risk include daycarc or school workers and parents. Infec- presence of rheumatoid factor and anti-CCP antibodies
tion may be asymptomatic or produce a flu-like Illness. makes a diagnosis of RA substantially more likely.
In adults, the rash may be absent or atypical The arthritis Spondyloarthrltls disorders share an overlapping set
begins acutely, with symmetric pain and stiffness of the of features, including Inflammation of the axial skeleton,
small joints of the hands and feet. as well as wriSts and tendons, and entheses; tendon and enthesis calcification;
knees, In a pattern often mimicking rheumatoid arthritis and an association with HLA-827 antigen. HLA-827 testing
(RA). This arthritis typlcaUy lasts for several weeks before can define a probability for spondyloarthritis but cannot
resolving. Circulating antl-parvovirus lgM antibodies pro- Independently confirm or exclude any specific diagnosis.
vide evidence of active disease; lgG antibodies indicate prior lhis patient does not have any features besides peripheral
infection and are found In many healthy persons. Treatment arthritis to suggest spondyloarthrltis.
is supportive. Parvovirus can cause polyarthralgla/arthritls but Is gen-
Antinuclear antibody testing may be appropriate if erally transient, lasting 3 weeks or less. and Is often accom-
systemic lupus erythematosus (SLE) or other related auto- panied by a net-like rash on the extremities and low-grade
immune diseases are suspected. Although some features fever. JgM serology would Indicate a recent Infection and
of parvovirus 819 may overlap with those of SLE (fever, has to be checked within 8 to 10 weeks of infection because
arthritis, cytopenias), other features of SLE (rashes, other it declines thereafter as lgG rises. An elevated lgC anti-
mucocutaneous abnormalities) are notably absent in this body would simply Indicate a past Infection with the virus.
patient. 1his patient's persistent joint findings lasting 8 weeks argue
Although HJV testing may be part of the complete against parvovirus Infection.
workup for this patient's arthritis, the patient's exposure Serum urate Is useful in thediagnoslsofgour In patients
to children and the resolved antecedent brief febrile illness with typical findings. Although chronic recurrent gout can
With an evanescent rash make parvovirus 819 much more be polyarticular, this patient does not have features that
likely. suggest underlying gout such as palpable tophaceous depo-
Only so•x. of patients with RA have detectable rheu- sitions, and gout Is rare In women before menopause.
matoid factor at onset, Increasing to 60'X. to so·x, in estab- Hypothyroidism can cause fatigue and uncommonly
lished disease. Conversely, up to 20'X, of patients With RA cause arthralgia and joint stiffness but is not associated
lack rheumatoid factor. Although rheumatoid factor test- with the active synovitis seen in this patient. Therefore,
Ing may be part of this patient's evaluation because she has measuring thyroid-stimulating hormone would not be
findings resembling RA, diagnosis requires that symptoms indicated.
be chronic, usually for at least 6 weeks. Additionally, RA
usually does not follow a viral Infection. lEY POIIIY
• Anti-cyclic cltrulllnated antibody testing Is beneficial
IIY POIU
In diagnosing rheumatoid arthritis.
• Parvovirus 819 infection should be suspected in
patients with an acute onset ofsmall· joint symmetric
BlbUography
polyarthritis following a febrile Illness With rash, who Ch~ng PV. 't'Jng CT. Cheng CH. Vu Kll. Dl:~g~~osttc performance of anti-cyclic
have exposure to children. cllrulllnaltd p~ptlde ~nd rhtumatold factor In patients wllh rheumlltold
nrthrllls. tnlJ ttheum Dis. 2016;19:88D-6.)1'MID: 259~09891

124
Answers and Critiq ues

Item 35 Answer: A with spiking fevers, pharyngitis, an evanescent salmon pink


rash that occurs In conjunction with fN!r, adenopathy, hep-
Educatio nal O bjed ive: Treat osteoarthritis with
atosplenomegaly With elevated liver enzymes, leukocytosis
duloxetlne.
with neutrophil predominance. inflammatory arthritis, and
Ouloxetine. a serotonin and norepinephrine reuprake inhibitor myalgia. Patients with AOSO can also develop other manifes-
With central nervous system effects. Is a good treatment option tations such as serositis. Extremely high serum ferritin levels
for this patient Withgenenllzed osteoarthtitls (OA). Duloxetlne are a typical feature. AOSD may also be seen during preg-
Is FDA approved for chronic musculoskeletal pain and has been nancy or in the postpartum period lhe diagnosis is clinical.
shown to have analgesic efficacy for chronic low back pain and other entities such as Infection and malignancy must be
and knee OA pain, implicating the role of central sensitization excluded.
In OA pain modulation. This patient has already tried mul- Cryoglobulinemic vasculitis can cause arthritis, rash,
tiple nonpharmacologie measures, as well as Intra-articular and fever and may be associated with liver abnormalities.
glucoco•1icold and hyaluronic injections. all with insufficient particularly if it is related to hepatitis C virus. However, the
symptomatic relief. Duloxetine is a reasonable choice given the rash of cryoglobulinemia is related to small-vessel vasculitis
patient's comorblditles and generalized musculoskeletal pain. characterized by palpable purpura of the lower extremities.
Gabapentln and pregabaUn are more effective than pla- Other organs characteristically involved include pertpheral
cebo in the treatment of neuropathic pain conditions such as nerves and the kidneys (glomerulonephritis). Cryoglobulin-
postherpeticneuralgia and diabetic neuropathy.1hesedrugs emia cannot explain the elevated serum ferritin level.
are expensive and both are associated with dose-dependent Lymphoma can cause arthralgia, fever, rash, lymphade-
dizziness and sedation. 1here IS no evidence of thei r etfec- nopathy. and organomegaly. Lymphoma would not explain
tivenessspecitlcally forchronicOA pain. the patient's pleural friction rub, arthritis, or elevated serum
NSAlDs inhibit cyclooxygenase (COX) enzymes. block- fenitln level.
ing the generation of the llpid prostaglandin~ (PGE). PG~ Microscopic polyangiitls is a form of ANCA-associated
stimulates Inflammation. vasodilation. smooth muscle con- vasculitis in which patients can develop fever, rash, arthri-
traction, pain, and fever. However, PGE2 also maintains gas- tis. and pulmonary disease. but this patient's rash (pink,
tric mucosa and promotes kidney sodium excretion and evanescent, Involving the trunk) Is not typical for vasculitis
glomerular filtration. Other COX products include t hrom· (palpable purpura. urtlcarta, livedo rettcularls, nodules). In
boxane A2, a prothrombotlc regulator of platelets, and pros- addition, kidney involvement Is nearly ubiquitous in micro-
t.acyclln, an antithrombotic and vasodilatory lipid. Because scopic polyangiitls, and the normal urinalysis argues against
NSAIOs inhibit all of these, the consequences of COX inhi- this diagnosis.
bition are complex and accompanied by multiple potential Similarly. systemic lupus erythematosus (SLE) can
side effects. Side-effect risk is increased in older patients and cause the same symptoms, but the rash associated with
those with preexisting comorbldltles. 1herefore, ibuprofen SLE is unlike that of AOSO. The malar rash of acute cutane-
is not an advisable choice in an elderly patient with peptic ous lupus erythematosus consists of bright erythematous
ulcer disease, hypertension, and heart disease. patches over both cheeks and the nasal bridge, almost always
Topical capsaicin may benefit locall:z.ed OA (for exam- spar1ng the nasolabial folds; the rash Is unrelated to fever. In
ple, knee only) but Is impractical In this case given the addition, SLE cannot explain the elevated ferritin level.
multiple areas of involvement. Furthem1ore, duloxetine is
KEY POINT
more likely to be efficacious given how many treatments this
patient has tried and failed . • Adult-onset Stll1 disease is characterized by spiking
fever, an evanescent salmon-colored rash on the
KEY POINT
trunk and extremities that occurs m conjunction
• Duloxetine is FDA appro\led for chronic musculoskele- with fever. arthritis. lymphadenopathy. and leukocy-
tal pain and has been shown to have analgesic efficacy tosis; an extremely high serum ferritin level Is char-
for chronic low back pain and knee osteoarthritis pain. acteristic

Bibllogr:aphy Bibliography
Mc:AIIndon 11!,/lannuru RR. Sullivan MC. An:len Nl<, Berenbaum F. Blenna- Cerfaud·Valeotln M, Jamilloux Y, lwaz J. ~..e P. Adult-oosel Stllrs disel!se.
ZclnsiTO SM, ct al. OARS I guidelines for the non-surgical management or Autolmmun Rev. 2014:13:708- 22. [PMIO: 24(>57513] dol:l0.10161j.
knee osleuarthrttls. Osteoarthritis Cartilage. 2014:22:363-88. [PMIO: autrev.2014.01.058
24462672.] doi:IO.l0161j.jool.2014.01 .003

C]ltem 36 Answer; A ftem 37 Answer: A


Educational Objedive: Diagnose adult-onStt Still disease. Educational Objective: Treat inflammatory bowel
d~-assoclated arthritis.
The most likely diagnosis is adult-onset Still disease (AOSD),
a systemic Inflammatory disease of multiple organ systems. Adalimumab Is the mostappropnatetreatment for this patient
This patient displays the typical signs and symptoms of AOSD. with Inflammatory bowel disease-associated arthritis. 'hrious

125
An sw ers a n d Critiques

pharmacologic agents may be useful in the treatment of both alone may be inadequate for diagnosis; serologies or other
intestinal manifestations and peripheral arthritis related to laboratory tests or synovial biopsy Is required to establish
Crohn disease and ulcerative colitis. including sulfasalazine, the diagnosis. In this patient. a synovial biopsy is the most
azathioprine. 6-mercaptopurtne. methotrexate. glucocort1- appropriate test to evaluate fbr all of the conditions in the
coids, and certain tumor necrosis factor (TNF)-a. inhibitors. differential diagnosis. The biggest concern here is Mycobac-
Adalimumab, certolizumab pegol, golimumab, and infliximab terium tuberculosis arthritis, given this patient's history of
are more effective than other TNF-a inhibitors in treating the latent tuberculosis and a high risk due to advancing age.
combination of bowel and joint manifestations. This patient Sarcoid osis is characterized by formation of nonca-
with Crohn disease has developed peripheral oligoarthritls and seating granulomas in multiple organs and tissues. These
dactylitis ("sausage digit"). She also has symptomatic bowel granulomas may increase angiotensin-converting enzyme
disease. She Is most likely to lmprove her bowel and joint dis- levels. A history of latent tuberculosis, calcified granulo-
ease by the addition of a TNF-a inhibitor. mas in the upper lobe, and a chronically inflamed joint are
Intra-articular glucocorticoid injections can be used more consistent with tuberculosis infection. In addition, the
to treat inflammatory arthritis. However, the duration of angiotensin-converting enzyme level is only 75% specific
symptom relief can be short term, practical use is limited by (25% of cases will be missed) and 90% specific (10% of posi-
the size and number of joints involved, and this therapy will tive results will be false positive). decreasing the usefulness
have no effect on the patient's dactylitis or inflammatory of this test for sarcoidosis.
bowel disease. Multiple small pulmonary nodules (<5 mm) found
Glucocorticoids are effective for inducing but not main- Incidentally In a patient without a known malignancy are
taining remission in Crohn disease. Although prednisone likely to be nonmalignant. Like solitary pulmonary nodules,
could be used to improve the patient's joint and howe! multiple pulmonary nodules may be further assessed with
symptoms on a short-term basis, it is unlikely to control her helical cr to better characterize their number, location, and
bowel and joint symptoms in the long term and is Increas- morphology. However, in this patient, the previous history of
ingly likely to be associated with significant side effects the tuberculosis, fever, and a chronic inflammatory monoarthri-
longer it Is used. tis strongly suggest tuberculosis, and a tissue diagnosis can
Rituxim ab depletes B cells and is used in combination be most easily established with synovial biopsy.
with methotrexate to treat rheumatoid arthritis in patients HLA- 827 testing may be useful in the evaluation of
who have not adeQuately responded to a TNF-a. Inhibitor. spondyloarthritis, which can present w1th chronic lower
Rituximab is not Indicated for this patient's arthritis or extremity Inflammatory monoarthrltis. However, rhis
bowel disease and is not recommended. patient has no other findings of spondyloarthritis such as
rash, bowel symptoms, or inflammatory back pain. Further-
KEY POINT
more, HLA-B27 lacks speciJkity and is unlikely to yield the
• Long-term treatment options for bowel and joint correct diognosis.
symptoms associated with inflammatory bowel dis-
ease include sulfasalazine, azathioprine, 6 mercap KEY POINT
topurine. methotrexate. and the tumor necrosis factor • Synovial tluid analys is can confirm intlammation but
a inhibitors adalimumab, certolizumab pegol. goli- may be inadequate for diagnosis of a chronic intlam-
mumab, and inniximab. matory monoarthritis; synovial b10psy may be
required.
Bibliography
Peluso R, Manguso!;: VItiello M, Jervollno S, Oi Minno MN. Management of Bibliography
arthropathy In intlammacory bowel diseases. Ther Adv Chronic Ois. Gering OM. Tak PP. How to perform and an3lyre ~novi~l biopsies. Best Pract
2015;6:65-77. (PMID: 25729557) doi:IO.lln/2040622314563929 Res Clln Rheumatol. 2013;27:195-207. [PMJI.): 23731931] doi:IO.IOI6/j.
berh.2013.03.006

Item 38 Answer: D
Educational Objective: Choose the most appropriate Item 39 Answer: A
test to diagnose tuberculosis as the cause of chronic Educa tional Objective: Diagnose erosive osteoarthr itis.
monoarthritls.
Erosive osteoarthritis (OA) is the most likely d iagnosis. Erosive
The most appropriate test to perform next Is a synovial biopsy. OA is thought to be an aggressive subset of h and osteoarthri-
This patient has persistent inflam matory swelling of the righ t tis that has periods of prominent inflammatory symptoms
knee, with synovial flu id leukocyte counts between 15,000 and characteristic erosive changes on imaging. Patients often
and 20,000/f..lL (15-20 x 109/L). The dltrerent1al d iagnosis experience pain, swelling, and joint deformities. Inflamma-
In this patient w ith chronic inflamm atory monoarthritis tory damage to soft-t1ssue structures can result in deviation
includes mycobacterial, fungal, or Borrelia burgdorferi infec- of the digits. Diagnosis Is mostly based on t he radiographic
tion and other systemic rheumatologic diseases such as sar- findings of articular sutface erosions at the centro! portion
coidosis. Frequently In t hese patients, synovial fluid analysis of the joint. Erosions are often symmetric and occur mainly

126
Answ e rs and Crit iques

in the distal Interphalangeal (DIP) and, to a lesser extent, anti-double-stranded DNA antibody titer. and low comple-
proximal interphalangeal (PlPl joints. Radiographs reveal a ment levels reliably diagnose an SLE flare. Elevated C-reactive
"gull-wing• deformity, resulting from marginal sclerosis and protein (CRP) Is common in inflammatory states except In
osteophyres on the distal side of the joints, whereas the proxi- SLE. in which CRP levels do not rise during SLE flare-up (but
mal side is centrally eroded or collapsed and thinned. Women do rise during an Infectious episode). Complements are espe·
are affected more frequently than men. Erythrocyte sedimen- clally helpful in patients with SLE. Most inflammatory states
tation rate (ESR), C-reactive protein (CRP), rheumatoid factor, are associated with increased complement levels except in
and anti-cyclic citrullinated peptide antibodies are usually certain immune complex- mediated diseases (SLE; cryoglob-
normal, as seen in this patient. Heberden nodes and squaring ulinemic and urticarial vasculitis), where the levels decrease
at the carpometacarpal joint also support a diagnosis of ero- due to excessive consumption during active disease state.
sive OA in this patient. C3 and C4 are the commonly measured complements, and
Although gout flares may be superimposed upon sHes their levels decrease during the flare-up. An Increase in the
of nodal OA, the inflamed DIP joints in this patient are less anti-double-stranded DNA antibody titer also Indicates an
likely to represent a gouty flare given the normal ESR, CRP, SLE flare and has the best correlation with llare-up of lupus
and urate levels. Moreover, radiographs of patients with nephritis. A kidney biopsy to evaluate SLE kidney inmlve·
tophaceous gout often show marginal and juxta-articular ment should also be considered here.
punched-out erosions, which are not seen in this patient. Antinuclear antibody (ANA) testing is a useful screen-
Psoriatic arthritis may be symmetric and affect the ing tool for SLE but does not correlate with disease activity.
hands, including the DIP joints. but the patient reports no Anti·Ro/SSA and anti-L.afSSB antibodies correlate with SLE
personal history of psoriasis, does not indicate significant ·rashes and photosensitivity but do not correlate with dis-
morning stiffness, has no rashes or nail pitting, and does ease activity. Anti-Smith antibodies are highly specific tor
not have the marginal erosions on radiograph that would the diagnosis of SLE but also do not correlate with disease
support this diagnosis. activity. Anti-Ul-ribonucleoprotein antibodies are found in
Although the patient has symmetric hand arthritis and patients with SLE and mixed connective tissue disease but
fits the typical demographics of a patient with rheumatoid do not correlate with disease activity.
arthritis (RA). the predominance of DIP symptoms, the lack
KEY POINT
of prolonged morning stiffness, the laboratory studies neg-
ative for inflam matory markers and/or RA autoan tibodies. • Elevation of the erythrocyte sedimentation rate, rising
and the typical "gull-wing" deformity on radiograph (rather anti- double-stranded DNA antibody titer. and low
than marginal erosions} do not support RA as the diagnosis. complement levels reliably diagnose a systemic lupus
erythematosus flare.
KEY POINT
• Diagnosis or erosive hand osteoart hrltis is based Blbliogtaphy
essentially on radiographic findings of' articular sur- Pisetsky DS Anti-DNA ~ntlbodles-quintessentlal biom~rk~r.; of SLE. N~t
face erosions at the central portion or the joint; ero- Rev Rheumalol. 2016:12:102·10. [PMID: 26581343] doi:IO.I0381
nrrheum.20JS.lS1
sions are often symmetric and occur mainly in the
distal Interphalangeal joints.

Bibliography
Kloppenburg M, Kwok W'l. Hand osteoarthlitls-a heterogeneous disorder.
Item 41 Answer: D
Educational Objective: Diagnose Immune-mediated
CJ
Nat Rev Rheumatol. 2011:8:12-31. [PMIO: 22tO.S241] doi:IO.I0381 necrotizing myopathy.
nrrheum.20ll.170
In addition to a musclebiopsy,anti-HMGCo-A reductase anti-
body testing is most likely to establish the diagnosis. A small
C:J item 40 Answer: A n umber of patients taking statins develop immune-mediated
necrotiZing myopathy (IMNM), which can be easily confused
Educational Objective: Diagnose a S)Stemlc lupus
w ith polymyositis. IMN M is thought to occur as a result of an
erytheptatosus flare.
immune response to HMGCR, the p harmacologic target of
Measurement of the anti- double-stranded DNA antibodies statins. IM NM can be a paraneoplaslic phenomenon related
will be most helpful In evaluating th is patient with a systemic to connective tissue disease, or drug related. most commonly
lupus erythematosus (SLE) flare. She has recent inflamma- statlns. Starin use m ay Initiate or worsen the syndrome. In
tory swelling of multiple joints. malar eruption. hyperten- patients tak ing statlns, myositis may rapidly progress and
sion. and new-onset edema. She also has an elevated urine persist despite drug d iscontinuation d ue to this abnormal
protein-creatinine ratio. suggesting the presence of active immune response and is often associated with the production
lupus nephritis. In this situation, the measurement of Inflam- of anti-HMG Co· A reductase a ntibodies. 1he histologic find-
matory m arkers and other tests associated w ith SLE flares can in g of necrotic muscle fibers w ith only m inimal inflammatory
be very useful in establishing a diagnosis and directing treat- cells that do not Invade the muscle fibers Is characteristic or
ment. Elevation of the e rythrocyte sedimentation rate. rising IMNM. and, given the patient's history of exposure to statins,

127
Answers and Critiques

a muscle biopsy and determination ofanti-HMG CO-A reduc- cr of the chest might identify findings to support an
Cl tase antibodies will most likely establish the diagnosis. lhe alternative diagnosis. For example, findings of hllar lymph-
CONT. condition may respond to immunosuppressive therapy. adenopathy or pulmonary nodules might support a diagno-
Anti- cyclic citrulllnated peptide antibodies are not seen sis of sarcoidosis. However, the yield would be low given the
in inflammatory myopathies: they have a relatively high normal chest radiograph.
specificity (>90'X,) for rheumatoid arthritis and may be use- Uke lip biopsy, parotid biopsy can provide evidence of
ful in the differential diagnosis of early polyarticular inflam- Sjogren syndrome; however. it Is more invasive and risks
matory arthritis. seventh cranial nerve damage. Its use Is currently limited to
Antl-cytosolic 5'-nucleotidase lA antibodies have been special situations.
identified in about half of patients with Inclusion body myo- A Schirmer test could conveniently confirm dry eyes,
sitis. Strong reactivity for these antibodies has relatively high but dry eyes have already been confirmed by a more rigorous
specificity for Inclusion body myositis compared with other and reproducible ophthalmologic examination.
muscle diseases. Sialography can assess the extent of mouth dryness,
Anti-histidyl-tRNA synthetase antibodies, particularly but the diagnosis of sicca has already been sufficiently
the subset anti- Jo-1 antibodies, are associated with the pres- confirmed.
ence of interstitial lung disease In patients with Inflamma-
kEY POINT
tory myopathies and, in particular. with antlsynthetase syn-
drome of interstitial lung disease, Raynaud phenomenon. • A llp biopsy should be considered in a patient with
inflammatory arthritis. and mechanic's hands. slcca and suspected SjOgren syndrome when Initial
serologic evaluation is uninJormative.
KlV POINT
• In patients with immune-mediated necrotizing myo- Bibliography
pathy, myositis may persist despite stalin discontinua- CIOvelli RA. Santos MC, Serrano ~V. Vallm V. Clinical charactertstics and
tion and is often associated with the production of biopsy accurocy in suspected cases of SjOgren's syndrome referred to
labial salivary gland biopsy. llMC Musculoskelet Dlsord. 2015;16:30.
anti-HMG 0:>-A reductase antibodies. (PMID: 258878881 dol:l0.1186/s1289J · Ol5·0482-9

Bibliography
Musset l. Allenbach V, Benveniste 0, Boyer 0, !lossuyt X. Bentow C, et ~1.
Anti-HMCCR antibodies as a biomarker for Immune-mediated n~rullz· Item 43 Answer: C
ing myopathies: a hislury of stallns and eXPerlent-e from a large lnterna· Educational Objective: Diagnose methotrexate- induced
Ilona! multi-center study. Autolmmun Rev. 2016:15:983-93. [PMID:
27491568] dol:10.1016/j.au!K'V.2016.07.023 anemia.
The most likely diagnosis is methotrexate-Induced anemia.
Item 42 Answer: B Methotrexate can cause stomatitis. hepatic inflammation and
fibrosis, and myelotoxlclty, including meg;l)oblastlc anemia
Educational Objective: Diagnose SJOgren syndrome.
and pancytopenia. Folic acid supplementation is mandatory
1he most appropriate diagnostic test to perform next is a lip in all patients receiving methotrexate and can prevent the
biopsy. This patient has parotid and lacrimal enlargement development of stomatitis and hepatotoxicity (as measured
and sicca. 1he differential diagnosis of sicca with parotid/ by elevated aminotransferase levels). Hematologic toxicity,
lacrimal enlargement Includes Sjogren syndrome, sarcoid- however, can occur even with folic acid supplementation.
osis, lgG4-related disease. HlY.-associated dift'use lympho- In this patient, a rtse in mean corpuscular volume (MCV)
cytic infiltrate syndrome, and others. lhe American College indicates a likely megaloblastic anemia. and methotrexate
of Rheumatology/European League Against Rheumatism has is the likely cause. Guidelines from the American College of
proposed diagnostic criteria for Sjogren syndrome. lhe most Rheumatology recommend periodic monitoring of the com-
heavily weighted crtten a Include focal lymphocytic sialadeni- plete blood count every 4 weeks during the first 3 months of
tis In labial salivary gland biopsy and the presence of anti-Ro/ therapy, every 8 to 12 weeks from 3 to 6 months, and every 8
SSA antibodies. 1he presence of both will establish the diag- to 12 weeks thereafter.
nosis. Alternatively, the diagnosis can be established with'(.lne Inflammatory anemia (anemia of chronic disease) Is a
of the t\-\Q major criteria and an abnormal ocular staining common manifestation of rheumatoid arthritiS and Is usu-
score of25, abnormal Schirmer test, or abnormal whole saliva ally a mild, normocytic anemia. Most patients experience
flow test. In addition, entitles in the differential diagnosis symptoms related to their underlying disease rather than the
should be excluded. The patient meets the criteria for kera- anemia. Inflammatory anemia would not present with this
toconjunctivitis s icca, but because of the absence of anti-Rot degree of anemia or macrocytosis.
SSA antibodies, additional support is needed. A lip biopsy to Typical features of iron deficiency are Identical to those
assess minor salivary gland pathology is relatively noninva- of any symptomatic anemia but may be subtle owing to an
sive; it can proVide evidence sufficient to meet the proposed insidious onset of the condition. Headache and plea (craving
diagnostic criteria and might assist In ruling out other entitles for typically undesirable trems such as tee, dirt, clay, paper,
in t he differential diagnosis. and laundry starch) are frequently associated symptoms;

128
Answers and Critiques

other less common symptoms include restless legs syn- The erythrocyte sedimentation rate (ESR) commonly
drome and hair loss. The hallmark of iron deficiency is a indicates the presence of inflammation. and elevations are
microcytic hypochromic anemia. However, this is usually used to identify and monitor disease activity in rheumato-
only seen in advanced iron deficiency, and anemia tends to logic diseases. However. obtaining an ESR will not provide
precede morphologic changes in the cells. The presence of additional infonnation not already gleaned from the phys-
macrocytosis makes iron deficiency unlikely. ical examination and will not establish a specific diagnosis.
Tocilizumab is not associated with a macrocyt ic anemia. MRI can detect soft-tissue abnormalities, inflammation,
Th rough its anti- inflammatory properties, lt may decrease and fluid collections but is not the most appropriate next
the likelihood of inflammatory- induced anemia in patients step to establish the cause of this patient's acute monoar-
with rheumatoid arthritis. thrltls because it cannot distinguish between infection and
KEY POINT acute gout.

• Methotrexate use can result in a megaloblastic anemia KEY POINT


or pancytopenia; periodic monitoring oft he complete • Joint aspiration and synovial fluid analysis for Gram
blood count is recommended. stain. cultures, and crystals are indicated to help diag-
nose the underlying cause of acute monoarthrltis.
Bibliography
Shea 1!, Swlnden MV, Gh.ogomu ET. Orllz. Z. Katchamart W, Rader T, et al. BlbUograpby
Folic acid and folinic acid for reducing side effects lo patients receiving fl.ec.ker JA, Daily JP. Pohlgeers KM . Acute monoarthrltls: diagnosis In adults.
methotrexate for rheumatoid arthritis. J Rheumatol. 2014;41:10~9-60.
Am Fam Physician. 2016;94:810-616. [PMID: 27929V7]
[PMID: 24737913) dol:l0.3699/jrhettm. l30?38

mIt em 4 4 Answer: A
Educat ional Objective: Evaluate acute monoarthritis
Item 45 Answer: A
Educational Objective: Prevent glucocorticoid-induced
osteoporosis .
w ith joint aspiration .
This patient at risk for glucocorticoid-induced osteoporosis
Aspiration of the right knee is the most appropriate test to should begin alendronate. One of the many risks and side
establish the diagnosis. This patient has isolated inflamma- e.ffects of chronic glucOCorticoid therapy is osteoporosis. The
tory swelling of the right knee. Synovial fluid aspiration is Fracture Risk Assessment Tool (FRAX) calculator defines the
essential when ewluating for infection and crystal-related 10-year fracture risk for patients with T-scores in the -1.0
disease and can distinguish between inflammatory and non- to -2.5 range. 1he FRAX calculator (www.sheiac.uk/FRAX)
inflammatory conditions. 1he most useful tests of synovial incorporates multiple risk factors, inclu ding sex, fracture his-
fluid for infection are leukocyte count, stains, and cultures, as tory, femoral neck bone mineral density, glucocorticoid use,
well as evaluation of synovial fluid for crystals under polar- sm oking, BMJ, age, and alcohol intake to determine projected
ized light. Synovial fluid leukocyte counts less than 200/J..IL fracture tisk. The American College of Rheumatology rec-
(0.2 x 109/L} are considered nonnaL between 200/J..IL and ommends tllat patients over the age of 40 years at moderate
2000/J..Il (0.2-2.0 x 109 /L) are associated with noninflamma- or high risk for osteoporotic fractures who are to be on at
tory conditions. and greater than 2000/J.IL (2.0 x 10~/L) are least 2.5 mg of prednisone daily for 3 months or more should
associated with inflammatory states. The higher the count is, begin prop hylactic bisphosphonate therapy w ith alendro-
the more inflammatory the fluid and the greater the suspicion nate, rlsedronate, or zoledronic add. In patients on chronic
for crystal-related or infectious disease. There is no absolute glucocorticoid therapy, prophylactic bisphosphonate therapy
cutoff value that distinguishes infection from crystal- related significantly increases bone mineral density compared with
disease. because some infections may have lower counts than placebo, and these patients also have fewer new vertebral
expected and crystal-related disease may have counts greater fmctures.
than 100,000/J..IL (100 x 109 /L}. Thus. the proper applica- In patien ts at high risk for major osteoporotic fracture
tiOn of synovial fluid leukocyte counts requires conservative (10-year risk greater than 20% or a T scores-2.5or a history
Interpretation. Generally, counts greater than 50,000/J.JL (SOx of a fragility fracture) taki ng a ny dose of glucocorticoids
109 /L) should be managed as infectious until explicitly proven for at least I month should receive prophylactic treatment
otherwise; if there is clinical suspicion for infection, fluid (although these patients should be tre ated regardless of glu-
should be sent for stains and cultures even in the setting of cocorticoid therapy). Alendronate, risedronate, zoledronic
counts less than 50,000/J.IL (SOx 109 /L). Synovial fluid analy- acid, or teriparatide are therapeutic options. This patient is
sis can alsocontinn or rule out hemarthrosis, which can have not in the high-risk category, a nd teriparatide is t herefore
a similar clinical presentation as in this case. not a recommended option.
Blood cultures are approptiate in this patient but are Dual-energy x- ray absorptiometry (DEXA) is useful at
only positive in 30'Y., to 40'X, of patients with Infectious the outset o f chron ic g lucocorticoid therapy to r isk-stratify
arthritis an d do not replace the need for arthrocentesis, patients and to provide baseline values. Although there Is
which is the test most likely to establish the diagnosis. no consensus on the optimal freq uency of monitoring bone

129
Answers and Critiques

mineral density with DEXA, there is no role for repeating the Conduction disturbances and arrhythmias are common and
study ln less than 1 year. probably relate to fibrosis or the conduction system. This
Because of her prolonged need for prednisone and risk uncommon complication is not the most likely cause of pro-
for osteoporotic fracture, no additional therapy would be gressive exertional dyspnea in this patient.
inupproprlute management for this patient There Is an increased rlsk for venous thromboembolism
in patients with systemic sclerosis, and risk is greatest during
KEY POINT
the first year after diagnosis. This patient's 3-month history
• Patients at moderate or high 10-year risk for a major of progressive exertional dyspnea Is unUkely due to acute
osteoporotic fracture taking at least 2.5 mg of pred- venous thromboembolic disease.
nisone daily for 3 months or more should begin pro-
KEY POINT
phylactic bisphosphonate therapy.
• Patients w1th systemic sclerosis, especially those with
Bibliography limited disease, are at r isk for pulmonary arterial
Buckley L. GuyattG, Fink HA, Cannon M. Grossman I. Hansen KE. et al. 2017 hypertension.
Amerk'dl\ College of Rheum2tology Guideline for the Prevention and
Treatment of Clucocortlcold-lnduced Osteopomsls. Arthritis Care Res
(Hoboken) . 201 7:69:1095-1110. [PMJO: 285854101 doi:JO.I002/acr.23279 Bibliography
Thakkar V, Lau £M. Conne(:tive tissue disease-related pulmonary arterial
hypertension. Best Pract Res Clln Rbeum:ltol. 2016:30:22-38. (I'MIU:
27421214] doi :10.10161J.berh.2016.03.004
Item 46 Answer: C
Educational Objective: Diagnose pulmonary arterial
hypertension in a patient with limited cutaneous systemic Item 47 Answer: C
sclerosis. · Educational Objective: Select the most appropriate
imaging study for probable osteoarthritis.
The most likely cause of her symptom is pulmonary arte-
rial hypertension. Pulmonary vascular disease occurs in Radiography is the most appropriate inftial imaging study.
up to 40% of patients with systemic sclerosis. Vascular dis- Radiography is used to assess inflammatory arthritis
ease leading to pulmonary arterlal hypertension may occur and osteoarthritis (OA). Radiography does not visualize
secondary to interstitial lung disease (typically in patients soft tissues nearly as well as bone, and due to the two-
with diffuse cutaneous systemic sclerosis) or as an isolated dimensional nature of the images, not all bone findings are
process (typically in limited cutaneous systemic sclerosis). visible on every view. Radiography may not detect early or
Patients are usually asymptomatic in early disease but later mild erosive arthritic changes. Despite these limitations,
develop dyspnea on exertion and diminished exercise tol- serial radiography can be useful for monitoring arthritis
erance. Severe disease can lead to rigbt-sided heart failure. disease progression. Radiography is relatively inexpensive
This patient wlth llmited cutaneous systemic sclerosis and and readily available. Despite low levels of ionizing radi-
progressive dyspnea has a normal chest radiograph and a ation, radiography is considered safe except for pregnant
normal pulmonary examination, but an increased pulmonic women. In this case, the most likely diagnosis is OA of
sound and a widened split of S2 on cardiac examination the right knee. Weight-bearing views should be obtained
compatible with pulmonary hypertension. Taken together, to more accurately assess the knee joint on radiographs.
the most likely cause of the patient's progressive dyspnea is Typical radiographic OA features are osteophytes, joint-
pulmonary arterlal hypertension. lnltial evaluation of this space narrowing, subchondral sclerosis, and cysts. Erosions
patient should proceed with pulmonary function tests and are uncommon unless the erosive OA subtype is present,
echocardlography. which usually is more aggressive and inflammatory on
Interstitial lung disease (ILD) usually develops sub- clinical evaluation.
acutely with progressive fibrosis. Patients with diffuse cuta- In contrast to radiography, cr permits multiple views
neous systemic sclerosis and anti-Sd-70 antibodies are at and orientations from a single study. cr Is more sensitive for
higher risk for developing ILD. The most common symptom detecting bony abnormalities, such as bone erosions, than
of ILD is slowly progressive dyspnea, at first on exertion, but radiography or MRI. cr is more expensive than radiography
later at rest Other symptoms include nonproductive cough, and exposes the patient to more radiation.
decreased exercise tolerance. and chest pain. Auscultation MRT is useful in detecting soft-tissue abnormalities,
reveals "Velcro"-like Inspiratory crackles, most prominent inflammation, and fluid collections, but is less effective
at the lung bases. This patient's symptom complex limited to than cr ln demonstrating bony abnormalities and erosive
exertional dyspnea and nonnal pulmonary examination and changes. MRI is more sensitive than radiography in detecting
chest radiograph make this diagnosis less likely. early spine and sacroiliac joint inflammation. MRI is more
Cardiac involvement in systemic sclerosis may be due expensive than radiography and cr and is generally ordered
to cardiac fibrosis or coronary artery disease or second- when assessment of soft- tissue imaging is required. Radiog-
arily due to systemic or pulmonary hypertension. Intrinsic raphy Is a better choice than MRT to detect typical changes
cardiac fibrosis is uncommon and usually asymptomatic. ofOA.

130
Ans w e r s and Crit iques

Ultrasonography can detect soft-tissue abnormalities KEY POINT


such as synovitis, tendinitis, bursitis, and joint fluid, and
• Pes anserine burs1tiS is an inllammatory condition
Doppler can assess for increased tissue blood flow con-
affectmg the bursa at the insenion of the conjoined
sistent with synovitis. Ultrasonography can diagnose and
medial knee tendons Into the anteromedial proximal
monitor disease, and can be used to guide arthrocentesis.
Ultrasonography is relatively inexpensive, and there is no tibia and should be considered when there is localized
ionizing radiation. Because inflammatory disease or joint pain inferomedial to the knee joint.
aspiration is not anticipated and radiography will be better
able to detect bony changes of OA, ultrasonography is not Bibliography
Alvarcc-Nemegyei J. OlnosoJJ I:.IIJdeneto·l.lased roll tissue rheuma~IO!O' IV.
Indicated. 11 n~r1ne bursili~ J Clln Rheumatoi2004;10:20S·6(PMID: 17043509)
KEY POINT
• Radlogr'dphy is typically used as the Initial imaging Item 49 Answer: C
modality to assess tntlammatory arthritis and osteo- Educational Objective: Diagnose relapsing
arthritis. p olychondritls.
1he most likely diagnosis is relapsing polychondritls (RP) .
Bibliography
RP Is characterized by Inflammation and damage of carti-
Jacobron JA, Clrl~h C. Jiang V, Sal.lb Dl. Radlogr<~phlc ~valuation of arthritis:
degenemuve jotnt dlseaw and vanauo~ Radiology. 2008;248:737-47 laginous tissues; tissues roost commonly affected include
(PM ID 18710973) dol:lO 1148/mdiol2483062112 the cartilaginous portions of the external and middle ear,
nose, tracheobronchial tree, and joints. Auricular involve-
ment affects the helix but spares the earlobes. Nasal chon-
ltem48 Answer: C dritis can result in collapse of the nasal bridge (saddle nose
Educational Objective: Diagnose pes anserine bursitis. deformity), which can also be seen In trauma. granuloma-
tosis with polyanglltts. cocaine use. congenital syphilis.
The most likely diagnosis is pes anserine bursitis, an inflam- and leprosy. AJrway stenosis from tracheal ling involve-
matory condition affecting the bursa at the insertion of the ment and aortitis/large-vessel vasculitis may occur and
conjoined medial knee tendons Into the anteromedial proxi- be life- threatening. RP Is diagnosed by its typical clini-
mal tibia. This condition has been known to affect athletes but cal manifestations. Laboratory tests are nonspecific; acute
can also occur In those with concurrent knee osteoarthritis phase reactants are elevated in 80%, and mild anemia is
(usually in obese women) even without a history of trauma present in 44%. Patients with RP should undergo imaging
or overuse. The diagnosis of pes anserine bursitis is clinical, (cr or MRI) to evaluate the large airways for inflammation/
based on a history of increased medial knee pain worsened stenosis. This patient has a history of recurrent inflam-
with climbing or descending stairs or rising from a seated mation of the cartilage of lhe right ear, inflammatory eye
position. Patients may also note morning pain and sti.ffuess disease, inflammatory arthritis, and bronchial strictures
lru."tlng more than 1 hour, and pain may worsen at night due to inflammation of bronchial cartilage, consistent with
PbyslcaJ examination showing severe point tenderness at the a diagnosis of RP.
insertion of the anserine tendon about 3 to 6 em (2 In) below Cogan syndrome (Interstitial kemtltls, with cochlear
the medial joint Une, often with local edema, supports the and vestibular dysfunction) Is an unlikely diagnosis because
diagnosis. Relief with Injection of a local anesthetic is also the patient has neither vestibular nor cochiear findings.
supportive. Polyarteritis nodosa, a medium-vessel vasculitis, affects
A medial collateral Ugament (MCL) tear may cause the mesenteric and renal arteries. Patients usually present
medial knee pain, which can also extend to where the MCL with abdominal symptoms, neurologic involvement, and
inserts on the anteromedial proximal tibia; however, an MCL skin findings. Polyarteritis nodosa does not Involve the eye or
injury is very painful and may cause difficulty ambulating. cartilage of the ear, nose, or airways, making this an unlikely
MCL injury is suspected when there is pain and/or laxity on diagnosis.
valgus stress of the knee. Besides the Inflammatory arthritis, this patient has no
A meniscal tear could cause medial knee pain, but the typical symptoms or signs of systemic lupus erythematosus
pain is usually around the joint line; patients may also have (SLE). ln addition, SLE does not typically cause inllamrna-
unusual sensations in the knee such as clicking, locking, tion and destruction of cartilaginous structures.
catching. or a sensation that the joint gives way. There are
several provocative maneuvers on physical examination that KEY POINT
may suggest a menlscal tear. • Relapsing polychondrltls Is characterized by inflam-
Quadriceps tendonitis occurs as a result of stress placed mation and damage of cartilaginous tissues; tissues
upon the quadriceps tendon, usually from sports-related most commonly affected include the cartilaginous
activities. Pain can be felt at the lower part of the thigh, just portions of the external and middle ear, nose, trache-
above the patella. There may also be swelling around the obronchial tree, and joints.
quadriceps tendon.

131
Answers and Critiques

Bibliography BibUograpby
VI !::II~ A. SocaJ, Rlgante 0, Lopalco G, Molinaro F. Messina M, etaI. Relapsi ng lnce- Askan H, OOiha ln RJ Pregn~ncy and rheumatoid ~cthritis. .Best l'rnl1
poi)'Chondr1tls ~n update on P,1thogeoesls. clinical feotures. dlilgnostlc Res Clln Rheumatol 2015:29:580 96. [PMIO: 26697768) doo:10.10 161J.
tools, ond therJpeutlc persptct!Yes. Curr ~heum3tol Rep. 2016:18:3 herh .2015.07.001
IPMIO: 26711694] tloi:IO.l007/s i1 926·01S-<lS49·S

Item 50 Answer: D
Item 51 Answer: D
Educational Objective: Treat gouty ceUulitis.
Cl
Educational Objective: Treat rheumatoid arthritis
during pregnancy. The most appropriate next step in management is treatment
with prednisone. Th.is patient has gouty cellulitis accom-
No change In therapy is needed for this pregnant patient with panying an attack of acute gouty arthritis of the ankle. This
rheum atoid arthrttts (RA). For women with estab lished RA, well recognized manifestation is often misdiagnosed as bac-
two th irds will go into rem ission or achieve low disease activ- terial celluJJtis and treated (unsuccessfully) with antibiotics.
ity during pregnancy; however, one third will not improve Prednisone should resolve both the acute gouty arthritis and
or will get worse. Improvement may depend on an HLA-DQ gouty ceUuUtis; a typical dose Is 40 mgld for 5 days.
mismatch between m other and child and/or other factors NSAIDs such as Indomethacin should be avoided In
such as microchlm erism (sharing of maternal cells w ith the this patient who Is anticoagulated with apixaban for atrial
fetus and vice versa). The benefit beg1ns in the first trimester fibrillation .
and continues t hroughout the pregnancy. Disease activity Antibiotics such as Intravenous clindamycin would be
typically returns, often with a flare. after delivery. Medica- indicated for bacterial cellulitis, for which there is no evi
tion management of a patient with RA Is a major issue, and dence. There Is not an expanding area of skin involvement,
pregnancy plans should be discussed with any woman of and two previous courses of antibiotics have not improved
childbearing age wh o will be placed on therapy. Hydroxychlo- symptoms. Additionally, cellulitis would not explain the
roquine is an antimalarial medication that appears to inhibit inflamed ankle joint. An Infected jolnt with concomitant
antigen processing. It can be used to treat RA and other forms ceUuUtts IS unlikely because of the protracted time course.
of inflammatory arthritis. Although its efficacy in arth ritis MRI of the lower leg would be appropriate to help
is modest, its excellent side-effect protue makes it a useful define chronic osteomyelitis. Thls is unlikely, however, given
adjunctive therapy. Hydroxychloroquine crosses the placenta: the patient's lack of clear predisposing factors such as dia ~
ho~NeVer, there does not appear to be fetal toxicity with doses
betes mellitus or an overlying skin ulcer. as well as normal
used for the treat ment of RA. This patien t currently has mini- plain rodlographs after a month o f symptoms.
mal disease activity an d is likely to improve during pregnancy; Surgical debridement is the Initial approa.c h for nec-
t herefore, she requires no change in therapy. rotizing fasciitis. Necrotizing fasciitis, however, has a more
Umited case studies suggest that use of tumor necro - fulminant course. It Is unlikely In this afebrile, nontoxic-ap-
sis factor a Inhibitors during pregnancy may be safe, but a pearing patient with nonnal vital signs, whose symptoms
relationship to rare birth defects has been raised. Different have not significantly worsened during the past 4 weeks.
agents may have different potential to cross the placenta.
Decisions regarding the u se of any biologic agent tn preg- KEY POINT
nancy should incorporate r1sk-beneftt analysis. Etane rcept • Prednisone is appropriate treatment for gouty cell uli-
could be used only If this patient has a significant flare of dis- tis accompanying an attac k of acute gouty arthritis.
ease during pregnancy but is not needed at the current time.
Le flunomide is contraindicated in this patient; this Bibliography
medication is extremely teratogenic and must not be used Perez-Rulz F. Castillo F~ Chinchilla SP. Herrero-Beltes AM. Cllnlc:~l manlres-
befo re/ during pregnancy. Pregnancy must be avoided until tuttons gnd di~gnosis o r gout Rheum Dis Clln Nnrt h Am. 2014:40:193-
20G. [PMIO: 247033-431 dol:l0.10161j. rdc.2014.0J.003
the d rug Is no longer d etectable in the serum; cholestyr-
amine may be used to hasten the elimination ofleflunom ide
from the body.
Methotrexate was discontinued prior to attempted con- Item 52 Answer: B
ception, w hich Is highly appropriate. Methotrexate is highly Educational Objective: Treat ftbromyalgia.
teratogenic and abortifacient, and it m ust be discontinued at
The addi tion of pregabalin Is the most appropriate treatment
least 3 months before pregn ancy.
This patient's current symptoms and history are consistent
UY POIIT With her prior diagnosis of llbromyalg1a. Patients with fibro-
• Methotrexate and leflunomlde are contraindicated in myalgla typically benefit from valldadon of their symptoms by
pregnant patients because these medications are a phy.slcian because many patients have previously had their
complaints dlsreg;~rded by caregivers or fam ily. In addition
high ly tera togenic and must not be used before/dur-
to psychological support and exercise, pharma<.'Olherapy is
ing pregnancy; hydroxychloroquine can be safely used
a mainstay of ftbromyalgla management. Three drugs a re
du ring p regnancy.
FDA approved for ftbromyalgla: pregabalin, duloxetine, and

132
Answers and Critiques

milnacipran. Each provides an average benefit of approxi- Improve, similar to cyclophosphamide, the skin thickening
mately 30% reduction in pain. The latter two agents can also associated with SSe in 71% of patients. This patient's FVC and
address coexisting mood disorder. This patient has already DLCO are down compared with previous determinations, and
been appropriately started on duloxetine, and has had sig- the high-resolution cr scan of the chest shows findings con-
nlflcant but insufficient pain improvement Because patients sistent with nonspecific interstitial pnerunonitis. Therefore,
with fibromyalgia often benefit from combination therapy, the treatment with mycophenolate mofetil is appropriate.
addition of a new treatment, acting through a complementary In head-to-head comparisons, cyclophosphamide was
mechanism, is indicated now for this patient. ln contrast to as effective as mycophenolate mofetil in improving respira-
the dual serotonin-norepinephrine reuptake inhibitor dulox- tory function and dyspnea, but mycophenolate mofetil was
etine, which enhances the suppression of pain signals, prega- better tolerated than cyclophosphamide based on a longer
balln Inhibits glutamate release to directly reduce the passage time to patient withdrawal and was associated with a better
of pain signals from the dorsal root gangLion. The combination safety profile, Including lower Incidence of leukopenia and
is frequently of benefit. thrombocytopenia.
Meloxicam is a long-acting NSAID that has better gas- High-dose glucocorticolds are frequently used in
trointestinal tolerability than some other NSAIDs. However. patients with SSc-ILD; however. there is no clear evidence of
NSAIDs have not shown reliable benefit for fibromyalgia their benefit, and their use may convey an increased risk for
pain. scleroderma renal crisis and should be avoided.
Gabapentin shares its mechanism of action with prega- Methotrexate does not have the data to support its use
balin; although its pharmacokinetics are Less reliable than in SSc-ILD and because it can cause pulmonary fibrosis, it is
pregabalin, and it Is not FDA approved for this indication, its not a good choice for this patient.
use in fibromyalgla is accepted; however, the discontinua-
KEY POINT
tion of duloxetlne would be inadvisable.
Although dual serotonin-norepinephrine reuptake • Mycophenolate mofetH is an appropriate treatment
inhibitors are beneficial in fibromyalgla, selective serotonin option for patients who have interstitial lung disease
reuptake inhibitors such as sertraline have not shown any associated with systemic sclerosis.
benefit when used as single agents.
Bibliography
KEY POINT
Tashkln OP. Roth MO. Clem~ls PJ. Furst DE, Khanna 0, Kleerup EC. el al:
• Combination pharmacologic therapy that takes Sderodema Lung Study II Investigators. My<:ophenolate mofetll versus
ornl cyclophosphamide rn sderoderma-reluted intersllllal lung disease
advantage of complementary mechanisms of action (SJ..S ll): n r:tndomised controlled, d.ouble-bllnd, parallel group t:rtal.
may be helpful in some patients with tibromyalgia. lancet Respir Med. 2016:4:?08·19. (PMID: 27469583) dol:lO.J0\6/$2213-
2600(16)30152·7

Bibliography
CJlron I, Chaparro LE. Tu D. Holden RR, Mllev R. Towheed T, et al.
Combination of preg;abalin with duloxl!tine for libmmy.J!gia: a rand- Item 54 Answer: A
omized controlled trial. P.aln. 2016:157:1532-40. [PMLD: 26982602]
doi:t 0 .1097/j.pain.0000000000000558
Educational Objective: Treat undifferentiated
connective tissue disease with hydroxychloroquine.
The addition of hydroxychloroquine is the most appropriate
Item 53 Answer: 0 treatment for this patienrs arthritis symptoms. This patient
has undifferentiated connectiVe tissue disease (UCfD). a term
Educational Objective: Treat lnterstJtlallung disease
used to describe an autoimmune disease that has clinical
associated with diffuse cutaneous systemic sclerosis.
manifestations of other specinc connective tissue diseases,
Mycophenolate mofetil is the most appropriate treatment for but not enough positive features to satisfy diagnostic or clas-
this patient who has interstitial lung disease (ILD) associated sification criteria for any one disease. She has Raynaud phe-
with diffuse cutaneous systemic sclerosis. In systemic sclero- nomenon, inflammatory arthritis, and positive antinuclear
sis (SSe), significant ILDoccurs In so<x, of patients with diffuse antibodies, but does not have enough clinical findings to
disease and In 35% of those with limited disease. The presence establish a more specific diagnosis. Over time, her condition
of anti...Scl-70 (DNA topoisomerase-1) antibodies increases may evolve into a specific c:onneetive tissue disease, most
the risk for ILD. Cyclophosphamide has been used to improve commonly systemic lupus erythematosus. Management of
and/or stabilize lung involvement in SSe, but its utility is lim- ucro is the same as other connective tissue diseases and is
ited because it can only be taken for a limited time and the based on the manifestations of the individual patient. This
positive results are lost in the year after stopping cyclophos- patient has not responded to a trial of an NSAJD and has
phamide. Recently; mycophenolate mofetll has been shown to impaired activities of dally living; thus, she is a candidate for
be as effective as cyclophosphamide and is a medication that hydroxychloroqulne.
can be used for many years. Data indicate that mycopheno- Discontinuing naproxen and starting ibuprofen is not
late mofetU can Improve FVC In 75% of patients with SSc-lLD indicated because Ibuprofen is not necessarily more effective
who are able to take this medication for 2 years. It can also than naproxen.

133
Answers and Critiques

Obtaining radJography of the feet in a patient wahout KEY POINT


symptoms or physical findings in the feel is unlikely to show
• Topical NSAlDs are recommended to treat osteoar-
any abnormality and thus will not provide any useful diag-
thritis in patients aged 75 years or older because they
nostic information.
provide similar pain relief as oral medications with
Repeating anti-double-stranded DNA antibody testing
greater gastrointestinal safety and tolerability, but
is not appropriate because the results are unlikely to have
changed over a 1-montb period. Repeat testing could be with an incre-Jsed incidence of skin irritation and
considered in the future if the patient's clinical condition rash.
changes.
Repeating antinuclear antibody testing will not provide Bibliography
further insight into the diagnosis, and the titer does not cor- Un J. Zhang W. Jones A, Doherty M. F.fficacy of topical non-steroidal anti-
rnnnmmntory drugs In rhe t rentment of osteoarthritis: meto-anolysls of
relate with disease activity. mndomrset.l controlled trials. BMJ. 200~:329:324. [PMID: IS2860S6)

KEY POINT
• Treatment of undifferentiated connective tissue dis-
Item 56 Answer: A
ease is the same as other connective tissue diseases
and is based on the manifestations of the indJvidual Educational Objective: Diagnose acute calcium
patient. pyrophosphate crystal arthritis.
The most likely diagnosis is acute calcium pyrophosphate
Bibliography (CPP) crystal arthritis (pseudogoutl. Acute CPP crystal
Untliffenmtf~ted CTO: ~wide spec-
Mosca M, Tani C. Olrtl L. Rombardieri S. arthritis occurs episodically and is generally a condition that
trum or autoimmune diseases. !lest Prncc Res Clln Rheumutol.
2012;26:73-7. [PMffi: 22424194) doi:IO.I016/j.be'rh.2012.01.005 occurs in the elderly. The most frequently involved joints are
knees and wrists. An acute attack is characterized by pain,
warmth, tenderness, and swelling; signs of systemic inflam-
Item 55 Answer: D mation may be present but are not universal. Attacks can last
for weeks to months (in general. longer than gout attacks).
Educational Objec:tive: Recognize the most common
Acute CPP crystal arthritis usually occurs in patients with
side effect of a topical NSAID.
osteoarthritis of involved joints. and plain rndiographs may
lhe patient should be warned about localized skin msh as demonstrote chondrocalcinosls, seen as a thin radiopaque
a possible side effect of topical NSAlDs recently prescribed line of CPP deposition at chondral surfaces (most easily
for her knee osteoarthritis (OA). Topical NSAIDs such as appreciated in the knees, pubic symphysis, and wrists).
dJclofenac (available as a solution, spray. geL or patch) provide Ideally, acute CPP crystal arthritis is definitively diagnosed
similar pain relief as oral medications with fewer gastroin- by identifYing positively refringent rhomboid-shaped CPP
testinal effects. The American College of Rheumatology cur- crystals from joint fluid. However, there is rarely fluid to
rently recommends topical NSAIDs rather than or.!l NSAIDs drain from a wrist, and even when fluid is obtainable, CPP
for patients aged 75 years or older because topical NSAIDs crystals can be difficult to identify.
are considered to provide similar OA pain relief as oral med- Gout appears less likely than acute CPP crystal arthri-
ications, With greater gastrointestinal safety and tolerability. tis given the joints involved and the lengthy nature of the
However, there are slightly more skin reactions (rash, itch, attacks. Initially, gout usually affects the metatarsophalan-
and burning) than placebo (rate ratio, 1.14 [9S'l'v CI, 0.51- geal joints, the mid foot, ankles, and knees, although in post-
2.55]). Topical diclotenac 1% gel may be Jess prone to produce menopausal women the finger joints may be affected (espe-
local skin reactions compared with the solution, but it may be cially those with pre-existing osteoarthritis). Even without
slightly less effective in reducing pain. treatment, most gout attacks resolve within 10 to 14 days.
Simple analgesics are neither anti-inflammatory nor Additionally, the low serum urate level makes gout less likely.
disease modi lYing but can help relieve pain in patients with Infectious arthritis is highly unlikely In this patient
arthritis. Topical analgesics (such as capsaicin and lidocaine) because symptoms began 2 weeks ago, she otherwise appears
may be useful and can limit systemic drug exposure when well, and her radiographs demonstrate no joint destruction
only a single area is painful. Topical capsaicin ls safe to use (which would be expe<..1ed 2 weeks Into an untreated joint
in the treatment of OA. but it can be associated with local infection). Additionally, she had two prior similar episodes,
irritation of the eyes and mucous membranes when drug and infectious arthritis is not self-limited or recurrent.
residue Is transferred from the fingers to these areas. Palindromic rheumatism is an uncommon arthritis that
Intra-articular glucocorticoids can reduce OA knee pain is often a precursor to rheumatoid arthritis and is charac-
within days to weeks. When a joint effusion is present, terized by acute episodes of monoarthritls that last from 24
glucocorticoid injections can be particularly helpful after to 72 hours; attacks migrnte between different appendJcular
drainage of the excessive joint fluid. local side effects include joints, especially the metacarpophalangeal and proximal
skin hypoplgmentation, subcutaneous tissue atrophy, and interphalangeal joints. The prolonged symptoms and joints
joint infection. in this case do not tit the pattern ofpalindromlc rheumatism.

134
Answers a nd Critiques

KEY POINT
• Acute calcium py rophosphate crystal arthritis (pseud-
Item 58 Answer: C
Educational Objective: Diagnose scleroderma renal
CJ
ogout) is characterized by the sudden onset of pain. crisis.
warmth. tenderness. and swelling of the aflected The most likely diagnosis is scleroderma renal crisis (SRC)
joint. usually a knee or w rist: attacks are typically in this patient with systemic sclerosis. Kidney involvement
longer than those of gout. is common in systemic sclerosis. Up to SO'l'. of patients have
mild proteinuria, elevation in the plasma creatinine con-
Bibliography centration. and/or hypertension. but most do not progress
Rosenthal AK. Ryan LM. Ca lcium Pyrophosphat e Deposition Disease. N Engl to chronic kidney disease. The most striking and life-threat-
J Med. 2016:374:2575-81. [PMID: 273SSS36l doi:IO.IOS6/NEJMralSII117
ening manifestation of kidney disease is SRC. SRC occurs
in 10% to IS')(, of patients. and is more frequent in diffuse
C] ltem 57 Answer: C cutaneous systemic sclerosis (DcSSc) and in patients with
anti-RNA polymerase Ill antibodies. SRC tends to occur early
Educational Objective: Diagnose rheumatoid arthritis-
in the disease course: if untreated, it carries a mortality rate
re lated pleural effusion.
approaching 90'%.. 'The typical presentation is acute kidney
Rheumatoid pleuritis is the most likely cause of this patient's injury and severe hypertension, mild proteinuria. urinalysis
pleural effusion. Pleuritis is the most common rheumatoid with few cells or casts. microangiopathtc hemolytic ane-
arthritis (RA) pulmonary manifestation but is frequently mia. and thrombocytopenia. Some patients develop pulmo-
asymptomatic; exudative pleural effusions may occur. nary edema and hypertensive encephalopathy. Occasionally.
According to the Light criteria. this pleural fluid is an exudate patients remain normotensive despite kidney dysfunction.
based upon an elevated pleural fluid lactate dehydrogenase ACE inhibitors significantly improve kidney survival and
(LDH) concentration greater than two thirds the upper limit decrease mortality among patients with SRC, regard less of
of normal for serum LDH. In rheumatoid effusions. pleural the serum creatinine level.
leukocyte count is typically Jess than 5000/tJL (5.0 x 109 /L). Disseminated intravascular coagulation (DIC) is charac-
pleural fluid glucose is less than 60 mg/dL (3.33 mmoi/L), and terized by abnormal actiV<Jtion of coagulation. generation of
pH is less than 7.3 lhis exudative pleural effusion is compat- thrombin. consumption of clotting factors. and peripheral
ible with rheumatoid pleurisy. malignancy. and tuberculosis destruction of platelets. DIC may result from various causes.
pleurisy. including sepsis. obstetric emergencies. acute leukemias
Malignancy is a possible diagnosis accounting tor this (especially acute promyelocytic leukemia), severe bums,
patient's pleural effusion. However. extremely low pleural venoms, and shock. It is not associated with a hypertensive
fluid glucose concentrations are found only in rheumatoid emergency or urgency.
pleurisy and empyema. Glucose concentration of approxi- Hemolytic uremic syndrome (HUSl is caused by some
mately 30 to 60 mg/dL (1.7- 3.3 mmoi/L) is more typical of strains of Escherichia colt, including the OL57:H7 strain that
malignancy. In addition, pleural fluid lymphocytosis, par- produces Shiga-like tox in that targets the vascular endothe-
ticularly with lymphocyte counts representing 85% or more lium of the glomerulus. causing cell death, breakdown of the
oft he total nucleated cells. is not commonly associated with endothelium, hemorrhage. and activation of platelets and
malignancies other than lymphoma. When in doubt, a chest inflammatory pathways resu lting in intravascular thrombo-
cr with contrast should be performed in patients with pleu- sis and hemolysis. It manifests as microangiopathic hemo-
ra I effusion ofindeterminate cause. lytic anemia. thrombocytopenia. and acute kidney injury
The low leukocyte count and lymphocytic predom- in the setting of dysentery. The absence of dysentery in this
inance is not compatible with an acute parapneumonic patient makes HUS an unlikely diagnosis.
effusion. Thro mbotic th rombocytopenic purpura is character-
Tumor necrosis factor a inhibitors such as etanercept ized by abnorm al activation of platelets and endothelial
increase t he risk for tuberculosis. However, a previous neg- cells, deposition of fibrin in the microvasculature, and
ative tuberculin skin test and low pleural fluid adenosine perip heral dest ruction of erythrocytes and platelets. The
deaminase measurement (<40 U!l ) effectively rule out this diagnosis is clinical and requ ires the presence of th rombo-
diagnosis. cytopenia and microangiopathic he molytic anemia in the
absence of other causes of these fi ndings such as SRC. In
KEY POINT
this patient w ith systemic sclerosis, the diagnosis ofSRC is
• Pleuritis is the m ost common rheumatoid arth ritis much more likely.
pulmonary manifestation but is frequently asympto-
KEY POINT
matic; exudative pleural effusions may occur.
• The typical presentation ofscleroderma renal crisis is
Bibliography acute kidney injury and severe hypertension, mild pro-
Kelly C. lqool K. lman·Gutlerrc"t L Evans P. Manchegowdo K. lung in\Oive- teinu ria. urinalysis w ith fiew cells or casts. m icroangio-
ment In lnOamn~:~tory rheumatic diseases. l!e:ll Pt-act Res Clln Rheumatol pathlc hemolytic anemia, and thrombocytopenia.
2016 Oct:JO(Sh870-888. doi: 10.1016/j.berh.2016.10.004. IPMID: 27904793)

135
Answers and Critiques

Bibliography Bibliography
Woodworth TC. Sullman YA. Furst OF.. C1ementq P. Scleroderma renal crisis lntern~tlonnl Team for the R~lsl()n of the lnlematton~l Criteria for lleh~rs
and rennl 1nV<Jlvemcnt in systemic sclerosis Nat Rev Nephrol Disease (ITR· ICf.IL>). 'rhe lnter!latlonal Criteria fnr Beh~ct's Disease
2016:12:678· 691 . [PMIf): 276411~5] doi:IO.l038/nnleph.2016.124 (ICBD): a collaborative study of27 countries on the sensitivity and~pecl-
11clty of the new criteria. J Eur Acud L>ennalol Venereol 2014:28:338-47.
[PMID: 23441863] dol:10.1111ijdv.l2107

CJ Item 59 Answer: A
Educational Objective : Diagnose Bdt~et syndrome. Item 60 Answer: C
The most likely diagnosis is Beh~et syndrome. a systemic Educational Objective: Treat chronic calcium
disease associated with inflammatory iniUu-atlon of multiple pyrophosphate arthropathy.
organs. The disease is characterized by recurrent painful oral Low-dose prednisone Is the most appropriate treatment for
and genital mucosal ulcerations. Inflammatory eye disease this patient with chronic calcium pyrophosphate arthrop-
(panuveitls, retinal vasculitis). and pathergy (an inflammatory athy, which may be present as two patterns: chronic cal-
response to skin prick with a sterile needle). Other clinical cium pyrophosphate (CPP) crystal inflammatory arthritis,
manifestations Include venous thrombosis; central nervous and osteoarthritis with calcium pyrophosphate deposi-
system (CNS) manifestations such as bralnstem lesions and tion (CPPD). Chronic CPP crystal inflammatory arthritis
aseptic meningitis, with the most common CNS symptoms is a rare polyarthritis involving the wrists and meta-
being headache and diplopia; inflammatory arthritis (usually carpophalangeal joints ("pseudo rheumatoid arthritis~),
In the knees); skin lesions: and gastrointestinal inflamma- with the absence of serologies associated with rheuma-
tion/ulceration indistinguishable from inflammatory bowel toid arthritis. The course may be marked by episodes of
disease. Beh~t syndrome is most common in countries along acute pseudogout, radiographic chondrocalcinosis, and
the Silk Road and is more prevalent in men. The International lack of chronic lntlammatton on examination . Osteo-
Criteria for Beh~t Disease are both sensitive and specific for arthritis with CPPD manifests as Lypical osteoarthritic
the diagnosis and include the foUowlng weighted elements: findings Involving joints not commonly associated with
genital aphthosis (2 points); ocular lesions (uveitis or retinal osteoarthritis (such as shoulders or metacarpophalangeal
vasculitis) (2 points); oral aphthosis (l point); skin lesions joints). Radiographic findings of osteoarthritis with CPPD
(pseudofolliculitls or erythema nodosum) (1 point): vascular (subchondral bone cysts, osteophytes, and subchondral
lesions (superficial phlebitis, deep vein thrombosis, large vein sclerosis, consistent with osteoarthritis) often precede the
thrombosis, arterial thrombosis, or aneurysm) (1 point); and onset of osteoarthritis. This patient presents with find-
pathergy (I point). Three points are required to establish the Ings on examination and radiograph consistent with both
diagnosis; this patient has 5 points. patterns of chronic calcium pyrophosphate arth ropathy.
Whereas Crohn disease can be associated with aph- Treatment of chronic calcium pyrophosphate arthropathy
thous ulcers, uveitis. and peripheral arthritis, the lack of can be difficult because there is a paucity of data. Options
gastrointestinal symptoms make this an unlikely diagnosis. include low-dose glucocorticoids, low-dose colchicine,
Sarcoidosis can manifest as acute arthritis, bilater-al or NSAIDs to prevent inflammatory manifestations of
hilar lymphadenopathy, and erythema nodosum, and a the disease. For this patient, low-dose daily prednisone is
more chronic form of arthritis affects 1% to 4% of patients appropriate.
that may involve shoulders. hands. wrists. knees. and Tumor necrosis factor (TNF)-a Inhibitors have not been
ankles. Uveitis is a common manifestation of sarcoidosis. investigated as a treatment for chronic calcium pyrophos-
Papular. nodular, and plaque-like skin lesions are com- phate arthropathy, and beginning a TN F-a. inhibitor such as
mon In sarcoidosis, but mucosal or genifal ulcer-ations adalimumab in not indicated. In this patient, the complete
are not found. absence of response to one TNF-a Inhibitor is another clue
Joint Involvement occurs in 9Q'X, of patients with sys- that rheumatoid arthritis was an incorrect diagnosis.
temic lupus erythematosus (SLE), most commonly poly- Allopurinol is a urate-lowering ther-apy, but calcium
arthralgia, with fr.rnk arthritis involving both small and pyrophosphate deposition results from calclwn pyrophos-
large peripheral joints found In 40% of patients. Bilateral phate rather than uric acid deposition. The radiographic
knee inflammatory arthritis would be an unusual initial presence of chondrocalclnosis and osteoarthritic changes, as
presentation for SLE. OraJ ulcers can occur but are clas- well as the absence of classic gouty erosions characterized by
sically painless, and genital ulcers typically do not occw-. punched-out lesions with overhanging edges, argue against
Uveitis is one of the many ocular compilations of SLE but the diagnosis of chronic tophaceous gout Finally, a low
is relatively rare. serum urate level and the absence of tophi also argue against
lEY POINT the diagnosis of chronic tophaceous gout
Methotrexate was recently found to be no better than
• Beh<;et syndrome is characterized by recurrent painful
placebo In a randomized trial for chronic calcium pyrophos-
oral and genital mucosal ulcerations. inflammatory
phate arthropathy.
eye disease, and pathergy.
Although daily NSAID therapy can be a reasonable
option, this patient is an elderly woman with hypertension

136
Answers a nd Critiq ues

and chronic kidney disease, making NSA1 Ds relatively con- Reducing the azathioprine dose is not likely to be of diagnos-
traindkated. tic or therapeutic benefit because proximal muscle weakness
is not an adverse effect of azathioprine.
KEY POINT
• Treatment options for chronic calcium pyrophosphate KfY POINT
arthropathy Include low- dose glucocorticoids, low- • Glucocorticoid myopathy should be suspected in a
dose colchicine, or NSAIDs to prevent inflammatory patient with recent initiation of high-dose glucocorti-
manifestations of the disease. coids. cushingoid features, initial clinical improve-
ment. and reduction In serum creatine kinase levels
Bibliography who has an Increase in weakness with reduction in
Rosenchal/\K.Ilyan LM. Coldum Pyropho~phate Oeposlllon Dlse:~se. N EngI the glucocorticoid dose; treatment Is appropriate dose
J Med. 2016; 37~:257S-84.II'MID: 27:lSSS36J doi: IO.JOS6/N8Mmi.SI1117
reduction or discontinuation if possible.

CJ item 6 1 Answer: D Bibliography


Fasano S,/\lves SC. Isenberg OA. Curren! pharmacologit·al treatment of idi-
Educational Objective: Treat drug-induced myopathy. opathic Inflammatory myop~thles. Expert Rev Clln Plumnacol. 2016:1-
l2.!PMID: 26708717]
The most appropriate management is reduction of the
prednisone dose in this patient with drug-induced myop-
athy caused by glucocorticoids. Glucocorticoid myopathy Item 62 Answer: E
is a common consequence of prolonged treatment with
Educational Objective: Diagnose Takayasu arteritis.
high-dose glucocorticoids, as seen in this patient treated
for polymyositis. Distinguishing between glucocorticoid- The most likely diagnosis is Takayasu arteritis, a rare
induced myopathy and active inflammatory myopathy granulomatous large-vessel vasculitis affecting the aorta
is important, particularly In older patients for whom and Its major branches and pulmonary arteries. This dis-
long- term glucocorticoid side effects can be particularly ease predominantly affects females typically Jess than age
debilitating. The history is suggestive of glucocorticoid 30 years and of Asian ancestry. Aneurysms and stenoses
myopathy when a patient has been started on high- dose of large arteries cause symptoms of claudication in the
glucocorticoids within a few months, has cushingoid extremities, discrepancies ln blood pressure between the
features. and has normal serum creatine kinase levels. arms, and reduced pulses. Involvement of the aorta can
No single feature Is diagnostic on electromyogram (EMG) lead to aortic insufficiency and heart failure. lhls rare
or muscle biopsy. Weakness can often be addressed complication is a leading cause of morbidity and mortality
by reducing the glucocorticoid dose and following the In patients with Takayasu arteritis. This patient has upper
patient over the next 3 to 4 weeks to assess for gains in extremity claudication, reduced radial pulse, evidence of
strength . Colchicine and hydroxychloroqulne are also Inflammation (elevared erythrocyte sedimentation rate,
implicated as a cause of drug-induced myopathy. Statins anemia) and new-onset aortic Insufficiency with dilated
are a frequent cause of myalgia and/or asymptomatic aortic root, all consistent with the diagnosis of Takayasu
serum creatine kinase elevations and. rarely, stalin use arteritis.
can result In an immune- mediated necrotizing myopa - Giant cell arteritis (GCA) is a large-vessel granuloma-
thy associated With antibodies to the enzyme HMG- CoA tous vasculitis that can also affect the great vessels of the
reductase. chest, leading to a similar presentation, but is unlikely given
An EMG could potentially reveal findingsoflow ampli- this patient's young age; GCA affects individuals over age
tude motor unit potentials or be normal in glucocorticoid SO years.
myopathy. However, the initial clinical improvement and lgA vasculitis is a small-vessel vasculitis mediated by
reduction in t he muscle enzymes With reduction in gluco- immune complexes predominantly containing lgA. It does
corticoid dose most strongly suggest drug- induced myop- not involve the large vessels andalmostalwayscauses a rash,
athy, and an EMG in this Situation Is unlikely to provide not seen in this patient.
helpful resul ts. Kawasaki disease is a medium-vessel vasculitis that
In glucocor ticoid-i nduced myopathy. nonspecific begins in childhood, manifesting with a rash and other
atrophy of type lib fibers without necrosis or inflamma- mucocutaneous findings. It can affect coronary vessels,
tion can be seen that would help substantiate the diag- leading to cardiac complications such as heart failure.
nosis. However, muscle biopsy is an invasive procedure This patient developed initial symptoms as an adult,
that is not indicated in a patient in whom initial clinical and has no rash, making Kawasaki disease an unlikely
improvement and normalization of muscle enzyme levels diagnosis.
has occurred. suggesting t he diagnosis of glucocorticoid- Polyartertt!snodosa (PAN) Is a medium-vessel vasculitis
induced myopat hy. that can affect multiple organ systems. Cardiac Involvement
Immunosuppressive therapy with azathioprine Is used as well as large-vessel involvement is not a typical manifes-
tor glucocorticoid- resistant disease or glucocorticoid sparing. tation of PAN, making this diagnosis unlikely.

137
Answers and Critiques

K£Y ,OINT Bibliography


She:! R. Swonden MV. Ghil(,>nmu ET. Ortiz Z. Kalrhamart W Rader T. et •I.
• Takayasu arteritis is a rare chronic granulomatous roUe acid and folinic acid for reducing sld~ effects tn p.ltlenls recelv1ng
vasculitiS seen m young Asian women that mainly methutrcxatc ror rheum~lold arthrtlls J Rheumalol. 2014:41:1049-60.
[PMIO: 247379131 doi-10.3899/Jrheum.l30?38
affects the aorta and 1ts maJor branches as well as the
coronary and pulmonary aneries. resulting in claudi-
cation, cardiac ISChemia, aortic or mitral regurgita-
tion. aortic dissection. and renal artery stenOSIS
Item 64 A nswer: E
Educational Objective: Diagnose LOfgren syndrome.
CJ
No further testing Is necessary. This patient has a triad ofbihilar
Bibliography adenopathy, arthritis. and erythema nodosum. consistent with
de Sout.a AW, de ('.arvdlho JF. Ofngnosnc nnd ctnssiOcannn criteria of LOfgren syndmme, a common rheumatologic manifestation of
Takayasu arter111s. J Auloommun. 2014;48·19:79 83. [PMIO: 24461381]
doi:10.1016/j.jaul.2014.01 0 11 sarcoidosis. Diagnostic specificity for sarcoidosis Is 9S'X, when
aU three parts oft he uiad are present. thus making further test-
ing unnecessary. TI1e most common joints involved are ankles,
Item 63 Answer: A followed by knees and wrists. LOfgren syndrome has a good
Educational Objective: Initiate folic acid in a patient prognosis with disease remission within 2 to 16 weeks.
with rheumatoid arthritis who is starting methotrexate. Biopsy of a hilar lymph node demonstrating noncaseat-
lng granulomas consistent with the dlagnosis of sarcoidosis
In addition to methotrexate, folic acid, 1 mg/d, should be
is unnecessary tn this patient who clinically has LOfgren
initiated in this patient with rheumatoid arthritis (RA). RA Is
syndrome
characterized by a chronic inflammatory polyarthritis affect-
Biopsy of a shin les1on will demonstrate septal pan·
ing large and small joints with a predilection for the small
nicuUtis without primary vasculitis that Is consistent with
joints of the hands and feet. Patients with RA typically report
erythema nodosum. The dlagnosis of erythema nodosum is
joint pain and Inflammatory symptoms, including sweUing
usually dimcal. but biopsy may be helpful in patients with
and morning stiffness oflcn lasting several hours. Joint swell-
atypical presentations, such as lesions located in areas other
ing (softness or bagginess of the affected joint) is palpable on than the anterior shins. In this case, biopsy will not add any
joint examination. Glucocorticoids, such as prednisone, act
further useful information.
rapidly to control inflammation and joint symptoms, and can
Rheumatoid arthritis (RA) IS a systemic autoimmune
be useful until slower-acting disease-modify1ng antirheu-
dlsorder that typically presents as a symmetric inflammatory
maticdrugs (DMAROs) achieve full effect. DMARD treatment
polyarthritis. Characteristically affected joints include the
typically begins with weekly administration of methotrexate,
proximal interphalangeal and metacarpophalangeal joints of
whlch is the anchor drug in RA and is used in both monother-
the hands and feet and the wrists. but other joints can also be
apy and combination thempy. Daily folic acid supplementa-
involved. Rheumatoid factor Is ordered as part of the evalua-
tion w ith 1 mg (or weekly folinic acid supplementation) has
tion of RA but Is unnecessary in a patient with acute arthritis.
been found to reduce the mucosal, hematologic, hepatic, and
erythema nodosum. and htlar lymphadenopathy.
gastrointestinal side effects of methotrexate. Folic acid sup -
Synovial fluid analysis is likely to demonstrate Inflam-
plementation also reduces discontinuation of methotrexate
mation but is not specilic for sarcoidosis versus other rheu-
for any reason. Methotrexate blocks the cellular utilization of
matologic diseases. Joint infection is unlikely In this patient
folic acid, and folate depletion is considered to be the cause of
with multiple swollen joints, rather than a single joint, and
most of the side effects associated with methotrexate therapy.
the presence of other findings specific for Lofgren syndrome.
Supplementation with folic acid reduces the Incidence of side
effects without any loss of methotrexate efficacy in treating KEY POINT
RA. Folic acid ca n be taken on the same day as methotrexate • The presence of acute arth ritis, bilateral hilar lym-
because folic acid and methotrexate enter the cell via differ- phadenopathy. and erythema nodosum is 95% specific
ent pathways. Folinic acid Is the reduced form of folate and is for LOfgren syndrome.
typically reserved for patients who have not had a satisfactory
response to folic acid. Folinic acid is considerably more expen-
Bibliography
sive than folic ac1d, and proper timing and administration of O'Rqpn A, Beon~n JS. Sarooldosts. Ann Intern Mec!. 2012;1S6:1TCs-l, ITCs·
folinic acid is complex. 2, ITCS-3, rTCS·4, rTCS·S. ITCS-6. ITCS·7.1TCS-8,1TCS-9, rTC5-lO,I'TCS-
Niacin, thl amine, and vitam in C have no known utility 11, rTCS-12. ITCS-13, ITCS-H. rTCS·IS: quiz ITCS-16. [PMIO: 22547~86]
doi:I0.732610003-4819-1S6- 9 · 20120SOIO-OIOOS
in treating RA or preventing methotrexate side effects.
UY POilU Item 65 Answer: D
• Folic acid supplementation should be initiated in Educational Objective: Diagnose lgA vasculitis.
patients beginning therapy with methotrexate to
reduce the risk of side effects and discontinuation of Skin biopsy with immunofluorescence is appropriate in this
patient with purpura. abdominal pain. and arthralgia, which is
methotrexate.
thedasslc triad for lgA vasculitis (Henoch-SchOnlein purpura).

138
Answers and Critiques

lgA vasculitis Is the most common childhood vasculitis (Inci-


CJ dence. 201100. 000) and tends to appear after upper respiratory
is characterized by DRESS (drug reaction, eosinophilia, and
systemic symptoms) syndrome and can result In kidney
CONl. infections. lgA vasculitis is rarer amongadults. lgA vasculitis Is failure and death; allopurinol should be discontinued at the
an lgA immune complex-mediated small-vessel vasculitis that first sign of a rash. 1he presence of the HLA-8• 5801 allele is
almost always affects the skin but frequently affects the bowel, a contraindication to prescribing allopurinol; therefore, this
leading to pain, bleeding. and occasionally intussusception; patient should be screened before beginnJng urate-lowering
less commonly, It affects the kidneys and rarely causes pulmo- therapy.
nary hemorrhage. Biopsy of the most accessible affected organ. The absence of the H LA-B. 5801 allele does not guaran-
In this case the skin, will establish the diagnosis by demon- tee protection against allopurinol hypersensitivity. Febux-
strating leukocytoclastic vasculitis with predominance of lgA ostat is a nonpurine, noncompetitive xanthine oxidase
deposits on immunofluorescence. Furthermore. skin biopsy is inhibitor, which is a viable alternative to allopurinol. It can
easily obtainable and does not expose the patient to radiation be used in pat ients with mild to moderate chronic kidney
or contrast agents or the risks of kidney biopsy. Kidney biopsy disease a nd is safe to t cy after an adverse reaction or failure
is reserved for patients in whom the diagnosis is uncertain or if with allopurinol. Febuxostat is approxi mately 10 times more
there is clinical evidence of severe kidney involvement Biopsy expensive than generic allopuri nol.
findings are characterized by lgA deposition in the mesanglum Probenecid should be avoided In patients w ith chronic
on immunofluorescence microscopy that is identical to that In kidney disease, as the drug requires intact kidney function.
lgA nephropathy. There is no suggestion of an an tinuclear antibody
Mesenteric angiography may demonstrate reduced (ANA)-related disease such as systemic lupus erythemato-
blood flow and possibly Irregularities in small vessels con- sus, and testing for ANA should not be performed in patients
sistent with vasculitis, but it will not establish the specific with low pretest probability of disease. The presence of ANA
diagnosis of lgA-mediated vasculitis. With the exception of does not correlate with t he risk for d rug hypersensitiVity.
angiography, radiologic testing is not specific for vasculitis; Therefore, there is no Indication for ANA testing.
the features of lgA vasculitis in the bowel such as edema
KEY POINT
from Inflammation or poor blood supply may occur In other
conditions such as infectious enterocolitis, inflammatory • Risk factors for allopu ri nol sensitivity include d iuretic
bowel disease. or ischemic bowel disease. use, chronic kidney d isease, and the presence of the
The diagnosis of lgA vasculitis is based on the clinical HLA s·ssot allele in certain Asian ethnic groups.
picture in both children and adults. L.aboratmy studies are
nonspecillc but confirm systemic inflammation. lgA levels Bibliography
are sometimes elevated in lgA vasculitis but are not sensitive Saokaew S, Tassaneeyakul w, MaenthaiS!mg R, Cnatyakunapruk N. Cost-
e!Tectiveness analysts of HLA-B"SSOl testing in pre-oenting a llopurinol -
or speci fie enough to establish the diagnosis. Induced SJSnEN in Thai population. PloS One. 20J-1:9:e94294. [PMIO:
24732692] doi:10.137lljournal.pone.0094294
KEY POINT
• The classic triad for lgA vasculitis (He noch-Schbnlein
purpura) is purpura. abdominal pain, and arthralgia;
Item 6 7 Answer: C
diagnosis is established wit h biopsy of the affected
organ. Educational Objective: Diagnose spondyloarthrltls In a
patient with enthesitis.
Blbllography Spondyloarthritis Is the most likely diagnosis in this patient
Audemard-Verger A. Plllebour r:. Gulllevln 1.. TheM! E, Terrter B. lgA vas- With enthesitis. Spondyloarthritis refers to a group of dis-
culilis (Henoch-ShOnleln purpura) in adults: Diagnostic and therapeutic
aspects. Autolmmun Rev. 2015:1~ :579-85. [PMIO: 2S68800l]dol:l0.1016/ orders tha t share a n overlapping set of features, Including
j.autrev.20JS.02.003 lntlammatlon of the axial skeleton, tendons, and entheses
(insertion of tendon to bone); tendon and enthesis calcifi-
cation; an association with HlA-B27; and mucocutaneous,
Item 66 Answer: D gastroin testinal, and ocular inflammation. The fo ur disor-
ders of spondyloarthritis are ankylosing spondylitis, psoriatic
Educational Objective: Screen for tbe HLA-
arthrl t!>, intlam matozy bowel disease- associated arthritis,
8 •5801 allele in a patient at high risk for allopurinol
and reactive arthritis. 1he enthesis is a complex structure at
hypersensitivity.
the site of inser tion of a tendon or ligament onto the bone.
HLA-B. 5801 allele testing is the most appropriate next step Inflammation of the en thesis (enthesitls) Is highly suggestive
in manage ment. This patient with gout is at high risk for of spondyloarthritis- When enthesitis is particularly severe,
allop urinol sensitivity. His risk facto rs Includ e d iuretic use, the Inflammation may extend along the associated tendon
ethniclty (Thai descent), and c hronic kidney disease. Patients and local ligaments, resulting In dactylitis ("sausage digitsft).
oflhal, Han Chinese, a nd Korean descent have a higher like- This patient has dactylitis, with diffuse Inflammatory swelling
lihood of having the HLA-s• 5801 allele, which confers a high of multiple digits, making spondyloarthrltis the likely cause of
risk for allopurinol sensitivity. Allop urino l hypersensitivity this patient's presentation.

139
Answers and Critiques

Infectious arthritis typically presents with pain, swell- c auses pain and sometimes sensory symptoms radiat-
ing, warmth, and erythema of a single joint, accompanied ing down the leg(s). This patient's constitutional symp-
by fever and constitutional symptoms, and is not associated toms of chills, weight loss, and low-grade fever. as well
with enthesitis/dactylitis. as image findings of erosion in the vertebral lx.Jdy and
Rheumatoid arthritis is characterized by a chronic paravertebral swelling, are not consistent with herniated
inflammatory polyarthritis affecting large and small joints intervertebral disk.
with a predilection for the small joints of the hands and feet Multiple myeloma is the most common primary maJig-
and is not associated with enthesitis/dactylitis. nant neoplasm of the skeletal system. and the vertebral
System lupus erythematosus (SLE) should be con- column is a commonly affected site. The classic radiographic
sidered in any patient who presents with unexplained appearance includes multiple. small, well-c ircumscribed,
multisystem disease. The most common early SLE mani- lytic, punched-out, round lesions that can be fo und in
festations include constitutional symptoms (fever, weight the skull, pelvis, and spine on skeletal survey, not erosive
loss, or severe fatigue), arthralgia/arthritis, and skin dis- changes as seen in this patient
ease. Joint involvement occurs in 90% of patients with Osteoporosis with vertebra! compression fracture may
SLE. wi.th inflammatory polyarthralgia the most common cause severe back pain and the thoracolumbar area is a
presentation. SLE is unlikely due to the lack of rash or common site. but patients do not typically experience neu-
other clinical features; furthermore, 1t is not associated rologic or constitutional symptoms. Erosive changes with
with enthesitis/dactylltis. surrounding fusiform paravertebral swelling suggest another
diagnosis.
KEY POINT
• Enthesitis is highly suggestive of spondyloanhritis; KEY POINT
when particularly severe, the inflammation may • In tuberculous vertebra! osteomyelitis, the lower tho-
extend along !he associated tendon and local liga- racic spine is the most frequently involved segment:
ments, resulting in dactylitis ("sausage digits"). common symptoms are back pain, fever, weight loss.
and neurologic abnormalities.
Bibliography
Ehrenfeld M. Spondyloarthropathles. Best Pract Res Clln Rheumatol. Bibliography
2012:26:135-45 . [PMIO: 22424199) doi:IO.IOl6/J.berh.20l2.01.002
Garg RK, Somwnshl OS. Spinal tubereulosls: a review. J Spinal Cord Med.
2011;34:440-54. (PMID: 22118251] dol:10. l179/204Sn23J IY.0000000023

CJ Item 68 Answer: 0
Item 69 Answer: A
Educational Objective: Diagnose tuberculous vertebral
osteomyelitis. Educational Objective: Diagnose fibromyalgla in a mao
in the absence of discrete tender points.
The most likely diagnosis is tuberculous vertebra! osteomyeli-
tiS (Pott's disease). the most common form of skeletal tuber- The most likely diagnosis is fibromyalgia. The characteristic
culosis. Skeletal tuberculosis accounts for 10% ro 35% of cases features of fibromyalgia are widespread chronic pain, fatigue,
of ex:trapulmonary tuberculosis. Spinal involvement usually and sleep disorders, which are frequently accompanied by
results from the hematogenous spread of Mycobacterium impaired cognitive function, mood disorders. and symptoms
tuberculosis into !he cancellous bone tissue of the vertebral such as headache, gastrointestinal symptoms, and paresthesia.
bodies, from a primary pulmonary focus or extrapulmo- Although this patient initially appears to have specific joint
nary foci such as the lymph nodes. Predisposing factors for symptoms. he Is actually experiendng widespread (including
skeletal tuberculosis include previous tuberculosis infeCtion, nonarticular) pain. This widespread pain, together with poor/
malnutrition, alcoholism, diabetes mellitus, and Hrv infec- nonrestorative sleep, fatigue, and cognitiVe dysfunction (in
tion. Delays in diagnosis are common because the onset of this case, difficulty concentrating), is consistent with fibro-
symptoms Is insidious, and disease progression is slow. The myalgia. The absence of tender polnts Is not relevant; men
duration of symptoms prior to diagnosis may range from with flbromyalgia frequently do not experience tender points.
weeks to years. The lower thoracic spine is the mOSt frequently Other features experienced by patients (bowel symptoms,
involved segment; common symptoms are back pain, fever, anxiety/depression, low-grade fever, low-titer/borderline
weight loss, and neurologic abnormalities. Paraplegia is the antinuclear antibodies) are not diagnostic offibromyalgia but
mOSt devastating complication. Diagnosis is suggested by typ- are consiStent with its presence.
ical clinical presentation. along with evidence of past expo- The clinical manifestations of hypothyroidism Include
sure to tuberculosis or concomitant viscera! tuberculosis. and fatigue, cold Intolerance, constipation, weight gain. impaired
findings on neuroimaging modallties- Bone tissue or abscess concentration, dry skin. edema, depression, mood changes,
samples from needle or surgical biopsy stained for acid-fast muscle cramps, and myalgia. A normal thyroid-stimulating
bacilli and/or positive cultures are most helpful in diagnosis. hormone level excludes hypothyroidism.
Herniated intervertebral disk usually occurs In the This patient's physical examination demonstrates full
lower lumbar spine (at l4-S or LS-Sllevels) and typically range of motion of the joints without crepitus, maki ng

140
Answers and Critiques

osteoarthr itis less likely. Additionally, osteoarthritis can- defined. fts use in this patient should not be considered
not explain his d iffuse body pain, difficulty s leeping, before having tried standard therapy.
chron ic fatigue. bowel symptoms, headache. and difficulty Cyclophosphamide may be considered in this patient
concentrating. and used appropriately In patients with lupus nephritis but
Rheumatoid arthritis (RA) Is characterized by pain and is not an appropriate first choice due to a higher rate of side
swelling in multiple (>3} small joints of the hands and for feet, effects compared with mycophenolate mofetil, as well as
along with morning stiffness lasting at least 1 hour. Initial its effect on reducing fertility and premature menopause.
symptoms often worsen gradually over weeks to months. RA Cyclophosphamide is typically reserved for severe active
frequently interferes With activities of daily living, Incl uding nephritis to induce remission, followed by mycophenolate
occupational and recreational activities. Constitutional symp- mofetil or possibly azathioprine as maintenance therapy.
toms such as Increased fatigue and malaise are common. Methotrexate Is not effective In lupus nephritis and may
Depression and myalgia may occur and, less often, fever, be associated with toxicity In a patient with kidney disease.
anorexia, and weight loss. Physical examination reveals ten-
KEY POINT
derness and swelling of the joints, sometimes with warmth
and erythema; symmetric joint involvement is common. This • Mycophenolate mofetil is the most appropriate Initial
patient lacks the synovitis characteristic of RA. immunosuppressive therapy in the treatment of
isolated class V lupus nephritis, especially without
KEY 'OINT
kidney dysfunction.
• The characteristic featu res of fibromyalgia a re wide-
spread chronic pain, fatigue, and sleep disorders, Bibliography
which are freq uently accom panied by Impaired Hahn OH, McMahon MA, Wilkinson/\, Wallace WI), Daikh OJ, FitzgeraldJO,
cognitive function, mood disorders, and symptoms et at; American College of Rheumatology. American College of
Rheumatology guidelines for screening, treatment. and management of
such as headache, gastroi ntestinal symptoms, and lupus nephritis. Arthritis Care Res (Hoboken). 2012:~ :797-808. [PMIO:
paresthesia . 22SS6l06) dot:IO.I002/acr.2t664

Bibliography Item 71 Answer: C


Clauw OJ. FlbromyalgJa: a clinical review. lAMA. 201~;311:1517-55. fPMID:
247373671 dol:l0.100lljama.201~.3266 Educational Objective: Treat psoriatic arthritis.
The most appropriate medication to add to this patient's treat-
[:J Item 70 Answer: E ment regimen Is a tumor necrosis factor (TNF)-a inhibitor
such as intliximab. In patients with psori atic arthritis who
Educational Objective: Treat class V (membranous)
have uncontrolled disease while taking methotrexate at a
lupus nephritis.
dose of 25 mg weekly, the addition of a TNF-a inhibitor is
Mycophenolate mofetil is the most appropriate treatment of indicated. Randomized controlled trials have established the
this patient's kidney disease. Classification of lupus nephritis efficacy of the combination of methotrexate and TNF-a inhib-
is based on findings by light microscopy. electron miCroscopy. itors for reducing symptoms, restoring function, and limiting
and immunofluorescence. lhls patient with recently diag- joint damage. Other biologics shown to be effective include
nosed systemic lupus erythematosus (SLE) now presents with ustekinumab and secukinumab.
proteinuria likely due to lupus nephritis (likely class V, mem- Abatacept is a biologic disease- modifying agent that
branous). Guidelines recommend aggressive therapy with targets the T-cell costimulation pathway and is used in the
immunosuppressives for significant kidney inmlvement. A treatment of rheum atoid arthritis. A single randomized con-
number of immunosuppressive therapies are beneficial in the t rolled trial has shown that abatacept could be efficacious
treatment ofSLE nephritis. including mycophenolate mofetil. in psoriatic arthritis. However. in a patient With psoriatic
cyclophosphamJde. azathioprine. and rituximab. fn the treat- arth ritis In whom a TNF-a inhibitor has not been used, aba-
ment of Isolated class V lupus nephritis. especially without tace ptls not the appropriate next choice in managemen t due
kldney dysfu nction, mycophenolate mofetil is the most appr<r to limited experience in these patients.
priate initial immunosuppressive therapy based on the guide- Hydroxychloroquine is a long-acting an ti- inflammatory
line recommendations. Importantly, mycophenolate mofetil is med ication that Interferes w ith the Innate immune sys-
teratogenic and has been associated with fetal harm and death; tem inflammatol)' response and is used to treat rheumatoid
It must be stopped 3 months before a planned pregnancy. arthritis, systemic lupus erythematosus, and dermatomy<r
Adallmu mab has not been shown to be effective in sitis. It has been reported to be associated w ith an increased
lupus nephritis and may potentially worsen the disease Incidence of severe cutaneous reactions in psoriasis and Is
based on animal data. not the best choice.
Belimumab may be considered in patients with con- Rituximab is a monoclonal antibody that targets C020•
tinued SLE activity after standard therapy has been tried B cells used in the treatment of rheumatoid arthritis, vascu-
and found to be ineffective. Its role in the treatment of litis, a nd various malignancies. It has not been used in the
lupus nephritis continues to evolve but is currently not well treatment of psoriatic arthritis.

141
Answers and Critiques

KEY POilU are ineffective and a TNF a inhibitor Is not available or


affordable
• In recalcitrant psoriatic arthritis. the combination of
methotrexate and a tumor necrOSIS factor a inhibitor UY POINT' - - - -
has shown efficacy in managing joint symptoms and • Continuous full-dose NSAIDs are tirst-line therapy for
slowing the progress1on of radtographic damage, ankylosing spondylitis.
includmg]olnt space narrowing and erosions.
Bibliography
Bibliography 'Thurog JO. Chhahr:1 A, Colbert RA. Ankylosing Spondylitis and Axial
Etyoussfl s, Thomas BJ, Clurtln C. Tailored treatment opttons for patients Spondylo~rthrltls. N t::ngl J Med. 2016:374:2563 -7-'1 . !PMIO, 27l5SS3SI
wHh psoriatic arthritis and )'lSOrlllsls review of esrabllshed and new dol:lO.l OS6/NEJMral406l82
biologic ond smllll m ole!eult thtrap~ Rheumatol Int. 20!6:36:603-12.
[PMID, 26892031] doi:IO.lOO?Is00296-016-3436-0
Item 73 Answer: 0
Educational Objective: Treat knee osteoarthritis with
Item 72 Answer: A
physical therapy and exercise.
Educational Objective: Treat ankylosing spondylitis
with NSAIDs. Physical therapy and a graduated leg muscle strengthening
exercise program are appropriate for this patient with knee
The most appropriate treatment Is an NSAID such as didofenac osteoarthritis. For patients with osteoarthritis, an individ-
for this patient with ankyloslng spondylitis, a chronic inflam- ualized management plan Includes education on osteoar-
matory disease affecting the axial skeleton (including sacro- thritis and joint protection, an exercise regimen, weight
iliac joints), entheses, and peripheral joints. It has a strong loss. proper footwear, and assistive devices as appropri-
familial predilection, the strongest association with HLA-827 ate. Physical actMty lncludes graduated aerobic exercise
among the forms of spondyloarthritis, and a male predomi- and strength training, with attention paid to bolstering
nance. ThiS man's history is suggestive of ankylosing spondy- strengthening periarticular structures and minimiZing
litis. with symptoms oflntlammatory back pain, the presence injury. The patient's ma in symptom is stiffn ess, not pain,
of HLA-827, a family history of ankylosing spondylitis, and a and he prefers nonpharmacologic therapy at this time,
change in hiS exercise routine. He Is early in the course of his which is most appropriate from a management perspective
disease and has not developed plain radiographic or labora- as well. Muscle weakness Is common in knee osteoarthritis.
tory abnormalities. Continuous full-dose NSAlDs are first-line possibly related to disuse. and muscle strengthening has
therapy and can help relieve pain and stiffness. Studies of been associated w ith reduced paJn and Improved function.
continuous full-dose NSA!Os demonstrate symptomatic relief All treatment guidelines agree that a muscle strengthening
as well as reduced sacroiliac and spine inflammation as seen exercise program should be part of the patient's manage-
on MRl in some patients. rn a young man with no medical m ent plan.
comorbldlties, they are associated with a low incidence ofside Because the patient has a history of peptic ulcer dis-
e[ects. Patients with ankyloslng spondylitis are more likely ease, ibuprofen is not advisable. He a lso chooses not to take
to respond to NSAIDs and do so more rapidly and completely medication for his knee symptoms, and it Is unclear whether
than patients with chronic low back pain from other causes. ibuprofen w ould help his majn symptom of stiffness.
Results from most randomized controlled trials of Intra-articular glucocorttcolds are useful agents in knee
methotrexate use In ankyloslng spondylitis have shown no osteoarthritis, particularly as second- line therapies. Similar
benefit, and it Is therefore not appropriate In this patient. to Ibuprofen, It is unclear whether this treatment would help
A considerable amount of more recent data suggests the patient with his stiffness.
that tumor necrosis factor (TNF)-a inhibitors can be partic- A knee MRI is not Indicated in a patient without knee
ularly helpful In the management of symptoms of ankylos- buckling or locking or signs of instnbility, all of which
lng spondylitis. However, NSAIDs remain first- line therapy. may indicate an unstable menlscal tear that needs surgical
There Is variability In the expression of disease, and some management. rncidental meniscal tears are common MRI
patients can be managed with complete symptom relief findings in the middle-aged population, and especially In
without Immunosuppression. The efficacy of TNF-a inhib- patients with knee osteoarthritis. Studies have shown that
itors for symptom relief needs to be weighed against their patients with knee osteoarthritis and meniscal tears experi-
cost, potential side effects, and the fact that these agents have ence similar long-term improvements In pain and function
not been definitively esl.abllshed to be disease modifying. with physical therapy compared with arthroscopic surgery,
Sulfasalazlne has historically been used in the treat- and that arthroscopic surgery may even pose higher harms.
ment of ankylosing spondylitis, but its use as second-line
therapy has declined as more effective agents, such as TN F-a. lEY POINT
inhibitors, have emerged. This patient's primary site of • Treatment guidelines recommend that a muscle
involvement is the spine, and many experts now relegate the strengthening exercise program should be part of the
use of sulfasalazine for treatment of the peripheral arthritiS management plan for knee osteoarthritis.
associated with ankylosing spondylitis and only if NSAJDs

142
Answers and Critiques

Answer: B
Bibliography
MeAlindon TE, Bannuru RR. Sullivan MC. Arden NK, Berenbaum I\ Bferma-
7.elnma SM. et al. OARS I guidelln51i:lr the non-surgical management of
Item 75
Educat ional Objective: Diagnose C1 -C2 subluxation In
Cl
knee osteoorthrltis. Osteoarthritis CQrtiloge. 201~;22:363·811. [PMID: a patient with rheumatoid arthrit is.
H462672) dul:l0.1016/).)oca.2014.01.003
lhe most appropriate diagnostic test to perform next is radi-
ography of the cervical spine with flexion/extension views.
Item 74 Answer: A Patients with long-standing (generally 10 years or more) rheu-
matoid arthritis (RAJ are at risk of de\leloping Q -C2 sublux-
Educat ional Objective: Diagnose acute cutaneous lupus
ation. Inflammation from RA can lead ro attenuation of the
erythematosus.
transverse ligament that normally limits the posterior motion
This patient has acute cutaneous lupus erythematosus (ACLE) of the odontoid process of the C2 vertebrae. Instability around
and most likely has systemic lupus erythematosus (SLE). 1he the odontoid can Impinge upon posterior structures. including
cutaneous manifestations of SLE are essential to recognize the spinal cord and vertebral arteries. Common symptoms
because skin findings are key diagnostic criteria. Because the include a sensation of the bead falling off, drop attacks, and
subtypes of SLE are important to distinguish, the dermatologic painless paresthesia ofthe hands and feet: rarely, sudden death
examination Is cruciaL The malar (butter/ly) rash of AUE con- can occur. Initial imaging should be radiography of the cervical
sists of bright erythematous patches over both cheeks and the spine with flexion/extension views: If results are abnormal,
nasal bridge, almostalwayssparlng the nasolabial folds (incan- MRI should then be performed. Consideration of swgery is
t~ to the more vi>laceous facial erythema ofderrnat~itis, necessary for patients with significant subluxation. especially
which often involves that crease). The malar rash of ACLE can if symptomatic Patients with long-standing RA undergoing
occursimultaneously with other find In~ ofSLE or may precede general anesthesia for any kind ofsurgery should have cervical
other symptoms of SLE by months or years. The rash can last a spine radiography with flexion/extension views to assess for
variable length of time from hours to weeks and typically recurs atlantoa.xlal subluxation. which can lead to neurologic com-
or worsens with sun exposure. Essentially aU patients with promise when the neck is extended during Intubation.
ACLE have or will develop SLE; therefore, all patients should Electromyography is the primary test used to diagnose
receive a thorough evaluation for systemic ln-.{)lvement. lower motor neuron conditions such as focal neuropathies,
Erysipelas is a superficial cellulitis involving the lym- polyneuropathies, motor neuron diseases. and myopathy.
phatics that often presents as a violaceous-red, edematous, Nerve conduction studies are used to assess pertphera I nerve
well-demarcated plaque on the face or lower extremities system function and can classify peripheral nerve conduc-
secondary to group A streptococci. Patients are extremely tion abnormalities due to axonal degeneration, demyelin-
uncomfortable and often have systemic symptoms such as ation. and conduction block. In most clinical situations
fever and malaise. When on the fuce, the presentation of involving peripheral nerve or muscle disease. nerve con-
erysipelas Is not symmetrically distributed as It Is with Aa. E. duction studies and electromyography are performed and
Rosacea often has inflammatory papules, small pus- interpreted at the same time. Electrodiagnostlc studies are
tules, and telangiectasias, and tends to involve the nasolabial not helpful in a myelopathy as seen with Cl-C2 subluxation.
folds. 1hese manifestations are rare to absent in SLE. The neurologic symptoms associated with vitamin 812
Seborrheic dermatitis Is often characterized by a greasy delklency typically include paresthesia and ataxia, more
scale and tends to involve the nasolabial folds, eyebrows, and often In the lower extremities. This patient has Intermittent
scalp. None of these manifestations is present in this patient. symptoms and no evidence of a peripheral neuropathy on
A less widely known subtype of cutaneous lupus is examination. A normal complete blood count makes vitamin
subacute cutaneous lupus erythematosus. 1he typical clin- 8 12 deficiency less likely, although neurologic complications
ical eruption consists of circular or polycyclic scaly ery- may occur without hematologic manifestations of deficiency.
thematous patches and papulosquamous plaques over the Hypothyroidism can cause distal peripheral neuropathy
forearms, chest, or especially the upper back. in a V-shaped (which is often painful) and carpal tunnel syndrome. The lack
configuration. 1bls pattern In not present in this patient. of painful paresthesias and a normal neurologic examination
make hypothyroidism neuropathy an unlikely diagnosis.
IIEY 'OINT
• The malar (butterfly) rash of acute cutaneous lupus ICIY POINT
erythematosus (ACLE) consists of bright eryt hema- • Patients with long-standing rheumatoid arthritis are at
tous parches over both cheeks and the nasal bridge, nsk for developing CI-C2 subluxation: symptoms
almost always sparing the nasolabial folds; essentially Include a sensation of the head falling off. drop
all patients with ACLE have or will develop systemic attacks, and painless paresthesia of the hands and feet.
lupus erythematosus.
Bibliography
Bibliography Od Cr~nde M, Del Ct"~nde ~: Carrino J. Rlll8ham CO 3nl Louie CH. Cervical
Okon l.C. Werth VP. Cutaneous lupus erythematosus: diagnosis and lreat- spine ln\IOI~ment early In the COut$e of rhcunu~toid arthrllls. Scmln
rnC!nt. Best Prnct RrsOin Rheurnatol. 2013;27:391-404 IPMID: 2.f23869SI Arth11tls Rheum. 2014;43:7311-44. I PMIO: 24444S9S] dol:10.10161j.
doi:IO l016/j.bem .20t3.0?.008 semarthnt.2013.12 001

143
Answ e rs and Critiques

C]ltem 76 Answer: 8 and oral mU<.:osal and lip changes. Coronary vessel vasculi-
tis, aneurysm formation, and other cardiac complications
Educational Objective: Diagnose neuropsychiatric
(such as heart failure, pe1icarditls, and arrhythmJas) may
!>')'stemlc lupus erythematosus.
develop. Up to 25':1. of untreated patients develop coronary
Neuropsychiatric systemic lupus erythematosus (NPSLE) is artery aneurysms as long-term sequelae of KD; intravenous
the most likely diagnosis in this patient with systemic lupus immunoglobulin (IVlG) treatment reduces the Incidence of
erythematosus (SLE). NPSLE prevalence is high in patients aneurysms, but 10~. to 20% of patients do not respond. Esti-
with SLE (75%) and may involve central and/or penpheral mates indicate that 4'Xo to 8% of patients treated with IVIC will
nervous systems. with the most common manifestations develop aneurysms. Thrombosis or stenosis of these aneu-
being headache. mild cognitive dysfunction, and mood dis- rysms can lead to iSchemia and myocardial infarction. Jt is
order. Acute presentations, Including seizures and psychosis. Important to Identify and follow patients at risk for long-term
happen infrequently (<S'X.) but require aggressive symptom- cardiac complications of KD. because they may require pre-
atic as well as disease-specific treatment This patient has no ventive therapy with aspirin or other antithrombotic medica-
sensory or motor deficits and presents with confusion, along tions. With the exception of the complications related to the
with a nonfocal neurologic examination. She also shows signs coronary arteries, most patients with KD completely recover
of active SLE disease, including clinical evidence (fatigue, from multisystem acU!e Inflammation. Patients with.o ut
rash, arthritis) and laboratory findings (low complements abnormalities need no specific cardiology surveillance, but
and a hlgh erythrocyte sedimentation rate). She has elevated those with persistent coronary aneurysm may require peri-
protein in the cerebral spinal fluid (CSF). and radiologic stud- odic electrocardiography, echocardiography, and/or coronary
ies show diffuse white matter changes. These findings suggest artery lmaglng as part of a long-term surveUiance program.
that NPSLE is the likely cause for the mental status changes/ Recommendations about the nature and frequency of
psychosis and aseptic meningitis. continued surveillance of patients recovering from KD, as well
This patient does not have acute bacterial meningitis as physical activity restrictions. are provided by the American
because she lacks fever (seen In >75% of patients) and neck Heart Association and are based upon the coronary artery
stiffness. and she has a nonnal cell count and glucose on CSF. status of the patient. In the absence of coronary aneurysm, the
This patient's clinical presentation is not suggestive of avallable data suggest that the risk for coronary artery disease
seizures: she Is alert and awake without involuntary move- is low. For example, a large cohort study showed that among
ments. Is following commands, and has no findings sugges- patients with KD, S'X. had persistent coronary aneurysm, and
tive of ongoing seizure/status epilepticus. long-term adverse cardiovascular events occurred in 8'1 with
Steroid-induced psychosis is seen in patients taking persistent coronary aneurysm versus 0% without persistent
high doses of glucocorticolds (usually > 1 mg/ kg/d) and Is aneurysm. However, the nature and frequency of long-term
very unusual at doses less than 20 mg/ d. This patient's dose surveillance are still unclear. An ongoing cohort study ofJap-
is only s mg/d, making steroid-induced psychosis unlikely. anese children with a history of KD growing into adulthood
will better infonn the medical community on the best lon g-
KlY 'OINT
term surveillance and management plan.
• Neuropsychiatric systemic lupus erythematosus prev- A past history of bundle branch block, myocarditis, or
alence Is high in patients with systemic lupus erythe- pericarditis does not appear to be associated with the same
matosus, with the most common manlfestalions risk for long-term card!ovascuJar events as does the presemce
being headache. mild cognitive dysfunction, and of coronary aneurysm.
mood disorder; severe acute presentations, including
KEY POINT
seizures and psychosis, occur infrequently.
• Persistent coronary artery aneurysm conveys the
Bibliography greatest risk for long-term cardiovascular events in
~u A. KllO AH. Neurop$)'clllall1c $)'$t~mlc lupus erythcmatosll$. CUrr survivors of childhood Kawasaki disease.
Neuropharmacot. 2011;9:449-57. (PMID: 22379459) doi:10.2174/
lS7015911796SS7984
BlbUogr2phy
McCrind~ BW, Rowley Al-l. Newburger JW. 1:1 al. Dtngnosls. Treatment. and
Long-Term Management of Kawa.sakl Disease: AScientific Statement for
Item 77 Answer: 8 Health Professionals From lhe Amerlt-an Hc:~rt ASsoctanon. Circulation.
2017;13S:e927-e999. IPMID: 28356445] doi:10.1161/CIR.
Educational Objective: PredJct long-term prognosis of 0000000000000484
Kawasaki disease.
A history of coronary artery an eurysm is the most import- Item 78 Answer: C
ant disease complication determining the patient's need for
Educational Objective: Prevent osteoartbrltis wt.th
extensive evaluation and his long-term prognosis. Kawasaki
weight loss.
disease (KD) Is a medium-vessel vasculitis that affects children
and Is very rare in adults. KD presents as fever, rash. cervical Recommending weight loss In order to lower the Ul<ellhood
lymphadenopathy, prominent nonexudative conjunctivitis, that she will develop osteoarthrltis (OA) Is the most appropriate

144
Answers and Critiques

choice. Obesity is the strongest modifiable risk factor for OA hypertensive emergency. headache. mlcroangiopathic hemo-
Incidence. Multiple studies ha\e shown that higher BMI catego- lytic anemia (schistocytes on the peripheral blood smear),
ries have proportionally higher risk of developing radiographic thrombocytopenia, elevated serum creatinine levels, and pro-
and symptomatic knee OA. A recent randomized clinical trial teinuria. Approximately 75'X. of cases of scleroderma renal cri-
showed that a reduction of <?:S kg or S'Y., of body weight over sis occur in the first 4 years ofSSc disease onset. Scleroderma
a 30-month period reduces the risk for the onset of radio- renal crisis can be seen in both limited and diffuse fonns of
graphic knee OA inmlddle-agedoverweightand obese women. SSe but more often In those with rapidly progressive diifuse
Another recent study of obese and overweight patients found disease. Anti- RNA polymerase Ill antibodies serve as a marker
that subjects who lost weight over 48 months shovved signiti- for increased rlsk for scleroderma renal crisis as well as exten-
camly lower knee cartilage degeneration, as assessed wlth MRI, sive skin disease. The use of ACE inhibitors has dropped
compared with those whose weight was stable. the 1-year mortality of sderodenna renal crisis from 76".<. to
Chondroitin sulfate supplementation has not been 15%. 'fhe mechanism of action of ACE inhibitors is believed
studied to lower the risk for developing incident OA. 1here to be mitigation of the eflect of interstitial fibrosis and vas-
are promising but Inconclusive data on whether chondroitin cular dysfunction in the glomerular arterial bed. Treatment
sulfate has a salutacy effect on the progression of structural with an ACE inhibitor (typically captopril) should be initiated
changes in established knee OA. promptly In SSe patients with even mild hypertension or
Limited studies have examined the association of vita- otherwise unexplained elevations in serum creatinine levels
minD with Incident OA. and available data thus far have not or acute kidney injury. Angiotensin receptor blockers (ARBs)
shown that low vitamin D confers a greater risk of incident are an alternative for patients whocannot take ACE inhibitors.
OA. There is no evidence that vitamin D supplementation although ARBs are less effective for managing scleroderma
reduces the incidence ofOA. renal crisis.
Suggesting that running will increase her risk of knee Cyclophosphamide is an immunosuppressive agent and
OA is not appropriate. Although not completely conclu- not effective in scleroderma renal crisis.
sive, the medical literature generally supports the idea that Glucocortlcoids, such as methylprednisolone, are Impli-
running does not contribute to the degeneration of artic- cated as potential risk factors for the development of sclero-
ular cartilage in joints that are neuroanatomically normal. derma renal crisis and are not indicated for this patient.
Some studies using advanced MRI techniques showed that In patients with hypertensive emergency, blood pres-
moderate exercise in subjects at high risk for OA may sure must be lowered quickly with short-acting Intrave-
be associated with favorable joint composition changes, nous antihypertensive infusions to limit end-organ damage.
which could have chondroprotective effects on the knee. Intravenous nitroprusside is often a first choice for patients
Additionally, some studies suggest that quadriceps muscle with hypertensive emergencies. It can be titrated quickly to
weakness may be a risk for knee OA incidence. Nonethe- achieve blood pressure control. However, it does nothing to
less. there is a clear association with knee OA and anteced- address the underlying physiological cause of scleroderma
ent knee injucy, such as menlscal or cruclate ligament renal crisis and is inferior to ACE inhibitors on reducing
tears, so while stopping running may not be a beneficial mortality.
recommendation. raking precaution to avoid injucy would
K£Y POINT
be prudent.
• Features of scleroderma renal crisis include hypertensive
KEY POINT
emergency, headache, microangiopathlc hemolytic
• Obesity Is the strongest modifiable risk factor for oste- anemia. thrombocytopenia, elevated serum creatinine
oarthritis incidence: weight loss can lower the risk for levels, and proteinuria; treatment Involves ACE
developing the disease. inhibitors, typically captopril.

Bibliography BibJiogmphy
Roos EM. Arden NK. Strategies for the prevention of knee osteoarthritis. Nat Wood~rth TG. Sultman VA. f'Urst DE. Clements P. Sderoderma renal crlsls
Rev Rheumatol. 2016:12:92-101. fPMID; 26·139406) dni:I0.1038/ and renal Involvement in systemic sclerosis. Nat Rev Nephrol.
nrrheum.201S.I3S 2016;12:678·691. (PMIO: 27MIJ35} doi:I0.1038/nrnepi1.2016.124

[:::J Item 79 Answer: A Item 80 Answer: C


Educational Objective: Treat scleroderma renal crisis. Educational Objective: Trea t early rheumatoid arthritis
with m ethotmcate
1he most appropriate treatment for this patient with sclero-
derma renal crisis Is the ACE inhibitor captoplil. This patient The most appropriate treatment at this time is methotrexate.
has features of systemic sclerosis (SSe}. including a hlstocy of 1his patient has early rheumatoid arthritis (RA) with moder-
Raynaud phenomenon, gastroesophageal reflux disease. and ate disease activity measured by the clinical disease activity
skin changes typical of this disease. She also has features that index(CDAJ).Methotrexateisan anchordrugin RAand is the
occur in patients with scleroderma renal crisis. including preferred initial monotherapy, as it both controls symptoms

145
Answers and Critiques

and prevents joint damage. Clinical trials have demonstrnted area of bone collapses, often leading to rapidly progressive
that 30% to SO% of patients treated with methotrexate mono- osteoarthritis. Patients with hip osteonecrosis present with
therapy will show no disease progression as measured by groin pain (that Is, true hip pain) occasionally radiating to
hand and foot radiographs. Close dose titrntion (up to as high the buttock, which worsens with ambulation; osteoarthritis
as 25 mg per week) based on her CDAl score is necessary, as can present in the same way, but would not be expected
is monitoring for the development of joint damage by rndi- in a 34-year-old patient Physical findings Include reduced
ogrnphy or ultrasonography. Ultrasonography is capable of range of motion of the hip, altered gait, and pain with weight
Identifying erosions earlier than radiography and can identify bearing. Plain radiography is the initial radiographic study of
continued inflammation even in asymptomalic patients with choice, although MRI is the gold standard for diagnosis and
no detectable synovitis on examination. Tf methotrexate alone may be required if plain radiography is not diagnostic. Early
does not lead to remission, a second disease-modifying treat- radiographic findings include bone density changes, sclerosis,
ment should be added. and, eventually, cyst formation. Subchondral radiolucency
Prednisone may improve symptoms and is disease producing the "crescent sign" indicates subchondral collapse.
modifYing in conjunction with methotrexate, but long-term End-stage disease is characteriZed by collapse of the femoral
side effects (such as diabetes mellitus and osteoporosis) limit head, joint-space narrowing, and degenerative changes.
its usefulness as primary thernpy in RA. Prednisone is pri- Dual-energy x-ray absorptiometry is useful to assess
marily used as short-term therapy for disease flares and for bone density in patients on chronic glucocorticoid therapy.
temporary symptom control while waiting for long- term Although it may establish the presence of osteoporosis, it
medications to become effective. Prednisone should be used cannot determine the cause of pain and has little role in the
at the lowest effective dose for the shortest period possible. evaluation of acute symptoms such as hip paln.
NSAIDs convey anti-Inflammatory, analgesic, and anti- MRI is the best method for detecting early bone edema
pyretic effects. As such, ibuprofen may address pain and caused by osteonecrosis when plain radiographs are normaL
swelling but Is not considered disease modifying and will not MRI is preferred for diagnostic use in patients with nondi-
slow the progression of disease. The major role for NSA!Ds in agnostic plain radiographs. However. for end-stage disease,
the treatment of RA is for temporary symptom control while MRI is more expensive than plain radiography, and it may be
waiting for the effect of disease-modifying treatments such unnecessary if plain radiography is diagnostic.
as methotrexate to be realized. Ullr4SOnography can be used to evaluate for trochan-
Although tlrst- Une therapy for lupus nephritis and teric bursitis as the cause of lateral hlp pain but would not
potentially useful in systemic sclerosis, mycophenolate be useful to check for osteonecrosis.
mofetil does not have a clearly defined role in the treatment
KEY POINT
of RA. A clinical trial of mycophenolate mofetil in RA showed
no benefit compared with placebo. • Plain radiography is the initial radiographic study of
choice for osteonecrosis, and MRl is the modality of'
KEY ,OINT
choice for sensitive evaluation of early disease when
• In rheumatoid arthritis, methotrexate is an anchor plain radiographs are normal.
drug and is the preferred initial monotherapy.
Bibliography
Bibliography Weinstein RS. Glucocorticoid-Induced osteonecrosis. Endocrine 2011:
Singh JA, Saag KC, Bridges Sl. Jr. Akl EA, Bmnuru RR, Sullivan MC, el al. 41:183-90. [PMlD: 22169965} dol:l0.1007/SI2020·011 · 9580· 0
201S American College of Rheumatology guideline for the t~eatment of
rheumatoid arthritis. Arthritis Rheumatol. 2016;68:1-26. IPMID:
26545940] dol:lOJ002/art.39480 It em 82 Answer: A
Educational Objective: Diagnose Infectious arthritis.
Cl
Item 8 1 Answer: C Arthrocentesis followed by intravenous antibiotics is the
appropriate next step in management of this lmrnunocom-
Educational Objective: Diagnose glucocorticoid-
promised patient with suspected Infectious arthrttls. This
Induced osteonecrosis.
patient Is immunosuppressed due to both his underlying
Plain radiography of the left hip is the most appropriate initial rheumatoid arthlitis (RA) and his medications. Additionally,
test for this patient with subacute hlp pain. She has lupus other than the left knee, It appears that his arthritis Is well
nephritis that requires treatment with glucocorticoids. Both controlled, raising concern for a cause other than a flare
glucocorticoid treatment and systemic lupus erythematosus of his RA, including Infectious arthritis. The hallmarks of a
are risk factors for osteonecrosis, which causes death of an joint infection are warmth, erythema, pain, and sweUing.
area of bone due to compromised blood supply. Other com- The typical bacterial Infectious arthritis develops over days.
mon risk factors include prior fracture or radiation exposure, Joint Infections are commonly limited to a single joint; the
excessive alcohol use, and sickle cell anemia. Osteonecro- knee Is the most common site, but the hlp, wrist, and ankle
sis typically affects the ends of long bones, including the are also commonly affected. Fever and rigors may occur but
femoral head, humeral head, and distal femur. The lnvol~ are not uniVersally present (60% of patients); absence of fever

146
Answer s a n d Critiques

therefore does not exclude an infected joinr. When infectious C Virus infection will also treat the cryoglobulinemia and the
CJ arthritis is suspected, the synovial fluid must be aspirated vasculitis.
CONr. and assessed for leukocyte count, Gram stain, culture, and In the absence of sclerodactyly. dilated capillary loops
crystal analysis. Joint aspiration should be performed before <lf fingernail beds, or digital pitting, the patient has no clin-
initiating antibiotics whenever possible. Infectious arthritis ical features aside from Raynaud phenomenon to support
must be expeditiously recognlzed and treated to prevent joint a diagnosis of systemic sclerosis; therefore, antlcentromere
damage; therefore, intravenous antibiotics should be initiated antibody testing iS not indicated.
right after arthrocentesis lfthe suspicion for Infectious arthri- Similarly, the patient has no joint swelling on physical
tis is high, as in this case. A synovial fluid leukocyte count of examination, making a diagnosis of rheumatoid arthritis
>50,000/ JJL (SOx 109 / L), and especially <!100,000/J.IL (100 x much less likely; therefore, anti-cyclic citrullinated peptide
10°/L). strongly suggests an infectious process. antibody testing is not needed.
Colchicine Is a useful medication for inflammatory Anti-Jo-1 antibodies are found in patients with anti-
arthritis caused by crystals composed of monosodium urate synthetase syndrome, which causes myositis and interstitial
or calcium pyrophosphate. The lack of response to naproxen lung disease. This patient lacks these din ical features, as well
suggests that crystal-associated arthritis is an unlikely diag- as inflammatory arthritis and mechanic's hands (hyper-
nosis. More importantly. infection must be ruled out with keratotic fissuring of the palmar and lateral surfaces of the
arthrocentesis before colchicine therapy Is Initiated. fingers) that are often part of the syndrome. Therefore. he
Starting prednisone for a presumed RA flare would should not be tested for these antibodies.
not be appropriate until Infectious arthritis was ruled out Anti-Ul-ribonucleoproteln antibodies is associated
by performing arthrocentesis and analyzing synovial fluid with l>ystemlc lupus erythematosus and miXed connective
for leukocyte count and cultures. The elevated leukocyte tissue disease, an overlap <tisease containing features of myo-
count and erythrocyte sedimentation rate cannot differenti- sitis, systemic sclerosis and lupus; however, the patient has
ate whether the underlying cause of the patient's arthritis is negative antinuclear antibodies and aside from Raynaud
infection or RA flare. phenomenon and joint pain has no other clinical features
Adding sulfasalazlne to the patient's medication regi - suggestive of these conditions.
men would only be appropriate if his RA was not well con-
UY POINT
trolled (swelllng. pain. and tenderness In multiple joints).
Single joint Involvement Is highly suspicious of another • Ninety percent of cases of mixed cryoglobulinemia are
diagnosis, particularly infection. associated with hepatitis C virus infection; therefore.
assessing for hepatitis C vims infection in an individ-
KEY POINT
ual with cryoglobulinemia is indicated.
• Arthrocentesis followed by intravenous antibiotics is
appropriate for patients with suspected infectious Bibliograp hy
arthritis. Ca<.vub P, Comannon<l C, Oomont f. Savey L. Saadoun 0. Cryoglobulinemia
vasculitis. Am 1 Med. 2015;128:950-5 . [PM!l): 25837517) doi:JO.J016/ j.
umjmed.201S.02.017
Bibliography
Wang Ot\, Tambynh PA. SepUcarthrltls In Immunocompetent and lmmuno·
suppressed hosts. Best Pract Res CllnRheumatol. 2015:29:275-89. [PMID:
26362744] doi:IO.JOJ6/J.berh.20JS.OS.008 Item 84 Answer: 8
Educational Objective: Diagnose psoriatic arthritis.

Item 83 Answer: E The most likely <tiagnosis is psoriatic arthrllis. Characteristic fea-
tures of psoriatic arthritis include psoriasis, enthesitis, dactylitis
Educational Objective: Diagnose hepatitis C virus-
("sausage digits•), tenosynoVitis, arthritis ot· the distal interpha-
related cryoglobuUnemJc vasculitis.
langeal joints, asymmetric oligoarthritls, and spondyllt1s. This
Hepatitis C antibody testing is the most appropriate test to patient has an asymmetric, oligoart1cular arthritis Involving the
perform next in this patient who likely has cryoglobuline- small joints of the hands (particularly the distal joints) and feet,
mic vasculitis. This patient has a small-vessel vasculitis doc- a history of dactylitis, negative autoantibodies, and new bone
umented on skin biopsy, along with Raynaud symptoms, fOrmation and pencil-in-cup deformities on radiographs, all of
arthralgia, sensory neuropathy, and abnormal urinalysis that which are compatible with the diagnosis of psoriatic arthritis.
may represent glomerulonephritis. The presence of cryoglob- Patien ts with a characteristic pattern of psoriatic joint involve-
ulins in this setting suggests cryoglobulinemic vasculitis: pos- ment (especially distal interphalangeal arthritis andlor dactyli-
itive rheumatoid factor and low C4 are characteristic. Ninety tis). but Without apparent psoriasis, should undergo a thorough
percent of cases of mixed cryoglobulinemia are associated examination tor occult psor1atic skin changes (for example.
with hepatitiS C virus infection; thls infection may go unrec- scalp. umblll.cus, gluteal cleft, genitals) and careful nail exam-
ognized for many years before the development of vasculitis. Ination. Although the characteristic skin rash is absent, the
It is important to assess for hepatitis C virus infection In an patient did have psoriatic nail changes characterized by ony-
individual with cryoglobulinemla; treatment of the hepatitis cholysis, pitting. and trachyonychia (roughened surface).

147
Answers and Critiques

Gout can occur as a monoarticular or oligoarticular significant prognostic and therapeutic implications. Kidney
inflammatory arthritis, most typically starting in the first biopsy is the diagnostic test of choice to assess and categorize
metatarsophalangeal joint but, subsequently. Involving joints the kidney disease and should be perfonned In most cases.
in the upper and lower extremities. An increasingly polyar- Indications for kidney biopsy are as follows: increasing serum
ticular pattern, however, is not likely to be seen in a patient creatinine without explanation, proteinuria >1000 mg/24 h.
without kidney disease, without hyperuricemia, and with- proteinuria >500 mg/24 h with hematuria, and proteinuria
out medication exposure that might be expected to increase >500 mg/24 h with cellular casts.
the serum urate leveL Testing for antlphospholipid antibodies is appropriate
Reactive arthritis is an inflammatory arthritis that to complete th.i.s patient's evaluation but will not help to
occurs in the setting of a defined antecedent infection, usu- further assess her kidney disease.
ally of the genitourinary or gastrointestinal tract These cases Cf of the abdomen and pelvis may reveal masses or
often present as an asymmetric oligoarthritis. Up to so·x. retroperitoneal obstruction but is unlikely to be helpful in
of cases resolve by 6 months, but 20% of patients prog- diagnosing the cause of the nephrotic syndrome. Renal vein
ress to chronic disease. This patient has arthritis lasting thrombosis can present with proteinuria but cannot account
almost a decade in the setting of psoriasis and should not be for this patient's rash, polYdrthritis, or hypocomplementemia.
attributed to reactive arthritis. Renal arteriography should be performed in patients with
Rheumatoid arthritis is characterized by a symmetric medium-vessel vasculitis such as polyarteritis nodosa. This
polyarticular inflammatory arthritis involving the proxi- patient hasSLE. and the wsculitis in SLE is small-vessel immune
mal interphalangeal and metacarpophalangeal joints in complex- mediated, in which arteriogram is usuaUy normal and
the hands and the analogous joints In the feet. and com- would unnecessarily expose the patient to nephrotoxic dye.
monly affecting the wrists and ankles. Radiographic find- Serum and urine protein electrophoresis can assess a
ings include periarticular osteopenia and marginal (near the myeloproliferative disorder, wh.ich is not suspected in this
edges of the joint) eroslons. This patient does not have phys- patient with msh, polyarthritis, and the nephrotic syndrome.
ical, radiographic, or laboratory findings (negative rheuma-
KEY POINT
toid factor and anti-cyclic citrullinated peptide antibodies)
to support this diagnosis. • Kidney biopsy is the diagnostic test of choice to assess
and categorize kidney disease in patients with sys-
KEY POINT
temic lupus erythematosus and is usually essential to
• Characteristic features of psoriatic arthritis include make therapeutic deCISions.
psoriasis, enthesitis, dactylitis. tenosynovitis. arthritis
of the distal interphalangeal joints, asymmetric oli- Bibliography
goarthritis, and spondylitis. i\lmaanl S. Me:~rn A. Rovln BH. Up<late on lupus nephritis. Oln J Am Soc
Nephrol. 2016. [PMIO: 27821390]

Bibliography
Raychaudhurl SP. Wilken R. Sukhov AC. RAychaudhuri SK, Maverakis E. Ite m 86 Answer: C
Management of psoriallcllrthritis: E<lrly diagnosis. monitoring of disease
severity and cutting edge therapies. J Autolmmun. 2017;76:21-37. (PMID: Educational Objective: Manage neuropathlc slde effects
27836567) dol:!O.!Ot6fj.jaut2016.10.009
oflefiunomlde.
The most appropriate next step in management is to stop leflun-
C] ltem 85 Answer: C omide. Letlunomide inhibits lymphocyte activation by block-
ing the pyrimidine synthesis pathway. It is approved to treat
Educational Objective: Evaluate kidney disease in
rheumatoid arthritis (RA), in which its efficacy is comparnble
systemic lupus erythematosus.
to methotrexate. Common toxidties include gastrointestinal
Kidney biopsy should be perfonned to assess this patient's upset, diarrhea. aminotransferase elewttons. cytopenias, infec-
kidney disease. Lupus nephritis occurs In up to 70% of tion, and teratogenesis. An uncommon side effect Is peripheral
patients with systemic lupus erythematosus (SLE); the pres- neuropathy, which is usually axonal and may include sensory,
ence of anti- double-stranded DNA antibodies is a marker motor, or mixed findings. The neuropathy can be severe but is
for risk. This patient's symptoms and laboratory findings. usually self-limited if the drug is discontinued, highlighting
including constitutional symptoms, inflammatory poly- the importance of early recognition. Low-dose methotrexate
arthritis, a malar (butterfly) rash, positive autoantibodies in combination with leflunomide is an infrequently employed,
(antinuclear, anti-Smith, and anti-double-stranded DNA but proven, strategy fbr treating refractory RA.
antibodies), hypocomplementemia, and proteinuria, are Beginning gabapenttn as a treatment for peripheral
suggestive of SLE. She likely has the nephrotic syndrome neuropathy would be an appropriate adjunctive therapy to
due to lupus nephritis, and further evaluation is needed. stopping leflunomlde, but the definitive treatment is to stop
Kidney disease is frequent in SLE and can be seen in lO% the offending agent.
to 30% of patients at initial presentation. There are six dif- High-dose glucocorticoids would be appropriate in
ferent pathologic classes of kidney involvement in SLE with the setting of rheumatoid vasculitis. Although peripheral

148
Answers and Critiques

neuropathy may be present in this rare complication of RA. level to less than 6.0 rng/dl (0.35 rnrnoi/U in patients With-
rheumatoid vasculitis invariably occurs in the setting of out tophi and Jess than 5.0 mg/dL (0.30 mmol/L) in patients
poorly controlled disease. In addition. other manifestations with tophi. Based on both the ACP and EULAR guidelines,
of vasculitis would typically be present, including cutaneous this patient meets criteria for more aggressive therapy.
findings (ulcers) and ocular disease (scleritis, episcleritis, Adding probenecid might be reasonable in some cases
uveitis). This patient has well-controlled RA and has no asso- but is contraindicated In this patient because of her esti-
ciated systemic or cutaneous findings of vasculitis, making mated glomerular filtration rate of less than 60 mUmin/
the diagnosis extremely unlikely and the need for increasing 1.73 m 2 , tophi, and recurrent nephrolithiasis.
the prednisone unnecessary. Pegloticase is a co~tly drug that should be considered
Methotrexate inhibits folic acid metabolism and only when other options have been exhausted.
increases extracellular adenosine levels. lt is the recom- Increasing colchicine to twice dally for prophylaxis of
mended initial clisease- modifying antirheumatic drug acute attacks offers no clear benefit over once daily dosing.
for most patients with RA. Methotrexate is administered More importantly, this would not affect the serum urate level
weekly along with dally folic acid supplementation, which or cause resorption of the tophi.
limits toxicity without affecting efficacy. Potential toxici-
KEY POINT
ties include hepatitis and bone marrow suppression (leu-
kopenia, anemia). Patients with liver disease should not • For patients with inadequately treated tophaceous
receive methotrexate. and limitlng alcohol intake is strongly gout wbo tolerate allopurinol, the dose can be titrated
advised. Methotrexate does not cause peripheral neuropathy to a maKimwn of 800 mgtd in 100-mg increments to
and should not be discontinued. alleviate symptoms.
KEY POINT
Bibliography
• An uncommon side effect of letlunomide is peripheral Qnseem A, HQrt!S RP. Forclea MA; Clinical Guidelines Committee of the
neuropathy, and definitive treatment is discontinua- American College or Physicians. Management of Acute and Recurrent
Gout: A Clinical Pnu:t!ce Guideline From the American College of
tion of the medication. Physicians. Ann Intern Med. 2017:166:58 · 68. IPMID: 2?802S08l
doi:I0.7326/M16-0570
Bibliography
Richards Bl, Spies J. McGill!\, Rltha•'ds CW, V.tlle J, Ill easel Jf. et al. Effect of
leflunomlde on the peripheral nerves in rheumatoidarthrlrls. Intern Mcd Item 88 Answer: C
1. 2007:37:101-7. IPMIO: 17229252] Education al Objective : Identify puJmonary arterial
hypertension as a leading c.a use of mortality In mixed
connective tissue disease.
Item 87 Answer: C
Pulmonary arterial hypertension (PAH) and interstitial lung
Educational Objective: Treat tophaceous gout with
disease are the two conditions most commonly associated
appropdate urate-lowering therapy.
with premature mortality in patients like this with mixed
Increasing allopurinol to 500 mg/d is the most appropriate connective tissue disease (MCfO). MCID is rare (1:1,000,000).
next step In management for this patient with symptomatic Age at onset is between 30 and SO years, with a 9:1 female pre-
tophaceous gout Although she has stage 3 chronic kidney dominance. Most patients have no known risk factors. MCfD
disease (with an estimated glomerular filtration rate <60 mU ls an overlap syndrome that includes features ofsystemic lupus
min/1.73 m 2), she is tolerating allopurinol, 400 rng/d, without erythematosus (SLE), systemic sclerosis, and/or polymyositis
issue and does not take a diuretic, so she is not at high risk tbr in the presence ofanti-Ul-ribonuc!eoproteln (RNP) antibod-
allopurinol hypersensitivity. Allopurinol can be titrated to a ies. More than SO'X. of patients With MCfD have hand edema
maximum of 800 mg/d In 100-mg increments. and synovitis at disease onset About one third develop myosi-
It is important to note that two recently published guide- tis, and nearly half develop decreased esophageal motility and
lines diiier regarding the role of pharmacologic urate-low- fibroslng alveolitls. PAH occurs In 20~•• with fatigue often the
ering therapy in patients with gout. The 2016 American lnltlaJ symptom. PAH (and Interstitial lung disease) is associ-
College of Physicians guidelin e (http://annals.org/aim/ ated With poor prognosis and is a major cause of mortality in
article/2578528/rnanagement-acute-recurrent-gout-dlni- patients with MCfD. The course ofMCTD is variable; the like-
cal-practice-guideline-from-american-college) notes a lack lihood of developing PAH, Interstitial lung disease, and/or car-
of evidence supporting a specific target l.evel for urate lower- cliovascular disease increases With disease duration. Regular
ing; this guideline stresses discussing the risks and benefits of follow- up and mon itoring for these conditions is warranted.
urate-lowering therapy with patients and suggests a "treat to Patien ts with MCTD should undergo high-resolution Cf of
avoid symptoms" approach w ithout specifically considering the chest, echocardiographY, and pu1monary function testing.
the serum urate levels. The 2016 European League Against Central nervous system (CNS) disease Is common in
RJleumatlsm (EULAR) recommendations support a "treat-to- MCfO, with trigeminal neuralgia occurring in up to 25'X, of
target" approach (consistent With the 2012 American College patients. CNS vasculitis ls extremely rare, and CNS disease
of Rheumatology gout guidelines). redudng the serum urate is not typically a cause of mortality in patients with MCTD.

149
Answers and Critiques

Pericarditis is likely the most common cardiac man- use of arthroscopy with debridement.'Ihis recommendation
ifestation ot' MCTD and while contributing to excess mor- is based on evidence from randomized trials and systematic
bidity. It does not result in premature death. As with other reviews that fail to show benefit for knee pain with or with -
chronic inflammatory d iseases, premature atherosclerosis is out radiographic knee OA.
common in patients with MCTD and can contribute to both Pregabalin, which binds to the a2-o subunit of volt-
morbidity and early mortality. age-gated calcium channels within the central nervous
Kidney disease is tbund in approximately 25% of system and inhibits excitatory neurotransmitter release,
patients with MCfD and mo~>t often presents as a membra- is not a recognized treatment for OA. It is currently FDA
nous nephropathy that typically does n ot lead to premature approved for flbromyaJgia, neuropathic pain, postherpetic
death. Glomerulonephritis is rarely observed. Renovascular neuralgia, and partial- onset seizures. Because it can cause
hypertension as seen in systemic sclerosis is a mre cause of euphoria, pregabalin Is categorized as a schedule V con-
mortality. trolled substance.
Trn madol is a centrally acting synthetic opioid analgesic
KEY POINT
that also weakly inhibits the reuptake of norepinephrine
• In patients with mixed connective tissue disease, pul· and serotonin. Tramadol may rarely be warranted to control
monary arterial hypertension and interstitial lung dis- pain in patients with OA who have not responded to other
case are associated with premature mortality; there- agents or are poor candidates for other interventions to treat
fore. regular follow up and monitoring for these painful joints, such as surgery. Compared with placebo,
conditions is warranted. tramadol results in a statistically significant but relatively
small improvement of OA- related !<nee pain. Slgnlticant side
BibUography effects sufficient to warrant drug discontinuation include
HajasA. Szodorny P. Nakken B. GaalJ, ZOld t::. L:!czik R, e r al. CIJnJcolcourse, nausea , vomiting, and constipation.
prognosis, and causes of death in mixed conn~ctive tissue dis«aSe.
J Rheumatol. 2013;40:U34·42. rPMTD: 23637328] doi:J0.~8991 KEY POINT
jrheum.121272
• Intra-articular glucocorticoid injection is a treatment
option for patients with localized osteoarthritis.
Item 89 Answer: C
Bibliography
Ed ucationa l Objective : Treat osteoarthritis with intra- JOni P. Hart R, Rutjes AW, Fischer R, Silletta MG, Reichenbach S, et al.
articular glucocorticoids. lnrrn-arlicular corticosttroid for knee ~t~rthritis. Cochmne uaraoose
Syst Rev. 2015:CDOOS3~8. [PMID, 26490760) doi:l0.1002/146S18SS.
Intra-articular glucocorticoid injection is an appropriat e CD005328.pub3
treatment option for this patient with knee osteoarthritis (OA)
not adequately controlled with acetaminophen or an exercise
program. Given the localized nature of her symptoms, and her Item 90 Answer: C
more specific desire for short-term relief while she travels, she
Educational Objective : Treat refractory systemic lupus
could benefit from targeted treatment. Despite a lack of high
erythematosus with bellmumab.
quality of evidence, tTials comparing intra-articular glucocor-
ttcoids and placebo generally show that the treatment is safe Belimumab Is the most appropriate therapeutic option to
and may sign ificantly improve pain and function for patients consider in thJs patient with a history of systemic lupus ery-
in the short term. Furthermore, U.S. and international OA thematosus (SLE) who continues tO have active disease man-
guidelines support the use of intra-articular glucocorlicoids ifestations primarily affecting the skin and joints while she Is
for knee OA. A recent 2-year study has called the efficacy on appropriate standard immunosuppressive therapy, includ-
and long-term safety of intra-articular glucocorticoid injec- ing glucocortlcoids. She has not been able to tolernte other
tions into question, with possible negative effects on cartilage immWlosuppressive agents commonly used to treat active
thickness, an outcome with unclear clinical meaning. Arthro- disease manifestations. Therefore, an alternative approach or
centesis and joint injections are safe procedures in patients escalation of the th erapy is appropriate for this patient. Beli-
receiving therapeutic doses of anticoagulation thempy, and mumab is FDA approved as an addition to standard therapy in
are rarely associated with significant bleeding. patients who have SLE with mild to moderately active disease.
Alendronate is an osteoporosis medication that inhibits Bellmumab is designed to inhibit B-lymphocyte stimulator
bone resorption via actions on osteoclasts or on osteoda:."t (BLyS) protein. BLyS plays a key role in the selection, matura-
precursors. Risedronate, another bisphosphonate, has been tion, and survival of B cells, and it has a significant role in the
studied in osteoarthritis, but results have not been favorable pathogenesis of SLE. The addition ofbelirnumab to standard
for OA symptom modification. Risedronate did reduce a therapy reduces d isease activity levels and delays flares in
market of cartilage degradation; nevertheless, it Is not a cur- patients with serologically active SLE or prior glucocorticoid
rently recommended OA treatment use. Its use is also associated with improved quality of life in
For patients with symptomatic knee OA, the American appropriate.ly selected patients. Belimumab has been found
Academy of Orthopaedic Surgeons recommends against the to be most effective In patients with mucocutaneous and jOint

150
Answers and Critiques

manifestations; Its role In other disease manifestations contin- and linear radiodensitiesof calcification within the cartilage
ues to be studied and evolve. on radiographs. It does not result in ossification of spinal
Abatacept is a T- cell costimulatory inhibitor that has ligaments and entheses.
been tried in SLE but not found to be effective. Psoriatic arthritis can affect the spine, with intlamma-
Adalimumab is a tumor necrosis factor (TNF)-a inhi· tory back pain symptoms similar to those listed with anky-
bitor etlectlve in the treatment of rheumatoid arthritis and losing spondylitis. Differences between psoriatic arthritis
spondyloarthritls. BlockingTNF In patients with SLE has not and DISH include that sacroiliac joints may be asymmetri-
been effective and may potentially worsen the disease. cally involved in psoriatic arthritis but are not involved in
The interleukin (lL)-17 inhibitor secukinumab Is an DISH, and the spondylitis is random and not necessarily
approved therapy for psoriatic arthritis and ankylosing continuous in psoriatic arthritis, whereas a defining char-
spondylitis. The role ofiL-17 and its blockade in SLE is under acteristic of DlSH is involvement of four or more vertebral
investigation but is currently not well elucidated in SLE. bo dies. DISH syndesmophytes (ligamentous ossification) are
Secuklnumab has not been studied in SLE and is not an located on the anterior and lateral margins of vertebrae, but
appropriate therapy. tltey tend to be nonmarglnalln psoriatic arthrjljs
KEY POINT KEY POINT
• Belimumab is FDA approved as an addition to stand· • Diffuse idiopathicskeletal hyperostosis is a noninflam-
ard therapy In patients who have systemic lupus ery- matory condition that involves ossification ofspinal lig-
themaLOsus with persistent mild to moderately active aments and entheses and usually presents as back pain
disease. and stiffness; characteristic radiographic changes
include confluent ossification of at least four contiguous
Bibliography vertebral levels, usually on the right side of the spine.
Gateia A. DeSanctis JB. A review ot cl!ntcnl trlalsofbellmurnah In the m~n­
~ement of Systemic lupus erythemntosus. Curr Pharm De:;.
2016;22:6306- 6312 . [PMTIJ: 2758720JJ dol:l0.2174/l38161282266616083ll Bibliography
03254 Mader R. Verlaan JJ. Buski111 D. OitTuse idiopathic s~let:ll hyperosto.~is:
clinical fe:Jturcs and pathogenic mechanisms. Nat Rev Rheumntol.
2013;9:741 -SO./PMID: 24189840) doi-IO.I038/nrrh~um.2013. 165

Item 91 Answer: C
Educational Objective: Diagnose dilfuse idiopathic ltem92 Answer: C
skeletal hyperostosis. Educational Objedive: Diagnose colchicine and
clarithro mycin drug interaction.
Diffuse Idiopathic skeletal hyperostosis (DISH) is the most
likely diagnosis. DISH is a noninflammatory condition involv- Discontin uation of colchicine and darithromycin is the most
ing ossification of spinal ligaments and entheses. DISH can be appropriate next step in management. The coadministration
asymptomatic or present In various ways, Including as pain of colchicine and clarithromycin can result in potentially fatal
and stiffness in the spine, with the thoracic spine being most colchicine toxicity that manifests as rhabdomyolysis. acute
often involved. Although features such as ligamentous ossifi- kidney Injury, and pancytopenia. Colchicine is metabolized in
cation are shared with ankylosing spondylitis, the sacroiliac the liver by the CYP3A4 cytochrome and should be avoided in
joints are not involved in DISH. The most characteristic radio- patients takingCYP3A4 inhibitors such as clarithromycin and
graphic change of DISH is confluent ossification of at least fluconazole. Coadministration with clarithromycin is particu-
four contiguous vertebral levels, usually on the right side of larly concerning. because there have been several case reportS
thespine.1he left-sided sparing is possibly related to mechan- of fatal outcomes with the combination (even when taken for
ical pressure from the aorta, and its pulsations, as a barrier a short time). Once the patient recovers. however, colchicine
toward production of bony hyperostosis on the len side. may be reinitiated (as it Is the combination of the two drugs
Ankylosing spondylitis is characterized by bilateral that Jed to the current scenario).
sacroiliac joint abnormalities on spinal radiographs. Other High-dose glucocorticoid therapy would be indicated
spinal sections may also be involved, but sacroiliac joint in the event of immune-mediated hemolytic anemia and
involvement is cardinal for diagnosis. Inflammatory back leukopenia (Evans syndrome). Colchicine toxicity. however.
pain Is the most common symptom and is usually comprised is not immune mediated and would not respond to gluco-
of four or five of the following features: onset of back dis- cortlcoids.
comfort before the age of 40 years; Insidious onset; Improve- Plasma exchange would be appropriate for throm-
ment with exercise; no improvement with rest; and pain at botic thrombocytopenic purpura (TIP). Although TIP can
night (with improvement upon arising). cause thrombocytopenia and acute kidney injury, the lack
Calcium pyrophosphate deposition may affect the axial of schlstocytes on blood smear a rgues against mlcroangio·
spine but more often affects the peripheral skeleton. It is pathic hemolytic anemia, and other features of TIP such
often asymptomatic, Is not known to have a predilection for as fever and mental status changes a re a lso absent. Plasma
a particular segment of the spine, and appears as punctate exchange is not effective for colchicine toxicity.

151
Answers and Critiques

There is no need to discontinue febuxostat; this drug face and presents as hypo and/or hyperpigmented, possibly
CJ can be continued in Jtdlients with chronic kidney disease erythel'l"'::tous, patches or thin plaques that may be variably
CO'll and would not cause the clinical picture described. atrophic or hyperkeratotic.
KEf POINT KEY POINT
• Coadministration of colchtcine and CYP3A4 inhibitors • Subacute cutaneous lupus erythematosus (SCLE) can
(such as clarlthromycln and fluconazole) should be present as annular with central clearing or papulo-
avotded because potentially fatal colchicine toxicity squamous with patchy erythematous plaques and
with kidney failure. rhabdomyolysis. and bone mar papules. and both forms can be seen in the same
row suppression may occur patient about 50'~ . of patients with SCLE also have
systemic lupus erythematosus.
Blbliography
Hung IF. Wu AK. Cheng VC, 'lang AS, T11 KW, Yeu11g CK, et ~l. F~tal inlerdc- BibUography
Uon between clarirhromycln and colchicine in pallents with renallnsuf- Okon I.G, Werth VP. Cutaneou~ lupus erythematosus: diagnosis and treat·
1\clency: n retrospecll\~ study. Clin Infect Dis. 2005;41:2~1-300. {PMID: ment. Best Pr3cl RcsCI In Rheumatnl. 2013:27:391·404. [PMID: 24238695]
16007523] dol:lO.l016/j.berh.2013 07.008

Item 94 Answer: D
Item 93 Answer: 0
Educational Objective : Diagnose ankytosing
Educational Objective : Diagnose subacute cutaneous
spondylitis.
lupus erythematosus.
The most appropriate test to perform next is plain radiog-
This patient hilS subacute cutaneous lupus erythematosus
raphy of the sacroiliac jolnts. This patient has features of
(SQ..E) and likely has systemic lupus erythematosus (SLE).
inflammatory back paJn (o~ before age 4S years. duration
She has a new rash on the torso w1th annular and patchy pap-
>3 months. Insidious onset. morning stiffness >30 minutes,
ular areas. This rash Is most likely SCLE. a photosensitive rash
Improvement with exercise, no Improvement With rest. awak-
occurring especially on the arms, neck, upper trunk. and face.
ing from pain with Improvement on arising. alternating but-
The rash can be annula r with centrnl dealing or papulosqua-
tock pain). as well as antecedent history of anterior uveitis. He
mous with patchy erythematous plaques and papules. ln
should therefore be evaluated for ankylosing spondyUtis. The
some patients, the rash may have both of the characteristics,
American College of Radiology appropriateness criteria rec-
as seen In this patient SCLE often has a fine scale that may
ommend radiography of the sacroiliac joints and spine as the
leave postlnflarnmatory hypo- or hyperpigmentation. SCLE
initial evaluation of patients With inflammatory sacroj.liac or
is associated with anti-Ro/SSA antibodies, w1th a prevalence
back symptoms for evidence of sacroiliitls, Including erosions,
of75%. About SO% of patients with SCLE also have SLE. These
evidence of sclerosis, and widening, narrowing, or partial
patients more typically have mild systemic symptoms, most
ankylosis of the sacroiliac joints. The presence of such findings
commonly arthritis and myalgias; patients with lupus vas-
would confirm the diagnosis of ankylosing spondylitis.
culitis. central nervous system lupus, and nephritis are found
A bone scan locks speciflclty for the changes of sac-
in less than lO'X, of patients with SCLE. Psoriasis can present
roililtls and Is not of value In the diagnosis of ankylosing
with similar lesions but are usually photoresponsive (improve
spondylitis.
with sun exposure). unlike this patient
Essentially 100')(, of palients with acute cutaneous lupus
cr of the sacrolliac joints is not considered an appro-
priate initial diagnostic test for ankylosing spondylitiS. It
erythematosus (ACLE) have SLE. ACLE may present in mul-
exposes the patlent to unnecessary amounts of radiation.
tiple forms. with the most commonly recognized being a
MRI Is also unnecessary 1n most cases of suspected
characteristic localized malar (butterfly) eruption. Less com-
ankylosing spondyliUs because the diagnosis can often be
monly It can appear as a generalized eruption, which typi-
confirmed with less costly radiography. Although MRl is
cally appears as an erythematous maculopapular eruption of
more sensitive than r.tdiography or cr in detecting sac-
sun-exposed skin such as the extensor surfaces of the arms roiUitis. it Is less speciflc and can result in a high yield
and hands. of false-positive test results. MRl may be useful to detect
Patients with dermatomyositis frequently have areas of
inflammation in patients w1th high clinical suspidon but
poiklloderma with iU-deftned patchy erythema and ·salt-and-
normal or equivocal radJographs.
pepper" dysplgmentatlon accompanied by telangiectasias on
the chest or upper back. There may be slightly violaceous UY POIIT
erythema over the •y• of the neck, chest, and the upper back • Plain radiography of the sacroiliac joints should be
(the "Shawl sign~) or on the lateral hips (the "holster sign"). obtained in patients with suspected ankylosing spon-
Discoid lupus erythematosus (OLE) occurs in 20'X. of dylitis to evaluate lor evidence of sacroiliitis. including
patients with SLE but more commonly occurs as an isolated, erosions. sclerosis. and widening, narrowing, or par-
nonsystemic finding; patients with isolated OLE usually do tial ankylosis of the sacroiliac joints.
not go on to develop SLE. OLE usually affects the scalp and

152
Answers and Critiques

Bibliography Bibliography
T3Utq! JO. Chhabr:~ II. Coll,..rt RJ\ llnkylusing spoodylitls and ax•al spon- l:uourte A. B:lrdln T. R•chelte I' Pror>hy1~xis for ll<'Ute gout !lues after Ini-
dyloarthrltis. N lingl J Med 2016.J 7-1.2563-74 . IPMIO: 27355535) tiatiOn ofurdte-l~rlng thm~py. RheumatolOGY (Oxford) 2014;53,192()..6.
dok i0.10561N ~:J Mra14061R.l [PMIO: 21758886) dOJ:I0.10931rhcumatology/keul57

Item 96 Answer: 8
Item 9 5 Answer: C
Educational Objective: Diagnose hlp osteonecrosis
Educational Objective: Prevent gout attacks during the related to systemic lupus erythematosus.
initiation of urate-lowering therapy.
MRl of the hlp Is the most appropriate test to perform
Discontinuing colchicine and beginning an NSAID such as next in this patient with systemic lupus erythematosus
meloxlcam Is the most appropriate next step to decrease (SLE) and a past exposure to high-dose glucocorticoids,
the frequency of this patient's gout attacks. He has experi- who now presents with unexplained groin pain. She is at
enced several acute gout attacks since beginning allopuri- high risk for developing osteonecrosis affecting the hlp/
nol 2 months ago. It Is common to experience "mobilization femur. Osteonecrosis Is a serious complication ofSLE that
attacks" at the Initiation of urate-lowering therapy, which most commonly affects the hips but can also involve other
occur when urate body stores shift from joints and soft large joints and should be suspected when there Is other-
tlssues to the vasculature. Hence, all patients beginning wise unexplained pain and/or reduced range of motion.
urate-lowering therapy should also receive a prophylac- Chronic prednisone doses (:>20 mg/d), severe/active SLE,
tic agent to prevent against mobilization flares for 6 to and vasculitis are all associated with Increased risk. Hip
12 months. without which the average patient will experi- radiography can detect advanced disease, but MRI is the
ence several acute attacks. COlchicine Is the primary pro- modality of choice for sensitive evaluation of early disease.
phylactic agent employed (once or twice dally) for this Small lesions can improve and resolve spontaneously, but
role, but low-dose glucocortlcolds and low-dose NSA1Ds larger lesions usually lead to bony collapse and structural
are also effective. Choice of therapy should be tailored to sequelae. MRI s hould be performed In this patient with
the individual's comorbid conditions. This patient is cur- nonnal pelvic and hlp radiographs.
rently taking colchicine, which Is most likely causing the The American COllege of Radiology (ACR) approach to
diarrhea. In this case, dlscontlnuation of colchicine and imaging for suspected osteonecrosis recommends CT only
initiating a low-dose NSAID such as meloxicam is the most for patients who cannot undergo MRI. CT is somewhat more
appropriate choice. All NSAIDs are considered equivalent sensitive than plain radiography In the diagnosis of osteone-
for this purpose. crosis but not nearly as sensitive as MRI.
Adding prednisone would be a reasonable option for Radlonuclide bone scan will detect increased bone
prophylaxis but Is not Ideal for this patient with insulin- turnover at the junction, osteonecrosis, and reactive bone
dependent type 2 diabetes mellitus, because even low-dose resulting In an Image oflncreascd tracer uptake surrounding
glucocortlcoids may result In hyperglycemia and increase a cold area (doughnut sign). However, radionuclide bone
insulin requirements. scan Is significantly less sensitive and less specific than MRI
Adding the uricosuric agent probenecid or Increasing in the detection of early osteonecrosis lesions and is not usu-
the allopurinol would not afford protection against mobi- ally recommended as a flrst-line diagnostic test.
lization flares because they are both urate-lowering agents, If MRi cannot be performed in patients with suspected
not prophylactic agents. Intensifying urate-lowering therapy early osteonecrosis, a reasonable alternative Is bone scan
would be a reasonable choice If the recutTent attacks are with single photon emission Cf (SPECT). This imaging
thought to represent a failure of urate-lowering therapy, modality is more sensitive than radiography, but the ACR
rather than mobilization attacks, as is suspected with this Appropriateness Cr1terta for Osteonecrosis of the Hlp des-
patient. ignated SPEer as a second-line test to MRI for the detection
Increasing colchicine would be a reasonable option, of early lesions.
were it not for the diarrhea that began with the institution
JC EY POI•T
of colchicine 2 months ago. Diarrhea Is one or the most
common side effects of colchicine, and Increasing the dose • Osteonecrosis is a complication of systemic lupus
would worsen gastrointestinal toxicity. erythematosus most commonly affecting the hips
and should be suspected when there Is otherwise
lEY POI•T
unexplained pain and/or reduced range of motion:
• All patients w1th gout begmnlng urate-lowering MRI Is the modality of choice for evaluation of early
therapy should also receive a prophylactic agent such disease.
as colchicine. low dose glucocortlcoids. or low-dose
NSA!Ds to prevent mobilization flares: the choice of Bibliography
prophylactic drug Is determined by patient comor- Contem RP. Bed~ ME. Ben011ente E. ROYer~ no SC. Palm SO. Osteonecrosis
bldltles. In systemic lupus erythematosus. Reumatol Clln. 201S:ll:lSI-S. IPMTO:
25-1414911 doi:lO.I0161).n:uma.20 I~.os.oos

153
Index

Nnrc, P:lge numbers followed Uy fand r indica~ figure and rablc. AntH.MSSA anllhodlcs. 61
respecrtvcly. Test qutStlons ~re lndlcat~ by Q Antinuclear anullodll!~ (ANA), S 61
Antiribosomal PantlbodiCl., 6t
A Anti-Ro/SSA nnllhodlcs, 61
Abol11cepl, 131 Anti-Scl-70 antibodies, 61
In pregnanty. lSI Anti-Smith antibodies. 6t
In meumarold nrthrlris. 21 Antisynthetase 'yndro01e, 45, Qrt
1\CF.Inhibitors Antllhyn>ld drugs. and drug-Induced lupus crythemalOS1.tS, 40t
~nd drug-induced lupus erythematosus. 40t Anti-Ul-RNf' antibodies, 6t
In sderod~m,a renal crisis. 54 Apremllnst, lOt, 11
Acetaminophen. 9 advel'!'e elfects o r, 11
In osteoarrhrt rts. 2/i 10 psoriatic arthril!s. 34
In pregn~ncy.ISI Arthritis
Acupuncrure, for Ohromynlgla. 28 rnonnnrthrltls. 1
Acute calcium pyrophosphate (·rystal arthnrls. S7r. 59, Q28. Q56. See also ollgoanhritts. 2
Calcium pyrophosphate deposition (CPPD) polymhrltls, 7
Acute cutaneous lupus erythemutc>sus (ACl.E). 35. 3Sf, 1\rthrllis·dermotllls sy11drome. 61 Ci2. 62f
Q7-t Arthrllls mutllan~. 11. See ul'iO ~nlarlc anhr111s
Acute lupus pneumnnltts, 37, Ql3 Arthnoccntesls. tn InfectiOus arthritis. 60, Q82
Adallmumab, 121 Artificial tears. 43. Q23
in 181) associated a rthritis. 34, Q37 Aspirin
Adult-onset Still disease (AOSD), 74, QJ6 In giant cell arter1tl~. 67
diagnostic crlter1o for, 74t In Kaw.~sakl dlse:lse. 69
treatment fnr. 7~ Athcmsdemtic heart disease. rhcumltoid arthntis and. 19
Aerobic exercise A! rial myxom~. 66r
In tlbmmyalgla. 28 Auroontibody tests s. 61
In ~eoorthrllfs, 26 Autoimmune ~molytic anemia. 17
Aerobic gram-negative bacilli. 62 Autolnn~mmatoryd~ases. 74. 7St
Alcohol use. 9. 44!, S8 Autologous ~maronoletlc ~rem celltronsplantalion. In systemic sclerosis,
Ali<Xlynlo. 'rl 54
1\llnpurinol. 12 13. Q66 M.athioprtne. 10. 101
advt'rse eCT~L'tS of, 1'I In Reh\l!t syndrome. 7.1
in gout. 58. Q87 In g~nulomatosls with polyanghll~. 70
In pregnancy, 1St In IBD·nssoclated al'thritls. 3.,
Amilrlptylfne, In Obromyalgla, 28 In lgG4 relaK"d dtsea.~. 78
Anakinm. 13! In lnOammatory myopathies. ~7
In gout olr..cks, sa. (lJ in mlcrc~pic poly:mglllfs, 71
In pregnancy. 1St In pregnancy. lSI
Analgesic>, 9. 5ee also Sp<.'Ciflc fiJf)e In relapstnp, polychondrltls, 7'1
in pregnnnly. lSI in systemic lupus erythematosus, 40t
ANCA-associared vasculitis. 69 71 roxiLity fro m, 10
c linical features or. 70t
n
enslnophllle gr.mu lomatosls with noly.lnglltls. 701. I
granulomatosis with pnlyangtltls. 69-70. 701 Buck pain, In spondylonrthrltls, 29
microscopic polyanglltls. 70-71.101. Q2 Bacterial arthritis. 61 . See also lnf~ctlous arthritis
Anemia oflnflnmmatlon. rheumatoid nrthr1t•s and, 19 Basic calcium phosphate (8CPl deposition, S7t, 60
Ankyloslng spondylitis, 30 -31 Reh~t syndrome, 73, QS9
and anterior uveitis, 34 dlagnost ic criteria for. 731
b.1mboo spine appe-.uance, 33, 33f gcn ita! mucos.11 ulcemtions In, 73, 73f
clinical features of, 10. 301 patherKY as..<nclared with. 73. 7lf
dblgnosls of, 32- 33, Q5, Q9, Q94 treatment for, 73
extm articular manlrestatlons oC 31 8eltmu01ab, 13t
radiographic Hndlngs ln. 7t in pregnan~y. lSI
treatment of, 34, Q72 In systemic lupus erythematosus. .,Ot, 41. Q9<l
Anllblotlc:s. fOf rnA:ctfous arthrlrts. 64-6S. 641 alosfmllar agents, 11
Anticentromtre anllhodles. 6t Blin<1ness.ln gi~nr cell arter1tls. 66
Anti-cyclic citrullinated peptfd~ (CCP) antlbodi~ S, 6t Horrello bur'fPorferl. seruloglc testing for. 62. Q16
In rheumatoid ;rthr1tls, 17. Q34 Bill in biopsy, In prima ry angiitis of central nerwus system, 69
Anll-double-stranded DNA anttbod~. 61, 39, 39! Bronchlollh.o;, In rheumatoid anhritls. I9
In SLF. flare. 39, 39r, Q40 Butterfly ~sh. 35. 3S f
Antlepileplics. In fihron1yalgla, 28
Antlhtstoncant1bodtes, 6t c
Anti-HMC Co-A reduct:IS( nnlfhodles. for lmmune-m~lated necrotizing Calcinoo;ls.ln syslemlc sclerosis. -19. Sl f
myopathy. 46-47,471. Q41 C:llcium channel hlocken~
Anll-tnnammatory agents. See also specific o~nt and drug-Induced lupus erythematosus, 40 1
colchicine, 9 In sclen!derma ~nal cr1sls. S4
glucocortlcolds. 8 Calcium pyrophosphate d~ltlno (C:PPn), 59-60
NSAI Os, 8, 8! acute Cf'P crystal arthritis. S7t. 59, Q211. QS6
In pregnancy, 1St asymptomatic, 59. S9f
Anti-Jn-1 antlbodieo;, 6t cartilage calclttes~tlon (chondrocalclne<f<) of' knee. 59, S9f

155
I n d ex

Caldum pyrophosphat( d(posll1on (CPI'IJ) (\.oncmued) D•n·use IdiopathiC ~kclet~l hypenxtosls (IJIS~il. 23, JJ. Q9t
chronic CPI' Cf'Y"I~IInfl~mmotory arthritis 59 r~diogr.tphlc find in~ ln. 7!. 23. 24f
~pklemlok>gy of. S9 OIIII.Jse infiltrutive lymphocyt<~is syndromt! (OILS), 63
management uf. 59. 601 IJigltal ulce"· In )ystemlc .sclem:;h. Sl
OSieoarthrltls wllh CPPO. S9 Di,;coid lupus erythemdiOSUS (IKE). 35- 36, 36f
pQthophys•ology nf. 59 Olse:~.~-modifylng ontlrheumatlcdru~ (DMARI)s), 9
radiograph!~ Ondml!$ '"· 71 In adult-onset Still dlstMe. 74
Camklnum;~b, 13t biologic. 11. 12r. 121, 13t
In gout auaoo. S8 non-TNF-a mhihlturs. II. t3t
in pregnancy. lSI in pn:gnancy, 1St
C·ANCA on!ibodll.'\, 6! TNF-n inhlbtlOrs. II. Ill
Captopril. In scleroderma renal crl~•~. 5~. Q79 nonblologlc. 9. tOt
Cartlla~ biopsy. 71 apremilost. u
Central sensiiiT.a!lon, 27 a7.athloprlne, 10
C.ereb~plnal fluid (C:SI'),In prlmnry anglltl~ oft-entral nervous~yslem. 69 C}ltlopho.~phamide. 10 II
C.ertolfzumah pq;ol. l~t C}ltlosporlnt, II
Cevlmellne. 43 hytl roxychloroquh1c. 'J
Chlkun~unya. arthritis ln. 63 lellunomlde. 10
Cholesterol emboll. 66t methotrexart, 9
Chondroitin sulf~te, In osteo<~rthrltls, 26 mycophenolnte mofetll. 11
Chronic graft-versus-host dtsense, SOl In PJ'e!tnancy. lSI
Cllrullln~. 16 •ulfasnlazlnt. 9
Cocaine. ~41 tacrullmus. 11
COgnitive behavioral theropy, for flbromyalgla, 28 tolilcl!lnlb.ll
Colchicine, 9, 44! in re3ctlve arthritis. 34
in lk~t syndrome, 73 tn relapsing polychondrltis, 74
and darithrom}ltln, 9. Q92 In TakayJsu arterltts, 67
In gout att:1cks, 58 Disequilibrium. prt8'Jbolln and. 9. Q30
n
in hypersensilivlly vasculitis. Olssemlnuted gonococc-~llnfectlon. 61 62. 62f. Q2S
tn pregnancy, I:it DMAROs. See l'>lsease-modlfylng antlrheumatlc drug-; (OMAROs)
In relapsing pol}ochondrllls. 74 DRESS (drug re-Jctfon with enslnophllia and systemic symptoms) !i)'ndrome.
slde etrecu of. 9 13,58
COmplementary and altmtatlve medldne. 14, 16 Drug-induced lupus erythematosus (DIU:), ~0. 401
C.omplemenr system. 4-S Orug-lmluccd myo!Xlthles. 441
Comeo1l n~lt. 19 Dry eye. SH K~~tocolljunctlvltl• <lt'Ca
Coronary ~nery dtse-ol~ (<.:AI.>), ~!ernie lupus erythematosus and. Ouloxetlne, 9
37 In fibmmyalgta. 28
C-~ctive protein (CRP), 4 in OSICOJrthntis. 26. Q3S
In rheumatoid anhrills. 18
Crlroaryt(nold arthritiS, rheumatoid ;lrthrt!ls and. 19 E
Cryoglohulinemlc wsculllls. 71-72 Ehlers Dan los :wndm~ (I:.IJS). 76 771
or.
clinical manlfeslatloM ond dlo~:nools 72 F:nduthelin- t, 18
management or. 72 t:nlhesls. 2
pathophysloiQRY ot 72 Enthesllls. 2
Cryoglobulins, 6!, 71 72 Enzyme-linked lmmul'l06()rbent assay. 5
Cl')'l'lal arthropatha Eosinophilic fal>C'Iitls. SOt
b:lslc calcium phosph~tc deposition. 60 F.oslnophlllc gnnulomatost~ with poly~nghlls (ECPA). 701.71
calcium pyrophosphate deposition. 59-60 Eplsderitls, rheum~ told orthrltts and. 19
golll, 55-59 Erosive ostronrthritt~. 22. Q39
cr. for rheum~tologlc dl~ases. s Erythrocyte sedimentation r~te (I:SR). 2-3
Cydobenzaprine, In 1\bromyai;Jia, 28 In rh~umatold arthritis. 18
Cycfooxygenosc (COX) lnhJbttors. 8. See also NSAIDs Etanercept. 12t
Cyclophosphamide. 10·11, lOt
in cryoglobullnemlc wscullll~. 72 F
In granulomatosis with polyat1(tlltb, 70 F'atigue. In rheumotologlc conditions. 2
In lgA vasculitis, 72 Febuxosw. 13
In microscopic poly.mglltis. 71 In gout. 58
in polyarteritis nodo.~. 68 in pregnancy, LSt
In pregnancy, tSI Felty syndrome. 19-20. QtS
In primary onglftls of central nervou~ system, 69. Q32 F~r. in rheumatologlecondltlons. 2
side elfect~ of, II Flbromyal),'fa, 'I1 28, QS2, Q69
In Sjogren syndrome, '13 2010 American College or Rheumatolr>gy Prelimln~ry Diagnostic Criteria
in systemic lupus erythematosus. 40t. 41 for. 27,281
in systemic scltrosls. 54 dl:~gnosls of. 21. 281
Cyclospor1ne, 101. It epidemiology or. 'I1
in Bch~et syndrome, 73 management ur
tn pregnancy, 1St nonph:lm1acologk' therapy. 'Il-28
in n:hlpslng polycbondr ltls, 7'1 pharmacologic theropy. 28
Cytopenfas. In systemic lupus ~thcm;~tosus, 37 parJ'tophystology of. 27
Axed erythtma, <lSI
D R>llc acid. tn rheumatoid arthritis, 9, 20, Q63
Dapsone fung~il arthritis. 62. Seeotso lnfc<:t1ous arthritis
In hypersensltlvlry vasculitis, 72
In relllpslng polychondr1Us. 74 G
Dengue.63 Gabapenlln. 9
DermatomyOsitis (DM). 43. 45, 45t. Seeolso lnnamm;~lory myopathies in llbromy.~lgta. 28
Dlclofcnoc, 8 Castric antral wscular ectasia (GI.VIi). 52. Q6
in ankyloslng spondylltts. Q72 Genetic diseases of ~•mnectlve tl~~ue. 76, 77r
topical. for elderly J)3!lenl wUh knee 0/\, Q20. QSS Ctant loell arteritis (CCA). 66-67. Q21
OffTuse aiiiCOiar hemorrho~Je. 37 clinical manlfcstalions and diagnosis or. 66
Olfl'usecutancous system•csclerosl' (DcSSc), -18, sot epidemiology or. 66

156
Index

man~gement or. 66 67 I
pathophysiology or. 66 lgA vascullll,, n. Q6S
subcr:~nlal. Q27 lgG4-related dls..oao;e, 76- 78, 78t
Glucoconlcold-lnduced n'tronecm,ls. 8. Q81 cllnlc-JI sign~. n
Glucorortimtd·lnducc<.l osteopumsls. 8 condition' associated wtlh. 78t
pre\'entJOn of, Q45 dlagnctsis or, 78
Glucoconlcolds, R retrnperlton~r.~l ttbrusls. o$$0Ciated worh, Ql7
m adult-onset Sttll dtsease, 74 tre.u:~nt or. 78
~nu.gesof.8 lliotiblll band syndrome, 25
~ru elfects of, 8 tmllglng 'tudles
in Beh<;et syndrome, 73 CT,5
tn cryoglubullnemlc vascultlls, 72 MRI.S
ln eo~inophillc: grJnulomatosls with polyangiltis, 71 rodiogmphy, 5, ?t
In glom cell arterllls, 66-67 for rheumatoid ~rthrltls. 18, rar. 19f
in gout attacks. 58 ultr.IS()nogroplly. s-o
In ~nulomal<~•ls w ith p<>ly\lnglllls, 70 lmmunecomplex-medlaled vasculitis, 71, 7tf
In hyperscn.,ltlvlty va~ulltis, 72 lnmlune-medldted necmtlting myopathy (IMNM). 43, 15-46, ~I. See also
In 181)-as.~lated arthritiS, 34 Inflammatory myopathies
in lgA wscullll~. 72 Immunosuppressive therapy
In Inflammatory myopathies, 47 ~reenlng cecommendatlons ~fore. 14
lntro-artkulnr. 26 in system ic lupu.~ erythematosus, 41
In mlcrtlllCO{llc polyangllll~. 71 Inclusion hody myositis (IBM). ~3. 46. Q22. See also lnllamm~tory
myopathy Induced by, 44t. Q61 myopathies
In polyarttrllls nOdllSII, 68 Indirect lmmunoOuOJ'escenre assay, 38
In pregnancy, lSI lnftetlous arthritis, 571
In primary :~nglltls of Ct'ntrul nervous syst~m. 69 c:austsor. 61 - 64
In reactive ~nhrtrls, 34 fuogallnfmtons, 62
In relapsing polythondrltls, 74 gram-negative Oll,oanlsms, 61 - 62, 62f
In rheumatoid arthritN, 21 grnm-poslrlve oog;tnlsm•. 61
In system lc lupus ~rythemato~us. 11 hepalllts, 63
In Takay~su anerltls. 67 HIV lnfecllon, 62- 63
Glucosamlne. In osteo:ulhrills, 26 Iyme arthritis. 62
Gollmumob, 12t mo.~ulto-bome viruses, 63
Gotrron rash, ~s. 451, 46f MycobactE'rlum rubercufosls. 62
Gout SS-59. Q9S parvovirus 819.63. Q33
~cute gnuty uthrilis. SS , 56 p~het1c: julnt Infections. 63
dlflhenllal dlagnosl~ of. S7t rubella tnfect1on, 63
chronlc recurrent gout. 56 dlnlcul manlfesratoons of. 60
clinical manlfestatoOO$, SS-56 laborntory and Imaging studies ln. 60- 61
diagnosis of, S6-S7. S7f management or. 64-6.5. 641
epidemiology of. ss nsk factors for. 611
hyperummia m, ss. S6t lnflamma.'IOmes, 74
interertllcol gout, 56 Inflammatory bowel diSease -liS.10dotcd arthrlhs. 30t. 32
Pllthophyslology or. ss treatment or, 34. Q37
r~diographlc findings In, 7t Inflammatory myopathies. 43 - 48
tnphaCI!ous gout, 56, S6f, Q3, Q87 autoantibodies in, 47, ~71
treatmtnt or. S7 -58 dermatumyosiHs, 43. 4.5, 4St
acute gout, 57- 58 dla~:no.~ls of. 46-47
prophylaxis, 58 differential diagnosis of. 43. 44t
urnte- lowering therapy, 12-14, ss. Q9S declromyogrJphy In, 47
Gouty cellulitis, 58, QSI epidemiology or: 44
Gr,mulomatosis with polyanglllls (GPA), 69-70. 701, Q29 Imaging studies In, 47
clinical manlf~totlons and dlogno.~ls of. 69-70 lmmune-mc<.llated necroll~lnn myop~thy, 43, 45- 46,
epidemiology and pathophysiology o r, 69 Q41
management or. 70 lnc:lusion body myositis. 43. 46. Q22
nnd malignancy, 46

"
l leberden and Bouchard nodes. 24
Hee.rfordt syndrome, 75
manag~menr or. 47-48
muscle biop~y ln. 47
muscle de.uuctlon In, 43
Heliotrope rash. 4St muscle-related enzymes In, 46-47
Henoch-SchOnlem purpura. See lgA vasculitis pathophysiology of, 44
Hepatllls 8 virus (HBV)-2ssoci~ted PAN, 68 polym)(lsltls. 43, 45
ll~llrls B virus Infection, and a rthritis, 63 prognosis In, 48
HepaUtiSCan tlbody te<;tlng, In c:ryoglobulfnemla. 72, Q83 serum c reatine kln3se tn. 43, ~7
HepallrlsCvlrus lnfi!Ctlon. and nrthrltts. 63 s kJ n blop5Y In, 47
HN-~ssociared arthritis, 62 lnOixlmab, 12t
HLA-8'5801 allele testtng,U- 13, Q66 In I SO-associated arthritis. 34
Hyaluronic acid lnjedlon, lntra-arlil1Jl3r. in o:.'teOal'thritls. 26-27 In psoriatic: arthritis,~. Q71
Hydral:rzlne, and drug- Induced lupus erythenutosus, 40t Interferon :alf2, In Be~t syndrome, 73
Hydrogen breath test, S2 lntcrnuktn {IU-6, 48
Hydroxydlloroqulne. 9, 101 lnterlcukin (IU·l~. ss
In pregnancy. 1St lntersntiallu ng d l - (ILO)
In rheumatoid arthritis, 21 dermatomyositis and, 4S
In Sjogren syndrome. 43 polymyos111s and, 'IS
In systemic lupus erythematosus. 40-41. 40t In rheumatoid arthritis. 19
In undiiTe~ntlated connect!~ !Issue disease, QS4 in systemk: sclerosis, 52, S3t, 54
Hyperalgestn, 27 lntn-artkula r glucocorticoid ln)ttllons
Hyperunsltlvlty vasculitis. 72 in ankyloslng spondyllrt:;. 34
Hyperurlttmla, 55 In gout , S8
cause; or. S6t in osteoarthritis, 26, Q89
and w:>ut. SS In reactl\le arthritis, 34

15 7
Ind ex

lnmvenous immunoglobulm epl~mcology and pathophysiology or, 70


In ~rmatomyosltls. 48 lll:ln:Jgement of, 71
In Kawasaki lllse:ase, 69 MilnaclpOIII, In nbromy:algla. 28
Ischemic opelc neump;~chy, 66 Milwaukee shoulder, 60
Mind-body iot~~ntlons. 16
J Minocydlne, and drug-induced lupus uythm>atosus. <tOt
J~ccoud arthropathy. 42 Mitochoodrlal myopathies, 4-Ct
Jocnt asplr:clion Mixed conn~vt tissue disease (MC!l>), 54, Q88
in :acute monoarthrUis, I, Q44 dlnlcal manlfesutlons and diagnosis of, 541, 55
and S)'ni)VIol Ruid a1101lysls, 7, 7t epidemiology of. 55
Jmnts pathophysiology o~ S5
In osteoarthritis, 74 prognosis of, 55
In rheumatoid arthritis, 18 t9, 181 Rllynnud phenomenon In, SS
In system ic lupus eryth~matosus, 36 treatment for, S4t, ss
undlfferenti~ted connective tissue disease, overlap syndrome. and, 54t
I Mono:~rthr1tls, 1
Kawasaki disease (KO), 69 acute, l, Q44
coronary artery aneurysm and, 69, Q77 chronic, l
prognosis of, 69, qn Morning stiffness
Kcmtttls (co meal lnnammatJon) osteoarthrtfu nnd, 24
rheumatoid arthritis and, 19 1n meumatologlc conditions. 2
KemtuconjunctlvillsSlcco Morphen, SOt
In rheumalold arthritis, 19 MRl,S
In SJOgren syndrome, 42 (see a/so SjOgren syndrome) in hlp osteonecrosis, 36, Q96
Kidney biopsy, In systemic lupus erythematosus. 37. QSS In Inflammatory myopathies, 47
In osteoartMcls, 25
l 1n primary angiitis or centro! nervous system, 69
Laboratory studlt!S, 2 In rhtumatold arthr1rls. 18
autoanlibody tests, S, 61 In spondyloorthrlfls, 33
complemenl components, 4-S Mwcle biopsy, In Inflammatory myopathies, 47
C-~cttv.! protein, 4 Muscular dystrophies. «t
erythrocyte sedlmentlltlon rate. 2-3 Musculoskelw.l paln
leflunomlde, 10, lOt articular, 1
tn pregnancy, lSI inllammatory versus noninflammatory, It
In pSOriatic arthritis, 34 periarticular. 1
In rheumatoid :uthrttls, 20-21 Mycobacterium tuberculosis, 62
sod~ clfects of, tO, Q86 Myoophenolatcmof~.lOt,ll
leslnurad, 13 ln granulomatosis with polyanghtls, 70
In gout. 58 lD lgG4·related disease, 78
Leokocytoclasllc 1135Culltls. 71 In lnflanun:~tory myopathies, 48
Ubm:an-Sack.~ endocanlltls, 37, Q26 1n lntemltlallung disease, S3t, QS3
Umlted cutllneous systemic sclerosis (LcSSc), 48, soc. Q18 in lupus nephr111s. 40t. 41, Q70
Linear scleroderma, SOt In mlcrosooplc poly:angllt!J. 71
Up biopsy. In suspected SjOgren SYOdrome, 43, Q42 In pregnancy, tSt
LOfgren SYn<lrom~. 75-76, Q64 side dfects o( u
Losartan, S8 In S)!Stemic lupus erythematosus. 40t. 41
t..umbosacralradiCillopgthy, 25 In systtmlc sclerosis, 54, QS3
Lupus nephritis, 36 Myocanltal rtbrosiS, 52
Lyme arthritis. 62. Q16 Myocardllls. 37
antibiotics In, 65 Myosltts-speclflc antibodies, 47, 471
Lymphomas, rheumatoid artht1tls and, 20
Lymphopenia/leukopenia. 37 It
NaUcnanges
M In dermatomyositis, 451
Malar rash, 35. 3Sf in }ll'IOrtatlc arthritis, 31, 31f
Marfan syndrome (MFS), 76, 77t Needle aspiration, 65
Ma.~ge.l6 Neisseria gonorrlloeae, 61
for tlbromyalgla, 28 Nephrogenic systt'1l\IC nbrosts. SOt
Mech.:lnlc's hands, ~St Neuropsychiatric systemic lupus erythematosus (NPSLE). 37, Q76
Medltallon, 16 Nlntedanib. 54
6-Mercaptopurlne, In 100-assoclated arthritis, 34 In Idiopathic puJmon:cry llbrosts, 54
Melabollc myopathies. 44t Nongonococcal gram-nea;attve organisms. 62
Metatarsophalangegl (MTP) joint, 56 Nonpharmac:ologlc tber.lpy
Methotrexate. 9, lOt for osteoarthritis, 26
in adult-onset Still disease. 74 for rheumatoid arthritis, t4, 16
and aom1la, 9, Q43 complementary and al!ematlve medlc1nt,l4, 16
disconnect effect, 2l occupational tllempy, 14
in giant cdl arteritis, 67 physlal therapy, 14
In IBD-assoclated 2.rthrit1s. 34 Nonscarrtng alopeda, 36
in lgVI- rtbted disease, 78 NSA1Ds,8
In lnflnmmatory myopathies, 47 ln adult-onset Still disease, 74
in pregnaocy.lSt In a nkyloslng spondylltls. 34. Qn
In psoriatic :~.rthr:ttts, 34 In gout attact<s, S8
In reactlvt arthritis, 34 in IBD·assoctated arthritis, 34
In relapsing po~hondrilis, 74 ln osteoarthr1tls, 26
In rheumatoid :~.rthrilis. 20, Q63, Q80 In pregnancy, 1St
side ~A'ecls of, 9 In psoriatic arthritis. 34
in SjOgren syndrome, 43 ln reactive arthritis, 34
:~nd TN F-a Inhibitors, 21 In relapsing polychondritls, 74
Microscopic polyanglltls (MPA), 70- 71, 701. Q2 In rlleumatold arthritis, 21
din leal mantresuuons and diagnosiS or, 70-71 In sysu:mJc lupus erythematosus, 401, 41

158
Index

topla.l, 26 Porphyromonosgingtoo/ls, 16
side ellects of. 8. QSS Prednisone, 8, Q61
toxk'fty from, 8t In chronic caklum pyrophosphate arthropathy. 59. 601, Q60
Nucleic acid ampiiRa.llon urine testing. rorchlam}'dtal reactive arthritiS, 61, Q4 In giant cell anerttls. 66, Q:ll
In gout, 58
0 In gouty cellulitis, 57- 58, QSI
Occupallona1therapy. 14 In lgG<t-related disease, 78
OUgoarthritls, 2 In polymyali!a rheumatlca, 67
Opiates. in pregnancy, lSI In relapsed polymyalgja rheumatica, 67, Q31
Os1~anhr1t1S (OA}, 21- 27, QlO In rhewnatold arthritis, 21
classllkallon or. 22- 24 In systemic lupus erythematosus. 401
clinical mnnlft:Statlons of, 24. 251 Pregatxlhn. 9
diagnosiS of, 24 26 d.isconUnuatlon of. 9, Q30
d!trcrenllal diagnosis ol, 25 in llbrornyaJgta, 28, QS2
epldem lology of, 22 side effects of, 9. Q30
erosive, 22. Q39 Pregnancy
·gul!-wlng• appearance In finger joints, 22, 23f rheumatoid anhrllls In, 21, Qso
genernlized, 24 rheumatologic medications In, effect of. 14, 1St
gout and, 25 systemic lupus erythenurosus during. 41
Imaging studies In, 25 systemic scltrosis In, 54
taborotory kstlng In, 24- 25 PremUost, in Bth~eJ syndrome, 73
localized. 24 Prlnury angiitis of central na'VOUS system (PACNS). 68-69. Q32
management ot; 26 clfnlcal manifestations and dlogn031s or. 69
lnrra art1l'll1ar Injections, 26-27 rrulrulgement of, 69
nonpharmacologlc therapy, 26, QB Probenecid. 13
oral and topical agents, 26, Q20 In gout, 58
pharmacologic ther~py. 26- 27 ln pregnancy, 1St
surgical therapy, Z1 Procalnamlde. and drug·lnduetd lupus erythematosus. 40t
pathophysiology of, 21- 22 Prosthetic Joint Infections, 63, Q1
primary, 22 !rea tment of, 6S
rndiogrophtc tlndlngs In, 71 Pseudogour, 571, S9, Q28. QS6
risk factors for, 22 P!>oeudomonas oeruglnoso, 62
secondary, 22, 23t ~a-rheumatoid arthritis. 59
weight loss, for OA pneven!ton, Q78 Psor1atlcarthrtlls, 29, 31· 32, Q71, Q84
Osteogenesis lmperfecta (01). 76, 771 dlntcal rearures of. 30t, 31, 31f
Osteonecrosis, 36, Q96 dlnlcal subtypes. 31
Overlap syndrome, S<l, S<lt In HIY, 63
Over·th~unter supplements, 14. 16 nail cmnges ln. 31. 31f
, pencll-ln-cup deformity, 33, 331
prognostic factors ln. 32
Pain radlognphlc findingS m, 7t
In fibromyalgia, 27 ~tment of, 3•
inRarnrnatory versus nontntlammatory. It Pulmonary arten:U hypertension (PAH)
In ost~arttuitls, 24 mixed connective !issue disease and, S4t, SS Q88
In spondyloanhrttls, 29 In systemic sclrmsls, S2. S3t. Q46
p-ANCA anttbudles, 6t Purulent arthritis, 62
Parvovlrus Bl9lnfectlon, 63, Q33
PeglotiC3$e, 13-14 II
In gout, 58, Q3 RA. See Rheumatoid arthr111s (RA)
In pregnancy. 151 Radlograpi\Y, S, 7t
Peptidylarglnine deimlnase (PADI), 16 In ankyloslng spondylitis, 32-33, Q94
PerlcaroJUs. systemic lupus erythematosus 11nd, :P of cervical spine with nexlonlextenslon vl~ws. 18, Q75
Pl:nodlc fever syndromes. See Autolnflammatory dlsea$es ln hlp pain, 8, Q81
Periodontal dtseose, and rheumatoid arthrltls, 16 In osteoarthr1tls, 25, Q47
Peripheral arthrllls. 62 In rheumatoid arthritis, 18, 18f. l9f. Q7S
Pl:s anserine bul'llltls, 25, Q48 In spondyloarthrltts, 33, 33f
Physical therapy, 14 ReactJv~ srthrtlls, 301. 32, Q4
In ankyloslng spondylitis, 34 In Hrv, 62- 63
In Inflammatory !Tl)'Opathles, 48 treatment of, 34
In knee oscooarthr1lls, 14, 26, Q73 Receptor activator of nuc1~~r factor kllppa B llg;lnd (RANKI..),J9
in ost~nrthr!tls, 26, QS !«lapsing pDiychondriUs (RP). 73-74, Q49
Pilocarpine, 43 Rheumatoid anht1tl$ (RA), 16-21, 25, Q12, Q34
Plrfenidone, 54 2010 American College of Rheumatology (ACR) and European League
In IdiOpathic pulmonary flbrosts, 54 Against Rheumatism (EUI.AR) criteria for. 16, 17t
Plasmapheresis, In CJ}'081obullnemlc vascuUtts, 72 Olnlcal Disease Acrlvlty Index (COAl), 20
Pleural effusions, :P clinical manlfe1>tatlons ot 16· 17, 171',18t
In rhcum:llold artJmrts. 19, Q57 d iagnosis or, 16·18
Polkllodermn, 49 Disease Ac!Mty Score 28 (DAS28), 20
Pol}'llttmds ~ (PAN), 68. QJ9 extr.a-:uUcu!:ar manifestations aod complk:ations of, 18-20
dlnlcal features of. 68, 681 blood Ye$SCI.s, 20
dlllgnosis of, 68 eyes,l9
nunagement of, 68 l=rt, 19
Polyanhrltls, 2 hematologlc,19·20
Polyarticular Infectious a rtlrrltls, 61 joints, 18-19
Polymyalgla r heumar:la (PMR), 67 lungs. 19
cUnicaJ nunlfestatklns and dlagn03ts or. 67 skln,19
gtant cell artrrltl.S and, 60 Imaging studies for, 18, tar, 19f
nun:lgt'ment o( 67 Inflammatory casade In, J2f
pathophysiology of, 67 laboratory studies for, 17 18
relapse or. 67, Q31 pathophysiology or, t6
Polymyositis {PM). 43. 45. Set! also lnOllnttn:ltory !Tl)'Opathles In ptl'gll.ancy, 21, QSO

159
Index

Rht'\lmaloid ~rthntis (RA) (Contlnul'Ci) Skin changes


r.~dlographlc flndmgs in. 7t In rheumatoid arlhrltls 19
risk factors for In systemic lupu:. erythematosus. 35-36
envlronmentallactors, 16 SLE. See Syslemic lupus eryt~matnsus (SLEl
genetic IJtllll"o. 16 Slit-lamp examination. 42
hormones. t6 Small Intestine bactenal <M:rgrowth, systemiC sclerosiS and, 52. Ql4
Infectious agents, 16 Small-vessel cutaneous vasculitiS, rheumatoid arthritis and, 20
smoking ttSSatinn. for p~nhon of RA. 16, Q12 Spondyloarthrit!s. 28-29. Q67
t~atment or. 20-21 clnsslllcatlon or. 29, 30t
biologiC DMAROs. 21 ankyloslng ipondylltls, 30-31. 301, QS
glucocorticmds, 21
gu~lln, 20
lnOdmmatory bowel disease-associated arthritis. 301. 32
psoriatic arthrllls, 301, :n-32, 3\f
t
lnil!al approach to. 20, 20f rtnctlvc arthrlll>, 30!, 32, Q4 I
nonblologec DMAROs. 20-21 clinical fcaturesot: 29-JO, 301. 31f
NSAIDs, 21 SOUSSIJle digitS, 29, 311
SU'l:k:nl thempy. 21 lmnglng srudil."$ fur, 33, 33f
Rheumatoid lilctor. 5, 61. 17 Inflammation ol'enthestsln. 2
Rheumatoid nodules. 19 laboratory studies for. 32 -33
Rheumatoid pieurlrls, J9, QS7 monngement of, 34 35
RheumatologJc disease. See Ol$0 specific disease pathophysiology of. 29
constitutional symptoms. 2 envlronmenlallllctors. 29
Imaging modlllllles for~~~ Imaging studi~) genetic f:lcturs, 29
Internal organ involvement In, 2. 4t Staphylococcus ou~us, 61
18borutory resrs for (.l'!'e l:lbomtory ~1udles) Staphyi<X'()("t:US epldermldls, 61
ocu~r manifestations of. 2. 31 St2Uns
skin IOIII)Ivemenlln. 2. 3t and drug-Induced lupus erythc:malosus, 40t
Rllonal't.'pl, 131 myopathlts Induced by. 44t, 46, 47
In p~nancy, 151 S!erold·lnduc~d myofY.lthy, 47
Rltuxomab, 131 Subacute cutnneous lupus erythematosus (SCLEl. 35, 36(,
In cryoglobulfnemlc vasculitiS, 72 Q93
in grnnulnmatosls with polyanglfl1s, 70. Q29 Subcrunial giant cell arteritis. 66, Q17
In JgC4-related dJse~. 78 Sugar-free ca~. 43. Q23
In inflammatory myopathies. 48 Sulfasalat.lne, 9.101
In mtcroscuptc pol)'llnghu~. 11 In 180-as.soc:lated arthritiS. 34
In pregnancy. 1St in pregnancy, 1St
ln S~n syndrome, 43 In psoriatic arthritis, 34
In systemtc sclerosis. 5-'1 In reactl~ arthr1tls, 34
Rubella arthrl!ls. 63 In rheumatoid arthrltls. 20
~I dralna~. for lnrected Joint. 65
5 SurgiCal lheropy, 16
S:lcmlhac joints In osteoarthritis, 17
MRiof,33.Q9 in rheumatoid arthritis. 21
rndlography or. 33. Q94 Synovectomy ofinflammalory pannus, 16
Sarcoidosis, ?S-76 Synovial biopsy, 1, 62, 6S, Q38
and U)fgren syndrome, 7S 76. Q64 Synovial fluid analysis. 7. 7t
radiographic Rndlngs In, 75, 76f Systemic lupus erythematosus (SLE), 3S 42, Q13
Sausage digits, 2 autoantibodies In, 38-39, 391
Schim1er rest. 42 cardiovascular lnlll)lvement ln. 37
Scler<:<l~ma, 501 ci:~Sl;if\catton criteria for, 38
Scler1tls, rheumatoid arthritis and, 19, Q24 ACR crll~rla. 38. 39t
Sclerodactyly. 49 diagnosis or. 38-40
Scleroderm~ rtnnl crisis, 52, 53t, QS8 differential diagnosis of. 39-40, 40t
Scleromyxedema, SOt epidemiology ol'. 35
Screening gastrointestinal lnvol~mentln, 38
for HIV lnl~tlon. 14 hematologic Involvement In, 37-38
for tuberculosis, 14 kidney inlll)lVem~nt In, 36 -37
Secukinumab, 131 l~boratory studies ror, 38-39, 39t
In pregnancy, lSI malignnnc:lcs assoclat~d with. 38
In psoriatic orthrltls, 311 management of, 40-41. 40t
Serotonin-norepinephrine reuplake lnhlblwrs (SNRis) mucocutaneous lnwllvt:mcnt In, 35-36. 3Sf, 36f
tn Hbrom)':llgia, 28 acute cutaneous lupus erythematosus, 35, 3Sf, Q74
Shawl sign, dermatomyositis, 4St discoid lupus erythematosus, 35-36, 36f
Shrinking lung syndrome, 37 subacute cutaneous lupus erylhematos~. 35, 36f, Q93
Slnlography, 42 musculoskeletal Involvement In, 36
SjOgren syndrome, .,2- 43. Q42 neuropsychiAtric Involvement In, 37, (!16
clink.~! manlf~'lah<ms or. .,2 and o:;tt:onecrosls. 36, Q96
extrnglandular ma nlfe81atlons. 42, 42t ~thophystology or. 35
diagnosis of, 42-43, 42t prqnancy and. ~I
dlff~n:ntlal d~is or. 43 prognosis In, ~1-42
epidemiology or. ~2 pulmon:~ry Involvement in, Y1
management or. 4J. Q23 Systemic rheumatologlc d isease, 4~t
and non-Hodgkin lymptlum~. 42 Systemic sclerosis (SSe). 48-54
JX!thophyslology or. 42 ac~teoi)'Sls and. 49, Slf
primary, 42 American College or Rheumatology/European League Against
prognosls for. 43 Rheumatism classlflcat1on crllerb for. 48, 49t
~ndary.42 autoantibodies In, SOt
slcc:a symptom.~ 42. Q23 cardiac lniii)Jvement In, 52
Skeletal tuberculosis. 62 cut:lneou.~ lnvulvt:mentln. 49, Slf
Skin biopsy diRgnosls or, 49, SOl
In dermatomyositis, 47 dlffertntlal diagnosis or, 48. SOt
In lgA wsculltis. n. Q6S dUfuse cutmcous, 48

160
Ind ex

epk.l~mlology ot. 48 u
&"•strointeshn:llln""l~mml in. 51 ·52, Q14 Ultr:lSOOOj!l'llphy. S-6
kldnc:y in,-olv.;,mcntln, 52 fur rheun1.1turd anhrill~. 18
lhnlled cutarn.~ous. ~8. Ql8 Und!lferenll.oled cotuto.•c:llvt 11\\UC: dtsr.tsc: (UCl'O). S4. ~I. QS4
lung involvc:ment on 52 Umte I()W('ring ;~gent'
mall;lgtment of. S3 .~4. S3t ~llupurlnol. 12-IJ. Q87
musculoskeletultnvolv<!mtnlln. 49. Sl . s t r febuxostal. IJ
pathophysiulogy or. 48 In gout. 58
pregnancy !Ind. s~ peglmicasc. 13 14
Rayno~ud phenomenon In, 411. Sl In pregnancy. lSI
systemic sclerosis sine '><leroderm~. ~8. SOt uricosurk agents. t:l
V>tscnlar involl.oement in . Sl. 521' Urlcosurk agents. 13
Systemic lo<'lerosls sine: sderodcrmn.18, SOt llll,'OUI, S8
Ustcklnumab, 131
T in pr<:Jln:'lncy. 1St
Tocrollmus, 11 In psmlatlr arthrlils. 34
Talcttl, 16 Uveoparotid rever. See lleerfunlt ~yndmme
tin tlbromyalgia. 28
lor osteo<~rthrltls, 26 v
Tnl1ayasu arterlli> (TA), 67, QC•2 Vacctnauon. before biologic DMARDs, 14
aortic angiogram In, 671 Vuncomycin.lor melhlc!llln-restscant S. ovreu~. 64t. 65
cll nlc-o~i manJrest3tlons nnd diagnosis ot: 67. 67f Vosculitis. 65
managemtnl ot: 67 dllferenllnl dl~gnc~~ols ol: 6c.t
pathophysiology or. 67 torge-vc:ssel. 66-68
Tempoml arteritis. SeeCiont cell arteritis (GCA) t~lnnt cell nrreriti~. 66 67
'JemporaJ artery biopsy. for gl:~nt «II ~rterllls, 66 polymyatglo rheumatka. 67
TeMflunomlde. 10 T'akoyasu arter1tls. 67.671. Q62
Thtotlde diuretics. ~nd drug lm.lu«d lupus erythematosus. 401 medium w~e1.68-69
11~'t·~ hlOpsy, 7-8 K:lw.ls.tki di<ense. 69, Qn
Tudl17umab, 131 polyarterlfls nod~. 68. Qt9
In giant tell >.trlerltls. 67 pMm:try anRiitl< nt «nl ral nervou.\ ~ystem, 68-69
In pregn~ ncy, ISt serondary, 65, 6SI
Tofaclllnlb, lOt, tt small·vesscl, 69-73
In pregn:~ncy, lSI ANCA·assodatt'd VO$CUIIIIS. 69 71
In rtte'urrullukl 3rlhrili~. 21 cryuglobullnemie v.c;culltis. 11-n
Tophi, S6 hy~rsen~ttlvity vao;cullfis, 72
ToptcJI agents, In pregn:tncy. 1.~1 lgA vJ<culltts, 72
Total hlp arthropl:tsty. 34 Immune cumplex-medlatt<! v~sculllis. 11. 71f
TO(al Joi nt onhropbsty. 16 Vertebral osteomyelitis (l'l>tt's diSC.,!oe) 62
Total Joint replattm«nt, 27 V sign. dermatomyoslll!l. 4St
Tramadol, 9
In ttbromyalgla. 2R w
in osteo:trthrltl\, 26 Wo!termelon Momach. S« Ca.,trll antrJI v:~scul.1r <:C't~Ja (GAVE)
tn pregnan<:y. 1St
Tricyclic antidepressants flt:As). in ftbrumyalgia. 28 )(
Triple tht'ropy, in rheumuttJid orthr1tls, 21 Xanthine oxldose Inhibitors
Troch;lnterlc bursitis, 25 In guut, 58
Tubttt:u lous ~rtebral osteom)'l!lllls. 62. Q68 and probenecid. 58
·rumor necrosis factor (TNF'>-a inhlhitors, II, l:lt X~rostoml:t. 42. S~tC' olso SjOKfen syndrome
In ankyluslng spondylltl$, 34
in 6eh~t syndrome, 7J y
und drug· induced lupus erylhemotosus. 401 Yoga, 16
and methotrexute, ~I (()r Nbromyalgt~. 28
In pregnancy. lSI
In psoriatic arthritis. :14 z
in reactive orthrllls. 34 Zidovudlne. 441
in rheumatoid arthrlll~. 21 Zlko vlru~. 63

161
~AcP·
NAME AND ADDRESS (Please complete.)
Medical
Knowledge
Self-Assessment
First Name Middle Initial American College of Physicians Program·1s
Leading Internal Medicine, Improving Uves

Address
TO EARN CME Credits and/or MOC Points
YOU MUST:
Address cont.
1. Answer a ll questions.
2. Score a minimum of 50% correct.
Ci1y State ZIP Code
------------------------------------------
------~------------ -- ----- - ---------------
TO EARN FREE INSTANTANEOUS
Country
CME Credits and/or MOC Points ONLINE:

1. Answer all of your questions.


Email address 2. Go to mksap.acponline.org and enter your
ACP Online username and password to access
an online answer sheet.
Order Number 3. Enter your answers.
4. You can also enter your answers directly at
(Use the 10-digit Order Number on your mksap.ac:ponline.org without first using this
MKSAP materials packing slip.) answer sheet.
To Submit Your Answer Sheet by Mail or FAX
for a $20 Administrative Fee per Answer Sheet:
1. Answer all of your questions and calculate
ACP ID Number your score.
(Refer to packing slip in your MKSAP materials 2. Complete boxes A-H .
for your 8-digit ACP ID Number.) 3. Complete payment information.
4. Send the answer sheet and payment information to
ACP, using the FAX number/address listed below.

• Required Submission Information if Applying for MOC

Birth Month and Day CD CD ABIM Candidate Number I I


MM DD

COMPLETE FORM BELOW ONLY IF YOU SUBMIT BY MAIL OR FAX


Last Name First Name Ml

Payment Information. Must remit in US funds, drawn on a US bank.


The processing fee for each paper answer sheet is $20.

0 Check, made payable to ACP, enclosed


Chargeto olvtSAI oe D.l l oFc~
Cam Number-------------------------------------------------------------------------------

Expiration Date ___________,/'-------- Security code (3 or 4 digit #s) -------------------


MM yy

Signature----------------------------------------------------------------------------------
Faxto: 215-351-2799 Mail to:
Member and Customer Service
American College of Physicians
190 N. lndepenoence Mall West
Philadelphia, PA 19106-1572
~ ~
Maximum
Number of
TEST TYPE CME CREDITS OR POINTS CLAIMED Enter your score here.
Credits ON SECTION Instructions for calculating
0 Card>ovascular Medic>ne 30 1 hour =1 credit or 1 point your own score are found in front
0 Dermatology 16 of the self-assessment test
0 Gartroenterology and Enter the number of in each book. You must receive
Hepatology 22 credits earned on a minimum score of 50% correct
0 Hematology and Oncology
0Neurology
33
22
the test to the nearest
quarter hour. Physicians
I JI I _ _ _ __ %
Credit Submission Date: _ _ _
0 Rheumatology 22 should claim only the
0 Endocrinology and Metabolism 19 credit commensurate
0 Generallntemai Med>elne 36 with the extent of
0 Infectious Disease 25 their participation in
the activity.
JIIOI want to submit for CME credits
0 Nephrology 25
0 Pulmonary and Critical
0 I want to submit for CME credits
Care Medicine 25 and MOC points.

1 QD <i> ~ <ID CID 46 00 (i) CO CID (I) 91 (A)Ci)~(Q)<D 136 00 (j) ~ (Q) ill
2 (A) (j) a:> <ID CE> n(A)(j)CQ@CD 92(A)Ci)C0@Cf.) 137(A)Ci)(C)@<I)
3 (A) (i) <0 (Q) (I) 48 (1) (i) <:0 CID CD 93 (A)(j)~(Q)<D n8 (A) Ci) co <ID <I)
4 <A> (j) ro CID oo 49 00 ()!) ~ CIDCD 94 00 CJD CO CID CD 139oo au ro <W CD
s oo rn ro av CID so 00 (i) <:0 <ID (I) 9s 00 (I) CCJ CID CD 140 00 Ci) (f) (Q) (!)

6 oo w ro av CID s1 ro ro
Cil <ID <D 96 oo oo ro
CiD <D t41oo ro
CID <ID <ID
7 (A) (i) ~ CiD <D 52 00 (i) co@(£) 97 oo Ci> co av oo 142 ® (i)ro <ID <D
8 (i) (j) ~ <ID CI> s3 m rn row CID 98 00 CiD CO <ID CD 1-uoo rn ro m CID
9 oo rn ro cw oo 54 CD CiD ID <ID (I) 99(A)(j)(f)(ID(£) 1" oo ro av
(j) <D
10 <A> Cli) ~ CID CI> ss OOC])COCIDCD 100 <D Ci) co
(Jj) <D 145 ® Cli) <0 <ID 00

11 CDClD~<IDOO 56 (D(j)(V@C£) 101 OOCID~@CD 146 QD CID ID CID <D


12 <A> CID ID <ID (I) 57 (A) (I) (C)@(£) 102 00 Ci) co(lj) <D 147 <D Ci> ro<ID <D
13 <A> Cli) co@ (I) sa <D<i>ID<ID CD 103 00 W ct:> CD) CID 148 <A> Cl> co<ID ([)
14 <A> Cl> (f) <ID (E) 59 <A> CiD© <ID CD 104 <A> w ro av CJD 149(i)(j)(O@(£)
15 (J;) (!) (t) <lD (E) 60 <D (])(f) <lD CE> 105 <!".)())co(lj) <D 1so <A> C1> IDem C1D

16<i)(i)~@C£) 61 (i)(i)(f)@(i) 106 a:> Ci> CO <.ID <D 1 s1 ® (ij) ro <.ID <D
11 <A> Cii) ~ CID CD 62 <A>CiliD CID CD 101 oo rn ro <ID CD 1s2 <A> CID ID CID CD
ta(i)(j)(C)@(£) 63 00 Ci> CO CID CD 1oe 00 (i) ID@ <D 153 <A> (j) ro av oo
19 (A) (j) CC) <ID 00 64 (1) (i) co
<ID (£) 1o9 <A> CiD ID <IV CD 154 oo CIDro@ <D
zoOO(i)CO<ID<D 65 <D 00 CO CID <D , o <A> oo ro av w 1 ss <A> (i) ID CID <D

21 <DCi>ID<IDCI> 66®(i)CO@<D 111 CA> rn ro <ID <D 1S6(i)(i) (C)@(£)


22 ooCil ro<IDCD 67 00 CID CO CID CID 112 (i) (i) (f) <ID <D 157(i)C])~(ID QD
u<A>CilCO<IDCD 68(J)(j)(f)@CD 113 <A> (j) (f) (ji) (!) 1sa 00 (I) CO CID <D
24 <A> Cll CO<JD 00 69 (1) (i) CO <lD CD 114 <A>(][) co <ID <D 1S9(1)(i)(O<JD([)
zs <A> Cli) ~ CID CID 70 <A> (j) co (ti) (£) 1u oo ro
CID CID m 160 <A> CID ~ CID <D

26 <A> (i) (f) <ID <D 11 <A> rn ro


<ID rn 11600(i)(I:)@CE) 161 <DCID~<IDCD
21 <A> CID CO CID CD 12 (1) Cli) ro
CID CD 111<D<1>CO<D'><D 162 oo w ro crv m
28 <A> CID ID <ID CID 73 <A> (i) COCID CD 11a oo w ro <ID <D 163 <D CID ID <ID <D
29 oo
(i) ro <ID rn 74 oo cv ro CID CD co
119 <A> (I) <Ji) (£) 164 <A>w ro m <W
30 00 (i) CO <ID CD 75 (A)(i)c.QCIDCD 120 <A> (I) CO <IV CD 165 ® <ID (f)@([)

31 (A)(]) co<ID (I) 76 <D Ci) ID (Q) CD 12t (i)<i)(f)(Q)([) 166 <A> (i) co(])) <I)
32<i)(I)(O@<D 77 <D (])co Cli) (£) 122 <i><ID co av <D 167 <A>(]) co (Jj)(I)
33 00 Cll CO CID CD 78 <A> (][) co <ri> CE> 123 00 (I) co <Ji) Cf) 168 oo (j)ro CID m
34 <A> (j) CO GD CD 79 <A> (i) <J:) (jj) CE> 124 (A) (j) (f) <ID (!) 169 00 (]) ID GD (£)
35 00 Cil W CID CD ao <A> (][) CO <ID CD 12S00Ci)~<fi)(ID 110 <A> Cli) ~ CID <D

36 CD (]) CO <ID CD 81 <A> (i) co


<li)(i) 126 <A> (i) CO CID <D 111 <A> (i) ID <ID <I)
37 00 (j) <:0 CID 00 U®CJ)CO(Q)(£) 121 <A>® ro m <ID 112 QD (I) CO (jj) CD
38 <D Cli) <0 CID CI> uCD(i)(f)<Ji)C£> 12a oo w co av m 173 <A> <D CO CID (£)
39 <A> (i) G!) CID CD 84(A)(i)(O<Ji)(ID 129 ® <JD a::> CID <D 174(A)(i)(O(ID(f)
40 <D Ci> <:0 crv m 85 <A>(]) co
<W <D 13000(J)<J:)@CD 175 ®(])CO <ID CD

41 <A>CV~CID<D 86 <A> (j) <0 CID <D 131 <A> (I)~ <ID (f.) t 76®<i>ID<IDCO
-42 CD (J) CO CID CD 87 <D (I) (0 (lj) (I) 132 <A> (][) (0 <Ji) CJD ' " ® (J) <:0 <Ji) <D
43 <D Cli) W CID CD 88 00 <I> <0 <ID CD t33 <A> (J) (0 (Q) <D 178 <A> (I)co (Jj) <D
44(i)(i)(f)(ID(j) n®(i)CO(Q)CE> 134 <A> (j) co <ID (i) 179 CD (i) (f) <ID CD
45 <i)(I)<D@ CD 90 (i) (j) co
(jj) (!) 135 (i)(]i) co@ <D tao®<DID<m<D

MK7010
ACP MKSAP.

ACP.
5-53-4

American College of Physicians


Leading Internal Medicine, Improving Lives 9 ~~n

S-ar putea să vă placă și